Big Ideas Math Answers Grade 2 Chapter 1 Numbers and Arrays

Big Ideas Math Answers Grade 2 Chapter 1

Big Ideas Math Answers 2nd Grade 1st Chapter Numbers and Arrays PDF is provided here. We have given the solutions to all the questions in Big Ideas Math Answers Grade 2 Chapter 1 Numbers and Arrays in pdf format. BIM Grade 2 Textbook Chapter 1 Numbers and Arrays Answer Key helps the students to complete their homework in time and also to enhance their knowledge. It will also useful to learn the concepts in depth.

Download Big Ideas Math Book 2nd Grade Answer Key Chapter 1 Numbers and Arrays PDF

Apply the real-time math examples by learning the tricks using BIM Grade 2 Chapter 1 Numbers and Arrays. Have a look at the topics before you start the preparation. The quick method of solving math questions si helpful to the students to save their time. We have provided the answers for each and every section of chapter 1 numbers and arrays in the following sections.

The topics covered in Bid Ideas Math Book Grade 2 Answer Key Chapter 1 Numbers and Arrays are Even and Odd Numbers, Model Even and Odd Numbers, Equal Groups, Use Arrays, and Make Arrays. The performance task given at the end helps you to test your skills. Make use of the below links and learn the basic topics covered here.

Vocabulary

Lesson 1: Even and Odd Numbers

Lesson 2: Model Even and Odd Numbers

Lesson 3: Equal Groups

Lesson 4: Use Arrays

Lesson 5: Make Arrays

Performance Task

Numbers and Arrays Vocabulary

Big Ideas Math Answers Grade 2 Chapter 1 Numbers and Arrays 1

Organize It

Use the review words to complete the graphic organizer.
Big Ideas Math Solutions Grade 2 Chapter 1 Numbers and Arrays 2
Answer: Plus sign
‘+’ is the mathematical symbol used to represent the notion of positive as well as the operation of addition.

Define It

Use your vocabulary cards to identify the word.

Question 1.
Big Ideas Math Solutions Grade 2 Chapter 1 Numbers and Arrays 3
Answer: Even

Question 2.
Big Ideas Math Solutions Grade 2 Chapter 1 Numbers and Arrays 4
Answer: Even

Question 3.
Big Ideas Math Solutions Grade 2 Chapter 1 Numbers and Arrays 5
Answer: Odd

Chapter 1 Vocabulary Cards

Big Ideas Math Solutions Grade 2 Chapter 1 Numbers and Arrays 6
Big Ideas Math Solutions Grade 2 Chapter 1 Numbers and Arrays 7
Big Ideas Math Solutions Grade 2 Chapter 1 Numbers and Arrays 8

Lesson 1.1 Even and Odd Numbers

Explore and Grow

Use linking cubes to model each story.
There are 6 students in the gym. Does each student have a partner?
There are 5 students in the library. Does each student have a partner?

Think and Grow

Big Ideas Math Solutions Grade 2 Chapter 1 Numbers and Arrays 9

Show and Grow

Question 1.
Big Ideas Math Solutions Grade 2 Chapter 1 Numbers and Arrays 10

Answer: Even

Big-Ideas-Math-Solutions-Grade-2-Chapter-1-Numbers-and-Arrays-10-1-Answer

Explanation: There are 6 parts, An even number can be shown as 2 equal parts.

Question 2.
Big Ideas Math Solutions Grade 2 Chapter 1 Numbers and Arrays 11

Answer: Odd
Big-Ideas-Math-Solutions-Grade-2-Chapter-1-Numbers-and-Arrays-11-Answer
Explanation: There are 9 parts, An odd number cannot be shown as 2 equal parts.

Color cubes to show the number. Circle even or odd.

Question 3.
Big Ideas Math Solutions Grade 2 Chapter 1 Numbers and Arrays 12

Answer: Odd
Big-Ideas-Math-Solutions-Grade-2-Chapter-1-Numbers-and-Arrays-12-Answer
Explanation: 11 is odd number and cannot be shown as 2 equal parts.

Question 4.
Big Ideas Math Solutions Grade 2 Chapter 1 Numbers and Arrays 13

Answer: Even
Big-Ideas-Math-Solutions-Grade-2-Chapter-1-Numbers-and-Arrays-13-Answer
Explanation: 16 is even number and can be shown as 2 equal parts.

Apply and Grow: Practice

Color cubes to show the number. Circle even or odd.

Question 5.
Big Ideas Math Solutions Grade 2 Chapter 1 Numbers and Arrays 14

Answer: Odd
Big-Ideas-Math-Solutions-Grade-2-Chapter-1-Numbers-and-Arrays-14-Answer
Explanation: 13 is odd number and cannot be shown as 2 equal parts.

Question 6.
Big Ideas Math Solutions Grade 2 Chapter 1 Numbers and Arrays 15

Answer: Even
Big-Ideas-Math-Solutions-Grade-2-Chapter-1-Numbers-and-Arrays-15-Answer
Explanation: 10 is even number and can be shown as 2 equal parts.

Is the number even or odd?

Question 7.
Big Ideas Math Solutions Grade 2 Chapter 1 Numbers and Arrays 16

Answer: Odd
Big-Ideas-Math-Solutions-Grade-2-Chapter-1-Numbers-and-Arrays-16-Answer
Explanation: 1 is odd number and cannot be shown as 2 equal parts.

Question 8.
Big Ideas Math Solutions Grade 2 Chapter 1 Numbers and Arrays 17

Answer: Even
Big-Ideas-Math-Solutions-Grade-2-Chapter-1-Numbers-and-Arrays-17-Answer
Explanation: 4 is even number and can be shown as 2 equal parts.

Question 9.
Big Ideas Math Solutions Grade 2 Chapter 1 Numbers and Arrays 18

Answer: Even
Big-Ideas-Math-Solutions-Grade-2-Chapter-1-Numbers-and-Arrays-18-Answer
Explanation: 18 is even number and can be shown as 2 equal parts.

Question 10.
Big Ideas Math Solutions Grade 2 Chapter 1 Numbers and Arrays 19

Answer: Odd
Big-Ideas-Math-Solutions-Grade-2-Chapter-1-Numbers-and-Arrays-19-Answer
Explanation: 17 is odd number and cannot be shown as 2 equal parts.

Question 11.
Big Ideas Math Solutions Grade 2 Chapter 1 Numbers and Arrays 20

Answer: Odd
Big-Ideas-Math-Solutions-Grade-2-Chapter-1-Numbers-and-Arrays-20-Answer
Explanation: 19 is odd number and cannot be shown as 2 equal parts.

Question 12.
Big Ideas Math Solutions Grade 2 Chapter 1 Numbers and Arrays 21

Answer: Even
Big-Ideas-Math-Solutions-Grade-2-Chapter-1-Numbers-and-Arrays-21-Answer
Explanation: 20 is even number and can be shown as 2 equal parts.

Question 13.
Number Sense
Circle even or odd to describe each group. Then write each number in the correct group.
Big Ideas Math Solutions Grade 2 Chapter 1 Numbers and Arrays 22

Answer: Big-Ideas-Math-Solutions-Grade-2-Chapter-1-Numbers-and-Arrays-22-Answer

Think and Grow: Modeling Real Life

There is an even number of students in your class. There are more than 16 but fewer than 20 students. How many students are in your class?
Big Ideas Math Solutions Grade 2 Chapter 1 Numbers and Arrays 23

Answer: 18 Students
Explanation: Between 16 and 20, Even number is 18.

Show how you know:

Show and Grow

Question 14.
There is an odd number of cows in a field. There are more than 13 but fewer than 17 cows. How many cows are in the field?
Big Ideas Math Solutions Grade 2 Chapter 1 Numbers and Arrays 24

Answer: 15 cows
Explanation: Between 13 and 17, Odd number is 15.

Question 15.
There are 14 geese on a farm. There are 2 more chickens than geese. Is there an even or odd number of chickens?
Big Ideas Math Solutions Grade 2 Chapter 1 Numbers and Arrays 25

Big Ideas Math Solutions Grade 2 Chapter 1 Numbers and Arrays 26

Answer: Even
Explanation: There are total 16 Chickens on a farm, which is Even number.

Even and Odd Numbers Home & Practice 1.1

Question 1.
Big Ideas Math Solutions Grade 2 Chapter 1 Numbers and Arrays 27

Answer: Odd
Big-Ideas-Math-Solutions-Grade-2-Chapter-1-Numbers-and-Arrays-27-Answer
Explanation: Total 11 parts, which cannot be shown as 2 equal parts.

Question 2.
Big Ideas Math Solutions Grade 2 Chapter 1 Numbers and Arrays 28

Answer: Even
Big-Ideas-Math-Solutions-Grade-2-Chapter-1-Numbers-and-Arrays-28-Answer
Explanation: Total 10 parts, which can be shown as 2 equal parts.

Color cubes to show the number. Circle even or Odd

Question 3.
Big Ideas Math Solutions Grade 2 Chapter 1 Numbers and Arrays 29

Answer: Odd
Big-Ideas-Math-Solutions-Grade-2-Chapter-1-Numbers-and-Arrays-29-Answer
Explanation: 9 is Odd number which cannot be shown in 2 equal parts.

Question 4.
Big Ideas Math Solutions Grade 2 Chapter 1 Numbers and Arrays 30

Answer: Even
Big-Ideas-Math-Solutions-Grade-2-Chapter-1-Numbers-and-Arrays-30-Answer
Explanation: 14 is Even number which can be shown in 2 equal parts.

Question 5.
Big Ideas Math Solutions Grade 2 Chapter 1 Numbers and Arrays 31

Answer: Even
Big-Ideas-Math-Solutions-Grade-2-Chapter-1-Numbers-and-Arrays-31-Answer
Explanation: 18 is Even number which can be shown in 2 equal parts.

Question 6.
Big Ideas Math Solutions Grade 2 Chapter 1 Numbers and Arrays 32

Answer: Odd
Big-Ideas-Math-Solutions-Grade-2-Chapter-1-Numbers-and-Arrays-32-Answer
Explanation: 15 is Odd number which cannot be shown in 2 equal parts.

Is the number even or odd?

Question 7.
Big Ideas Math Solutions Grade 2 Chapter 1 Numbers and Arrays 33

Answer: Even
Big-Ideas-Math-Solutions-Grade-2-Chapter-1-Numbers-and-Arrays-33-Answer
Explanation: 2 is Even number which can be shown in 2 equal parts.

Question 8.
Big Ideas Math Solutions Grade 2 Chapter 1 Numbers and Arrays 34

Answer: Odd
Big-Ideas-Math-Solutions-Grade-2-Chapter-1-Numbers-and-Arrays-34-Answer
Explanation: 5 is Odd number which cannot be shown in 2 equal parts.

Review & Refresh

Question 9.
DIG DEEPER!
You break apart a linking cube train to make two equal parts. There is 1 cube left over. Is the number of cubes even or odd? Explain.
Big Ideas Math Solutions Grade 2 Chapter 1 Numbers and Arrays 35

Answer: Odd
Explanation: There are 5 parts in cube train, when it is broken in 2 equal parts then there is 1 cube left (2+2+1) which is Odd number.

Question 10.
Modeling Real Life
There is an even number of eggs in a nest. There are more than 10 but fewer than 14 eggs. How many eggs are in the nest?
Big Ideas Math Solutions Grade 2 Chapter 1 Numbers and Arrays 36

Answer: 12 eggs
Explanation: 12 is even number and also between 10 and 14, 12 is the only even number.

Question 11.
Modeling Real Life
You have 6 green crayons. You have 1 more blue crayon than green crayons. Do you have an even or an odd number of blue crayons?
Big Ideas Math Solutions Grade 2 Chapter 1 Numbers and Arrays 37
Big Ideas Math Solutions Grade 2 Chapter 1 Numbers and Arrays 38

Answer: Odd
Big-Ideas-Math-Solutions-Grade-2-Chapter-1-Numbers-and-Arrays-37-Answer
Explanation:  Total 7 blue crayon which is Odd number so it cannot be shown in 2 equal parts.

Review & Refresh

Question 12.
6 + 6 = ___
Answer: 12
Explanation: 12 is Even number which can be shown in 2 equal parts.

Question 13.
9 + 9 = ___
Answer: 18
Explanation: 18 is Even number which can be shown in 2 equal parts.

Question 14.
7 + 7 = ___
Answer: 14
Explanation: 14 is Even number which can be shown in 2 equal parts.

Question 15.
8 + 8 = ___
Answer: 16
Explanation: 16 is Even number which can be shown in 2 equal parts.

Lesson 1.2 Model Even and Odd Numbers

Explore and Grow

Use linking cubes to model each sum. Is the sum even or odd?

Big Ideas Math Solutions Grade 2 Chapter 1 Numbers and Arrays 19.1

Answer: Even
Big-Ideas-Math-Solutions-Grade-2-Chapter-1-Numbers-and-Arrays-20.1-Answer
Explanation: 4+4=8, 8 is Even number which can be shown in 2 equal parts.

Big Ideas Math Solutions Grade 2 Chapter 1 Numbers and Arrays 20.1

Answer: Odd
Big-Ideas-Math-Solutions-Grade-2-Chapter-1-Numbers-and-Arrays-20.1--Answer
Explanation: 5+4=9, 9 is Odd number which cannot be shown in 2 equal parts.

Show and Grow

Question 1.
Big Ideas Math Solutions Grade 2 Chapter 1 Numbers and Arrays 22.1

Answer: Odd
Big-Ideas-Math-Solutions-Grade-2-Chapter-1-Numbers-and-Arrays-22.1-Answer
Explanation: 7=4+3, Odd number

Question 2.
Big Ideas Math Solutions Grade 2 Chapter 1 Numbers and Arrays 23.1

Answer: Even
Big-Ideas-Math-Solutions-Grade-2-Chapter-1-Numbers-and-Arrays-23.1-Answer
Explanation: 10=5+5, Even number

Question 3.
Big Ideas Math Solutions Grade 2 Chapter 1 Numbers and Arrays 24.1
Answer: Even

Big-Ideas-Math-Solutions-Grade-2-Chapter-1-Numbers-and-Arrays-24.1-Answer
Explanation: 14=7+7, Even number

Question 4.
Big Ideas Math Solutions Grade 2 Chapter 1 Numbers and Arrays 25.1
Answer: Odd

Big-Ideas-Math-Solutions-Grade-2-Chapter-1-Numbers-and-Arrays-25.1-Answer

Explanation: 17=9+8, Odd number

Question 5.
Big Ideas Math Solutions Grade 2 Chapter 1 Numbers and Arrays 26.1

Answer: Even
Big-Ideas-Math-Solutions-Grade-2-Chapter-1-Numbers-and-Arrays-26.1-Answer
Explanation: 18=9+9, Even number

Question 6.
Big Ideas Math Solutions Grade 2 Chapter 1 Numbers and Arrays 27.1

Answer: Odd
Big-Ideas-Math-Solutions-Grade-2-Chapter-1-Numbers-and-Arrays-27.1-Answer
Explanation: 13=7+6, Odd number

Question 7.
Big Ideas Math Solutions Grade 2 Chapter 1 Numbers and Arrays 28.1

Answer: Odd
Big-Ideas-Math-Solutions-Grade-2-Chapter-1-Numbers-and-Arrays-28.1-Answer
Explanation: 15=8+7, Odd number

Question 8.
Big Ideas Math Solutions Grade 2 Chapter 1 Numbers and Arrays 29.1

Answer: Even
Big-Ideas-Math-Solutions-Grade-2-Chapter-1-Numbers-and-Arrays-29.1-Answer

Explanation: 20=10+10, Even number

Question 9.
YOU BE THE TEACHER
Descartes uses doubles plus 1 to model an odd number. Is he correct? Explain.
Big Ideas Math Solutions Grade 2 Chapter 1 Numbers and Arrays 30.1

Answer: Yes
Explanation: 3+4=7, Odd number which cannot be shown as 2 equal parts.

Question 10.
You do 6 sit-ups on Saturday and 7 on Sunday. Do you do an even or odd number of sit-ups in all?
Big Ideas Math Solutions Grade 2 Chapter 1 Numbers and Arrays 31.1

Answer: Big-Ideas-Math-Solutions-Grade-2-Chapter-1-Numbers-and-Arrays-31.1-Answer
Explanation: 6+7=13, It is an Odd number

Think and Grow: Modeling Real Life

There is an even number of marbles in one bag and an odd number of marbles in another bag. Is there an even or an odd number of marbles in all?
Big Ideas Math Solutions Grade 2 Chapter 1 Numbers and Arrays 32.1

Which equation could match the story?
Big Ideas Math Solutions Grade 2 Chapter 1 Numbers and Arrays 33.1
There is an ___ number of marbles in all.
Answer: Odd
Explanation: 8+7=15, 8 is even and 7 is odd number, in total 15 is Odd number of marbles.

Show and Grow

Question 11.
Two buckets each have an odd number of seashells. Is there an even or an odd number of seashells in all?
Big Ideas Math Solutions Grade 2 Chapter 1 Numbers and Arrays 34.1
Which equation could match the story?
Big Ideas Math Solutions Grade 2 Chapter 1 Numbers and Arrays 35.1
There is an __ number of seashells in all.
Answer: Odd
Explanation: 9+5=14, 9 and 5 are odd numbers which in total 14 is Even number.

Question 12.
DIG DEEPER!
You have an odd number of flowers. You and your friend have an even number of flowers in all. Does your friend have an even or an odd number of flowers?
Big Ideas Math Solutions Grade 2 Chapter 1 Numbers and Arrays 36.1
Your friend has an ___ number of flowers.
Answer: Even
Explanation: Friend has only even number of flowers.

Model Even and Odd Numbers Homework & Practice 1.2

Question 1.
Big Ideas Math Solutions Grade 2 Chapter 1 Numbers and Arrays 37.1
Answer: Even
Big-Ideas-Math-Solutions-Grade-2-Chapter-1-Numbers-and-Arrays-37.1-Answer
Explanation: 8=4+4, Even number

Question 2.
Big Ideas Math Solutions Grade 2 Chapter 1 Numbers and Arrays 38.1

Answer: Odd
Big-Ideas-Math-Solutions-Grade-2-Chapter-1-Numbers-and-Arrays-38.1-Answer
Explanation: 17=9+8, Odd number

Question 3.
Big Ideas Math Solutions Grade 2 Chapter 1 Numbers and Arrays 39

Answer: Odd
Big-Ideas-Math-Solutions-Grade-2-Chapter-1-Numbers-and-Arrays-39-Answer
Explanation: 9=5+4, Odd number

Question 4.
Big Ideas Math Solutions Grade 2 Chapter 1 Numbers and Arrays 40

Answer: Even
Big-Ideas-Math-Solutions-Grade-2-Chapter-1-Numbers-and-Arrays-40-Answer
Explanation: 16=8+8, Even number

Question 5.
Reasoning
Fill in the blanks using even or odd.
The sum of two even numbers is ___.
Answer: Even
The sum of two odd numbers is ___.
Answer: Even
The sum of an even number and an odd number is __.
Answer: Odd

Question 6.
You do 10 jumping jacks on Saturday and 10 on Sunday. Do you do an even or odd number of jumping jacks in all?
Big Ideas Math Solutions Grade 2 Chapter 1 Numbers and Arrays 41

Answer: Even
Explanation: 10+10=20, which is Even number.

Question 7.
Modeling Real Life
You and your friend each have an even number of googly eyes. Do you and your friend have an even or an odd number of googly eyes in all?
Big Ideas Math Solutions Grade 2 Chapter 1 Numbers and Arrays 42

Which equation could match the story?
Big Ideas Math Solutions Grade 2 Chapter 1 Numbers and Arrays 43
You have an __ number of googly eyes in all.
Answer: 4+6=10
Explanation: 4 and 6 are Even numbers. So totally 10 googly eyes which is even number.

Question 8.
DIG DEEPER!
You hop an even number of times. You and your friend hop an odd number of times in all. Does your friend hop an even or an odd number of times?
Your friend hops an ___ number of times.
Answer: Your Friend hops an Odd number of times.
Explanation: you hop an even and overall odd number, so your friend has odd number of times.

Review & Refresh

Circle the shape that shows equal shares.

Question 9.
Big Ideas Math Solutions Grade 2 Chapter 1 Numbers and Arrays 44
Answer: Big-Ideas-Math-Solutions-Grade-2-Chapter-1-Numbers-and-Arrays-44-Answer

Question 10.
Big Ideas Math Solutions Grade 2 Chapter 1 Numbers and Arrays 45

Answer: Big-Ideas-Math-Solutions-Grade-2-Chapter-1-Numbers-and-Arrays-45-Answer

Lesson 1.3 Equal Groups

Explore and Grow

Circle groups of two oranges. Complete the sentence.
Big Ideas Math Solutions Grade 2 Chapter 1 Numbers and Arrays 46
Big Ideas Math Solutions Grade 2 Chapter 1 Numbers and Arrays 47

Answer: 4 groups of 2 is 8.

Big-Ideas-Math-Solutions-Grade-2-Chapter-1-Numbers-and-Arrays-46-Answer

Show and Grow

Question 1.
Big Ideas Math Solutions Grade 2 Chapter 1 Numbers and Arrays 49

Answer: 5 groups of 2
5+5=10

Question 2.
Big Ideas Math Solutions Grade 2 Chapter 1 Numbers and Arrays 50

Answer: 2 groups of 3
2+2+2=6

Question 3.
Circle groups of 3. Write a repeated addition equation.
Big Ideas Math Solutions Grade 2 Chapter 1 Numbers and Arrays 51

Answer: 5 groups of 3
5+5+5+5+5=25
Big-Ideas-Math-Solutions-Grade-2-Chapter-1-Numbers-and-Arrays-51-Answer

Apply and Grow: Practice

Question 4.
Circle groups of 5. Write a repeated addition equation.
Big Ideas Math Answers Grade 2 Chapter 1 Numbers and Arrays 52

Answer: 3 groups of 5
5+5=15
Big-Ideas-Math-Answers-Grade-2-Chapter-1-Numbers-and-Arrays-52-Answer

Question 5.
Circle groups of 4. Write a repeated addition equation.
Big Ideas Math Answers Grade 2 Chapter 1 Numbers and Arrays 53

Answer: 4 groups of 4
Answer: 4+4+4+4=16
Big-Ideas-Math-Answers-Grade-2-Chapter-1-Numbers-and-Arrays-53-Answer

Question 6.
YOU BE THE TEACHER
Newton says he can circle 5 equal groups. Is he correct? Explain.
Big Ideas Math Answers Grade 2 Chapter 1 Numbers and Arrays 54

Answer:
Big-Ideas-Math-Answers-Grade-2-Chapter-1-Numbers-and-Arrays-54-Answer
Yes he is correct. Newton can circle 3 groups of 5 parts. So, 3+3+3+3+3=15

Think and Grow: Modeling Real Life

You have 3 boxes. There are 5 pencils in each box. How many pencils are there in all?
Big Ideas Math Answers Grade 2 Chapter 1 Numbers and Arrays 55
Model:
Repeated addition equation:
Big Ideas Math Answers Grade 2 Chapter 1 Numbers and Arrays 56

Answer: 15 pencils in total. 5+5+5=15.

Show and Grow

Question 7.
You have 5 bags. There are 4 notebooks in each bag. How many notebooks are there in all?
Big Ideas Math Answers Grade 2 Chapter 1 Numbers and Arrays 57
Big Ideas Math Answers Grade 2 Chapter 1 Numbers and Arrays 58

Answer: 20 notebooks in total. 5+5+5+5=20.

Question 8.
DIG DEEPER!
There are 4 boxes. Each box has the same number of glue sticks. There are 16 in all. How many glue sticks are in each box?
Big Ideas Math Answers Grade 2 Chapter 1 Numbers and Arrays 59

Answer: There are 4 glue sticks in each box.

Question 9.
Explain how you solved Exercises 7 and 8. What did you do differently?
_______________________________________
_______________________________________

Answer: Adding the number with the given number of times.

Equal Groups Homework & Practice 1.3

Question 1.
Big Ideas Math Answers Grade 2 Chapter 1 Numbers and Arrays 60

Answer: 3 groups of 2
3+3=6

Question 2.
Big Ideas Math Answers Grade 2 Chapter 1 Numbers and Arrays 61

Answer: 4 groups of 3
4+4+4=12

Question 3.
Circle groups of 2. Write a repeated addition equation.
Big Ideas Math Answers Grade 2 Chapter 1 Numbers and Arrays 62

Answer: 4 groups of 2
Big-Ideas-Math-Answers-Grade-2-Chapter-1-Numbers-and-Arrays-62-Answer
Answer: 2+2+2+2=8

Question 4.
Structure
Show two different ways to put the buttons in equal groups.
One Way:
Big Ideas Math Answers Grade 2 Chapter 1 Numbers and Arrays 63
Another Way:
Big Ideas Math Answers Grade 2 Chapter 1 Numbers and Arrays 64

Answer: Big-Ideas-Math-Answers-Grade-2-Chapter-1-Numbers-and-Arrays-63-Answer

Question 5.
Modeling Real Life
You have 3 jars of paint brushes.There are 6 paint brushes in each jar. How many paint brushes are there in all?
Big Ideas Math Answers Grade 2 Chapter 1 Numbers and Arrays 65

Answer: 18 paintbrushes
Explanation: 6+6+6=18
Review & Refresh

Question 6.
Big Ideas Math Answers Grade 2 Chapter 1 Numbers and Arrays 66
How many students chose baseball? ___

Answer: 5 students chose baseball
Which sport is the most favorite? Big Ideas Math Answers Grade 2 Chapter 1 Numbers and Arrays 67

Answer: Favorite sport is Soccer

Lesson 1.4 Use Arrays

Explore and Grow

How many equal groups are there? Write an addition equation to tell how many cars there are in all.
Big Ideas Math Answers Grade 2 Chapter 1 Numbers and Arrays 67.1
Number of equal groups: ___
Answer: 2
Addition equation:
3+3=6

Show and Grow

Question 1.
Big Ideas Math Answers Grade 2 Chapter 1 Numbers and Arrays 68

Answer: 2 rows of 4
4+4=8

Question 2.
Big Ideas Math Answers Grade 2 Chapter 1 Numbers and Arrays 69

Answer: 3 rows of 4
4+4+4=12

Question 3.
Big Ideas Math Answers Grade 2 Chapter 1 Numbers and Arrays 70

Answer: 4 rows of 4
4+4+4+4=16

Apply and Grow: Practice

Question 4.
Big Ideas Math Answers 2nd Grade Chapter 1 Numbers and Arrays 71

Answer: 2 rows of 5
5+5=10

Question 5.
Big Ideas Math Answers 2nd Grade Chapter 1 Numbers and Arrays 72

Answer: 3 rows of 3
3+3+3=9

Question 6.
Big Ideas Math Answers 2nd Grade Chapter 1 Numbers and Arrays 73

Answer: 3 rows of 5
5+5+5=15

Question 7.
Big Ideas Math Answers 2nd Grade Chapter 1 Numbers and Arrays 74

Answer: 5 rows of 4
5+5+5+5=20

Question 8.
Logic
Which arrays show the same number of circles?
Big Ideas Math Answers 2nd Grade Chapter 1 Numbers and Arrays 75

Answer: Red and Blue circles show the same number.

Think and Grow: Modeling Real Life

The arrays show the desks in two classrooms. Which classroom has more desks?
Big Ideas Math Answers 2nd Grade Chapter 1 Numbers and Arrays 76
Repeated addition equations:
Big Ideas Math Answers 2nd Grade Chapter 1 Numbers and Arrays 77

Classroom A: 5+5+5+5+5=25
Classroom B: 7+7+7+7=28
Answer: Classroom B has more number of desks.

Show and Grow

Question 9.
The arrays show gardens of green and yellow pepper plants. Are there more green pepper plants or yellow pepper plants?
Big Ideas Math Answers 2nd Grade Chapter 1 Numbers and Arrays 78

Answer: Green pepper plants
Explanation: There are 15 Green pepper plants and 12 Yellow pepper plants.

Use Arrays Homework & Practice 1.4

Question 1.
Big Ideas Math Answers 2nd Grade Chapter 1 Numbers and Arrays 79

Answer: 3 rows of 2
2+2+2=6

Question 2.
Big Ideas Math Answers 2nd Grade Chapter 1 Numbers and Arrays 80

Answer: 2 rows of 2
2+2=4

Question 3.
Big Ideas Math Answers 2nd Grade Chapter 1 Numbers and Arrays 81

Answer: 1 rows of 1
1+1+1+1=4

Question 4.
Big Ideas Math Answers 2nd Grade Chapter 1 Numbers and Arrays 82

Answer: 3 rows of 6
6+6+6= 18

Question 5.
Number Sense
Use the array to complete the equation.
Big Ideas Math Answers 2nd Grade Chapter 1 Numbers and Arrays 83

Answer: 6+6=12

Question 6.
Modeling Real Life
The arrays show toy cars. Are there more orange cars or blue cars?
Big Ideas Math Answers 2nd Grade Chapter 1 Numbers and Arrays 84

Answer: Orange cars
Explanation: There are 16 orange cars and 15 blue cars.

Question 7.
DIG DEEPER!
The arrays show a sheet of stickers separated into two pieces. How many rows and columns of stickers did the sheet have before it was separated?
Big Ideas Math Answers 2nd Grade Chapter 1 Numbers and Arrays 85

Answer: 5 rows and 5 columns

Review & Refresh

Question 8.
Big Ideas Math Answers 2nd Grade Chapter 1 Numbers and Arrays 86

Answer: 6 flat surfaces
8 vertices
12 edges

Question 9.
Big Ideas Math Answers 2nd Grade Chapter 1 Numbers and Arrays 87

Answer: 0 flat surfaces
0  vertices
0 edges

Lesson 1.5 Make Arrays

Explore and Grow

Use counters to model the story. Write an addition equation to match.

There are 4 rows of students. There are 3 students in each row. How many students are there in all?

Addition equation:

Big Ideas Math Answers 2nd Grade Chapter 1 Numbers and Arrays 87.1

Answer: 12 students
Explanation: 3+3+3+3=12 students.

Show and Grow

Question 1.
A photo album has 3 rows of photos. There are 2 photos in each row. How many photos are there in all?
Big Ideas Math Answers 2nd Grade Chapter 1 Numbers and Arrays 88
Big Ideas Math Answers 2nd Grade Chapter 1 Numbers and Arrays 89

Answer: 6 photos
Explanation: 2+2+2=6

Question 2.
You have 4 rows of stickers. There are 5 stickers in each row. How many stickers do you have in all?
Big Ideas Math Answers 2nd Grade Chapter 1 Numbers and Arrays 90

Answer: 20 stickers
Explanation: 5+5+5+5=20

Apply and Grow: Practice

Question 3.
An ice cube tray has 5 rows. There are 2 ice cubes in each row. How many ice cubes are there in all?
Big Ideas Math Answers 2nd Grade Chapter 1 Numbers and Arrays 91
Big Ideas Math Answers 2nd Grade Chapter 1 Numbers and Arrays 92

Answer: 10 ice cubes
Explanation: 2+2+2+2+2=10

Question 4.
A bookcase has 3 shelves. There are 5 stuffed animals on each shelf. How many stuffed animals are there in all?
Big Ideas Math Answers 2nd Grade Chapter 1 Numbers and Arrays 93

Answer: 15 stuffed animals
Explanation: 5+5+5=15 stuffed animals

Question 5.
A closet has 4 shelves. There are 2 games on each shelf. How many games are there in all?
Big Ideas Math Answers 2nd Grade Chapter 1 Numbers and Arrays 94

Answer: 8 games
Explanation: 2+2+2+2=8

Question 6.
Structure
Make an array to match the equation.
5 + 5 + 5 + 5 = 20

Answer: Big-Ideas-Math-Answer-Key-Grade-2-Chapter-1-Numbers-and-Arrays-Lesson 1.5 (6)-Answer

Think and Grow: Modeling Real Life

A marching band has 3 equal rows of drummers. There are 15 drummers in all. How many drummers are in each row?
Big Ideas Math Answers 2nd Grade Chapter 1 Numbers and Arrays 95
Model:
Repeated addition equation:
Big Ideas Math Answers 2nd Grade Chapter 1 Numbers and Arrays 96

Answer: 5 Drummers
Explanation: 5+5+5=15

Show and Grow

Question 7.
A quilt has 4 equal rows of patches. There are 24 patches in all. How many patches are in each row?
Big Ideas Math Answer Key Grade 2 Chapter 1 Numbers and Arrays 97

Answer: 6 patches
Explanation: 6+6+6+6=24

Question 8.
A building has 3 equal rows of windows. There are 18 windows in all. How many columns are there?
Big Ideas Math Answer Key Grade 2 Chapter 1 Numbers and Arrays 98

Answer: 6 columns
Explanation: 3+3+3+3+3+3=18

Make Arrays Homework & Practice 1.5

Question 1.
A parking lot has 3 rows. There are 5 parking spots in each row. How many parking spots are there in all?
Big Ideas Math Answer Key Grade 2 Chapter 1 Numbers and Arrays 99

Answer: 15 parking spots
Explanation: 5+5+5=15

Question 2.
A bookcase has 4 shelves. There are 3 books on each shelf. How many books are there in all?
Big Ideas Math Answer Key Grade 2 Chapter 1 Numbers and Arrays 100

Answer: 12 books
Explanation: 3+3+3+3=12

Question 3.
Reasoning
Newton has 10 tokens. Which equations can Newton use to make an array with his tokens?
Big Ideas Math Answer Key Grade 2 Chapter 1 Numbers and Arrays 101

Answer: 2+2+2+2+2=10  and 5+5=10

Question 4.
Modeling Real Life
A theater has 4 equal rows of seats. There are 16 seats in all. How many seats are in each row?
Big Ideas Math Answer Key Grade 2 Chapter 1 Numbers and Arrays 102

Answer: 4 seats
Explanation: 4+4+4+4=16

Question 5.
Modeling Real Life
A chorus has 5 equal rows of singers. There are 30 singers in all. How many singers are in each row?
Big Ideas Math Answer Key Grade 2 Chapter 1 Numbers and Arrays 103

Answer: 6 singers
Explanation: 6+6+6+6+6=30

Review & Refresh

Question 6.
There are 9 lions in all. How many lions are in the den?
Big Ideas Math Answer Key Grade 2 Chapter 1 Numbers and Arrays 104

Answer: 0 lions are in den.
Explanation: All 9 lions are outside the den. Therefore, 0 lions are in the den.

Numbers and Arrays Performance Task

Question 1.
Your art supplies are packaged in boxes as described below.
Big Ideas Math Answer Key Grade 2 Chapter 1 Numbers and Arrays 105
a. What do you have the most of ?
Big Ideas Math Answer Key Grade 2 Chapter 1 Numbers and Arrays 106

Answer: 16 Crayons
Explanation: Colored Pencils= 5+5+5=15
Markers= 7+7=14
Crayons= 4+4+4+4=16
Crayons are more than Colored Pencils and Markers.
b. Do you have an even or an odd number of markers?
Big Ideas Math Answer Key Grade 2 Chapter 1 Numbers and Arrays 107

Answer: Even number of markers

Question 2.
a. You have 5 equal rows of paint bottles. You have 20 paint bottles in all. How many paint bottles are in each row?
Big Ideas Math Answer Key Grade 2 Chapter 1 Numbers and Arrays 108

Answer: 4 paint bottles are in each row.
b. You add another row of paint bottles. How many paint bottles do you have now?
Answer: 24 paint bottles
c. Describe another way to arrange the paint bottles you have now.
Big Ideas Math Answer Key Grade 2 Chapter 1 Numbers and Arrays 109

Answer: 6+6+6+6=24 paint bottles

Numbers and Arrays Activity

Array Flip and Find

To Play: Place the Array Flip and Find Cards face down in the boxes. Take turns flipping two cards. If your cards show the same total, keep the cards. If your cards show different totals, flip the cards back over. Play until all matches are made.
Big Ideas Math Answer Key Grade 2 Chapter 1 Numbers and Arrays 110

Numbers and Arrays Chapter Practice

1.1 Even and Odd Numbers

Question 1.
Big Ideas Math Answer Key Grade 2 Chapter 1 Numbers and Arrays 111

Answer:  Odd number
Big-Ideas-Math-Answer-Key-Grade-2-Chapter-1-Numbers-and-Arrays-111-Answer
Explanation: 5 cube part

Question 2.
Big Ideas Math Answer Key Grade 2 Chapter 1 Numbers and Arrays 112

Answer: Even number
Big-Ideas-Math-Answer-Key-Grade-2-Chapter-1-Numbers-and-Arrays-112-Answer
Explanation: 14 cube part

Color cubes to show the number. Circle even or odd.

Question 3.
Big Ideas Math Answer Key Grade 2 Chapter 1 Numbers and Arrays 113
Answer:
Big-Ideas-Math-Answer-Key-Grade-2-Chapter-1-Numbers-and-Arrays-113-Answer

Question 4.
Big Ideas Math Answer Key Grade 2 Chapter 1 Numbers and Arrays 114
Answer:
Big-Ideas-Math-Answer-Key-Grade-2-Chapter-1-Numbers-and-Arrays-114-Answer

Question 5.
Modeling Real Life
You see an odd number of boats. There are more than 15 but fewer than 19 boats. How many boats do you see?
Big Ideas Math Answer Key Grade 2 Chapter 1 Numbers and Arrays 115

Show how you know:

Answer: 17 boats
Explanation: Between 15 and 19 there is Odd number 17.

1.2 Model Even and Odd Numbers

Question 6.
Big Ideas Math Answer Key Grade 2 Chapter 1 Numbers and Arrays 116

Answer: 9=5+4
Big-Ideas-Math-Answer-Key-Grade-2-Chapter-1-Numbers-and-Arrays-116-Answer

Question 7.
Big Ideas Math Answer Key Grade 2 Chapter 1 Numbers and Arrays 117

Answer: 18=9+9
Big-Ideas-Math-Answer-Key-Grade-2-Chapter-1-Numbers-and-Arrays-117-Answer

Question 8.
Big Ideas Math Answer Key Grade 2 Chapter 1 Numbers and Arrays 118

Answer: 10=5+5
Big-Ideas-Math-Answer-Key-Grade-2-Chapter-1-Numbers-and-Arrays-118-Answer

Question 9.
Big Ideas Math Answer Key Grade 2 Chapter 1 Numbers and Arrays 119

Answer: 7+6=13
Big-Ideas-Math-Answer-Key-Grade-2-Chapter-1-Numbers-and-Arrays-119-Answer

1.3 Equal Groups

Question 10.
Big Ideas Math Answer Key Grade 2 Chapter 1 Numbers and Arrays 120

Answer: 4 groups of 2
Answer: 4+4=8

Question 11.
Circle groups of 3. Write a repeated addition equation.
Big Ideas Math Answer Key Grade 2 Chapter 1 Numbers and Arrays 121

Answer: 4 groups of 3
Answer: 3+3+3+3=12
Big-Ideas-Math-Answer-Key-Grade-2-Chapter-1-Numbers-and-Arrays-121-Answer

Question 12.
Structure
Show two different ways to put the balls in equal groups.
One Way:
Big Ideas Math Answer Key Grade 2 Chapter 1 Numbers and Arrays 122
Another Way:
Big Ideas Math Answer Key Grade 2 Chapter 1 Numbers and Arrays 123
Answer: Big-Ideas-Math-Answer-Key-Grade-2-Chapter-1-Numbers-and-Arrays-122-Answer

1.4 Use Arrays

Question 13.
Big Ideas Math Answer Key Grade 2 Chapter 1 Numbers and Arrays 124

Answer: 3 rows of 2
Answer: 2+2+2=6

Question 14.
Big Ideas Math Answers Grade 2 Chapter 1 Numbers and Arrays 125

Answer: 2 rows of 4
Answer: 4+4=8

Question 15.
Big Ideas Math Answers Grade 2 Chapter 1 Numbers and Arrays 126

Answer: 5 rows of 3
Answer: 3+3+3+3+3=15

Question 16.
Big Ideas Math Answer Key Grade 2 Chapter 1 Numbers and Arrays 127

Answer: 4 rows of 5
Answer: 5+5+5+5=20

1.5 Make Arrays

Question 17.
A cupboard has 4 shelves. There are 3 glasses on each shelf. How many glasses are there in all?
Big Ideas Math Answers 2nd Grade Chapter 1 Numbers and Arrays 128

Answer: 12 glasses
Answer: 3+3+3+3=12

Question 18.
A bingo card has 5 rows. There are 5 squares in each row. How many squares are there in all?
Big Ideas Math Answer Key Grade 2 Chapter 1 Numbers and Arrays 129

Answer: 25 squares
Answer: 5+5+5+5+5=25

Question 19.
Modeling Real Life
A pet store has 5 equal rows of fish tanks. There are 20 fish tanks in all. How many fish tanks are in each row?
Big Ideas Math Answer Key Grade 2 Chapter 1 Numbers and Arrays 130

Answer: 4 fish tanks
Explanation: 4+4+4+4+4=20

Conclusion:

We wish that the data provided in Big Ideas Math Book Grade 2 Chapter 1 Numbers and Arrays Answer Key is satisfactory for all students. This Big Ideas Math Answers Grade 2 Chapter 1 Numbers and Arrays is not only useful for the students but also for the parents. To clear your doubts, please write a comment in the below comment section. Stay tuned to our site bigideasmathanswers.com to get the answer key of other grades and grade 2 chapters.

Big Ideas Math Answers Grade 8 Chapter 4 Graphing and Writing Linear Equations

Big Ideas Math Answers Grade 8 Chapter 4

Are you searching everywhere regarding the Big Ideas Math 8th Grade Answer Key Chapter 4 Graphing and Writing Linear Equations? If so, halt your search as this is the one-stop destination for all your needs. Practice using the Graphing and Writing Linear Equations Big Ideas Math Grade 8 Answers and understand the concepts easily. Begin your preparation right away and seek the homework help needed right after class in a matter of seconds.

Big Ideas Math Book 8th Grade Answer Key Chapter 4 Graphing and Writing Linear Equations

Make the most out of the handy resources available for Big Ideas Math Ch 4 Graphing and Writing Linear Equations and stand out from the rest of the crowd. BIM Book 8th Grade Chapter 4 Solutions include questions belonging to Lessons 4.1 to 4.7, Cumulative Practice, Assessment Tests, Review Tests, etc. Big Ideas Math 8th Grade Chapter 4 Solution Key is given by subject experts after extensive research. Access the quick links over here during your preparation and get the assistance needed at the comfort of your home.

Performance Task

Lesson: 1 Graphing Linear Equations

Lesson: 2 Slope of a Line

Lesson: 3 Graphing Proportional Relationships

Lesson: 4 Graphing Linear Equations in Slope-Intercept Form

Lesson: 5 Graphing Linear Equations in Standard Form

Lesson: 6 Writing Equations in Slope-Intercept Form

Lesson: 7 Writing Equations in Point-Slope Form

Chapter: 4 – Graphing and Writing Linear Equations

Graphing and Writing Linear Equations STEAM Video/Performance Task

STEAM Video

“Hurricane
A hurricane is a storm with violent winds. How can you prepare your home for a hurricane?
Watch the STEAM Video “Hurricane!” Then answer the following questions.
Big Ideas Math Answer Key Grade 8 Chapter 4 Graphing and Writing Linear Equations 1
1. Robert says that the closer you are to the eye of a hurricane, the stronger the winds become. The wind speed on an island is 50 miles per hour when the eye of a hurricane is 140 miles away.
a. Describe the wind speed on the island when the eye of the hurricane is 100 miles away.
b. Describe the distance of the island from the eye of the hurricane when the wind speed on the island is 25 miles per hour.
c. Sketch a line that could represent the wind speed y (in miles per hour) on the island when the eye of x the hurricane is miles away from the island. Wind speed
2. A storm dissipates as it travels over land. What does this mean?

Performance Task

Anatomy of a Hurricane
After completing this chapter, you will be able to use the concepts you learned to answer the questions in the STEAM Video Performance Task. You will be given information about the atmospheric pressure inside a hurricane.
Big Ideas Math Answer Key Grade 8 Chapter 4 Graphing and Writing Linear Equations 2
You will be asked to use a model to find the strength of a hurricane after x hours of monitoring. Why is it helpful to predict how strong the winds of a hurricane will become?

Graphing and Writing Linear Equations Getting Ready for Chapter 4

Chapter Exploration
1. Work with a partner.
a. Use the equation y = \(\frac{1}{2}\)x + 1 to complete the table. (Choose any two x-values and find the y-values.)
b. Write the two ordered pairs given by the table. These are called solutions of the equation.
Big Ideas Math Answer Key Grade 8 Chapter 4 Graphing and Writing Linear Equations 3.1
c. PRECISION Plot the two solutions. Draw a line exactly through the points.
d. Find a different point on the line. Check that this point is a solution of the equation y = \(\frac{1}{2}\)x + 1.
e. LOGIC Do you think it is true that any point on the line is a solution of the equation y = \(\frac{1}{2}\)x + 1? Explain.
f. Choose five additional x-values for the table below. (Choose both positive and negative x-values.) Plot the five corresponding solutions. Does each point lie on the line?
Big Ideas Math Answer Key Grade 8 Chapter 4 Graphing and Writing Linear Equations 3
g. LOGIC Do you think it is true that any solution of the equation y = \(\frac{1}{2}\)x + 1 is a point on the line? Explain.
h. Why do you think y = ax + b is called a linear equation?

Vocabulary
The following vocabulary terms are defined in this chapter. Think about what each term might mean and record your thoughts.
linear equation
slope
y-intercept
solution of a linear equation
x-intercept

Lesson 4.1 Graphing Linear Equations

EXPLORATION 1

Creating Graphs
Work with a partner. It starts snowing at midnight in Town A and Town B. The snow falls at a rate of 1.5 inche sper hour.
a. In Town A, there is no snow on the ground at midnight. How deep is the snow at each hour between midnight and 6 A.M.? Make a graph that represents this situation.
Big Ideas Math Answer Key Grade 8 Chapter 4 Graphing and Writing Linear Equations 4.1 1
b. Repeat part(a) for TownB, which has 4 inches of snow on the ground at midnight.
c. The equations below represent the depth y(in inches) of snow x hours after midnight in Town C and Town D. Graph each equation.
Town C y = 2x + 3
Town D y = 8
Big Ideas Math Answer Key Grade 8 Chapter 4 Graphing and Writing Linear Equations 4.1 2
d. Use your graphs to compare the snowfall in each town.
Answer:

Big Ideas Math Answer Key Grade 8 Chapter 4 Graphing and Writing Linear Equations 4.1 3

Try It

Graph the linear equation.
Question 1.
y = 3x
Answer:
Make to table of values
Replace x with a number and find the value of y
Big ideas math answers grade 8 chapter 4 img_1
Plot the values of x and y obtained above, on the graph
Big ideas math answers grade 8 chapter 4 img_1.1

Draw the line through the points

Big ideas math answers grade 8 chapter 4 img_1.2

Question 2.
y = – 2x – 1
Answer:
Big ideas math answers grade 8 chapter 4 img_2.1
Plot the values of x and y
Big ideas math answers grade 8 chapter 4 img_2.2
Now the line through the points
Big ideas math answers grade 8 chapter 4 img_2.3

Question 3.
y = –\(\frac{1}{2}\)x + 2
Answer:
Big ideas math answers grade 8 chapter 4 img_3.1
Plot the ordered pairs
Big ideas math answers grade 8 chapter 4 img_3.2

Graph the linear equation.
Question 4.
y = 3
Answer:
The graph of y = 3 is a horizontal like passing through (0, 3)
Draw a horizontal line through this point.
Big ideas math answer key grade 8 chapter 4 img_4

Question 5.
y = – 1.5
Answer:
The graph of y = -1.5 is a horizontal line passing through (0, -1.5)
Draw a horizontal line through this point.
Big ideas math answer key grade 8 chapter 4 img_5

Question 6.
x = – 4
Answer:
The graph of x = – 4 is a vertical line passing through (-4, 0)
Draw a vertical line through this point.
BIM Grade 8 Answers Chapter 4 img_6

Question 7.
x = \(\frac{1}{2}\)
Answer:
The graph of x = \(\frac{1}{2}\) is a vertical line passing through (\(\frac{1}{2}\), 0)
Draw a vertical line through this point.
Big Ideas Math 8th Grade Solution Key Chapter 4 img_7

Self-Assessment for Concepts & Skills
Solve each exercise. Then rate your understanding of the success criteria in your journal.

GRAPHING A LINEAR EQUATION Graph the linear equation.
Question 8.
y = – x + 1
Answer:
Make of a table of values
Replace x with a number and find the value of y
Big Ideas Math 8th Grade Solution Key Chapter 4 img_8.1
Plot the values of x and y obtained, on the graph,
Big Ideas Math 8th Grade Solution Key Chapter 4 img_8.2

Question 9.
y = 0.8x – 2
Answer:
Replace x with a number and find the value of y
Big Ideas Math 8th Grade Solution Key Chapter 4 img_9.1
Big Ideas Math 8th Grade Solution Key Chapter 4 img_9.2

Question 10.
x = 2.5
Answer:
The graph of x = 2.5 is a vertical line passing through (2.5, 0)
Draw a vertical line through this point.
Big Ideas Math Answers Grade 8 Chapter 4 img_10.1

Question 11.
y = \(\frac{2}{3}\)
Answer:
The graph of y = \(\frac{2}{3}\) is a horizontal line passing through (0, \(\frac{2}{3}\))
Draw a horizontal line through this point.
BIM Grade 8 Answer Key Chapter 4 img_11

Question 12.
WHICH ONE DOESN’T BELONG?
Which equation does not belong with the other three? Explain your reasoning.
Big Ideas Math Answer Key Grade 8 Chapter 4 Graphing and Writing Linear Equations 4.1 4
Answer:
y = x – 2
4x + 3 = y
y = x² + 6
x + 5 = y

Self-Assessment for Problem Solving
Solve each exercise. The rate your understanding of the success criteria in your journal.

Question 13.
A game show contestant earns y dollars for completing a puzzle in x minutes. This situation is represented by the equation y = – 250x + 5000. How long did a contestant who earned $500 take to complete the puzzle? Justify your answer.
Answer:
Given,
A game show contestant earns y dollars for completing a puzzle in x minutes.
This situation is represented by the equation y = – 250x + 5000.
y = -250x + 5000
500 = -250x + 5000
500 – 5000 = -250x + 5000 – 5000
-4500 = -250x
x = 18

Question 14.
The total cost y (in dollars) to join a cheerleading team and attend x competitions is represented by the equation y = 10x + 50.
Big Ideas Math Answer Key Grade 8 Chapter 4 Graphing and Writing Linear Equations 4.1 5
a. Graph the linear equation.

Answer:
Big Ideas Math Answers 8th Grade Chapter 4 img_14

b. You have $75 to spend. How many competitions can you attend?
Answer:
75 ≤ 10x + 50
75 – 50 ≤ 10x
25 ≤ 10x
2.5 ≥ x
By this I can say that I can attend 2 competitions if I have $75 to spend.

Question 15.
The seating capacity for a banquet hall is represented by y = 8x + 56, where x is the number of extra tables you need. How many extra tables do you need to double the original seating capacity?
Answer:
Given,
The seating capacity for a banquet hall is represented by y = 8x + 56, where x is the number of extra tables you need.
y = 8x + 56
2 × 56 = 8x + 56
112 = 8x + 56
8x = 112 – 56
8x = 56
x = 7 tables

Graphing Linear Equations Homework & Practice 4.1

Review & Refresh

Tell whether the triangles are similar. Explain.
Question 1.
Big Ideas Math Answer Key Grade 8 Chapter 4 Graphing and Writing Linear Equations 4.1 6
Answer:
x° + 46° + 95° = 180°
x° + 141° = 180°
x° = 180° – 141°
x° = 39°
Thus the angles of the triangle are 39°, 46°, 95°
y° + 39° + 46° = 180°
y° + 75° = 180°
y° = 180° – 75°
y° = 95°
Three angles of the triangle are 39°, 46°, 95°
The triangles have two pairs of congruent angles.

Question 2.
Big Ideas Math Answer Key Grade 8 Chapter 4 Graphing and Writing Linear Equations 4.1 7
Answer:
x° + 40° + 51° = 180°
x° + 91° = 180°
x° = 180° – 91°
x° = 89°
Three angles of the triangle are 40°, 51°, 89°
y° + 40° + 79° = 180°
y° + 119° = 180°
y° = 180° – 119°
y° = 61°

Describe the translation of the point to its image.
Question 3.
(1, – 4) → (3, 0)
Answer:
A(1, -4) = A'(1 + 2, -4) = (3, -4)
A'(3, 4) = B(3, -4 + 4) = (3, 0)
Translate 2 units right and 4 units up.

Question 4.
(6, 4) → (- 4, – 6)
Answer:
We are given the points
(6, 4) → (- 4, – 6)
A(6, 4) = A'(6 – 10, 4) = (-4, 4)
A'(-4, -4) = B(-4, 4 – 10) = (-4, -6)

Question 5.
(4, – 2) → (- 9, 3)
Answer:
We are given the points
A(4, -2)
B(-9, 3)
A(4, -2) = A'(4 – 13, -2) = (-9, -2)
A'(-9, -2) = B(-9, -2 + 4) = (-9, 3)

Concepts, Skills, & Problem Solving

CREATING GRAPHS Make a graph of the situation. (See Exploration 1, p. 141.)
Question 6.
The equation y = – 2x + 8 represents the amount (in fluid ounces) of dish detergent in a bottle after x days of use.
Answer:
Bigideas math answers grade 8 chapter 4 img_15

Question 7.
The equation y = 15x + 20 represents the cost (in dollars) of a gym membership after x months.
Answer:
Bigideas math answers grade 8 chapter 4 img_16

PRECISION Copy and complete the table with two solutions. Plot the ordered pairs and draw the graph of the linear equation. Use the graph to find a third solution of the equation.
Question 8.
Big Ideas Math Answer Key Grade 8 Chapter 4 Graphing and Writing Linear Equations 4.1 8
Answer:
Bigideas math answers grade 8 chapter 4 img_17
(x, y) = (2, 5)
Bigideas math answers grade 8 chapter 4 img_18

Question 9.
Big Ideas Math Answer Key Grade 8 Chapter 4 Graphing and Writing Linear Equations 4.1 9
Answer:
Bigideas math answers grade 8 chapter 4 img_19
(x, y) = (3, 3)
Bigideas math answers grade 8 chapter 4 img_20

GRAPHING A LINEAR EQUATION Graph the linear equation.
Question 10.
y = – 5x
Answer:
Bigideas math answers grade 8 chapter 4 img_21

Question 11.
y = 9x
Answer:
Bigideas math answers grade 8 chapter 4 img_22

Question 12.
y = 5
Answer:
The graph of y = 5 is a horizontal line passing through (0, 5)
Draw a horizontal line through this point.
Bigideas math answers grade 8 chapter 4 img_23

Question 13.
x = – 6
Answer:
Bigideas math answers grade 8 chapter 4 img_24

Question 14.
y = x – 3
Answer:
Bigideas math answers grade 8 chapter 4 img_25

Question 15.
y = – 7x – 1
Answer:
Bigideas math answers grade 8 chapter 4 img_26

Question 16.
y = – \(\frac{x}{8}\) + 4
Answer:
Bigideas math answers grade 8 chapter 4 img_27

Question 17.
y = 0.75x – 0.5
Answer:
Bigideas math answers grade 8 chapter 4 img_28

Question 18.
y = – \(\frac{2}{3}\)
Answer:
Bigideas math answers grade 8 chapter 4 img_29

Question 19.
y = 6.75
Answer:
Bigideas math answers grade 8 chapter 4 img_30

Question 20.
x = – 0.5
Answer:
The graph of x = -0.5 is a vertical line passing through (-0.5, 0)
Draw a vertical line through this point.
Big Ideas Math Answers Grade 8 Ch 4 img_26

Question 21.
x = \(\frac{1}{4}\)
Answer:
The graph of x = \(\frac{1}{4}\) is a vertical line passing through (\(\frac{1}{4}\), 0)
Draw a vertical line through this point.
Big Ideas Math Answers Grade 8 Ch 4 img_27

Question 22.
YOU BE THE TEACHER
Your friend graphs the equation y = 4. Is your friend correct? Explain your reasoning.
Answer:
Big Ideas Math Answers Grade 8 Ch 4 img_28
No my friend is not correct because the graph for the equation y = 4 is a  horizontal line not a vertical line, and it passes through the point (0, 4) not (4, 0)

Question 23.
MODELING REAL LIFE
The equation y = 20 represents the cost y (in dollars) for sending x text messages in a month. Graph the linear equation. What does the graph tell you about your texting plan?
Big Ideas Math Answer Key Grade 8 Chapter 4 Graphing and Writing Linear Equations 4.1 10
Answer:
Big Ideas Math Answers Grade 8 Ch 4 img_29

Question 24.
MODELING REAL LIFE
The equation y = 2x + 3 represents the cost y (in dollars) of mailing a package that weighs x pounds.
Big Ideas Math Answer Key Grade 8 Chapter 4 Graphing and Writing Linear Equations 4.1 11
a. Use a graph to estimate how much it costs to mail the package.
b. Use the equation to find exactly how much it costs to mail the package.
Answer:
Given the equation y = 2x + 3
The ordered pairs will be (0, 3), (2,7), (4, 11)
Now plot the ordered pairs
Big Ideas Math Answers Grade 8 Ch 4 img_30
y = 2(1.126) + 3
= 5.252 ≈ 5.25

SOLVING A LINEAR EQUATION Solve for y. Then graph the linear equation.
Question 25.
y – 3x = 1
Answer:
y – 3x = 1
y = 3x + 1
Big Ideas Math Answers Grade 8 Ch 4 img_31
Draw a line through the points
Big Ideas Math Answers Grade 8 Ch 4 img_32

Question 26.
5x + 2y = 4
Answer:
5x + 2y = 4
2y = 4 – 5x
y = – \(\frac{5}{2}\)x + 2
BIM 8th Grade Solution Key Chapter 4 img_33

Question 27.
– \(\frac{1}{3}\)y + 4x = 3
Answer:
– \(\frac{1}{3}\)y + 4x = 3
– \(\frac{1}{3}\)y = 3 – 4x
y = 12x – 9
BIM 8th Grade Solution Key Chapter 4 img_34

Question 28.
x + 0.5y = 1.5
Answer:
x + 0.5y = 1.5
0.5y = -x + 1.5
y = -2x + 3
Big Ideas Math Grade 8 Answers Chapter 4 img_35

Question 29.
MODELING REAL LIFE
The depth y (in inches) of a lake after x years is represented by the equation y = 0.2x + 42. How much does the depth of the lake increase in four years? Use a graph to justify your answer.
Big Ideas Math Answer Key Grade 8 Chapter 4 Graphing and Writing Linear Equations 4.1 12
Answer:
y = 0.2x + 42
Depth of the lake now: y = 0.2(0) + 42 = 42
Depth of the lake after 4 years: y = 0.2(4) + 42 = 42.8
Big Ideas Math Grade 8 Answers Chapter 4 img_36
42.8 – 42 = 0.8 inches

Question 30.
MODELING REAL LIFE
The amount y (in dollars) of money in your savings account after x months is represented by the equation y = 12.5x + 100.
a. Graph the linear equation.

Answer:
Big Ideas Math Grade 8 Answers Chapter 4 img_37
b. How many months will it take you to save a total of $237.50?
Answer:
y = 12.5x + 100
237.5 = x + 100
237.5 – 100 = 12.5x + 100 – 100
12.5x = 137.5
x = 11

Question 31.
PROBLEM SOLVING
The radius y (in millimeters) of a chemical spill after x days is represented by the equation y = 6x + 50.
Big Ideas Math Answer Key Grade 8 Chapter 4 Graphing and Writing Linear Equations 4.1 13
a. Graph the linear equation.

Answer:
Big Ideas Math Grade 8 Answers Chapter 4 img_38
b. The leak is noticed after two weeks. What is the area of the leak when it is noticed? Justify your answer.
Answer:
y = 6(14) + 50
y = 84 + 50
y = 134 mm
2πr = 2π = 841.95 sq. mm

Question 32.
GEOMETRY
The sum S of the interior angle measures of a polygon with n sides is S = (n – 2) • 180°.
a. Plot four points (n, S) that satisfy the equation. Is the equation a linear equation? Explain your reasoning.

Answer:
Big Ideas Math 8th Grade Answer Key for Chapter 4 img_39
b. Does the value n = 3.5 make sense in the context of the problem? Explain your reasoning.
Answer:
The value n = 3.5 does not make sense because the number of angles cannot be other than integer greater or equal to 2.

Question 33.
DIG DEEPER!
One second of video on your cell phone uses the same amount of memory as two pictures. Your cell phone can store 2500 pictures.
a. Create a graph that represents the number y of pictures your cell phone can store when you take x seconds of video.

Answer:
Big Ideas Math 8th Grade Answer Key for Chapter 4 img_40
b. How many pictures can your cell phone store in addition to a video that is one minute and thirty seconds long?
Answer:
Determine the number of pictures you can store in addition to a video of 1 min 30 seconds.
1 min 30 seconds = (60 + 90) 3 seconds = 90 seconds
2500 – (2 . 90)
2500 – 180 = 2320 pictures

Lesson 4.2 Slope of a Line

EXPLORATION 1

Measuring the Steepness of a Line
Work with a partner. Draw any nonvertical line in a coordinate plane.
a. Develop a way to measure the steepness of the line. Compare your method with other pairs.
b. Draw a line that is parallel to your line. What can you determine about the steepness of each line? Explain your reasoning.
Answer:

EXPLORATION 2

Using Right Triangles
Work with a partner. Use the figure shown.
Big Ideas Math Answers 8th Grade Chapter 4 Graphing and Writing Linear Equations 4.2 1
a. △ABC is a right triangle formed by drawing a horizontal line segment from point A and a vertical line segment from point B. Use this method to draw another right triangle, △DEF, with its longest side on the line
b. What can you conclude about the two triangles in part(a)? Justify your conclusion. Compare your results with other pairs.
c. Based on your conclusions in part(b), what is true about \(\frac{BC}{AC}\) and the corresponding measure in △DEF? Explain your reasoning. What do these values tell you about the line?
Answer:

Big Ideas Math Answers 8th Grade Chapter 4 Graphing and Writing Linear Equations 4.2 2

Try It

Find the slope of the line.
Question 1.
Big Ideas Math Answers 8th Grade Chapter 4 Graphing and Writing Linear Equations 4.2 3
Answer:
(x1, y1) = (-2, 3)
(x2, y2) = (3, 2)
m = (y2 – y1)/(x2 – x1)
m = (2 -3)/(3 – (-2))
m = -1/5
Thus slope = -1/5

Question 2.
Big Ideas Math Answers 8th Grade Chapter 4 Graphing and Writing Linear Equations 4.2 4
Answer:
(x1, y1) = (-4, -1)
(x2, y2) = (2, 1)
m = (y2 – y1)/(x2 – x1)
m = (1 – (-1))/(2 – (-4))
m = 2/6
Thus slope = 1/3

Find the slope of the line through the given points.
Question 3.
(1, -2), (7, -2)
Answer:
(x1, y1) = (1, -2)
(x2, y2) = (7, -2)
m = (y2 – y1)/(x2 – x1)
m = (-2 – (-2))/(7 – 1)
m = 0/6
Thus slope = 0

Question 4.
(-3, -3), (-3, -5)
Answer:
(x1, y1) = (-3, -3)
(x2, y2) = (-3, -5)
m = (y2 – y1)/(x2 – x1)
m = (-5 + 3)/(-3 + 3)
m = -2/0
Thus slope = undefined

Question 5.
WHAT IF
The blue line passes through (-4, -3) and (-3, 2). Are any of the lines parallel? Explain.
Answer:
(x1, y1) = (-4, -3)
(x2, y2) = (-3, 2)
m = (y2 – y1)/(x2 – x1)
m = (2 + 3)/(-3 + 4)
m = 5/1
m = 5
The slpe of the blue line is 5 and the slope of the red line is also 5.
The blue lines and red lines have same slopes so they are parallel.

Self-Assessment for Concepts & Skills
Solve each exercise. Then rate your understanding of the success criteria in your journal.

Question 6.
VOCABULARY
What does it mean for a line to have a slope of 4?
Answer:
If a line have a slope of 4 it means that the line rises 4 units for every 1 units it runs.

FINDING THE SLOPE OF A LINE Find the slope of the line through the given points.
Question 7.
(1, -1), (6, 2)
Answer:
(x1, y1) = (1, -1)
(x2, y2) = (6, 2)
m = (y2 – y1)/(x2 – x1)
m = (2 – (-1))/(6 – 1)
m = 3/5

Question 8.
(2, -3), (5, -3)
Answer:
(x1, y1) = (2, -3)
(x2, y2) = (5, -3)
m = (y2 – y1)/(x2 – x1)
m = (5 – 2)/(-3 + 3)
m = 3/0
m = undefined

Question 9.
FINDING SLOPE
Are the lines parallel? Explain your reasoning.
Big Ideas Math Answers 8th Grade Chapter 4 Graphing and Writing Linear Equations 4.2 5
Answer:
Red line:
(x1, y1) = (-1, 0)
(x2, y2) = (1, -2)
m = (y2 – y1)/(x2 – x1)
m = (-2 – 0)/1 – (-1))
m = -2/2
m = -1
Blue Line:
(x1, y1) = (-1, 3)
(x2, y2) = (1, -1)
m = (y2 – y1)/(x2 – x1)
m = (-1 – 3)/(1 – (-1))
m = -4/2
m = -2
The slope of the blue line and red line are not the same. So they are not parallel.

Self-Assessment for Problem Solving
Solve each exercise. Then rate your understanding of the success criteria in your journal.

Question 10.
The table shows the lengths (in inches) of your hair months after your last haircut. The points in the table lie on a line. Find and interpret the slope of the line. After how many months is your hair 4 inches long?
Big Ideas Math Answers 8th Grade Chapter 4 Graphing and Writing Linear Equations 4.2 6
Answer:
Determine the slope of the line using two points from the table:
(2, 1), (4, 2)
m = (2 – 1)/4 – 2
m = 1/2
m = 0.5
This means that each month the hair grows 0.5 inches
As the hair grows 0.5 inches/ month, it will be 4 inches long after 4/0.5 = 8 months.

Question 11.
A customer pays an initial fee and a daily fee to rent a snowmobile. The total payment for 3 days is 92 dollars. The total payment for 5 days is 120 dollars. What is the daily fee? Justify your answer.
Answer:
Given,
A customer pays an initial fee and a daily fee to rent a snowmobile.
The total payment for 3 days is 92 dollars. The total payment for 5 days is 120 dollars.
m = (120 – 92)/5 – 3
m = 28/2
m = 14

Question 12.
You in-line skate from an elevation of 720 feet to an elevation of 750 feet in 30 minutes. Your friend in-line skates from an elevation of 600 feet to an elevation of 690 feet in one hour. Compare your rates of change in elevation.
Big Ideas Math Answers 8th Grade Chapter 4 Graphing and Writing Linear Equations 4.2 7
Answer:
Given,
You in-line skate from an elevation of 720 feet to an elevation of 750 feet in 30 minutes.
Your friend in-line skates from an elevation of 600 feet to an elevation of 690 feet in one hour.
(750 – 720)/30 = 30/30 = 1 ft/min
(690 – 600)/60 = 90/60 = 1.5 ft/min

Slope of a Line Homework & Practice 4.2

Review & Refresh

Graph the linear equation.
Question 1.
y = 4x – 3
Answer:
Big Ideas Math Grade 8 Chapter 4 Solution Key img_41

Question 2.
x = -3
Answer:
Big Ideas Math Grade 8 Chapter 4 Solution Key img_42

Question 3.
y = 2
Answer:
Big Ideas Math Grade 8 Chapter 4 Solution Key img_43

Question 4.
y = \(\frac{3}{2}\)x – \(\frac{1}{2}\)
Answer:
Big Ideas Math Grade 8 Chapter 4 Solution Key img_44

Find the missing values in the ratio table.
Question 5.
Big Ideas Math Answers 8th Grade Chapter 4 Graphing and Writing Linear Equations 4.2 8
Answer:
Big-Ideas-Math-Answers-8th-Grade-Chapter-4-Graphing-and-Writing-Linear-Equations-4.2-8
x/10 = 1/3
x = 10/3
x = 3.33
1/3 = 5/y
y = 5 × 3
y = 15
1/3 = 7/z
z = 3 × 7
z = 21

Question 6.
Big Ideas Math Answers 8th Grade Chapter 4 Graphing and Writing Linear Equations 4.2 9
Answer:
Big-Ideas-Math-Answers-8th-Grade-Chapter-4-Graphing-and-Writing-Linear-Equations-4.2-9

Concepts, Skills, &Problem Solving

USING RIGHT TRIANGLES Use the figure shown (See Exploration 2, p. 147.)
Big Ideas Math Answers 8th Grade Chapter 4 Graphing and Writing Linear Equations 4.2 10
Question 7.
Find the slope of the line.
Answer:
(x1, y1) = B(-4, 2)
(x2, y2) = A(-2, 1)
m = (y2 – y1)/(x2 – x1)
m = (1 – 2)/(-2 – (-4))
m = -1/2
Thus the slope m = -1/2

Question 8.
Let point D be at (-4, 1). Use the sides of △BDA to find the slope of the line.
Answer:
Big Ideas Math Grade 8 Chapter 4 Answers img_45
m = -BD/DA = -1/2

FINDING THE SLOPE OF A LINE Find the slope of the line.
Question 9.
Big Ideas Math Answers 8th Grade Chapter 4 Graphing and Writing Linear Equations 4.2 11
Answer:
(x1, y1) = (-2, 0)
(x2, y2) = (2, 3)
m = (y2 – y1)/(x2 – x1)
m = (3 – 0)/(2 – (-2))
m = 3/4

Question 10.
Big Ideas Math Answers 8th Grade Chapter 4 Graphing and Writing Linear Equations 4.2 11
Answer:
(x1, y1) = (-2, 5)
(x2, y2) = (2, 0)
m = (y2 – y1)/(x2 – x1)
m = (0 – 5)/(2 – (-2))
m = -5/4

Question 11.
Big Ideas Math Answers 8th Grade Chapter 4 Graphing and Writing Linear Equations 4.2 13
Answer:
(x1, y1) = (-4, 1)
(x2, y2) = (1, -2)
m = (y2 – y1)/(x2 – x1)
m = (-2 – 1)/(1 + 2)
m = -3/5

Question 12.
Big Ideas Math Answers 8th Grade Chapter 4 Graphing and Writing Linear Equations 4.2 14
Answer:
(x1, y1) = (-5, -4)
(x2, y2) = (1, -3)
m = (y2 – y1)/(x2 – x1)
m = (-3 – (-4))/(1 – (-5))
m = 1/6

Question 13.
Big Ideas Math Answers 8th Grade Chapter 4 Graphing and Writing Linear Equations 4.2 15
Answer:
(x1, y1) = (-1, 3)
(x2, y2) = (3, 3)
m = (y2 – y1)/(x2 – x1)
m = (3 – 3)/(3 – (-1))
m = 0/4
m = 0

Question 14.
Big Ideas Math Answers 8th Grade Chapter 4 Graphing and Writing Linear Equations 4.2 16
Answer:
(x1, y1) = (1, 3)
(x2, y2) = (1, -2)
m = (y2 – y1)/(x2 – x1)
m = (-2 – 3)/(1 – 1)
m = -5/0
m = undefined

FINDING THE SLOPE OF A LINE Find the slope of the line through the given points.
Question 15.
(4, -1), (-2, -1)
Answer:
(x1, y1) = (4, -1)
(x2, y2) = (-2, -1)
m = (y2 – y1)/(x2 – x1)
m = (-1 – (-1))/(-2 – 4)
m = 0/-6
m = 0

Question 16.
(5, -3), (5, 8)
Answer:
(x1, y1) = (5, -3)
(x2, y2) = (5, 8)
m = (y2 – y1)/(x2 – x1)
m = (8 – 3)/(5 – 5)
m = 5/0
m = undefined

Question 17.
(-7, 0), (-7, -6)
Answer:
(x1, y1) = (-7, 0)
(x2, y2) = (-7, -6)
m = (y2 – y1)/(x2 – x1)
m = (-6 – 0)/(-7 – (-7))
m = -6/0
m = undefined

Question 18.
(-3, 1), (-1, 5)
Answer:
(x1, y1) = (-3, 1)
(x2, y2) = (-1, 5)
m = (y2 – y1)/(x2 – x1)
m = (5 – 1)/(-1 + 3)
m = 4/2
m = 2

Question 19.
(10, 4), (4, 15)
Answer:
(x1, y1) = (10, 4)
(x2, y2) = (4, 15)
m = (y2 – y1)/(x2 – x1)
m = (15 – 4)/(4 – 10)
m = 11/-6
m = -11/6

Question 20.
(-3, 6), (2, 6)
Answer:
(x1, y1) = (-3, 6)
(x2, y2) = (2, 6)
m = (y2 – y1)/(x2 – x1)
m = (6 – 6)/(2 – (-3))
m = 0/5
m = 0

Question 21.
REASONING
Draw a line through each point using slope of m = \(\frac{1}{4}\). Do the lines intersect? Explain.
Big Ideas Math Answers 8th Grade Chapter 4 Graphing and Writing Linear Equations 4.2 17
Answer:
Big Ideas math Grade 8 Answer Key Chapter 4 img_46
The 2 lines are parallel because they have the same slope and they do not intersect.

Question 22.
YOU BE THE TEACHER
Your friend finds the slope of the line shown. Is your friend correct? Explain your reasoning.
Big Ideas Math Answers 8th Grade Chapter 4 Graphing and Writing Linear Equations 4.2 18
Answer:
Big Ideas math Grade 8 Answer Key Chapter 4 img_47
No my friend is not correct because the denominator should be 2 – 4
(x1, y1) = (2, 3)
(x2, y2) = (4, 1)
m = (y2 – y1)/(x2 – x1)
m = (1 – 3)/(4 – 2)
m = -2/2
m = -1

IDENTIFYING PARALLEL LINES Which lines are parallel? How do you know?
Question 23.
Big Ideas Math Answers 8th Grade Chapter 4 Graphing and Writing Linear Equations 4.2 19
Answer:
Blue line:
(x1, y1) = (-5, 2)
(x2, y2) = (-4, -1)
m = (y2 – y1)/(x2 – x1)
m = (-1 – 2)/(-4 – (-5))
m = -3/1
m = -3
Red line:
(x1, y1) = (-2, 1)
(x2, y2) = (-1, -2)
m = (y2 – y1)/(x2 – x1)
m = (-2 – 1)/(-1 – (-2))
m = -3/1
m = -3
Green Line:
(x1, y1) = (1, 3)
(x2, y2) = (2, -1)
m = (y2 – y1)/(x2 – x1)
m = (-1 – 3)/(2 – 1)
m = -4/1
m = -4
Blue line and red line have slope of -3, so they are parallel.

Question 24.
Big Ideas Math Answers 8th Grade Chapter 4 Graphing and Writing Linear Equations 4.2 20
Answer:
Blue line:
(x1, y1) = (-2, 3)
(x2, y2) = (-5, -2)
m = (y2 – y1)/(x2 – x1)
m = (-2 – 3)/(-5 – (-2))
m = -5/-3
m = 5/3
Red line:
(x1, y1) = (1, 2)
(x2, y2) = (-2, -2)
m = (y2 – y1)/(x2 – x1)
m = (-2 – 2)/(-2 – 1)
m = -4/-3
m = 4/3
Green Line:
(x1, y1) = (4, 1)
(x2, y2) = (1, -3)
m = (y2 – y1)/(x2 – x1)
m = (-3 – 1)/(1 – 4)
m = -4/-3
m = 4/3
Red line and green line have slope of 4/3 by this we can say that they are parallel.

IDENTIFYING PARALLEL LINES Are the given lines parallel? Explain your reasoning.
Question 25.
y = -5, y = 3
Answer:
8th Grade Big Ideas Math Answers Chapter 4 img_47
Both lines are horizontal and have slope = 0

Question 26.
y = 0, x = 0
Answer:
8th Grade Big Ideas Math Answers Chapter 4 img_48
The line y = 0 have slope = 0 and are horizontal lines.
The line x = 0 have slope = undefined and are vertical lines.
So, they are not parallel.

Question 27.
x = -4, x = 1
Answer:
8th Grade Big Ideas Math Answers Chapter 4 img_49
Both lines are vertical and have an undefined slope.

FINDING SLOPE The points in the table lie on a line. Find the slope of the line.
Question 28.
Big Ideas Math Answers 8th Grade Chapter 4 Graphing and Writing Linear Equations 4.2 21
Answer:
m = (y2 – y1)/(x2 – x1)
m = (10 – 2)/(3 – 1) = (18 – 10)/(5 – 3) = (26 – 18)/(7 – 5)
m = 8/2 = 8/2 = 8/2
m = 4 = 4 = 4
Slope = 4

Question 9.
Big Ideas Math Answers 8th Grade Chapter 4 Graphing and Writing Linear Equations 4.2 22
Answer:
m = (y2 – y1)/(x2 – x1)
m = (2 – 0)/(2 – (-3)) = (4 – 2)/(7 – 2) = (6 – 4)/(12 – 7)
m = 2/5 = 2/5 = 2/5
m = 2/5

Question 30.
MODELING REAL LIFE
Carpenters refer to the slope of a roof as the pitch of the roof. Find the pitch of the roof.
Big Ideas Math Answers 8th Grade Chapter 4 Graphing and Writing Linear Equations 4.2 23
Answer:
Pitch of the roof = rise/run
= 4/12 = 1/3

Question 31.
PROJECT
The guidelines for a wheelchair ramp suggest that the ratio of the rise to the run be no greater than 1 : 12.
Big Ideas Math Answers 8th Grade Chapter 4 Graphing and Writing Linear Equations 4.2 24
a. CHOOSE TOOLS Find a wheelchair ramp in your school or neighborhood. Measure its slope. Does the ramp follow the guidelines?

Answer:
rise/run < 1/12
m = 0.06
1/12 = 0.0833
0.06 < 0.0833
As m < 1/12 the wheelchair ramp follows the guides.

b. Design a wheelchair ramp that provides access to a building with a front door that is 2.5 feet above the sidewalk. Illustrate your design.
Answer:
AC/AB = 1/12
2.5/AB = 1/12
AB = 2.5 × 12
AB = 30
So the end of the ramp should be placed at least 30 feet from the front door.

USING AN EQUATION Use an equation to find the value of k so that the line that passes through the given points has the given slope.
Question 32.
(1, 3), (5, k); m = 2
Answer:
A(1, 3)
B(5, k)
m = 2
2 = (k – 3)/(5 – 1)
2 × 4 = k – 3
8 = k – 3
k = 8 + 3
k = 11

Question 33.
(-2, k), (2, 0); m = -1
Answer:
Given,
A(-2, k)
B(2, 0)
m = -1
-1 = (0 – k)/2 – (-2)
-1 = -k/4
-4 = -k
k = 4

Question 34.
(-4, k), (6, -7); m = –\(\frac{1}{5}\)
Answer:
Given,
A(-4, k)
B(6, -7)
m = –\(\frac{1}{5}\)
–\(\frac{1}{5}\) = (-7 – k)/6 – (-4)
-2 = -7 – k
-2 + 7 = -k
5 = -k
k = -5

Question 35.
(4, -4), (k, -1); m = \(\frac{3}{4}\)
Answer:
\(\frac{3}{4}\) = (-1 – (-4))/(k – 4)
4 = k – 4
k = 4 + 4
k = 8

Question 36.
MODELING REAL LIFE
The graph shows the numbers of prescriptions filled over time by a pharmacy.
Big Ideas Math Answers 8th Grade Chapter 4 Graphing and Writing Linear Equations 4.2 25
a. Find the slope of the line.
Answer:
(0, 0), (20, 5)
m = (5 – 0)/(20 – 0)
m = 5/20
m = 1/4
b. Explain the meaning of the slope as a rate of change.
Answer:
This means that every 4 minutes a prescription is filled.

Question 37.
CRITICAL THINKING
Which is steeper: the boatramp, or a road with a 12% grade? Note: Explain. (Road grade is the vertical increase divided by the horizontal distance.)
Big Ideas Math Answers 8th Grade Chapter 4 Graphing and Writing Linear Equations 4.2 26
Answer:
Mramp = rise/run = 6/36 = 1/6
Mroad = 12% = 12/100 = 0.12
0.16 = 0.166… > 0.12
Mramp > Mroad
Therefore the slope of the ramp is steeper than the slope of the road.

Question 38.
REASONING
Do the points A(-2, -1), B(1, 5), and C(4, 11) lie on the same line? Without using a graph, how do you know?
Answer:
Given,
A(-2, -1), B(1, 5), and C(4, 11)
mAB = (5 – (-1))/(1 – (-2)) = 6/3 = 2
mBC = (11 – 5)/(4 – 1) = 6/3 = 2
By seeing the slopes we can say that the points A, B, C lie on the same line.

Question 39.
PROBLEM SOLVING
A small business earns a profit of $6500 in January and $17,500 in May. What is the rate of change in profit for this time period? Justify your answer.
Answer:
Pjan = 6500
Pmay = 17,500
Pmay – Pjan/5 – 1
= (17,500 – 6500)/4
= 11,000/4 = 2750

Question 40.
STRUCTURE
Choose two points in the coordinate plane. Use the slope formula to find the slope of the line that passes through the two points. Then find the slope using the formula \(\frac{y_{1}-y_{2}}{x_{1}-x_{2}}\). Compare your results.
Answer:
P1(2, 5)
P2(3, 10)
m1 = (10 – 5)/(3 – 2) = 5/1 = 5
m2 = (5 – 10)/(1 – 3) = -5/-1 = 5
m1 = m2

Question 41.
DIG DEEPER!
The top and the bottom of the slide are level with the ground, which has a slope of 0.
Big Ideas Math Answers 8th Grade Chapter 4 Graphing and Writing Linear Equations 4.2 27
a. What is the slope of the main portion of the slide?
b. Describe the change in the slope when the bottom of the slide is only 12 inches above the ground. Explain your reasoning.
Answer:
18 inches = 1.5 feet
mMC = rise/run = (8 – 1.5)/(12 – 1 – 1) = 6.5/10 = 0.65
AD = 1
mMC = CR/MR
= (8 – 1)/(12 – 1 – 1) = 7/10 = 0.7
The slope increases from 0.65 to 0.70 because the rise increasses, while the run stays the same.

Lesson 4.3 Graphing Proportional Relationships

EXPLORATION 1

Using a Ratio Table to Find Slope
Work with a partner. The graph shows amounts of vinegar and water that can be used to make a cleaning product.
a. Use the graph to make a ratio table relating the quantities. Explain how the slope of the line is represented in the table.
Big Ideas Math Answers Grade 8 Chapter 4 Graphing and Writing Linear Equations 4.3 1
b. Make a ratio table that represents a different ratio of vinegar to water. Use the table to describe the slope of the graph of the new relationship.
Answer:

EXPLORATION 2

Deriving an Equation
Work with a partner. Let (x, y) represent any point on the graph of a proportional relationship.
Big Ideas Math Answers Grade 8 Chapter 4 Graphing and Writing Linear Equations 4.3 2
a. Describe the relationship between the corresponding side lengths of the triangles shown in the graph. Explain your reasoning.
b. Use the relationship in part(a) to write an equation relating y, m, and x. Then solve the equation for y. How can you find the side lengths of the triangles in the graph?
c. What does your equation in part(b) describe? What does represent? Explain your reasoning.
Answer:

Try It

Question 1.
WHAT IF
The cost of frozen yogurt is represented by y = 0.75x. Graph the equation and interpret the slope.
Answer:
The equation shows that the slope m is 0.75. So the graph passes through the points (0, 0) and (1, 0.75).
Plot the ordered pairs and draw the graph.
Big Ideas Math Grade 8 Answer Key Chapter 4 img_48
The slope indicates that the unit cost is $0.75 per ounce.

Question 2.
How much would a spacecraft that weighs 3500 kilograms on Earth weigh on Titan?
Answer:
y = 1/7 x
y = 1/7 × 3500
y = 500 kg
So a spacecraft would weigh 500 kg on Titan.

Self-Assessment for Concepts & Skills
Solve each exercise. Then rate your understanding of the success criteria in your journal.

GRAPHING A PROPORTIONAL RELATIONSHIP Graph the equation.
Question 3.
y = 4x
Answer:
Big Ideas Math Grade 8 Answer Key Chapter 4 img_49

Question 4.
y = -3x
Answer:
Big Ideas Math Grade 8 Answer Key Chapter 4 img_50

Question 5.
y = 8x
Answer:
Big Ideas Math Grade 8 Answer Key Chapter 4 img_51

Question 6.
WRITING AND USING AN EQUATION
The number of objects a x machine produces is proportional to the time (in minutes) that the machine runs. The machine produces five objects in four minutes.
a. Write an equation that represents the situation.

Answer:
As 5 objects are produced in 4 minutes, the slope of the line is m = 5/4.
The equation that represents the situation is
y = 5/4 x
y = 1.25 x

b. Graph the equation in part (a) and interpret the slope.

Answer:
Use the slope. The equation shows that the slope m is 1.25. So the graph passes through the points (0, 0) and (1, 1.25)

c. How many objects does the machine produce in one hour?
Answer:
Big Ideas Math Grade 8 Answer Key Chapter 4 img_52

Self-Assessment for Problem Solving
Solve each exercise. Then rate your understanding of the success criteria in your journal.

Question 7.
The amount y (in liters) of water that flows over a natural waterfall in x seconds is represented by the equation y = 500x. The graph shows the number of liters of water that flow over an artificial waterfall. Which waterfall has a greater flow? Justify your answer.
Big Ideas Math Answers Grade 8 Chapter 4 Graphing and Writing Linear Equations 4.3 3
Answer:
Given the equation y = 500x
15000 – 3000 = 12000
12000/4 = 3000
Mnatural = 500
3000 > 500
Therefore the artificial waterfall has greater flow.

Question 8.
The speed of sound in air is 343 meters per second. You see lightning and hear thunder 12 seconds later.
a. Is there a proportional relationship between the amount of time that passes and your distance from a lightning strike? Explain.

Answer:
y = kx
where k is the speed of sound, x the time and y the distance.
Yes, there is a proportional relationship between the amount of time that passes and your distance from the lightning strike as the further you are, the more time will pass until the sound reaches you.

b. Estimate your distance from the lightning strike.
Answer:
y = 343 × 12
= 4116 meters

Graphing Proportional Relationships Homework & Practice 4.3

Review & Refresh

Find the slope of the line.
Question 1.
Big Ideas Math Answers Grade 8 Chapter 4 Graphing and Writing Linear Equations 4.3 4
Answer:
(x1, y1) = (0, -3)
(x2, y2) = (3, 0)
m = (y2 – y1)/(x2 – x1)
m = (0 – (-3))/(3 – 0)
m = (0 + 3)/(3 – 0)
m = 3/3
m = 1

Question 2.
Big Ideas Math Answers Grade 8 Chapter 4 Graphing and Writing Linear Equations 4.3 5
Answer:
(x1, y1) = (0, 1)
(x2, y2) = (3, -5)
m = (y2 – y1)/(x2 – x1)
m = (-5 – 1)/(3 – 0)
m = -6/3
m = -2

Question 3.
Big Ideas Math Answers Grade 8 Chapter 4 Graphing and Writing Linear Equations 4.3 6
Answer:
(x1, y1) = (0, 0)
(x2, y2) = (2, 8)
m = (y2 – y1)/(x2 – x1)
m = (8 – 0)/(2 – 0)
m = 8/2
m = 4

Solve the equation. Check your solution.
Question 4.
2x + 3x = 10
Answer:
Given the equation
2x + 3x = 10
5x = 10
x = 10/5
x = 2

Question 5.
x + \(\frac{1}{6}\) = 4 – 2x
Answer:
Given the equation
x + \(\frac{1}{6}\) = 4 – 2x
x + 2x = 4 – \(\frac{1}{6}\)
3x = 4 – \(\frac{1}{6}\)
3x = \(\frac{23}{6}\)
x = \(\frac{23}{18}\)

Question 6.
2(1 – x) = 11
Answer:
2(1 – x) = 11
2 – 2x = 11
2 – 11 = 2x
2x = -9
x = -9/2

Concepts, Skills, & Problem Solving

USING EQUIVALENT RATIOS The graph shows amounts of water and flour that can be used to make dough. (See Exploration 1, p. 155.)
Big Ideas Math Answers Grade 8 Chapter 4 Graphing and Writing Linear Equations 4.3 7
Question 7.
Use the graph to make a ratio table relating the quantities. Explain how the slope of the line is represented in the table.
Answer:
BIM Grade 8 Solution Key Chapter 4 img_53
m = rise/run
= (10 – 5)/(6 – 3)
= 5/3
That means to every 5 cups of flour there is an increase of 3 cups of water.
The slope m is 5/3.

Question 8.
Make a ratio table that represents a different ratio of flour to water. Use the table to describe the slope of the graph of the new relationship.
Answer:
BIM Grade 8 Solution Key Chapter 4 img_54
From the table we find that for every increase of 7 cups of flour there is an increase of 4 cups of water.
The slope is 7/4.

Question 9.
GRAPHING AN EQUATION
The amount y(in dollars) that you raise by selling fundraiser tickets is represented by the equation y = 5x. Graph the equation and interpret the slope.
Answer:
BIM Grade 8 Solution Key Chapter 4 img_55
The slope indicates that the unit cost is $5 per ticket.

IDENTIFYING PROPORTIONAL RELATIONSHIPS Tell whether and are in a proportional relationship. Explain your reasoning. If so, write an equation that represents the relationship.
Question 10.
Big Ideas Math Answers Grade 8 Chapter 4 Graphing and Writing Linear Equations 4.3 8
Answer:
The graph doesn’t represent a proportional relationship because it doesn’t pass through the point (0, 0).

Question 11.
Big Ideas Math Answers Grade 8 Chapter 4 Graphing and Writing Linear Equations 4.3 9
Answer:
The graph represents a proportional relationship because it is linear and passes through the point (0, 0)
(0, 0), (2, 8)
m = (8 – 0)/(2 – 0)
m = 8/2
m = 4
The equation is y = 4x

Question 12.
Big Ideas Math Answers Grade 8 Chapter 4 Graphing and Writing Linear Equations 4.3 10
Answer:
(2 – 1)/(6 – 3) = 1/3
(3 – 2)/(9 – 6) = 1/3
(4 – 3)/(12 – 9) = 1/3
As the rate of change is constant, it means that the graph is a line.
(1 – y)/(3 – 0) = 1/3
(1 – y)/3 = 1/3
1 – y = 1
y = 1 – 1
y = 0
Therefore the point (0, 0) belomgs to the graph.
So the table represents a proportional relationship
y = 1/3 x

Question 13.
Big Ideas Math Answers Grade 8 Chapter 4 Graphing and Writing Linear Equations 4.3 11
Answer:
(8 – 4)/(5 – 2) = 4/3
(13 – 8)/(8 – 5) = 5/3
(23 – 13)/10 – 8 = 10/2 = 5

Question 14.
MODELING REAL LIFE
The cost y (in dollars) to rent a kayak is proportional to the number x of hours that you rent the kayak. It costs $27 to rent the kayak for 3 hours.
Big Ideas Math Answers Grade 8 Chapter 4 Graphing and Writing Linear Equations 4.3 12
a. Write an equation that represents the situation.
b. Interpret the slope of the graph of the equation.
c. How much does it cost to rent the kayak for 5 hours? Justify your answer.
Answer:
y = kx
27 = k × 3
k = 27/3
k = 9
The equation is k = 9x
b. The slope k = 3 shows that the cost of renting the kayak per hour is $9.
c. y = 9 × 5
y = 45

Question 15.
MODELING REAL LIFE
The distance y (in miles) that a truck travels on x gallons of gasoline is represented by the equation y = 18x. The graph shows the distance that a car travels.
Big Ideas Math Answers Grade 8 Chapter 4 Graphing and Writing Linear Equations 4.3 13
a. Which vehicle gets better gas mileage? Explain how you found your answer.

Answer:
y = 18x
(0, 0), (2, 50)
m = (50 – 0)/(2 – 0)
m = 50/2
m = 25
25 > 18
Therefore the car has better mileage.

b. How much farther can the vehicle you chose in part(a) travel on 8 gallons of gasoline?
Answer:
y = 25 × 8 – 18 × 8
= 200 – 144
= 56 miles

Question 16.
PROBLEM SOLVING
Toenails grow about 13 millimeters per year. The table shows fingernail growth.
Big Ideas Math Answers Grade 8 Chapter 4 Graphing and Writing Linear Equations 4.3 14
a. Do fingernails or toenails grow faster? Explain.

Answer:
y = 0.25x
m = (1.4 – 0.7)/(2 – 1)
m = 0.7
y = 0.7x
Because 0.7 > 0.25, the fingernails grow faster.

b. In the same coordinate plane, graph equations that represent the growth rates of toenails and fingernails. Compare and interpret the steepness of each graph.
Answer:
BIM Answer Key Grade 8 Chapter 4 img_57

Question 17.
REASONING
The quantities and are in a proportional relationship. What do you know about the ratio of y to x for any point (x, y) on the graph of x and y?
Answer:
y = kx
where k is constant
y/x = k
This means the ratio of y to x is constant.

Question 18.
DIG DEEPER!
The graph relates the temperature change y (in degrees Fahrenheit) to the altitude change x (in thousands of feet).
Big Ideas Math Answers Grade 8 Chapter 4 Graphing and Writing Linear Equations 4.3 15
a. Is the relationship proportional? Explain.

Answer: The relationship is proportional because the graph is linear and passes through the origin.

b. Write an equation of the line. Interpret the slope.

Answer:
(0,0), (10, -35)
m = (-35 – 0)/(10 – 0)
= -35/10
= -3.5
y = -3.5x

c. You are at the bottom of a mountain where the temperature is 74°F. The top of the mountain is 5500 feet above you. What is the temperature at the top of the mountain? Justify your answer.
Answer:
x = 5.5 – 0 = 5.5 thousand feet
y = -3.5x = -3.5(5.5) = -19.25
74 – 19.25 = 54.75°F

Question 19.
CRITICAL THINKING
Consider the distance equation d = rt, where d is the distance (in feet), r is the rate (in feet per second), and t is the time (in seconds). You run for 50 seconds. Are the distance you run and the rate you run at proportional? Use a graph to justify your answer.
Answer:
d = rt
d = 50r
Having the form y = kx the equation represents a proportional relationship.
BIM Answer Key Grade 8 Chapter 4 img_58

Lesson 4.4 Graphing Linear Equations in Slope-Intercept Form

EXPLORATION 1

Deriving an Equation
Work with a partner. In the previous section, you learned that the graph of a proportional relationship can be represented by the equation y = mx, where m is the constant of proportionality.
Big Ideas Math Solutions Grade 8 Chapter 4 Exponents and Scientific Notation 4.4 1
a. You translate the graph of a proportional relationship 3 units up as shown below. Let (x, y) represent any point on the graph. Make a conjecture about the equation of the line. Explain your reasoning.
Big Ideas Math Solutions Grade 8 Chapter 4 Exponents and Scientific Notation 4.4 2
b. Describe the relationship between the corresponding side lengths of the triangles. Explain your reasoning.
c. Use the relationship in part(b) to write an equation relating y, m, and x. Does your equation support your conjecture in part(a)? Explain.
d. You translate the graph of a proportional relationship b units up. Write an equation relating y, m, x, and b. Justify your answer.
Answer:

Big Ideas Math Solutions Grade 8 Chapter 4 Exponents and Scientific Notation 4.4 3

Try It

Find the slope and the y-intercept of the graph of the linear equation.
Question 1.
y = 3x – 7
Answer:
Given the equation
y = 3x – 7
Write the equation in slope – intercept form: y = mx + b
The slope of the line is m and the y – intercept of the line is b.
y = 3x – 7
Slope = 3 and y – intercept = -7

Question 2.
y – 1 = –\(\frac{2}{3}\)x
Answer:
Write the equation in slope – intercept form: y = mx + b
The slope of the line is m and the y – intercept of the line is b.
y – 1 = –\(\frac{2}{3}\)x
y = –\(\frac{2}{3}\)x + 1
Slope = –\(\frac{2}{3}\) and y – intercept = 1

Graph the linear equation. Identify the x-intercept.
Question 3.
y = x – 4
Answer:
y = x – 4
Comparing the above equation with slope – intercept equation.
slope = 1, y-intercept = -4
Ploy y – intercept and slope
slope = rise/run = 1/1
Plot the point that is 1 unit right and 1 unit up from (0, -4) = (1, -3)
Grade 8 BIM Answers Chapter 4 img_59
Thus the intercept is 4.

Question 4.
y = –\(\frac{1}{2}\)x + 1
Answer:
y = –\(\frac{1}{2}\)x + 1
Comparing the above equation with slope – intercept equation.
Slope = –\(\frac{1}{2}\), y-intercept = 1
y-intercept = 1. So plot (0, 1)
Slope = rise/run = -1/2
Plot the point that is 2 units right and 1 unit down from (0, -4) = (2, 0)
Grade 8 BIM Answers Chapter 4 img_60
So, the x-intercept is 2.

Self-Assessment for Concepts & Skills
Solve each exercise. Then rate your understanding of the success criteria in your journal.

Question 5.
IN YOUR OWN WORDS
Consider the graph of the equation y = mx + b.
a. How does changing the value of m affect the graph of the equation?

Answer:
The value of m is the slope of the graph. If the value of m changes it means the slope of the graph is changing, whether it will rise or fall from left or right is dependent on the value of m.

b. How does changing the value of b affect the graph of the equation?
Answer:
The value of b is the y-intercept of the graph. If the value of b changes it means it affects where the graph crosses the y – axis.

IDENTIFYING SLOPE AND y-INTERCEPT Find the slope and the y-intercept of the graph of the linear equation.
Question 6.
y = -x + 0.25
Answer:
y = mx + c
slope = -1 and y – intercept = 0.25

Question 7.
y – 2 = –\(\frac{3}{4}\)x
Answer:
Given the equation
y – 2 = –\(\frac{3}{4}\)x
y = –\(\frac{3}{4}\)x + 2
slope = –\(\frac{3}{4}\) and y – intercept = 2

GRAPHING A LINEAR EQUATION Graph the linear equation. Identify the x-intercept.
Question 8.
y = x – 7
Answer:
Grade 8 BIM Answers Chapter 4 img_61
The line crosses the x-axis at (7, 0)
So, the x – intercept is 7.

Question 9.
y = 2x + 8
Answer:
Grade 8 BIM Answers Chapter 4 img_62
The line crosses the x – axis at (-4, 0)
So, the x – intercept is -4.

Self-Assessment for Problem Solving
Solve each exercise. Then rate your understanding of the success criteria in your journal.

Question 10.
The height y (in feet) of a movable bridge after rising for seconds is represented by the equation y = 3x + 6. Graph the equation. Interpret the y-intercept and slope. How many seconds does it take the bridge to reach a height of 76 feet? Justify your answer.
Big Ideas Math Solutions Grade 8 Chapter 4 Exponents and Scientific Notation 4.4 4
Answer:
Given,
y = 3x + 6.
slope = 3, y – intercept = 16
Grade 8 BIM Answers Chapter 4 img_63
The y – intercept is 16. So, the initial height of the bridge is 16 feet.
The slope is 3. So, the bridge rises 3 feet per second.
The bridge will reach a height of 76 feet in 20 seconds.

Question 11.
The number of perfume bottles in storage after x months is represented by the equation y = -20x + 460. Graph the equation. Interpret the y-intercept and the slope. In how many months will there be no perfume bottles left in storage? Justify your answer.
Big Ideas Math Solutions Grade 8 Chapter 4 Exponents and Scientific Notation 4.4 5.1
Answer:
Given the equation
y = -20x + 460
Slope = -20, y-intercept = 460
Grade 8 BIM Answers Chapter 4 img_64
The y-intercept is 460. So, the initial number of perfume in the storage is 460.
The slope is -20. So, the number of perfume bottle decrease with 20 bottles per months.
There will be no perfume bottle left in the storage in 23 months.

Graphing Linear Equations in Slope-Intercept Form Homework & Practice 4.4

Review & Refresh

Tell whether x and y are in a proportional relationship. Explain your reasoning. If so, write an equation that represents the relationship.
Question 1.
Big Ideas Math Solutions Grade 8 Chapter 4 Exponents and Scientific Notation 4.4 5
Answer:
(8 – 6)/(2 – 1) = 2/1 = 2
(10 – 8)/(3 – 2) = 2/1 = 2
(12 – 10)/(4 – 3) = 2/1 = 2
The rate of change in the table is constant.
(6 – y)/(1 – 0) = 2
6 – y = 2
y = 6 – 2
y = 4
Therefore the graph does not pass through the origin.
So x and y are not proportional.

Question 2.
Big Ideas Math Solutions Grade 8 Chapter 4 Exponents and Scientific Notation 4.4 6
Answer:
(4 – 0)/(-8 – 0) = 4/-8 = -1/2 = -0.5
(2 – 4)/(-4 – (-8)) = -2/4 = -1/2 = -0.5
(-2 – 2)/(4 – (-4)) = -4/8 = -1/2 = -0.5
(-4 – (-2))/(8 – 4) = -2/4 = -1/2 = -0.5
As the rate of change is constant, x and y are in a proportional relationship.
y = -0.5x

Solve the equation for y.
Question 3.
x = 4y – 2
Answer:
Given the equation
x = 4y – 2
x – 2 = 4y
y = x/4 + 1/2

Question 4.
3y = -6x + 1
Answer:
Given the equation
3y = -6x + 1
y = -2x + 1/3

Question 5.
1 + y = –\(\frac{4}{5}\)x – 2
Answer:
Given the equation
1 + y = –\(\frac{4}{5}\)x – 2
y = –\(\frac{4}{5}\)x – 3

Question 6.
2.5y = 5x – 5
Answer:
Given the equation
2.5y = 5x – 5
y = 2x – 2

Question 7.
1.3y + 5.2 = -3.9x
Answer:
Given the equation
1.3y + 5.2 = -3.9x
1.3y = -3.9x – 5.2
y = -3x – 4

Question 8.
y – \(\frac{2}{3}\)x = -6
Answer:
Given the equation
y – \(\frac{2}{3}\)x = -6
y = \(\frac{2}{3}\)x -6

Concepts, Skills, &Problem Solving

GRAPHING A LINEAR EQUATION Graph the equation. (See Exploration 1, p. 161.)
Question 9.
The graph of y = 3.5x is translated up 2 units.
Answer:
Given the equation
y = 3.5x
The line obtained by translating the graph of the line y = 3.5x up 2 units has the same slope (3.5) and y – intercept 2 units greater, which means b = 0 + 2 = 2
Big Ideas Math Grade 8 Answer Key Chapter 4 img_60

Question 10.
The graph of y = -5x is translated down 3 units.
Answer:
y = -5x
The line obtained by translating the graph of the line y = -5x down 3 units has the same slope and the y – intercept 3 units smaller, which means b = 0 – 3 = -3
Big Ideas Math 8th Grade Answer Key for Chapter 4 img_61

MATCHING EQUATIONS AND GRAPHS Match the equation with its graph. Identify the slope and the y-intercept.
Question 11.
y = 2x + 1
Answer:
Given the eqation
y = 2x + 1
slope = 2 and y – intercept = 1
Big Ideas Math 8th Grade Answer Key for Chapter 4 img_62

Question 12.
y = \(\frac{1}{3}\)x – 2
Answer:
slope = 1/3 and y – intercept = -2
Big Ideas Math 8th Grade Answer Key for Chapter 4 img_63

Question 13.
y = –\(\frac{2}{3}\)x + 1
Answer:

Big Ideas Math Solutions Grade 8 Chapter 4 Exponents and Scientific Notation 4.4 7

Answer:
Slope = -2/3 and y – intercept = 1
The graph which passes through the point (0, 1) and has a negative slope is the matching graph of the given equation.

IDENTIFYING SLOPES AND y-INTERCEPTS Find the slope and the y-intercept of the graph of the linear equation.
Question 14.
y = 4x – 5
Answer:
y = mx + b
slope = 4 and y — intercept = -5

Question 15.
y = -7x + 12
Answer:
y = -7x + 12
y = mx + b
slpoe = -7 and y – intercept = 12

Question 16.
y = –\(\frac{4}{5}\)x – 2
Answer:
y = mx + b
slope = -4/5
y – intercept = -2

Question 17.
y = 2.25x + 3
Answer:
y = mx + b
slope = 2.25 and y – intercept = 3

Question 18.
y + 1 = \(\frac{4}{3}\)x
Answer:
y = mx + b
y + 1 = \(\frac{4}{3}\)x
y = \(\frac{4}{3}\)x – 1
slope = \(\frac{4}{3}\), y – intercept = -1

Question 19.
y – 6 = \(\frac{3}{5}\)x
Answer:
y = mx + b
y – 6 = \(\frac{3}{5}\)x
y = \(\frac{3}{5}\)x + 6
slope = 3/8 and y – intercept = 6

Question 20.
y – 3.5 = -2x
Answer:
y = mx + b
y – 3.5 = -2x
y = -2x + 3.5
slope = -2 and y – intercept = 3.5

Question 21.
y = -5 – \(\frac{1}{2}\)x
Answer:
y = mx + b
y = -5 – \(\frac{1}{2}\)x
y =- \(\frac{1}{2}\)x – 5
slope = – \(\frac{1}{2}\) and y – intercept = -5

Question 22.
y = 11 + 1.5x
Answer:
y = mx + b
y = 1.5x + 11
slope = 1.5 and y – intercept = 11

Question 23.
YOU BE THE TEACHER
Your friend finds the slope and y-intercept of the graph of the equation y = 4x – 3. Is your friend correct? Explain your reasoning.
Big Ideas Math Solutions Grade 8 Chapter 4 Exponents and Scientific Notation 4.4 8
Answer:
y = 4x – 3
No my friend is not correct because the y – intercept is -3.

Question 24.
MODELING REAL LIFE
The number y of seasonal allergy shots available at a facility x days after receiving a shipment is represented by y = -15x + 375.
a. Graph the linear equation.
b. Interpret the slope and the y-intercept.
Answer:
y = -15x + 375
x = 0
y = -15(0) + 375 = 375
y = 0
0 = -15x + 375
15x = 375
x = 375/15
x = 25
BIM Grade 8 Solution Key Chapter 4 img_64
The slope shows that the number of seasonal allergy shots decrease by 15 shots each day.
The y – intercept shows that the number of shots immediately after receiving a shipment is 375.

GRAPHING AN EQUATION Graph the linear equation. Identify the x-intercept.
Question 25.
y = x + 3
Answer:
Given the equation
y = x + 3
slope = 1 and y – intercept = 3
Slope = rise/run = 1/1
Plot the point that is 1 unit right and 1 unit up from (0, 3) = (1, 4)
BIM Grade 8 Solution Key Chapter 4 img_65
So, the x – intercept is -3.

Question 26.
y = 4x – 8
Answer:
y = 4x – 8
Comparing the above equation with slope – intercept equation.
slope = 4 and y – intercept = -8
Slope = rise/run = 4/1 = 4
Plot the point that is 1 unit right and 4 unit up from (0, -8) = (1, -4)
BIM Grade 8 Solution Key Chapter 4 img_66

Question 27.
y = -3x + 9
Answer:
y = -3x + 9
slope = -3 and y – intercept = 9
slope rise/run = -3/1 = -3
BIM Grade 8 Solution Key Chapter 4 img_67

So, the intercept is 3.

Question 28.
y = -5x – 5
Answer:
y = -5x – 5
slope = -5 and y – intercept = -5
slope = rise/run = -5/1
Plot the point that is 1 unit right and 5 unit up from (0, -5) = (1, -10)
BIM Grade 8 Solution Key Chapter 4 img_68
So, the x – intercept is -1.

Question 29.
y + 14 = -7x
Answer:
y + 14 = -7x
y = -7x – 14
slope = -7 and y – intercept = -14
Slope = rise/run = -7/1
Plot the point that is 1 unit right and 7 unit down from (0, -14) = (1, -21)
BIM Grade 8 Solution Key Chapter 4 img_69
So, the x – intercept is -2.

Question 30.
y = 8 – 2x
Answer:
Given the equation
y = 8 – 2x
y = -2x + 8
slope = -2 and y – intercept = 8
slope = rise/run = -2/1
Plot the point 1 unit right and 2 units down from (0, 8) = (1, 6)
BIM Grade 8 Solution Key Chapter 4 img_70
So, the x – intercept is 4.

Question 31.
PRECISION
You go to a harvest festival and pick apples.
a. Which equation represents the cost (in dollars) of going to the festival and picking x pounds of apples? Explain.
Big Ideas Math Solutions Grade 8 Chapter 4 Exponents and Scientific Notation 4.4 9
b. Graph the equation you chose in part(a).
Answer:
Picking a pound of apples costs $0.75, therefore x pounds cost 0.75 × x = 0.75x
y = 0.75x + 5
BIM Grade 8 Solution Key Chapter 4 img_71

Question 32.
REASONING
Without graphing, identify the equations of the lines that are parallel. Explain your reasoning.
Big Ideas Math Solutions Grade 8 Chapter 4 Exponents and Scientific Notation 4.4 10
Answer:
The lines which area parallel are those having the same slope.
y = 2x + 4
y = 2x – 3
y = 2x + 1
y = 1/2x + 1
y = 1/2x + 2

Question 33.
PROBLEM SOLVING
A skydiver parachutes to the ground. The height y (in feet) of the skydiver after x seconds is y = -10x + 3000.
Big Ideas Math Solutions Grade 8 Chapter 4 Exponents and Scientific Notation 4.4 11
a. Graph the linear equation.
b. Interpret the slope, y-intercept, and x-intercept.
Answer:
y = -10x + 3000
x = 0
y = -10(0) + 3000 = 3000
y = 0
0 = -10 + 3000
10x = 3000
x = 3000/10 = 300
BIM Grade 8 Solution Key Chapter 4 img_72
b. The slope shows that each second the skydiver descends 10 feet.
The y – intercept shows that the skydiver begins its dive from 3000 feet.
The x – intercept shows that he reaches the ground after 300 seconds.

Question 34.
DIG DEEPER!
Six friends create a website. The website earns money by selling banner ads. It costs $120 a month to operate the website.
a. A banner ad earns $0.005 per click. Write a linear equation that represents the monthly profit after paying operating costs.
b. Graph the equation in part(a). On the graph, label the number of clicks needed for the friends to start making a profit. Explain.
Answer:
y = 0.005x – 120
x = 0
y = 0.005(0) – 120
y = -120
y = 0
0 = 0.005x – 120
0.005x = 120
x = 24000
BIM Grade 8 Solution Key Chapter 4 img_73
x > 24,000

Lesson 4.5 Graphing Linear Equations in Standard Form

EXPLORATION 1

Using Intercepts
Work with a partner. You spend $150 on fruit trays and vegetable trays for a party.
Big Ideas Math Answer Key Grade 8 Chapter 4 Graphing and Writing Linear Equations 4.5 1
a. You buy x fruit trays and y vegetable trays. Complete the verbal model. Then use the verbal model to write an equation that relates x and y.
Big Ideas Math Answer Key Grade 8 Chapter 4 Graphing and Writing Linear Equations 4.5 2
b. What is the greatest number of fruit trays that you can buy? vegetable trays? Can you use these numbers to graph your equation from part (a) in the coordinate plane? Explain.
Big Ideas Math Answer Key Grade 8 Chapter 4 Graphing and Writing Linear Equations 4.5 3
c.Use a graph to determine the different combinations of fruit trays and vegetable trays that you can buy. Justify your answers algebraically.
d. You are given an extra $50 to spend. How does this affect the intercepts of your graph in part(c)? Explain your reasoning.
Answer:

Big Ideas Math Answer Key Grade 8 Chapter 4 Graphing and Writing Linear Equations 4.5 4

Try It

Graph the linear equation.
Question 1.
x + y = -2
Answer:
Given the equation
y = mx + b
x + y = -2
y = -x – 2
Comparing the value of b and m from y = mx + b
m = -1 and b = -2
Plot y – intercept = (0, b) = (0, -2)
Slope = -1
run/rise = -1/1
Plot the point 1 unit down and 1 unit to the right = (1, -3)
Now plot the points and draw the graph
BIM Grade 8 Solution Key Chapter 4 img_74

Question 2.
–\(\frac{1}{2}\)x + 2y = 6
Answer:
–\(\frac{1}{2}\)x + 2y = 6
2y = 6 + \(\frac{1}{2}\)x
y = 0.25x + 3
Comparing the value of b and m from y = mx + b
m = 0.25 and b = 3
Plot y – intercept = (0, b) = (0, 3)
Slope = 0.25
run/rise = 0.25/1
Plot the point 0.25 unit up and 1 unit to the right = (1, 3.25)
Now plot the points and draw the graph

BIM Grade 8 Solution Key Chapter 4 img_75

Question 3.
–\(\frac{2}{3}\)x + y = 0
Answer:
–\(\frac{2}{3}\)x + y = 0
y = \(\frac{2}{3}\)x
Comparing the value of b and m from y = mx + b
m = \(\frac{2}{3}\) and b = 0
Plot y – intercept = (0, b) = (0, 0)
Slope =\(\frac{2}{3}\)
run/rise = \(\frac{2}{3}\)
Plot the point 0.25 unit up and 1 unit to the right = (3, 2)
Now plot the points and draw the graph
BIM Grade 8 Solution Key Chapter 4 img_76

Question 4.
2x + y = 5
Answer:
2x + y = 5
y = -2x + 5
Comparing the value of b and m from y = mx + b
m = -2 and b = 5
Plot y – intercept = (0, b) = (0, 5)
Slope = -2
run/rise = \(\frac{-2}{1}\)
Plot the point 0.25 unit up and 1 unit to the right = (1, 3)
Now plot the points and draw the graph
BIM Grade 8 Solution Key Chapter 4 img_77

Graph the linear equation using intercepts.
Question 5.
2x – y = 8
Answer:
y = 0
2x – y = 8
2x – 0 = 8
2x = 8
x = 4
The x – intercept is (4, 0)
Y – intercept :
x = 0
2x – y = 8
2(0) – y = 8
y = -8
BIM Grade 8 Solution Key Chapter 4 img_78

Question 6.
x + 3y = 6
Answer:
X-intercept:
y = 0
x + 3y = 6
x + 3(0) = 6
x + 0 = 6
x = 6
The x – intercept is (6, 0)
Y – intercept:
x = 0
x + 3y = 6
0 + 3y = 6
y = 2
The y – intercept is (0, 2)
BIM Grade 8 Solution Key Chapter 4 img_79

Self-Assessment for Concepts & Skills
Solve each exercise. Then rate your understanding of the success criteria in your journal.

STRUCTURE Determine whether the equation is in standard form. If not, rewrite the equation in standard form.
Question 7.
y = x – 6
Answer:
y = x – 6
The standard form of equation is: Ax + By = C
The given equation is not in the standard form.
y = x – 6
x – y = 6

Question 8.
y – \(\frac{1}{6}\)x + 5 = 0
Answer:
The standard form of equation is: Ax + By = C
The given equation is not in the standard form.
y – \(\frac{1}{6}\)x + 5 = 0
\(\frac{1}{6}\)x – y = 5

Question 9.
4x + y = 5
Answer:
The standard form of equation is: Ax + By = C
The given equation is in the form of the standard form.

Question 10.
WRITING
Describe two ways to graph the equation 4x + 2y = 6.
Answer:
The two ways to graph the equation:
1. Graph the equation using standard form
2. Graph the equation using intercept.

GRAPHING A LINEAR EQUATION Graph the linear equation.
Question 11.
4x + y = 5
Answer:
Given the equation
4x + y = 5
y = -4x + 5
Comparing the value of b and m from y = mx + b
m = -4 and b = 5
Plot y – intercept = (0, b) = (0, 5)
Slope = -4
run/rise = \(\frac{-4}{1}\)
Plot the point 4 unit down and 1 unit to the right = (1, 1)
Now plot the points and draw the graph
BIM Grade 8 Solution Key Chapter 4 img_81

Question 12.
\(\frac{1}{3}\)x + 2y = 8
Answer:
X – intercept:
y = 0
\(\frac{1}{3}\)x + 2y = 8
\(\frac{1}{3}\)x + 2(0) = 8
\(\frac{1}{3}\)x = 8
x = 24
The x – intercept is (24, 0)
Y – intercept:
x = 0
\(\frac{1}{3}\)x + 2y = 8
\(\frac{1}{3}\)(0) + 2y = 8
2y = 8
y = 4
The y – intercept is (0, 4)
BIM Grade 8 Solution Key Chapter 4 img_82

Question 13.
5x – y = 10
Answer:
X – intercept:
y = 0
5x – 0 = 10
5x = 10
x = 2
The x-intercept is (2, 0)
Y – intercept:
x = 0
5x – y = 10
5(0) – y = 10
-y = 10
y = -10
The y – intercept is (0, -10)
BIM Grade 8 Solution Key Chapter 4 img_83

Question 14.
x – 3y = 9
Answer:
X – intercept:
y = 0
x – 3(0) = 9
x = 9
The x – intercept is (9, 0)
Y – intercept:
x = 0
0 – 3y = 9
-3y = 9
y = -3
The y – intercept is (0, -3)
BIM Grade 8 Solution Key Chapter 4 img_84

Self-Assessment for Problem Solving
Solve each exercise. Then rate your understanding of the success criteria in your journal.

Question 15.
You have $30 to spend on paint and clay. The equation 2x + 6y = 30 represents this situation, where x is the number of paint bottles and y is the number of tubs of clay. Graph the equation. Interpret the intercepts. How many bottles of paint can you buy if you buy 3 tubs of clay? Justify your answer.
Answer:
Given,
You have $30 to spend on paint and clay.
The equation 2x + 6y = 30 represents this situation,
where x is the number of paint bottles and y is the number of tubs of clay.
X – intercept:
y = 0
2x + 6y = 30
2x + 6(0) = 30
2x = 30
x = 15
The x – intercept is (15, 0)
Y – intercept:
x = 0
2x + 6y = 30
2(0) + 6y = 30
6y = 30
y = 5
The y – intercept is (0, 5)
BIM Grade 8 Solution Key Chapter 4 img_85
From the graph, I can buy 6 bottles of point if I buy 3 tubs of clay.
BIM Grade 8 Solution Key Chapter 4 img_86

Question 16.
You complete two projects for a class in 60 minutes. The equation x + y = 60 represents this situation, where x is the time (in minutes) you spend assembling a birdhouse and y is the time (in minutes) you spend writing a paper.
Big Ideas Math Answer Key Grade 8 Chapter 4 Graphing and Writing Linear Equations 4.5 5
a. Graph the equation. Interpret the intercepts.

Answer:
x + y = 60
y = -x + 60
BIM Grade 8 Solution Key Chapter 4 img_87

b. You spend twice as much time assembling the birdhouse as you do writing the paper. How much time do you spend writing the paper? Justify your answer.
Answer:
We are given,
y = 2x
2x = -x + 60
2x + x = 60
3x = 60
x = 20
y = 2 (20)
y = 40

Graphing Linear Equations in Standard Form Homework & Practice 4.5

Review & Refresh

Find the slope and the y-intercept of the graph of the linear equation.
Question 1.
y = x – 1
Answer:
y = mx + b
Slope = -1 and y – intercept = -1

Question 2.
y = -2x + 1
Answer:
y = -2x + 1
y = mx + b
Slope = -2 and y – intercept = 1

Question 3.
y = \(\frac{8}{9}\)x – 8
Answer:
y = \(\frac{8}{9}\)x – 8
y = mx + b
Slope = \(\frac{8}{9}\) and y – intercept = -8

Tell whether the blue figure is a reflection of the red figure.
Question 4.
Big Ideas Math Answer Key Grade 8 Chapter 4 Graphing and Writing Linear Equations 4.5 6
Answer:
The blue figure is not a reflection of the red figure because, for example the reflection of the upper leg of the upper leg of the red triangle across the y-axis is the top vertex of the blue triangle, not a point.

Question 5.
Big Ideas Math Answer Key Grade 8 Chapter 4 Graphing and Writing Linear Equations 4.5 7
Answer:
The blue figure is a reflection of the red figure because to each point in the red figure corresponds a symmetrical point in the blue figure.

Question 6.
Big Ideas Math Answer Key Grade 8 Chapter 4 Graphing and Writing Linear Equations 4.5 8
Answer:
The blue figure is a reflection of the red figure because to each point in the red figure corresponds a symmetrical point in the blue figure.

Concepts, Skills, &Problem Solving

USING INTERCEPTS Define two variables for the verbal model. Write an equation in slope-intercept form that relates the variables. Graph the equation using intercepts. (See Exploration 1, p. 167.)
Question 7.
Big Ideas Math Answer Key Grade 8 Chapter 4 Graphing and Writing Linear Equations 4.5 9
Answer:
x = amount of peaches (in pounds)
y = the amount of apples (in pounds)
2x + 1.5y = 15
y = 0 = 2x + 1.5(0) = 15
2x = 15
x = 7.5
x = 0
2(0) + 1.5y = 15
1.5y =15
y = 10
BIM Grade 8 Solution key Chapter 4 img_88

Question 8.
Big Ideas Math Answer Key Grade 8 Chapter 4 Graphing and Writing Linear Equations 4.5 10
Answer:
x = the biked distance (in miles)
y = the walked distance (in miles)
y = 0
16x + 2(0) = 32
16x = 32
x = 2
x = 0
16(0) + 2y = 32
2y = 32
y = 16
BIM Grade 8 Solution key Chapter 4 img_89

REWRITING AN EQUATION Write the linear equation in slope-intercept form.
Question 9.
2x + y = 17
Answer:
Given the equation
2x + y = 17
y = 17 – 2x
y = -2x + 17

Question 10.
5x – y = \(\frac{1}{4}\)
Answer:
Given the equation
5x – y = \(\frac{1}{4}\)
-y = \(\frac{1}{4}\) – 5x
y = 5x – \(\frac{1}{4}\)

Question 11.
–\(\frac{1}{2}\)x + y = 10
Answer:
Given the equation
–\(\frac{1}{2}\)x + y = 10
y = \(\frac{1}{2}\)x + 10

GRAPHING AN EQUATION Graph the linear equation.
Question 12.
-18x + 9y = 72
Answer:
Given the equation
-18x + 9y = 72
X – intercept:
y = 0
-18x + 9(0) = 72
-18x = 72
x = -4
The x – intercept is (-4, 0)
Y – intercept:
x = 0
-18x + 9y = 72
-18(0) + 9y = 72
9y = 72
y = 8
BIM Grade 8 Answers Chapter 4 img_90

Question 13.
16x – 4y = 2
Answer:
Given the equation
16x – 4y = 2
X – intercept:
y = 0
16x – 4y = 2
16x – 4(0) = 2
16x = 2
x = 0.125
The X – intercept is (0.125, 0)
Y – intercept:
x = 0
16(0) – 4y = 2
-4y = 2
y = -2
BIM Grade 8 Answers Chapter 4 img_91

Question 14.
\(\frac{1}{4}\)x + \(\frac{3}{4}\)y = 1
Answer:
Given the equation
\(\frac{1}{4}\)x + \(\frac{3}{4}\)y = 1
x + 3y = 4
y = 0
x + 3(0) = 4
x = 4
x = 0
0 + 3y = 4
3y = 4
y = 4/3
BIM Grade 8 Answers Chapter 4 img_93

MATCHING Match the equation with its graph.
Question 15.
15x – 12y = 60
Answer:
y = 0
15x – 12(0) = 60
15x = 60
x = 60/15
x = 4
x = 0
15(0) – 12y = 60
-12y = 60
y = -5
The graph having the x – intercept 4 and y – intercept -5

Question 16.
5x + 4y = 20
Answer:
Given the linear equation
5x + 4y = 20
y = 0
5x + 4(0) = 20
5x = 20
x = 4
x = 0
5(0) + 4y = 20
4y = 20
y = 5

Question 17.
10x + 8y = -40
Answer:

Big Ideas Math Answer Key Grade 8 Chapter 4 Graphing and Writing Linear Equations 4.5 11
10x + 8y = -40
y = 0
10x + 8(0) = -40
10x = -40
x = -4
x = 0
10(0) + 8y = -40
8y = -40
y = -5

Question 18
YOU BE THE TEACHER
Your friend finds the x-intercept of -2x + 3y = 12. Is your friend correct? Explain your reasoning.
Big Ideas Math Answer Key Grade 8 Chapter 4 Graphing and Writing Linear Equations 4.5 12
Answer:
-2x + 3y = 12
y = 0
-2x + 3(0) = 12
-2x = 12
x = -6
Your friend is not correct because the x – intercept is the value of x corresponding to y = 0.
Your friend computed the y – intercept.

Question 19.
MODELING REAL LIFE
A charm bracelet costs $65, plus $25 for each charm. The equation -25x + y = 65 represents the cost y (in dollars) of the bracelet, where x is the number of charms.
a. Graph the equation.
b. How much does a bracelet with three charms cost?
Answer:
BIM Grade 8 Answers Chapter 4 img_94
y = 25x + 65
Substitute the value of x in the equation
y = 25(3) + 65
y = 75 + 65
y = 140

USING INTERCEPTS TO GRAPH Graph the linear equation using intercepts.
Question 20.
3x – 4y = -12
Answer:
Given the equation
3x – 4y = -12
3x – 4(0) = -12
3x = -12
x = -4
The x – intercept is (-4, 0)
Y – intercept:
x = 0
3(0) – 4y = -12
-4y = -12
y = 3
The y – intercept is (0, 3)
BIM Grade 8 Answers Chapter 4 img_95

Question 21.
2x + y = 8
Answer:
X – intercept:
y = 0
2x + y = 8
2x + 0 = 8
2x = 8
x = 4
The x – intercept is (4, 0)
Y – intercept:
x = 0
2x + y = 8
2(0) + y = 8
y = 8
The y – intercept is (0, 8)
BIM Grade 8 Answers Chapter 4 img_96

Question 22.
\(\frac{1}{3}\)x – \(\frac{1}{6}\)y = –\(\frac{2}{3}\)
Answer:
X – intercept:
y = 0
\(\frac{1}{3}\)x – \(\frac{1}{6}\)(0) = –\(\frac{2}{3}\)
\(\frac{1}{3}\)x = –\(\frac{2}{3}\)
x = -2
The x – intercept is (-2, 0)
Y – intercept:
x = 0
\(\frac{1}{3}\)(0) – \(\frac{1}{6}\)y = –\(\frac{2}{3}\)
y = 4
The y – intercept is (0, 4)
BIM Grade 8 Answers Chapter 4 img_97

Question 23.
MODELING REAL LIFE
Your cousin has $90 to spend on video games and movies. The equation 30x + 15y = 90 represents this situation, where x is the number of video games purchased and y is the number of movies purchased. Graph the equation. Interpret the intercepts.
Answer:
30x + 15y = 90
x = 0
30(0) + 15y = 90
15y = 90
y = 6
y = 0
30x + 15(0) = 90
30x = 90
x = 3
BIM Grade 8 Answers Chapter 4 img_98
The x – intercept shows that 3 video games are purchased when no movies are purchased.
The y – intercept shows that 6 movies are purchased when no video games are purchased.

Question 24.
PROBLEM SOLVING
A group of friends go scuba diving. They rent a boat for x days and scuba gear for y people, represented by the equation 250x + 50y = 1000.
Big Ideas Math Answer Key Grade 8 Chapter 4 Graphing and Writing Linear Equations 4.5 13
a. Graph the equation and interpret the intercepts.
b. How many friends can go scuba diving if they rent the boat for 1 day? 2 days?
c. How much money is spent in total?
Answer:
250x + 50y = 1000
x = 0
250(0) + 50y = 1000
50y = 1000
y = 20
when y = 0
250x + 50(0) = 1000
250x = 1000
x = 4
BIM Grade 8 Answers Chapter 4 img_99
b.
250(1) + 50y = 1000
250 + 50y = 1000
50y = 1000 – 250
50y = 750
y = 15
when x = 2
250(2) + 50y = 1000
500 + 50y = 1000
50y = 1000 – 500
50y = 500
y = 500/50
y = 10

Question 25.
DIG DEEPER!
You work at a restaurant as a host and a server. You earn $9.45 for each hour you work as a host and $3.78 for each hour you work as a server.
Big Ideas Math Answer Key Grade 8 Chapter 4 Graphing and Writing Linear Equations 4.5 14
a. Write an equation in standard form that models your earnings.
b. Graph the equation.
Answer:
You earn $9.45 for each hour you work as a host and $3.78 for each hour you work as a server.
Number of hours worked as host + $3.78.
Number of hours worked as server = $113.40
9.45x + 3.78y = 113.40
x = 0
9.45(0) + 3.78y = 113.40
3.78y = 113.40
y = 30
when y = 0
9.45x + 3.78(0) = 113.40
9.45x = 113.40
x = 12
BIM Grade 8 Answers Chapter 4 img_100

Question 26.
LOGIC
Does the graph of every linear equation have an x-intercept? Justify your reasoning.
Answer:
y = mx + b
y = 0
0 = mx + b
mx = -b
x = -b/m for m ≠ 0
If m = 0 the equation has no solution. Therefore the equation y = b has no x – intercept.

Question 27.
CRITICAL THINKING
For a house call, a veterinarian charges $70, plus $40 per hour.
Big Ideas Math Answer Key Grade 8 Chapter 4 Graphing and Writing Linear Equations 4.5 15
a. Write an equation that represents the total fee y (in dollars) the veterinarian charges for a visit lasting x hours.

b. Find the x-intercept. Does this value make sense in this context? Explain your reasoning.
c. Graph the equation.
Answer:
Total fee = fixed charge + number of hours . cost per hour
y = 70 + 40x
y = 0
0 = 70 + 40x
-70 = 40x
x = -1.75
x = 0
y = 70 + 40(0)
y = 70
BIM Grade 8 Solutions Chapter 4 img_101

Lesson 4.6 Writing Equations in Slope-Intercept Form

EXPLORATION 1

Writing Equations of Lines
Work with a partner.For each part, answer the following questions.

  • What are the slopes and the y-intercepts of the lines?
  • What are equations that represent the lines?
  • What do the lines have in common?

Big Ideas Math Answers 8th Grade Chapter 4 Graphing and Writing Linear Equations 4.6 1
Answer:

EXPLORATION 2

Interpreting the Slope and the y-Intercept
Work with a partner. The graph represents the distance y (in miles) of a car from Phoenix after t hours of a trip.
Big Ideas Math Answers 8th Grade Chapter 4 Graphing and Writing Linear Equations 4.6 2
a. Find the slope and the y-intercept of the line. What do they represent in this situation?
b. Write an equation that represents the graph.
c. How can you determine the distance of the car from Phoenix after 11 hours?
Answer:

Try It

Write an equation in slope-intercept form of the line that passes through the given points.
Question 1.
Big Ideas Math Answers 8th Grade Chapter 4 Graphing and Writing Linear Equations 4.6 3
Answer:
m = (y2 – y1)/(x2 – x1)
= (4 – 2)/(1 – 0)
= 2/1
= 2
Because the line crosses the y – axis at (0, 2)
y = mx + b
y = 2x + 2

Question 2.
Big Ideas Math Answers 8th Grade Chapter 4 Graphing and Writing Linear Equations 4.6 4
Answer:
m = (y2 – y1)/(x2 – x1)
= (-1 – 3)/(0 – (-3))
= -4/3
Because y = -1 when x = 0, the y – intercept is -1
y = mx + b
y = -4/3 x – 1

Write an equation of the line that passes through the given points.
Question 3.
Big Ideas Math Answers 8th Grade Chapter 4 Graphing and Writing Linear Equations 4.6 5
Answer:
m = (y2 – y1)/(x2 – x1)
= (5 – 5)/(0 – (-4))
= 0/4
Because y = 5 when x = 0, the y – intercept is 5
y = mx + b
y = (0)x + 5
y = 5

Question 4.
Big Ideas Math Answers 8th Grade Chapter 4 Graphing and Writing Linear Equations 4.6 6
Answer:
m = (y2 – y1)/(x2 – x1)
= (1 – 1)/(3 – 0)
= 0/3
= 0
Because the line crosses the y – axis at (0, 1) the y – intercept is 1
y = mx + b
y = (0)x + 1
y = 1

Self-Assessment for Concepts & Skills
Solve each exercise. Then rate your understanding of the success criteria in your journal.

WRITING EQUATIONS IN SLOPE-INTERCEPT FORM Write an equation in slope-intercept form of the line that passes through the given points.
Question 5.
Big Ideas Math Answers 8th Grade Chapter 4 Graphing and Writing Linear Equations 4.6 7
Answer:
m = (y2 – y1)/(x2 – x1)
= (5 – 2)/(1 – 0)
= 3/1
= 3
Because y = 2 when x = 0, the y – intercept is 2
y = mx + b
y = (3)x + 2
y = 3x + 2

Question 6.
Big Ideas Math Answers 8th Grade Chapter 4 Graphing and Writing Linear Equations 4.6 8
Answer:
m = (y2 – y1)/(x2 – x1)
= (-1 – 5)/(1 – (-1))
= -6/2
= -3
Because the line crosses the y – axis at (0, 2) the y – intercept is 2
y = mx + b
y = -3x + 2

Question 7.
WRITING AN EQUATION
Write an equation of the line that passes through (0, -5) and (2, -5).
Answer:
m = (y2 – y1)/(x2 – x1)
= (-5 – (-5))/(2 – 0)
= 0/2
= 0
Because y = -5 when x = 0, the y – intercept is -5
y = mx + b
y = (0)x + -5
y = -5

Self-Assessment for Problem Solving
Solve each exercise. Then rate your understanding of the success criteria in your journal.

Question 8.
You load boxes onto an empty truck at a constant rate. After 3 hours, there are 100 boxes on the truck. How much longer do you work if you load a total of 120 boxes? Justify your answer.
Answer:
Let x be the number of hours you work if you load a total of 120 boxes.
100/3 = 120/x
100x = 3 × 120
x = 360/100
x = 3.6 hours
3.6 – 3 = 0.6 hours

Question 9.
The table shows the amounts (in tons) of waste left in a landfill after x months of waste relocation. Interpret the slope and the y-intercept of the line that passes through the given points. How many months does it take to empty the landfill? Justify your answer.
Big Ideas Math Answers 8th Grade Chapter 4 Graphing and Writing Linear Equations 4.6 9
Answer:
m = (12 – 15)/ (6 – 0)
m = -3/6
m = -0.5
b = 15
The y – intercept shows that there are 150 tons of waste in the beginning.
y = -0.5x + 15
y = 0
0 = -0.5x + 15
x = 30
So the ladfill will be emptied after 30 months.

Question 10.
DIG DEEPER!
A lifetime subscription to a website costs $250. A monthly subscription to the website costs $10 to join and $15 per month. Write equations to represent the costs of each plan. If you want to be a member for one year, which plan is less expensive? Explain.
Answer:
Given,
A lifetime subscription to a website costs $250. A monthly subscription to the website costs $10 to join and $15 per month.
Total cost for plan 1 = the lifetime subscription
y = 250
Total cost for Plan 2 = Fixed tax + Number of months . monthly cost
y = 10 + 15x
Plan 1: y = 250
Plan 2: y = 10 + 15(12) = 190
As 190 < 250, plan 1 is less expensive.

Writing Equations in Slope-Intercept Form Homework & Practice 4.6

Review & Refresh

Write the linear equation in slope-intercept form.
Question 1.
4x + y = 1
Answer:
Given the equation
4x + y = 1
y = -4x + 1

Question 2.
x – y = \(\frac{1}{5}\)
Answer:
Given the equation
x – y = \(\frac{1}{5}\)
x – \(\frac{1}{5}\) = y

Question 3.
–\(\frac{2}{3}\)x + 2y = -7
Answer:
Given the equation
–\(\frac{2}{3}\)x + 2y = -7
2y = -7 + \(\frac{2}{3}\)x
y = \(\frac{1}{3}\)x – \(\frac{7}{2}\)

Plot the ordered pair in a coordinate plane.
Question 4.
(1, 4)
Answer:
8th Grade BIM Answers Chapter 4 img_102

Question 5.
(-1, -2)
Answer:
8th Grade BIM Answers Chapter 4 img_103

Question 6.
(0, 1)
Answer:
8th Grade BIM Answers Chapter 4 img_104

Question 7.
(2, 7)
Answer:
8th Grade BIM Answers Chapter 4 img_105

Concepts, Skills, & Problem Solving

INTERPRETING THE SLOPE AND THE y-INTERCEPT The graph y represents the cost (in dollars) to open an online gaming account and buy x games. (See Exploration 2, p. 173.)
Big Ideas Math Answers 8th Grade Chapter 4 Graphing and Writing Linear Equations 4.6 10
Question 8.
Find the slope and the y-intercept of the line. What do they represent in this situation?
Answer:
(0, 15), (3, 45)
m = (45 – 15)/(3 – 0)
m = 30/3 10
Thus the slope of the line is m – 3.
b = 15
The slope represents the cost of one game, while the y – intercept is the cost of opening the gaming account.

Question 9.
Write an equation that represents the graph.
Answer:
m = 10
b = 15
y = mx + b
y = 10x + 15

Question 10.
How can you determine the total cost of opening an account and buying 6 games?
Answer:
y = 10x + 15
y = 10(6) + 15
y = 60 + 15
y = 75

WRITING EQUATIONS IN SLOPE-INTERCEPT FORM Write an equation in slope-intercept form of the line that passes through the given points.
Question 11.
Big Ideas Math Answers 8th Grade Chapter 4 Graphing and Writing Linear Equations 4.6 11
Answer:
m = (y2 – y1)/(x2 – x1)
= (4 – 3)/(0 – (-1))
= 1/1
= 1
Because the line crosses the y – axis at (0, 4) the y – intercept is 4
y = mx + b
y = (1)x + 4
y = x + 4

Question 12.
Big Ideas Math Answers 8th Grade Chapter 4 Graphing and Writing Linear Equations 4.6 12
Answer:
m = (y2 – y1)/(x2 – x1)
= (6 – 0)/(-3 – 0)
= 6/-3
= -2
Because the line crosses the y – axis at (0, 2) the y – intercept is 2
y = mx + b
y = -2x + 0
y = -2x

Question 13.
Big Ideas Math Answers 8th Grade Chapter 4 Graphing and Writing Linear Equations 4.6 13
Answer:
m = (y2 – y1)/(x2 – x1)
= (2 – 1)/(4 – 0)
= 1/4
Because the line crosses the y – axis at (0, 1) the y – intercept is 1
y = mx + b
y = 1/4 x + 1

Question 14.
Big Ideas Math Answers 8th Grade Chapter 4 Graphing and Writing Linear Equations 4.6 14
Answer:
m = (y2 – y1)/(x2 – x1)
= (1 – 2)/(0 – (-2))
= -1/2
Because y = 1 when x = 0, the y – intercept is 1
y = mx + b
y = -1/2 x + 2

Question 15.
Big Ideas Math Answers 8th Grade Chapter 4 Graphing and Writing Linear Equations 4.6 15
Answer:
m = (y2 – y1)/(x2 – x1)
= (-3 – (-4))/(0 – (-3))
= 1/3
Because y = -3 when x = 0, the y – intercept is -3
y = mx + b
y = 1/3 x – 3

Question 16.
Big Ideas Math Answers 8th Grade Chapter 4 Graphing and Writing Linear Equations 4.6 16
Answer:
m = (y2 – y1)/(x2 – x1)
= (-1 -4)/(0 – (-2))
= -5/2
Because y = -1 when x = 0, the y – intercept is -1
y = mx + b
y = -5/2 x – 1

WRITING EQUATIONS Write an equation of the line that passes through the given points.
Question 17.
(-1, 4), (0, 2)
Answer:
m = (y2 – y1)/(x2 – x1)
= (2 – 4)/(0 – (-1))
= -2/1
= -2
Because y = 2 when x = 0, the y – intercept is 2
y = mx + b
y = -2x + 2

Question 18.
(-1, 0), (0, 0)
Answer:
m = (y2 – y1)/(x2 – x1)
= (0 – 0)/(0 – (-1))
= 0/1
= 0
Because y = 0 when x = 0, the y – intercept is 0
y = mx + b
y = 0

Question 19.
(0, 4), (0, -3)
Answer:
Both points belong to the y-axis. Therefore the equation of the line passing through them is
x = 0

Question 20.
YOU BE THE TEACHER
Your friend writes an equation of the line shown. Is your friend correct? Explain your reasoning.
Big Ideas Math Answers 8th Grade Chapter 4 Graphing and Writing Linear Equations 4.6 17
Answer:
Because in the given graph, y = -2 when x = 0, so the y – intercept is -2. The equation of the line should be: y = 1/2 x – 2
No my friend is NOT correct.

Question 21.
MODELING REAL LIFE
A boa constrictor is 18 inches long at birth and grows 8 inches per year. Write an equation in slope y-intercept form that represents the length (in feet) of a boa constrictor that is x years old.
Big Ideas Math Answers 8th Grade Chapter 4 Graphing and Writing Linear Equations 4.6 18
Answer:
Given,
A boa constrictor is 18 inches long at birth and grows 8 inches per year.
Length after x years = birth length + number of years . Growth per year
y = 18 + 8x
y = 8x + 18
Convert it into feet
y = 2/3 x + 3/2

Question 22.
MODELING REAL LIFE
The table shows the speeds y (in miles per hour) of a car after x seconds of braking. Write an equation of the line that passes through the points in the table. Interpret the slope and the y-intercept of the line.
Big Ideas Math Answers 8th Grade Chapter 4 Graphing and Writing Linear Equations 4.6 19
Answer:
m = (y2 – y1)/(x2 – x1)
= (60 – 70)/(1 – 0)
= -10/1
= -10
Because y = 70 when x = 0, the y – intercept is 70
y = mx + b
y = -10x + 70
Slope = -10 represents the decrease in the speed of the car each seconds after breaking.
The y – intercept of 70 represents the initial speed of the car.

Question 23.
MODELING REAL LIFE
A dentist charges a flat fee for an office visit, plus an additional fee for every tooth removed. The graph shows the total cost y (in dollars) for a patient when the dentist removes x teeth. Interpret the slope and the y-intercept.
Big Ideas Math Answers 8th Grade Chapter 4 Graphing and Writing Linear Equations 4.6 20
Answer:
(2, 500), (4, 900)
m = (900 – 500)/(4 – 2)
m = 400/2
m = 200
y = mx + b
500 = 200(2) + b
500 = 400 + b
b = 500 – 400
b = 100
The slope shows that the amount charged for each removed tooth is $200.
The y – intercept shows that the flat fee for an office visit is $100.

Question 24.
MODELING REAL LIFE
One of your friends gives you $10 for a charity walkathon. Another friend gives you an amount per mile. After 5 miles, you have raised $13.50 total. Write an equation that represents the amount y of money you have raised after x miles.
Answer:
Given,
One of your friends gives you $10 for a charity walkathon.
Another friend gives you an amount per mile. After 5 miles, you have raised $13.50 total.
y = mx + b
b = 10
13.50 = 5m + 10
13.50 – 10 = 5m
3.50 = 5m
m = 3.50/5
m = 0.7
y = 0.7x + 10

Question 25.
PROBLEM SOLVING
You have 500 sheets of notebook paper. After 1 week, you have 72% of the sheets left. You use the same number of sheets each week. Write an equation that represents the number y of sheets remaining after x weeks.
Answer:
y = mx + b
500 – 0.72 × 500 = 500 – 360 = 140 sheets
m = -140
b = 500
y = -140x + 500

Question 26.
DIG DEEPER!
The palm tree on the left is 10 years old. The palm tree on the right is 8 years old. The trees grow at the same rate.
Big Ideas Math Answers 8th Grade Chapter 4 Graphing and Writing Linear Equations 4.6 21
a. Estimate the height y (in feet) of each tree.
b. Plot the two points (x, y), where x is the age of each tree and y is the height of each tree.
c. What is the rate of growth of the trees?
d. Write an equation that represents the height of a palm tree in terms of its age.
Answer:
a. estimate
left: 18
right: 12
plot y = 1.8x

Lesson 4.7 Writing Equations in Point-Slope Form

EXPLORATION 1

Deriving an Equation
Work with a partner. Let (x1, y1) represent a specific point on a line. Let (x, y) represent any other point on the line.
Big Ideas Math Answers Grade 8 Chapter 4 Graphing and Writing Linear Equations 4.7 1
a. Write an equation that represents the slope m of the line. Explain your reasoning.
b. Multiply each side of your equation in part(a) by the expression in the denominator. What does the resulting equation represent? Explain your reasoning.
Answer:

EXPLORATION 2

Writing an Equation
Work with a partner.
For 4 months, you saved $25 a month. You now have $175 in your savings account.
Big Ideas Math Answers Grade 8 Chapter 4 Graphing and Writing Linear Equations 4.7 2
a. Draw a graph that shows the balance in your account after t months.
b.Use your result from Exploration 1 to write an equation that represents the balance A after t months.
Answer:

Big Ideas Math Answers Grade 8 Chapter 4 Graphing and Writing Linear Equations 4.7 3

Try It
Write an equation in point -slope form of the line that passes through the given point and has the given slope.
Question 1.
(1, 2); m = -4
Answer:
y – y1 = m(x – x1)
y – 2 = -4(x – (1))
y – 2 = -4(x – 1)

Question 2.
(7, 0); m = 1
Answer:
y – y1 = m(x – x1)
y – 0 = 1(x – (7))
y – 0 = 1(x – 7)

Question 3.
(-8, -5); m = –\(\frac{3}{4}\)
Answer:
y – y1 = m(x – x1)
y – (-5) = –\(\frac{3}{4}\)(x – (-8))
y + 5 = –\(\frac{3}{4}\)(x + 8)

Write an equation in slope-intercept form of the line that passes through the given points.
Question 4.
(-2, 1), (3, -4)
Answer:
Slope(m) = (-4 – 1)/(3 – (-2))
= -5/5
m = -1
y – y1 = m(x – x1)
y – 1 = -1(x – (-2))
y – 1 = -1(x + 2)
y – 1 = -x – 2
y = -x – 1

Question 5.
Big Ideas Math Answers Grade 8 Chapter 4 Graphing and Writing Linear Equations 4.7 4
Answer:
Slope(m) = (3 – 5)/(-3 – (-5))
= -2/2
m = -1
y – y1 = m(x – x1)
y – 1 = -1(x – (-1))
y – 1 = -1(x + 1)
y – 1 = -x – 1
y = -x – 1 + 1
y = -x

Self-Assessment for Concepts & Skills
Solve each exercise. Then rate your understanding of the success criteria in your journal.

WRITING AN EQUATION Write an equation in point-slope form of the line that passes through the given point and has the given slope.
Question 6.
(2, 0); m = 1
Answer:
y – y1 = m(x – x1)
y – 0 = 1(x – (2))
y – 0 = 1(x – 2)

Question 7.
(-3, -1); m = –\(\frac{1}{3}\)
Answer:
y – y1 = m(x – x1)
y – (-1) = –\(\frac{1}{3}\)(x – (-3))
y + 1 = –\(\frac{1}{3}\)(x + 3)

Question 8.
(5, 4); m = 3
Answer:
y – y1 = m(x – x1)
y – 4 = 3(x – (5))
y – 4 = 3(x – 5)

Question 9.
WRITING AN EQUATION
Write an equation of the line that passes through the points given in the table.
Big Ideas Math Answers Grade 8 Chapter 4 Graphing and Writing Linear Equations 4.7 5
Answer:
Slope(m) = (-2 – 1)/(5 – 3)
= -3/2
m = -1
y – y1 = m(x – x1)
y – (-5) = -3/2(x – 7)
y + 5 = -3/2(x – 7)
y + 5 = -3/2 x + 21/2
y = -3/2 x + 11/2

Question 10.
DIFFERENT WORDS, SAME QUESTION
Which is different? Sketch “both” graphs.
Big Ideas Math Answers Grade 8 Chapter 4 Graphing and Writing Linear Equations 4.7 6
Answer:
y – 7 = 4x – 4
y = 4x + -4 + 7
y = 4x + 3
Graph line passes through the points (4, 5) and (5, 9)

Self-Assessment for Problem Solving
Solve each exercise. Then rate your understanding of the success criteria in your journal.

Question 11.
A writer finishes a project that a coworker started at a rate of 3 pages per hour. After 3 hours,25% of the project is complete.
a. The project is 200 pages long. Write and graph an equation for the total number y of pages that have been finished after the writer works for x hours.
b. The writer has a total of 45 hours to finish the project. Will the writer meet the deadline? Explain your reasoning.
Answer:
m = 3
y = 3x + b
b + 9 = 25%(200)
b + 9 = 0.25(200)
b + 9 = 50
b = 50 – 9
b = 41
y = 3x + 41
BIM 8th Grade Solution Key Ch 4 img_106
y = 3x + 41
y = 3(45) + 41 = 176 pages
As 176 < 200, the writer will not meet the deadline.

Question 12.
DIG DEEPER!
You and your friend begin to run along a path at different constant speeds.After 1 minute,your friend is 45 meters ahead of you. After 3 minutes, your friend is 105 meters ahead of you.
a. Write and graph an equation for the distance y (in meters) your friend is ahead of you after x minutes. Justify your answer.

Answer:
y = mx + b
45 = m + b
105 = 3m + b
105 – 45 = (3m + b) – (m + b)
60 = 2m
m = 30
45 = 30 + b
b = 45 – 30
b = 15
y = 30x + 15
BIM 8th Grade Solution Key Ch 4 img_107

b. Did you and your friend start running from the same spot? Explain your reasoning.
Answer:
The distance between you and your friend in the initial moment is b = 15 meters. So you are ahead your friend by 15 meters at the starting point.

Writing Equations in Point-Slope Form Homework & Practice 4.7

Review & Refresh

Write an equation in slope-intercept form of the line that passes through the given points.
Question 1.
Big Ideas Math Answers Grade 8 Chapter 4 Graphing and Writing Linear Equations 4.7 7
Answer:
Slope(m) = (5 – 4)/(0 – (-2))
= 1/2
m = 1/2
Because y = 5 when x = 0, the y – intercept is 5.
y = mx + b
y = 1/2 x + 5

Question 2.
Big Ideas Math Answers Grade 8 Chapter 4 Graphing and Writing Linear Equations 4.7 8
Answer:
Slope(m) = (5 – (-1))/(2 – (-2))
= (5 + 1)/(2 + 2)
m = 6/4
m = 3/2
From the graph, the line crosses the y – axis at (0, 2)
y = mx + b
y = 3/2 x + 2

Solve the equation. Check your solution, if possible.
Question 3.
2x + 3 = 2x
Answer:
Given the equation
2x + 3 = 2x
3 = 2x – 2x
3 ≠ 0

Question 4.
6x – 7 = 1 – 3x
Answer:
Given the equation
6x – 7 = 1 – 3x
6x + 3x = 1 + 7
9x = 8
x = 8/3

Question 5.
0.1x – 1 = 1.2x – 5.4
Answer:
Given the equation
0.1x – 1 = 1.2x – 5.4
0.1x – 1.2x = 1 – 5.4
-1.1x = -4.4
x = 4

Concepts, Skills, &Problem Solving

WRITING AN EQUATION The value of a new car decreases $4000 each year. After 3 years, the car is worth $18,000. (See Exploration 2, p. 179.)
Question 6.
Draw a graph that shows the value of the car after t years.
Answer:
BIM 8th Grade Solution Key Chapter 4 img_111

Question 7.
Write an equation that represents the value V of the car after t years.
Answer:
y = -4000t + b
where b is the original price
18,000 = -4000(3) + b
18,000 + 12,000 = b
b = 30,000
y = -4000t + 30,000

WRITING AN EQUATION Write an equation in point-slope form of the line that passes through the given point and has the given slope.
Question 8.
(3, 0); m = –\(\frac{2}{3}\)
Answer:
y – y1 = m(x – x1)
y – (0) = -2/3(x – 3)
y – 0 = -2/3(x – 3)

Question 9.
(4, 8); m = \(\frac{3}{4}\)
Answer:
y – y1 = m(x – x1)
y – (8) = 3/4(x – 4)
y – 8 = 3/4(x – 4)

Question 10.
(1, -3); m = 4
Answer:
y – y1 = m(x – x1)
y – (-3) = 4(x – 1)
y + 3 = 4(x – 1)

Question 11.
(7, -5); m = –\(\frac{1}{7}\)
Answer:
y – y1 = m(x – x1)
y – (-5) = –\(\frac{1}{7}\)(x – 7)
y + 5 = –\(\frac{1}{7}\)(x – 7)

Question 12.
(3, 3); m = \(\frac{5}{3}\)
Answer:
y – y1 = m(x – x1)
y – (3) = \(\frac{5}{3}\)(x – 3)
y – 3 = \(\frac{5}{3}\)(x – 3)

Question 13.
(-1, -4); m = -2
Answer:
y – y1 = m(x – x1)
y – (-4) = -2(x – (-1))
y + 4 = -2(x + 1)

WRITING AN EQUATION Write an equation in slope-intercept form of the line that passes through the given points.
Question 14.
(-1, -1), (1, 5)
Answer:
Slope(m) = (5 – (-1))/(2 – (-1))
= (5 + 1)/(1 + 1)
m = 6/2
m = 3
y – y1 = m(x – x1)
y – (5) = 3(x – (1))
y – 5 = 3x – 3
y = 3x + 2

Question 15.
(2, 4), (3, 6)
Answer:
Slope(m) = (6 – 4)/(3 – 2)
m = 2/1
m = 2
y – y1 = m(x – x1)
y – (4) = 2(x – (2))
y – 4 = 2x – 4
y = 2x

Question 16.
(-2, 3), (2, 7)
Answer:
Slope(m) = (7 – (3))/(2 – (-2))
= (7 – 3)/(2 + 2)
m = 4/4
m = 1
y – y1 = m(x – x1)
y – (3) = 1(x – (-2))
y – 3 = x + 2
y = x + 5

Question 17.
(4, 1), (8, 2)
Answer:
Slope(m) = (2 – (1))/(8 – (4))
= (2 – 1)/(8 – 4)
m = 1/4
y – y1 = m(x – x1)
y – (1) = 1/4(x – (4))
y – 1 = 1/4 x – 1
y = 1/4 x

Question 18.
(-9, 5), (-3, 3)
Answer:
Slope(m) = (3 – (5))/(-3 – (-9))
= (3 – 5)/(-3 + 9)
m = -2/6
m = -1/3
y – y1 = m(x – x1)
y – (3) = -1/3(x + 3)
y – 3 = -1/3 x – 1
y = -1/3 x + 2

Question 19.
(1, 2), (-2, -1)
Answer:
Slope(m) = (2 – (1))/(8 – (4))
= (-1 – 2)/(-2 – 1)
m = -3/-3
m = 1
y – y1 = m(x – x1)
y – (2) = 1(x – (1))
y – 2 = x – 1
y = x + 1

Question 20.
MODELING REAL LIFE
At 0° C, the volume of a gas is 22 liters. For each degree the temperature T (in degrees Celsius) increases, the volume V (in liters) of the gas increases by \(\frac{2}{25}\). Write an equation that represents the volume of the gas in terms of the temperature.
Answer:
The equation modeling the situation has the form:
V = mT + b
m = 2/25
22 = 2/25(0) + b
b = 22
V = 2/25 T + 22

WRITING AN EQUATION Write an equation of the line that passes through the given points in any form. Explain your choice of form.
Question 21.
Big Ideas Math Answers Grade 8 Chapter 4 Graphing and Writing Linear Equations 4.7 9
Answer:
m = (y2 – y1)/(x2 – x1)
= (2.5 – 1.5)/(0 – (-1))
= 1/1
= 1
Because the line crosses the y – axis at (0, 2.5), the y – intercept is 2.5
y = mx + b
y = (1)x + 2.5
y = x + 2.5

Question 22.
Big Ideas Math Answers Grade 8 Chapter 4 Graphing and Writing Linear Equations 4.7 10
Answer:
m = (y2 – y1)/(x2 – x1)
= (3.5 – 1.5)/(2 – (1))
= 2/1
= 2
y – y1 = m(x – x1)
y – (1.5) = 2(x – (1))
y – 1.5 = 2x – 2
y = 2x – 0.5

Question 23.
Big Ideas Math Answers Grade 8 Chapter 4 Graphing and Writing Linear Equations 4.7 11
Answer:
m = (y2 – y1)/(x2 – x1)
= (-1.5 – 4.5)/(1 – (-1))
= -6/2
= -3
y – y1 = m(x – x1)
y – (-1.5) = -3(x – (1))
y + 1.5 = -3x + 3
y = -3x + 1.5

Question 24.
Big Ideas Math Answers Grade 8 Chapter 4 Graphing and Writing Linear Equations 4.7 12
Answer:
m = (y2 – y1)/(x2 – x1)
= (-0.5 – 3.5)/(1 – (-1))
= -4/2
= -2
y – y1 = m(x – x1)
y – (-0.5) = -2(x – (1))
y + 0.5 = -2x – 2
y = -2x – 2.5

Question 25.
Big Ideas Math Answers Grade 8 Chapter 4 Graphing and Writing Linear Equations 4.7 13
Answer:
m = (y2 – y1)/(x2 – x1)
= (1 – (-1))/(0 – (-3))
= (1 + 1)/(0 + 3)
= 2/3
Because y = 1 when x = 0, the y – intercept is 1.
y = mx + b
y = 2/3 x + 1

Question 26.
Big Ideas Math Answers Grade 8 Chapter 4 Graphing and Writing Linear Equations 4.7 14
Answer:
m = (y2 – y1)/(x2 – x1)
= (4 – 6)/(-3 – (-7))
= -2/4
= -1/2
y – y1 = m(x – x1)
y – (2) = -1/2(x – (1))
y – 2 = -1/2x + 1/2
y = -1/2 x + 5/2

Question 27.
REASONING
Write an equation of the line that passes through the point (8, 2) and is parallel to the graph of the equation y = 4x – 3.
Answer:
y = 4x – 3
Comparing the given equation with y = mx + b, we get
m = 4
y – y1 = m(x – x1)
y – 2 = 4(x – 8)
y – 2 = 4x – 32
y = 4x – 32 + 2
y = 4x – 30

Question 28.
MODELING REAL LIFE
The table shows the amount y (in fluid ounces) of carpet cleaner in a tank after x minutes of cleaning.
Big Ideas Math Answers Grade 8 Chapter 4 Graphing and Writing Linear Equations 4.7 15
a. Write an equation that represents the amount of cleaner x in the tank after minutes.
b. How much cleaner is in the tank when the cleaning begins?
c. After how many minutes is the tank empty? Justify your answer.
Answer:

Question 29.
DIG DEEPER!
According to Dolbear’s law, you can predict the temperature T (in degrees Fahrenheit) by counting the number x of chirps made by a snowy tree cricket in 1 minute.When the temperature is 50°F, a cricket chirps 40 times in 1 minute. For each rise in temperature of 0.25°F, the cricket makes an additional chirp each minute.
a. You count 100 chirps in 1 minute. What is the temperature?
b. The temperature is 96°F.How many chirps do you expect the cricket to make? Justify your answer.
Answer:

Question 30.
PROBLEM SOLVING
The Leaning Tower of Pisa in Italy was built between 1173 and 1350.
a. Write an equation that represents the yellow line.
b. The tower is 56 meters tall. How far from the center is the top of the tower? Justify your answer.
Big Ideas Math Answers Grade 8 Chapter 4 Graphing and Writing Linear Equations 4.7 16
Answer:

Graphing and Writing Linear Equations Connecting Concepts

Using the Problem-Solving Plan
Question 1.
Every item in a retail store is on sale for 40% off. Write and graph an equation that represents the sale price of an item that has an original price of x dollars.
Big Ideas Math Answers Grade 8 Chapter 4 Graphing and Writing Linear Equations 4.7 cc 1
Understand the problem.
You know the percent discount of items in a retail store.You are asked to write and graph an equation that represents the sale price of an item that has an original price of x dollars.
Make a plan.
Selling an item for 40% off is the same as selling an item for 60% of its original price. Use this information to write and graph an equation that represents the situation.
Solve and check.
Use the plan to solve the problem. Then check your solution.
Answer:
40% = 0.40 and to find a percent of a number you multiply the number by the percent in decimal form.
So, the equation is d = 0.4p
BIM 8th Grade Answers img_111

Question 2.
Two supplementary angles have angle measures of x° and y°. Write and graph an equation that represents the relationship between the measures of the angles.
Answer:

Question 3.
A mechanic charges a diagnostic fee plus an hourly rate. The table shows the numbers of hours worked and the total costs for three customers.A fourth customer pays $285. Find the number of hours that the mechanic worked for the fourth customer.
Big Ideas Math Answers Grade 8 Chapter 4 Graphing and Writing Linear Equations 4.7 cc 2
Answer:

Performance Task

Anatomy of a Hurricane
At the beginning of this chapter, you watched a STEAM Video called “Hurricane!” You are now ready to complete the performance task related to this video, available at BigIdeasMath.com. Be sure to use the problem-solving plan as you work through the performance task.
Big Ideas Math Answers Grade 8 Chapter 4 Graphing and Writing Linear Equations 4.7 cc 3

Graphing and Writing Linear Equations Chapter Review

Review Vocabulary
Write the definition and give an example of each vocabulary term.
Big Ideas Math Answers Grade 8 Chapter 4 Graphing and Writing Linear Equations cr 1

Graphic Organizers
You can use a Definition and Example Chart to organize information about a concept. Here is an example of a Definition and Example Chart for the vocabulary term linear equation.
Big Ideas Math Answers Grade 8 Chapter 4 Graphing and Writing Linear Equations cr 2

Choose and complete a graphic organizer to help you study the concept.
Big Ideas Math Answers Grade 8 Chapter 4 Graphing and Writing Linear Equations cr 3
1. slope
2. slope of parallel lines
3. proportional relationship
4. slope-intercept form
5. standard form
6. point-slope form

Chapter Self-Assessment

As you complete the exercises, use the scale below to rate your understanding of the success criteria in your journal.
Big Ideas Math Answers Grade 8 Chapter 4 Graphing and Writing Linear Equations 4.7 cr 1

4.1 Graphing Linear Equations (pp. 141–146)
Learning Target: Graph linear equations.Graph the linear equation.

Question 1.
y = \(\frac{3}{5}\)x
Answer:
Bigideas Math Answers 8th Grade chapter 4 img_112

Question 2.
y = -2
Answer:
Bigideas Math Answers 8th Grade chapter 4 img_113

Question 3.
y = 9 – x
Answer:
Bigideas Math Answers 8th Grade chapter 4 img_114

Question 4.
y = -0.25x + 4
Answer:
Bigideas Math Answers 8th Grade chapter 4 img_115

Question 5.
y = \(\frac{2}{3}\)x + 2
Answer:
Bigideas Math Answers 8th Grade chapter 4 img_116

Question 6.
x = -5
Answer:
Bigideas Math Answers 8th Grade chapter 4 img_117

Question 7.
The equation y = 0.53x + 3 represents the cost y (in dollars) of riding in a taxi x miles.
Big Ideas Math Answers Grade 8 Chapter 4 Graphing and Writing Linear Equations 4.7 cr 2
a. Use a graph to estimate how much it costs to ride 5.25 miles in a taxi.
b. Use the equation to find exactly how much it costs to ride 5.25 miles in a taxi.
Answer:
Bigideas Math Answers 8th Grade chapter 4 img_118
y = 0.5x + 3
y = 0.5(5.25) + 3
y ≈ 5.6

Question 8.
The equation y = 9.5x represents the earnings y (in dollars) of an aquarium gift shop employee that works x hours.
Big Ideas Math Answers Grade 8 Chapter 4 Graphing and Writing Linear Equations 4.7 cr 8
a. Graph the linear equation.
b. How much does the employee earn for working 40 hours?
Answer:
Bigideas Math Answers 8th Grade chapter 4 img_119
Determine y for x = 40:
y = 9.5x
y = 9.5(40) = 380

Question 9.
Is y = x2 a linear equation? Explain your reasoning.
Answer:
y = x2
The graph of the given equation passes through the origin, but is not linear, therefore it is not a linear equation.
So, the answer is no.

Question 10.
The sum S of the exterior angle measures of a polygon with n sides is S = 360°.
a. Plot four points (n, S) that satisfy the equation. Is the equation a linear equation? Explain your reasoning.
b. Does the value n = 2 make sense in the context of the problem? Explain your reasoning.
Answer:
8th Grade Big Ideas Math Answer Key Chapter 4 img_120
The value n = 2 does not make sense in the context of the problem because a polygon has at least 3 sides.

4.2 Slope of a Line (pp. 147–154)
Learning Target: Find and interpret the slope of a line.

Describe the slope of the line. Then find the slope of the line.
Question 11.
Big Ideas Math Answers Grade 8 Chapter 4 Graphing and Writing Linear Equations 4.7 cr 11
Answer:
(x1, y1) = (3, 1)
(x2, y2) = (-3, -3)
m = (y2 – y1)/(x2 – x1)
m = (-3 – 1)/(-3 – 3)
m = -4/-6
m = 2/3

Question 12.
Big Ideas Math Answers Grade 8 Chapter 4 Graphing and Writing Linear Equations 4.7 cr 12
Answer:
(x1, y1) = (0, 4)
(x2, y2) = (2, -2)
m = (y2 – y1)/(x2 – x1)
m = (-2 – 4)/(2 – 0)
m = -6/2
m = -3
The slope is negative

Find the slope of the line through the given points.
Question 13.
(-5, 4), (8, 4)
Answer:
(x1, y1) = (-5, 4)
(x2, y2) = (8, 4)
m = (y2 – y1)/(x2 – x1)
m = (4 – 4)/(8 – (-5))
m = 0/13
m = 0

Question 14.
(-3, 5), (-3, 1)
Answer:
(x1, y1) = (-3, 5)
(x2, y2) = (-3, 1)
m = (y2 – y1)/(x2 – x1)
m = (1 – 5)/(-3 + 3)
m = -4/0
m = undefined

The points in the table lie on a line. Find the slope of the line.
Question 15.
Big Ideas Math Answers Grade 8 Chapter 4 Graphing and Writing Linear Equations 4.7 cr 15
Answer:
(x1, y1) = (0, -1)
(x2, y2) = (1, 0)
m = (y2 – y1)/(x2 – x1)
m = (0 – (-1))/(1 – 0)
m = 1/1
m = 1

Question 16.
Big Ideas Math Answers Grade 8 Chapter 4 Graphing and Writing Linear Equations 4.7 cr 16
Answer:
(x1, y1) = (-2, 3)
(x2, y2) = (0, 4)
m = (y2 – y1)/(x2 – x1)
m = (4 – 3)/(0 – (-2))
m = 1/2

Question 17.
How do you know when two lines are parallel? Use an example to justify your answer.
Answer:
Two lines are parallel when their slopes are the same. In order for the two lines not to coincide, we must add the condition that their y – intercepts.
Example 1:
d1: y = 3x – 6
d2: 3x – y = 6
The lines d1 and d2 have the same slope and the same y – intercept, therefore they coincide.

Question 18.
Draw a line through the point (-1, 2) that is parallel to the graph of the line in Exercise 11.
Answer:
y = 2/3 x – 1
A (-1, 2)
y = 2/3 x + b
y = 2/3 (-1) + b
b = 8/3
The equation of d1 is:
y = 2/3 x + 8/3
Determine the x intercept of d1:
0 = 2/3 x + 8/3
0 = 2x + 8
2x = -8
x = -8/2
x = -4
Bigideas math answers grade 8 ch 4 img_121

4.3 Graphing Proportional Relationships (pp. 155–160)
Learning Target: Graph proportional relationships.

Tell whether x and y are in a proportional relationship. Explain your reasoning. If so, write an equation that represents the relationship.
Question 19.
Big Ideas Math Answers Grade 8 Chapter 4 Graphing and Writing Linear Equations 4.7 cr 19
Answer:
x and y are not in a proportional relationship because the line does not pass through the origin.

Question 20.
Big Ideas Math Answers Grade 8 Chapter 4 Graphing and Writing Linear Equations 4.7 cr 20
Answer:
x and y are in a proportional relationship because the line does passes through the origin.
Determine the slope k using two points from the graph
k = (10 – 0)/(2 – 0)
k = 10/2
k = 5x

Question 21.
The cost y (in dollars) to provide food for guests at a dinner party is proportional to the number x of guests attending the party. It costs $30 to provide food for 4 guests.
a. Write an equation that represents the situation.
b. Interpret the slope of the graph of the equation.
c. How much does it cost to provide food for 10 guests? Justify your answer.
Answer:
y = kx
30 = 4k
k = 30/4
k = 7.5
y = 7.5x
b. The slope 7.5 represents the unit cost for a guest.
y = 7.5 × 10
y = 75
c. Determine y for x = 10
So it costs $75 to provide food for 10 guests.

Question 22.
The distance y (in miles) you run after weeks is represented by the equation y =8x. Graph the equation and interpret the slope.
Answer:
y = 8x
Big Ideas Math 8th Grade Answer Key for Chapter 4 img_122

Question 23.
You research that hair grows 15 centimeters per year on average. The table shows your friend’s hair growth.
Big Ideas Math Answers Grade 8 Chapter 4 Graphing and Writing Linear Equations 4.7 cr 23
a. Does your friend’s hair grow faster than average? Explain.

Answer:
The rate of growth on average is
15/12 = 1.25 cm/month
The slope/rate of growth for your friend is
(6 – 3)/(4 – 2) = 3/2 = 1.5 cm/month
As 1.5 > 1.25, your friends hair grows faster than average.

b. In the same coordinate plane, graph the average hair growth and the hair growth of your friend. Compare and interpret the steepness of each of the graphs.
Answer:
The equation for the average growth is
y = 1.25x
The equation for the friends growth is
y = 1.5 x
Big Ideas Math 8th Grade Answer Key for Chapter 4 img_123

4.4 Graphing Linear Equations in Slope-Intercept Form (pp. 161–166)
Learning Target: Graph linear equations in slope-intercept form.

Find the slope and the -intercept of the graph of the linear equation.
Question 24.
y = -4x + 1
Answer:
y = mx + b
slope = -4 and y – intercept = 1

Question 25.
y = \(\frac{2}{3}\)x – 12
Answer:
y = mx + b
slope = \(\frac{2}{3}\) and y – intercept = -12

Question 26.
y – 7 = 0.5x
Answer:
Given the equation
y – 7 = 0.5x
y = 0.5x + 7
slope = 0.5 and y – intercept = 7

Graph the linear equation. Identify the -intercept.
Question 27.
y = 2x – 6
Answer:
Given the equation
y = 2x – 6
Comparing the above equation with slope – intercept equation
slope = 2, y – intercept = -6
Slope = rise/run = 2/1
Plot the point that is 1 unit right and 2 units up from (0, -6) = (1, -4)
BIM Grade 8 Answers Chapter 4 img_124
The line crosses the x – axis at (3, 0)
So, the x – intercept is 3.
BIM Grade 8 Answers Chapter 4 img_125

Question 28.
y = -4x + 8
Answer:
y = -4x + 8
slope = -4 and y – intercept = 8
So plot (0, 8)
Slope = rise/run = -4/1
plot the point that is 1 unit right and 4 units down from (0, 8) = (1, 4)
BIM Grade 8 Answers Chapter 4 img_126
The line crosses the x- axis at (2, 0)
So the x – intercept is 2.
BIM Grade 8 Answers Chapter 4 img_127

Question 29.
y = -x – 8
Answer:
Given the equation
y = -x – 8
comparing the above equation with sloope – intercept equation.
Slope = -1 and y – intercept = -8
Slope = rise/run = -1/1
Plot the point that is 1 unit right and 1 unit down from (0, -8) = (1, -9)
BIM Grade 8 Answers Chapter 4 img_128
The line crosses the x-axis at (-8, 0)
So, the intercept is -8.
BIM Grade 8 Answers Chapter 4 img_129BIM Grade 8 Answers Chapter 4 img_129

Question 30.
The cost y (in dollars) of one person buying admission to a fair and going on x rides is y = x + 12.
Big Ideas Math Answers Grade 8 Chapter 4 Graphing and Writing Linear Equations 4.7 cr 30
a. Graph the equation.
b. Interpret the y-intercept and the slope.
Answer:
y = x + 12
Comparing the above equation with slope – intercept equation.
Slope = 1 and y – intercept = 12
So plot (0, 20)
Slope = rise/run = 1/1
Plot the point that is 1 unit right and 1 unit up from (0, 12) = (1, 13)
BIM Grade 8 Answers Chapter 4 img_130
The y – intercept is 12 so the initial cost of admission is $12.
The slope is 1 so for each ride the cost of the person increases $1 per ride.

Question 31.
Graph the linear equation with slope -5 and y-intercept 0.
Answer:
y – intercept = 0. So plot (0, 0)
Plot the point that is 1 unit right and 5 unit down from (0, 0) = (1, -5)
BIM Grade 8 Answers Chapter 4 img_131

4.5 Graphing Linear Equations in Standard Form (pp. 167–172)
Learning Target: Graph linear equations in standard form.

Write the linear equation in slope-intercept form.
Question 32.
4x + 2y = -12
Answer:
4x + 2y = -12
2y = -12 – 4x
y = -6 – 2x
y = -2x – 6

Question 33.
x – y = \(\frac{1}{4}\)
Answer:
Given the equation
x – y = \(\frac{1}{4}\)
y = x – \(\frac{1}{4}\)

Graph the linear equation.
Question 34.
\(\frac{1}{4}\)x + y = 3
Answer:
\(\frac{1}{4}\)x + y = 3
y = 3 – \(\frac{1}{4}\)x
y = –\(\frac{1}{4}\)x + 3
Slope = –\(\frac{1}{4}\) and y – intercept = 3
So plot (0, 3)
Slope = rise/run = –\(\frac{1}{4}\)
Plot the point that is 4 units right and 1 unit down from (0, 3) = (4, 2)
Big Ideas Math Answers 8th Grade Chapter 4 img_132

Question 35.
-4x + 2y = 8
Answer:
-4x + 2y = 8
2y = 8 + 4x
y = 2x + 4
Slope = 2 and y – intercept = 4
So plot (0, 4)
Slope = rise/run = 2/1
Plot the point that is 1 unit right and 2 units up from (0, 4) = (1, 6)
Big Ideas Math Answers 8th Grade Chapter 4 img_133

Question 36.
x + 5y = 10
Answer:
x + 5y = 10
5y = -x + 10
y = -1/5 x + 2
Slope = -1/5 and y – intercept = 2
So, plot (0, 2)
Slope = rise/run = -1/5
Plot the point that is 5 unit right and 1 unit down from (0, 2) = (5, 1)
Big Ideas Math Answers 8th Grade Chapter 4 img_134

Question 37.
–\(\frac{1}{2}\)x + \(\frac{1}{8}\)y = \(\frac{3}{4}\)
Answer:
–\(\frac{1}{2}\)x + \(\frac{1}{8}\)y = \(\frac{3}{4}\)
\(\frac{1}{8}\)y = \(\frac{3}{4}\)  + \(\frac{1}{2}\)x
y = 4x + 6
Slope = 4 and y – intercept = 6
So plot (0, 6)
Slope = rise/run = 4/1
Plot the point that is 1 unit right and 4 units up from (0, 6) = (1, 10)
Big Ideas Math Answers 8th Grade Chapter 4 img_135

Question 38.
A dog kennel charges $30 per night to board your dog and $6 for each hour of playtime. The amount of money you spend is given by 30x + 6y = 180, where x is the number of nights and y is the number of hours of playtime. Graph the equation and interpret the intercepts.
Answer:
Given,
A dog kennel charges $30 per night to board your dog and $6 for each hour of playtime.
The amount of money you spend is given by 30x + 6y = 180,
where x is the number of nights and y is the number of hours of playtime.
30x + 6y = 180
6y = -30x + 180
y = -5x + 30
BIM Answers Grade 8 Chapter 4 img_136
The x – intercept is 6, which means that the dog can stay for 6 nights when there is no playtime.
The y – intercept is 30, which means the dog can play for 30 hours when he does not spend any night at the kennel.

4.6 Writing Equations in Slope-Intercept Form (pp. 173–178)
Learning Target: Write equations of lines in slope-intercept form.

Write an equation in slope-intercept form of the line that passes through the given points.
Question 39.
Big Ideas Math Answers Grade 8 Chapter 4 Graphing and Writing Linear Equations 4.7 cr 39
Answer:
m = (y2 – y1)/(x2 – x1)
m = (1 – (-2))/(3 – 0)
m = (1 + 2)/(3 – 0)
m = 3/3
m = 1
We have to find the y – intercept because the line crosses the y – axis at (0, -2)
y = mx + b
y = x – 2

Question 40.
Big Ideas Math Answers Grade 8 Chapter 4 Graphing and Writing Linear Equations 4.7 cr 40
Answer:
m = (y2 – y1)/(x2 – x1)
m = (4 – 2)/(0 – 4)
m = 2/-4
m = -1/2
We have to find the y – intercept because the line crosses the y – axis at (0, 4)
y = mx + b
y =-1/2 x + 4

Question 41.
Big Ideas Math Answers Grade 8 Chapter 4 Graphing and Writing Linear Equations 4.7 cr 41
Answer:
m = (y2 – y1)/(x2 – x1)
m = (-2 – 1)/(2 – 0)
m = -3/2
We have to find the y – intercept because y = 1 when x = 0, the y – intercept is 1.
y = mx + b
y = -3/2 x + 1

Question 42.
Big Ideas Math Answers Grade 8 Chapter 4 Graphing and Writing Linear Equations 4.7 cr 42
Answer:
m = (y2 – y1)/(x2 – x1)
m = (-1 – (-3))/(1 – 0)
m = 2/1
m = 2
We have to find the y – intercept because y = -3 when x = 0, the y – intercept is -3.
y = mx + b
y = 2x + (-3)
y = 2x – 3

Question 43.
Write an equation of the line that passes through (0, 8) and (6, 8).
Answer:
m = (y2 – y1)/(x2 – x1)
m = (8 – 8)/(6 – 0)
m = 0/6
m = 0
We have to find the y – intercept because y = 8 when x = 0, the y – intercept is 8.
y = mx + b
y = (0) x + 8
y = 8

Question 44.
Write an equation of the line that passes through (0, -5) and (-5, -5).
Answer:
m = (y2 – y1)/(x2 – x1)
m = (-5 – (-5))/(-5 – 0)
m = 0/-5
m = 0
We have to find the y – intercept because y = -5 when x = 0, the y – intercept is -5
y = mx + b
y = (0) x + (-5)
y = -5

Question 45.
A construction crew is extending a highway sound barrier that is 13 miles long. The crew builds \(\frac{1}{2}\) of a mile per week. Write an equation in slope -intercept form that represents the length y (in miles) of the barrier after x weeks.
Answer:
Given,
A construction crew is extending a highway sound barrier that is 13 miles long.
The crew builds \(\frac{1}{2}\) of a mile per week.
y = mx + b
m = \(\frac{1}{2}\)
b = 13
y = \(\frac{1}{2}\)x + 13

4.7 Writing Equations in Point-Slope Form (pp. 179–184)
Learning Target: Write equations of lines in point-slope form.

Write an equation in point-slope form of the line that passes through the given point and has the given slope.
Question 46.
(4, 4); m = 3
Answer:
y – y1 = m(x – x1)
Substitute m value, x and y value in the equation
y – (4) = 3(x – 4)
y – 4 = 3(x – 4)

Question 47.
(2, -8); m = –\(\frac{2}{3}\)
Answer:
y – y1 = m(x – x1)
Substitute m value, x and y value in the equation
y – (-8) = –\(\frac{2}{3}\)(x – 2)
y + 8 = –\(\frac{2}{3}\)(x – 2)

Write an equation in slope-intercept form of the line that passes through the given points.
Question 48.
(-4, 2), (6, -3)
Answer:
m = (y2 – y1)/(x2 – x1)
m = (-3 – 2)/(6 – (-4))
m = -5/10
m = -1/2
y – y1 = m(x – x1)
Substitute m value, x and y value in the equation
y – 2 = –\(\frac{1}{2}\)(x – (-4))
y – 2 = –\(\frac{1}{2}\)(x + 4)
y = –\(\frac{1}{2}\)x

Question 49.
Big Ideas Math Answers Grade 8 Chapter 4 Graphing and Writing Linear Equations 4.7 cr 49
Answer:
m = (y2 – y1)/(x2 – x1)
m = (5 – 1)/(3 – 2)
m = 4/1
m = 4
y – y1 = m(x – x1)
Substitute m value, x and y value in the equation
y – (-3) = 4(x – 1)
y + 3 = 4x – 4
y = 4x – 7

Question 50.
The table shows your elevation y (in feet) on a ski slope after x minutes.
Big Ideas Math Answers Grade 8 Chapter 4 Graphing and Writing Linear Equations 4.7 cr 50
a. Write an equation that represents your elevation after x minutes.

Answer:
m = (600 – 800)/(2 – 1)
m = -200
800 = -200(1) + b
800 = -200 + b
800 + 200 = b
b = 1000 feet

b. What is your starting elevation?

Answer:
The starting elevation is the y – intercept
b = 1000 feet

c. After how many minutes do you reach the bottom of the ski slope? Justify your answer.
Answer:
0 = -200x + 1000
0 – 1000 = -200x
-1000 = -200x
200x = 1000
x = 5 minutes

Question 51.
A company offers cable television at$29.95 per month plus a one-time installation fee. The total cost for the first six months of service is $214.70. a. Write an equation in point-slope form that represents the total cost you pay for cable television after x months.
b. How much is the installation fee? Justify your answer.
Answer:
y – y1 = m(x – x1)
m = 29.95
y – 214.70 = 29.95(x – 6)
y – 214.70 + 214.70 = 29.95x – 179.97 + 2147.70
y = 29.95x + 35
b = 35

Question 52.
When might it be better to represent an equation in point-slope form rather than slope-intercept form? Use an example to justify your answer.
Answer:
When we are given the slope and a point that is the y – intercept, then the easiest way is to use the slope – intercept form y = mx + b
Example:
m = 2
(0, 5)
y = 2x + 5
m = 2
(1, 3)
y – 3 = 2(x – 1)
Easier when given the slope and a point that is not the y – intercept.

Graphing and Writing Linear Equations Practice Test

Find the slope and the -intercept of the graph of the linear equation.
Question 1.
y = 6x – 5
Answer:
y = 6x – 5
Slope = 6 and y – intercept = -5

Question 2.
y – 1 = 3x + 8.4
Answer:
Given the equation
y – 1 = 3x + 8.4
y = 3x + 8.4 + 1
y = 3x + 9.4
Slope = 3 and y – intercept = 9.4

Question 3.
–\(\frac{1}{2}\)x + 2y = 7
Answer:
Given the equation
–\(\frac{1}{2}\)x + 2y = 7
y = \(\frac{1}{4}\)x + \(\frac{7}{2}\)
Slope = \(\frac{1}{4}\) and y – intercept = \(\frac{7}{2}\)

Graph the linear equation.
Question 4.
y = –\(\frac{1}{2}\)x – 5
Answer:
Given the equation
y = –\(\frac{1}{2}\)x – 5
Slope = –\(\frac{1}{2}\) and y – intercept = -5
So plot (0, -5)
Plot the point that is 2 units right and 1 unit down from (0, -5) = (2, -6)
Draw a line through the two points.
BIM Grade 8 Answers Chapter 4 img_108

Question 5.
-3x + 6y = 12
Answer:
Given the equation
-3x + 6y = 12
6y = 3x + 12
y = \(\frac{1}{2}\)x + 2
Slope = \(\frac{1}{2}\), y – intercept = 2
Slope = rise/run = \(\frac{1}{2}\)
Plot the point that is 2 units right and 1 unit up from (0, 2) = (2, 3)
BIM Grade 8 Answers Chapter 4 img_109

Question 6.
y = \(\frac{2}{3}\)x
Answer:
Given the equation
y = \(\frac{2}{3}\)x
Slope = \(\frac{2}{3}\), y – intercept = 0
Slope = rise/run = \(\frac{2}{3}\)
Plot the point that is 3 units right and 2 unit up from (0, 0) = (3, 2)
Big Ideas Math Answers Grade 8 Ch 4 img_109

Question 7.
Which lines are parallel? Explain.
Big Ideas Math Solutions Grade 8 Chapter 4 Exponents and Scientific Notation pt 7
Answer:
Red line:
(x1, y1) = (-4, 1)
(x2, y2) = (2, 4)
m = (y2 – y1)/(x2 – x1)
m = (4 – 1)/(2 – (-4))
m = 3/6
m = 1/2
Blue line:
(x1, y1) = (-4, -1)
(x2, y2) = (2, 0.5)
m = (y2 – y1)/(x2 – x1)
m = (0.5 – (-1))/(2 – (-4))
m = 1.5/6
m = 1/4
Green Line:
(x1, y1) = (-2, -4)
(x2, y2) = (2, -2)
m = (y2 – y1)/(x2 – x1)
m = (-2 – (-4))/(2 – (-2))
m = 2/4
m = 1/2
Red lines and Green lines are parallel because both have same slope = 1/2

Question 8.
The points in the table lie on a line. Find the slope of the line.
Big Ideas Math Solutions Grade 8 Chapter 4 Exponents and Scientific Notation pt 8
Answer:
(x1, y1) = (-1, -4)
(x2, y2) = (0, -1)
m = (y2 – y1)/(x2 – x1)
m = (-1 – (-4))/(0 – (-1))
m = 3/1
m = 3

Write an equation in slope-intercept form of the line that passes through the given points.
Question 9.
Big Ideas Math Solutions Grade 8 Chapter 4 Exponents and Scientific Notation pt 9
Answer:
(x1, y1) = (-4, 1)
(x2, y2) = (2, 4)
m = (y2 – y1)/(x2 – x1)
m = (-5 – (-1))/(3 – 0)
m = -4/3
Because the line crosses the y – axis at (0, -1), the y – intercept is -1.
y = mx + b
y = -4/3x – 1

Question 10.
Big Ideas Math Solutions Grade 8 Chapter 4 Exponents and Scientific Notation pt 10
Answer:
m = (y2 – y1)/(x2 – x1)
m = (2 – 2)/(0 – (-2))
m = 0/2
m = 0
Because y = 2 when x =0, the y – intercept is 2.
y = mx + b
y = 2

Question 11.
Write an equation in point-slope form of the line that passes through (-4, 1) and (4, 3).
Answer:
m = (y2 – y1)/(x2 – x1)
m = (3 – 1)/(4 – (-4))
m = 2/8
m = 1/4
y – y1 = m(x – x1)
y – 1 = 1/4(x – (-4))
y – 1 = 1/4(x + 4)

Question 12.
The number y of new vocabulary words that you learn after x weeks is represented by the equation y = 15x.
a. Graph the equation and interpret the slope.
b. How many new vocabulary words do you learn after 5 weeks?
c. How many more vocabulary words do you learn after 6 weeks than after 4 weeks?
Answer:
a. 8th Grade Big Ideas Math Answer Key Chapter 4 img_109
b. y = 15 . 5
y = 75 words
c. 15 . 6 – 15 . 4 = 90 – 60 = 30 words

Question 13.
You used $90 worth of paint for a school float. The amount of money you spend is given by 18x + 15y = 90, where x is the number of gallons of blue paint and y is the number of gallons of white paint. Graph the equation and interpret the intercepts.
Answer:
Given,
18x + 15y = 90
15y = -18x + 90
y = -6/5 x + 6
8th Grade Big Ideas Math Answer Key Chapter 4 img_110
The x – intercept is 5 and shows that 5 gallons of blue paint might be bought when no gallon of the white pants is bought.
The y – intercept is 6 and shows that 6 gallons of white paint might be bought when no gallon of blue is bought.

Graphing and Writing Linear Equations Cumulative Practice

Big Ideas Math Solutions Grade 8 Chapter 4 Exponents and Scientific Notation cp 1
Question 1.
Which equation matches the line shown in the graph?
Big Ideas Math Solutions Grade 8 Chapter 4 Exponents and Scientific Notation cp 2
A. y =2x – 2
B. y = 2x + 1
C. y = x – 2
D. y = x + 1
Answer:
m = (-2 – 0)/(0 – 1)
m = 2
y = 2x – 2
Thus the correct answer is option A.

Question 2.
Which point lies on the graph of 6x – 5y = 14?
F. (-4, -1)
G. (-2, 4)
H. (-1, -4)
I. (4, -2)
Answer:
6x – 5y = 14
F. 6(-4) – 5(-1) = 14
-24 + 5 = 14
-19 ≠ 14
G. 6(-2) – 5(4) = 14
-12 – 20 = 14
-32 ≠ 14
H. 6(-1) – 5(-4) = 14
-6 + 20 = 14
14 = 14
Thus the correct answer is option H.

Question 3.
You reflect the triangle in the x-axis. What are the coordinates of the image?
Big Ideas Math Solutions Grade 8 Chapter 4 Exponents and Scientific Notation cp 3
A. X'(4, 1), Y'(2, 3), Z'(-2, 1)
B. X'(4, -1), Y'(2, -3), Z'(-2, -1)
C. X'(-4, -1), Y(-2, -3), Z'(2, -1)
D. X'(1, 4), Y'(3, 2), Z'(1, -2)
Answer:
BIM 8th Grade Solution Key ch 4 img_108
Thus the correct answer is option C.

Question 4.
Which of the following is the equation of a line parallel to the line shown in the graph?
Big Ideas Math Solutions Grade 8 Chapter 4 Exponents and Scientific Notation cp 4
Answer:
m = (-4 – 2)/(6 – 4)
m = -6/2
m = -3
Two lines parallel if they have the same slope.
From the given equations, the one having the slope -3 is y = -3x + 5
Thus the correct answer is option H.

Question 5.
What is the value of x?
Big Ideas Math Solutions Grade 8 Chapter 4 Exponents and Scientific Notation cp 5
Answer:
122 = 47 + x
47 + x = 122
x = 122 – 47
x = 75

Question 6.
An emergency plumber charges $49.00 plus $70.00 per hour of the repair. A bill to repair your sink is $241.50. This can be modeled by 70.00 h + 49.00 = 241.50, where h represents the number of hours for the repair. How many hours did it take to repair your sink?
A. 2.75 hours
B. 3.45 hours
C. 4.15 hours
D. 13,475 hours
Answer:
70.00 h + 49.00 = 241.50
70h = 241.50 – 49
70h = 192.5
h = 2.75 hours
Thus the correct answer is option A.

Question 7.
It costs $40 to rent a car for one day. In addition, the rental agency charges you for each mile driven, as shown in the graph.
Big Ideas Math Solutions Grade 8 Chapter 4 Exponents and Scientific Notation cp 7
Part A Determine the slope of the line joining the points on the graph.
Part B Explain what the slope represents.
Answer:
m = (50 – 40)/(100 – 0)
m = 10/100
m = 0.1

Question 8.
What value of makes the equation true?
Big Ideas Math Solutions Grade 8 Chapter 4 Exponents and Scientific Notation cp 8
7 + 2x = 4x – 5
Answer:
7 + 2x = 4x – 5
2x – 4x = -5 – 7
-2x = -12
x = 6

Question 9.
Trapezoid KLMN is graphed in the coordinate plane shown.
Big Ideas Math Solutions Grade 8 Chapter 4 Exponents and Scientific Notation cp 9
Rotate Trapezoid KLMN 90° clockwise about the origin. What are the coordinates of point M’, the image of point M after the rotation?
F. (-3, -2)
G. (-2, -3)
H. (-2, 3)
I. (3, 2)
Answer: M'(-3, -2)
Thus the correct answer is option F.

Question 10.
Solve the formula K = 3M – 7.
A. M = K + 7
B. M = \(\frac{K+7}{3}\)
C. M = \(\frac{K}{3}\) + 7
D. M = \(\frac{K-7}{3}\)
Answer:
K = 3M – 7
K + 7 = 3M
M = \(\frac{K+7}{3}\)
Thus the correct answer is option B.

Question 11.
What is the distance across the canyon?
Big Ideas Math Solutions Grade 8 Chapter 4 Exponents and Scientific Notation cp 11
F. 3.6 ft
G. 12 ft
H. 40 ft
I. 250 ft
Answer:
100/30 = d/12
3d = 12 × 10
3d = 120
d = 40 feet
Thus the correct answer is option H.

Conclusion:

All the solutions in the above article are beneficial for all the students of middle school students. All the solutions are prepared by the math professionals. The solutions are given clearly with step by step explanations. If you have any doubts regarding the chapter we are always ready to clarify your doubts. All you have to do is to post the comments in the below comment box.

Big Ideas Math Answers Grade 1 Chapter 9 Add Two-Digit Numbers

Big Ideas Math Answers Grade 1 Chapter 9

Get the free pdf link to Download Big Ideas Math Answers Grade 1 Chapter 9 Add Two-Digit Numbers from here. This Big Ideas Math Book 1st Grade Answer Key Chapter 9 Add Two-Digit Numbers was designed by subject experts to assist the primary school students. We have provided the Big Ideas Math Answers Grade 1 Chapter 9 Add Two-Digit Numbers in the pdf format so that you can prepare offline.

Big Ideas Math Book 1st Grade Answer Key Chapter 9 Add Two-Digit Numbers

Start your preparation by taking help from Big Ideas Math Book 1st Grade Solution Key Chapter 9 Add Two-Digit Numbers and learn the concepts from practice tests, cumulative tests, and others. Big Ideas Math Answers Grade 1 Chapter 9 Add Two-Digit Numbers helps both Students and Teachers out there to get all the concepts underlying. Score good marks in your exams with the help of Big Ideas Math Book 1st Grade Answer Key Chapter 9 Add Two-Digit Numbers.

Vocabulary

Lesson: 1 Add Tens and Ones

Lesson: 2 Add Tens and Ones Using a Number Line

Lesson: 3 Make a 10 to Add

Lesson: 4 Add Two-Digit Numbers

Lesson: 5 Practice Addition Strategies

Lesson: 6 Problem Solving: Addition

Chapter-9: Add Two-Digit Numbers

Add Two-Digit Numbers Vocabulary

Organize It

Review Words:
120 chart
column
ones
row
tens

Big Ideas Math Answer Key Grade 1 Chapter 9 Add Two-Digit Numbers 1
Answer:
Tens and ones.

Explanation:
The first row represents the tens and second row represents the ones
In tens row there are 3 lines 3 x 10 = 30.
In ones row there 7 boxes 7 x 1 = 7
so, total there are 37

Define It

Use the review words to complete the puzzle.

Big Ideas Math Answer Key Grade 1 Chapter 9 Add Two-Digit Numbers 2
Answer:
1) 120Chart
2) Column
3)Row

Explanation:
First figure shows the numbers from 1 to 120.
The vocabulary for the figure is 120 chart which is represented in the figure horizontally.
Second figure shows columns colored
The vocabulary for second figure is column
Third figure shows rows colored
The vocabulary for the third figure is row

Lesson 9.1 Add Tens and Ones

Explore and Grow

Show how you can use a model to solve.
Big Ideas Math Answer Key Grade 1 Chapter 9 Add Two-Digit Numbers 3
32 + 7 = _________

Big Ideas Math Answer Key Grade 1 Chapter 9 Add Two-Digit Numbers 4
Answer:
32 + 7= 39 Three tens and 9 ones.
Explanation:
The first row represents the tens and second row represents the ones
In tens row there are 3 lines 3 x 10 = 30.
In ones row there 9 boxes 9 x 1 = 9
so, total there are 39

Show and Grow

Question 1.
25 + 12 = _________
Big Ideas Math Answer Key Grade 1 Chapter 9 Add Two-Digit Numbers 5
Answer:
25 + 12 = 37 Three tens and 7 ones
Explanation:
The first row represents the tens and second row represents the ones
In tens row there are 2 lines 2 x 10 = 20.
In ones row there 5 boxes 5 x 1 = 5
so, total there are 25.
The first row represents the tens and second row represents the ones
In tens row there are 1 lines 1 x 10 = 10.
In ones row there 2 boxes 2 x 1 = 2
so, total there are 12
By adding both the numbers the answer is 37.

Question 2.
36 + 3 = __________
Big Ideas Math Answer Key Grade 1 Chapter 9 Add Two-Digit Numbers 6
Answer:
36 +3= 39, three tens and 9 ones
Explanation:
The first row represents the tens and second row represents the ones
In tens row there are 3 lines 3 x 10 = 30.
In ones row there 6 boxes 6 x 1 = 6
so, total there are 36.
The first row represents the tens and second row represents the ones
In tens row there are 0 lines 0 x 10 = 0.
In ones row there 3 boxes 3 x 1 = 3
so, total there are 3
By adding both the numbers the answer is 39.

Question 3.
21 + 8 = __________
Big Ideas Math Answer Key Grade 1 Chapter 9 Add Two-Digit Numbers 7
Answer:
21+ 8 = 29, 2 tens and 9 ones.
Explanation:
The first row represents the tens and second row represents the ones
In tens row there are 2 lines 2 x 10 = 20.
In ones row there 1 boxes 1 x 1 = 1
so, total there are 1.
The first row represents the tens and second row represents the ones
In tens row there are 0 lines 0 x 10 = 0.
In ones row there 8 boxes 8 x 1 = 8
so, total there are 8
By adding both the numbers the answer is 29.

Question 4.
22 + 24 = __________
Big Ideas Math Answer Key Grade 1 Chapter 9 Add Two-Digit Numbers 8
Answer:
22 + 24 = 46, 4 tens and 6 ones
Explanation:
The first row represents the tens and second row represents the ones
In tens row there are 2 lines 2 x 10 = 20.
In ones row there 2 boxes 2 x 1 = 2
so, total there are 22.
The first row represents the tens and second row represents the ones
In tens row there are 2 lines 2 x 10 = 20.
In ones row there 4 boxes 4 x 1 = 4
so, total there are 24
By adding both the numbers the answer is 46.

Apply and Grow: Practice

Question 5.
34 + 4 = __________
Big Ideas Math Answer Key Grade 1 Chapter 9 Add Two-Digit Numbers 9
Answer:
34 + 4 = 38, 3 tens and 8 ones.
Explanation:
The first row represents the tens and second row represents the ones
In tens row there are 3 lines 3 x 10 = 30.
In ones row there 4 boxes 4 x 1 = 4
so, total there are 34.
The first row represents the tens and second row represents the ones
In tens row there are 0 lines 0 x 10 = 0.
In ones row there 4 boxes 4 x 1 = 4
so, total there are 4
By adding both the numbers the answer is 38.

Question 6.
43 + 15 = __________
Big Ideas Math Answer Key Grade 1 Chapter 9 Add Two-Digit Numbers 10

Answer:
43 + 15 = 58, 5 tens and 8 ones.

Explanation:
The first row represents the tens and second row represents the ones
In tens row there are 4 lines 4 x 10 = 40.
In ones row there 3 boxes 3 x 1 = 3
so, total there are 43.
The first row represents the tens and second row represents the ones
In tens row there are 1 lines 1 x 10 = 10.
In ones row there 5 boxes 5 x 1 = 5.
so, total there are 15
By adding both the numbers the answer is 58.

Question 7.
71 + 20 = __________
Answer:
71 + 20 = 91,
9 tens and 1 ones.

Explanation:
The number of choclates with rim is 71
The number of choclates with jim is 20
Total number of choclates is 91.

Question 8.
93 + 6 = __________
Answer:
93 + 6 = 99,
9 tens and 9 ones.

Explanation:
The number of choclates with rim is 93
The number of choclates with jim is 6
Total number of choclates is 99

Question 9.
55 + 23 = __________
Answer:
55 + 23 = 78, 7 tens and 8 ones.

Explanation:
The number of choclates with rim is 55
The number of choclates with jim is 23
Total number of choclates is 78.

Question 10.
62 + 32 = __________
Answer:
62 + 32 = 94, 9 tens and 4 ones.

Explanation:
The number of choclates with rim is 62
The number of choclates with jim is 32
Total number of choclates is 94.

Question 11.
MP Reasoning
Circle the number to complete the equation.
Big Ideas Math Answer Key Grade 1 Chapter 9 Add Two-Digit Numbers 11
Answer:
41 + 5 = 46,
Circled the number 5.

Think and Grow: Modeling Real Life

You watch television for 24 minutes in the morning and 32 minutes at night. How many minutes do you spend watching television in all?
Big Ideas Math Answer Key Grade 1 Chapter 9 Add Two-Digit Numbers 12
Addition equation:

Model:

___________ minutes
Answer: 56

Explanation:
Models: television,
24 minutes in the morning and 32 minutes in the night
24 + 32 = 56.

Show and Grow

Question 12.
You do 42 jumping jacks in the morning and 46 at night. How many jumping jacks do you do in all?
Big Ideas Math Answer Key Grade 1 Chapter 9 Add Two-Digit Numbers 13
Addition equation:

Model:

___________ jumping jacks
Answer: 88

Explanation:
Models : Jumping jacks,
42 jumping jacks in the morning and 46 in the night
42 + 46 = 88. 88 Jumping jacks

Add Tens and Ones Practice 9.1

Question 1.
42 + 7 = _________
Big Ideas Math Answer Key Grade 1 Chapter 9 Add Two-Digit Numbers 14
Answer:
42 + 7 = 49, 4 tens and 9 ones.

Explanation:
The first row represents the tens and second row represents the ones
In tens row there are 4 lines 4 x 10 = 40.
In ones row there 2 boxes 2 x 1 = 2.
so, total there are 42.
The first row represents the tens and second row represents the ones
In tens row there are 0 lines 0 x 10 = 0.
In ones row there 7 boxes 7 x 1 = 7
so, total there are 7
By adding both the numbers the answer is 49.

Question 2.
61 + 35 = __________
Big Ideas Math Answer Key Grade 1 Chapter 9 Add Two-Digit Numbers 15
Answer:
61 + 35 = 96, 9 tens and 6 ones.

Explanation:
The first row represents the tens and second row represents the ones
In tens row there are 6 lines 6 x 10 = 60.
In ones row there 1 boxes 1 x 1 = 1
so, total there are 61.
The first row represents the tens and second row represents the ones
In tens row there are 3 lines 3 x 10 = 30.
In ones row there 5 boxes 5 x 1 = 5
so, total there are 35
By adding both the numbers the answer is 96.

Question 3.
74 + 11 = __________
Answer:
74 + 11 = 85, 8 tens and 5 ones.

Explanation:
The number of choclates with rim is 74
The number of choclates with jim is 11
Total number of choclates is 85.

Question 4.
86 + 2 = ___________
Answer:
86 + 2 = 88, 8 tens and 8 ones.

Explanation:
The number of choclates with rim is 86
The number of choclates with jim is 2
Total number of choclates is 88.

Question 5.
MP Reasoning
Circle the number to complete the equation.
Big Ideas Math Answer Key Grade 1 Chapter 9 Add Two-Digit Numbers 16
Answer:
22 + 70 =  92, circled the 70 to complete the number.

Question 6.

Modeling Real Life
You eat 33 grapes. Your friend eats 23 grapes. How many grapes do you and your friend eat in all?
Big Ideas Math Answer Key Grade 1 Chapter 9 Add Two-Digit Numbers 17

___________ grapes
Answer: 55

Explanation:
Me ate 33 and my friend ate 23 in all
33 + 23 = 55.
We bote ate 55 in all.

Review & Refresh

Question 7.
Big Ideas Math Answer Key Grade 1 Chapter 9 Add Two-Digit Numbers 18
__________ tens and _________ ones is _________ .
Answer:
8 tens and 2 ones is 82

Explanation:
The first row represents the tens and second row represents the ones
In tens row there are 8 lines 8 x 10 = 80.
In ones row there 2 boxes 2 x 1 = 2
so, total there are 82.

Question 8.
Big Ideas Math Answer Key Grade 1 Chapter 9 Add Two-Digit Numbers 19
__________ tens and _________ ones is _________ .
Answer:
4 tens and 6 ones is 46

Explanation:
The first row represents the tens and second row represents the ones
In tens row there are 4 lines 4 x 10 = 40.
In ones row there 6 boxes 6 x 1 = 6
so, total there are 46.

Lesson 9.2 Add Tens and Ones Using a Number Line

Explore and Grow

Color to show how you can use the hundred chart to find the sum.

23 + 34 = ___________

Big Ideas Math Answers 1st Grade 1 Chapter 9 Add Two-Digit Numbers 20
Answer:
23 + 34 = 57,
5 tens and 7 ones.

Explanation:
In the below figure 120 chart is shown in that 23 and 34 are colored
that two numbers are added
to get the total sum.

Show and Grow

Question 1.
22 + 7 = __________
Big Ideas Math Answers 1st Grade 1 Chapter 9 Add Two-Digit Numbers 21
Answer: 29

Explanation:
22 + 7 = 29,
the addition is represented on number line

Question 2.
35 + 41 = __________
Big Ideas Math Answers 1st Grade 1 Chapter 9 Add Two-Digit Numbers 22
Answer:
35 + 41 = 76.

Explanation:
35 + 41 is represented on the number line.

Apply and Grow: practice

Question 3.
53 + 40 = _________
Big Ideas Math Answers 1st Grade 1 Chapter 9 Add Two-Digit Numbers 23
Answer:
53 + 40 = 93.

Question 4.
82 + 12 = __________
Big Ideas Math Answers 1st Grade 1 Chapter 9 Add Two-Digit Numbers 24
Answer: 82 + 12 = 94.

Question 5.
48 + 31 = ___________
Big Ideas Math Answers 1st Grade 1 Chapter 9 Add Two-Digit Numbers 24
Answer: 48 + 31 = 79.
From the above equation 48 is marked and from that point
3 points with 10 are taken and 1 with one is taken
48 + 31 = 79

Question 6.
MP Structure
Write an equation that matches the number line.
Big Ideas Math Answers 1st Grade 1 Chapter 9 Add Two-Digit Numbers 25
Answer: 37 + 23 = 60

Explanation:
From the point 37 to 60
2 points with 10 are taken and 3 points with 1 are taken
so, the total equation is 37 + 23 = 60.

Think and Grow: Modeling Real Life

The home team scores 37 points. The visiting team scores 22 more. How many points does the visiting team score?
Big Ideas Math Answers 1st Grade 1 Chapter 9 Add Two-Digit Numbers 26
Addition equation:

Model:
Big Ideas Math Answers 1st Grade 1 Chapter 9 Add Two-Digit Numbers 27

__________ points
Answer: 37 + 22 = 59

Explanation:
A point on the number line is taken as 37
From there 2 poins with 10 that is 20
From there 2 more points with 2 are taken
total it makes 22

Show and Grow

Question 7.
Your friend scores 63 points. You score 25 more than your friend. How many points do you score?
Big Ideas Math Answers 1st Grade 1 Chapter 9 Add Two-Digit Numbers 28
Addition equation:

Model:
Big Ideas Math Answers 1st Grade 1 Chapter 9 Add Two-Digit Numbers 27

__________ points
Answer: 63 + 25 = 88
Explanation:
A point on the number line is taken as 63
From there 2 poins with 10 that is 20
From there 5 more points with 5 are taken
total it makes 88.

Add Tens and Ones Using a Number Line Practice 9.2

Question 1.
13 + 60 = __________
Big Ideas Math Answers 1st Grade 1 Chapter 9 Add Two-Digit Numbers 29
Answer: 73

Question 2.
81 + 8 = _____________
Big Ideas Math Answers 1st Grade 1 Chapter 9 Add Two-Digit Numbers 30
Answer: 89

Question 3.
56 + 42 = ____________
Big Ideas Math Answers 1st Grade 1 Chapter 9 Add Two-Digit Numbers 30
Answer: 98

Question 4.
MP Structure
Write an equation that matches the number line.
Big Ideas Math Answers 1st Grade 1 Chapter 9 Add Two-Digit Numbers 31
Answer: 17 + 41= 58

Question 5.
Modeling Real Life
There are 36 black keys on a piano. There are 16 more white keys than black keys. How many white keys are there?
Big Ideas Math Answers 1st Grade 1 Chapter 9 Add Two-Digit Numbers 32
Big Ideas Math Answers 1st Grade 1 Chapter 9 Add Two-Digit Numbers 33

____________ white keys
Answer: 52 white keys

Review & Refresh

Question 6.
3 + 7 + 4 = ___________
Answer: 14

Question 7.
4 + 5 + 6 = ___________
Answer: 15

Lesson 9.3 Make a 10 to Add

Explore and Grow

How can you use the model to solve?

38 + 6 = ___________

Big Ideas Math Answers Grade 1 Chapter 9 Add Two-Digit Numbers 34
Answer:
38 + 6 = 44

Explanation:
The first row represents the tens and second row represents the ones
In tens row there are 3 lines 3 x 10 = 30.
In ones row there 8 boxes 8 x 1 = 8
so, total there are 38.
The first row represents the tens and second row represents the ones
In tens row there are 0 lines 0 x 10 = 0.
In ones row there 6 boxes 6 x 1 = 6
so, total there are 6
By adding both the numbers the answer is 44.
Count one numbers from both the sides 8 + 6 = 14
It has 10 in it

Show and Grow

Question 1.
41 + 7 = ___________
Big Ideas Math Answers Grade 1 Chapter 9 Add Two-Digit Numbers 35
Make a 10?           Yes         No
Answer: No

Explanation:
By adding ones place from both the sides
1 + 7 = 8
So, the answer is no

Question 2.
56 + 8 = ___________
Big Ideas Math Answers Grade 1 Chapter 9 Add Two-Digit Numbers 36
Make a 10?           Yes         No
Answer: Yes

Explanation:
By adding ones place from both the sides
6 + 8 = 14
So, there is 10 in it.

Apply and Grow: Practice

Question 3.
72 + 4 = _________
Big Ideas Math Answers Grade 1 Chapter 9 Add Two-Digit Numbers 37
Make a 10?           Yes         No
Answer: No

Explanation:
By adding ones place from both the sides
2 + 4 = 6
So, There is no 10 in it.

Question 4.
63 + 9 = ___________
Big Ideas Math Answers Grade 1 Chapter 9 Add Two-Digit Numbers 38
Make a 10?           Yes         No
Answer: Yes

Explanation:
By adding ones place from both the sides
3  + 9 = 11
So, there is 10 in it.

Question 5.
14 + 6 = __________
Make a 10?           Yes         No
Answer: Yes

Explanation:
By adding ones place from both the sides
4 + 6 = 10
so, there is 10 in it.

Question 6.
27 + 5 = __________
Make a 10?           Yes         No
Answer: Yes

Explanation:
By adding ones place from both the sides
7 + 5 = 12
So, there is 10 in it.

Question 7.
46 + 7 = ___________
Make a 10?           Yes         No
Answer: Yes

Explanation:
By adding ones place from both the sides
6 + 7 = 13
So, there is 10 in it

Question 8.
81 + 8 = ___________
Make a 10?           Yes         No
Answer: No

Explanation:
By adding ones place from both the sides
1 + 8 = 9
so, there is 10 in it.

MP Logic
Complete.
Question 9.
Big Ideas Math Answers Grade 1 Chapter 9 Add Two-Digit Numbers 39

Answer: 62

Explanation:
56 + 6 = 62
56 + 4 makes 60 and 4 + 2 = 6
6 is divided as 4 + 2
60 + 2

Question 10.
Big Ideas Math Answers Grade 1 Chapter 9 Add Two-Digit Numbers 40
Answer: 48


Explanation:
39 + 9 = 48
39 + 1 = 40, 9 is divided to 1 and 8.

Think and Grow: Modeling Real Life

You put 17 puzzle pieces together. There are 7 left. How many puzzle pieces are there in all?
Big Ideas Math Answers Grade 1 Chapter 9 Add Two-Digit Numbers 41
Addition equation:

Model:

Make a 10?           Yes         No

___________ Puzzle pieces
Answer: Yes.

Explanation:
17 puzzle pieces are together
7 added to it
17 + 7 = 24
The ones place of both sides are added 7 + 7 = 14
so, there is ten in it.

Show and Grow

Question 11.
You color 46 states. There are 4 left. How many states are there in all?
Big Ideas Math Answers Grade 1 Chapter 9 Add Two-Digit Numbers 42
Addition equation:

Model:

Make a 10?           Yes         No

___________ States
Answer: 50

Explanation:
Colored states are 46
uncolored states are 4
Total states are 46 + 4 = 50
6 + 4 = 10
Both the sides ones places are added

Make a 10 to Add Practice 9.3

Question 1.
66 + 5 = __________
Big Ideas Math Answers Grade 1 Chapter 9 Add Two-Digit Numbers 43
Make a 10?           Yes         No
Answer: yes

Explanation:
Number of lines which represents 10 is 6 that in tens place
in ones place both the sides 6 + 5 = 11
so, there is 10 in it

Question 2.
74 + 3 = ____________
Big Ideas Math Answers Grade 1 Chapter 9 Add Two-Digit Numbers 44
Make a 10?           Yes         No
Answer: No

Explanation:
Number of lines which represents 10 is 7 that in tens place
in ones place both the sides 4 + 3 = 7.
so, there is no 10 in it.

Question 3.
28 + 8 _____________
Make a 10?           Yes         No
Answer: Yes

Explanation:
2 represents tens place 2 x 10 = 20
In ones place from both the sides 8 + 8 = 16
so, There is ten in it.

Question 4.
52 + 9 = ______________
Make a 10?           Yes         No
Answer: Yes

Explanation:
5 represents tens place 5 x 10 = 50
In ones place from both the sides 2 + 9 = 11
so, There is ten in it.
52 + 9 = 61.

Question 5.
26 + 7 = ____________
Make a 10?           Yes         No
Answer: yes

Explanation:
2 represents tens place 2 x 10 = 20
In ones place from both the sides 6 + 7 = 13
so, There is ten in it.
26 + 7 = 33.

Question 6.
41 + 6 = ____________
Make a 10?           Yes         No
Answer: No

Explanation:
4 represents tens place 4 x 10 = 40
In ones place from both the sides 1 + 6 = 7
so, There is no ten in it.
41 + 6 = 47.

MP Logic
Complete.
Question 7.
Big Ideas Math Answers Grade 1 Chapter 9 Add Two-Digit Numbers 45
Answer: 41


Explanation:
37 + 4 = 41
4 is divided to 3 and 1
37 + 3 = 40
40 plus 1 is 41

Question 8.
Big Ideas Math Answers Grade 1 Chapter 9 Add Two-Digit Numbers 46
Answer: 54

Explanation:
48 + 6 = 54
6 is divided in to 2 and 4
48 + 2 = 50 So, there is ten in it
50 + 4 = 54

Question 9.
Modeling Real Life
A snake lays 24 eggs. Another snake lays 9 eggs. How many eggs are there in all?
Big Ideas Math Answers Grade 1 Chapter 9 Add Two-Digit Numbers 47
Answer:
33 eggs

Explanation:
A snake lays 24 eggs
another snake lays 9
there are 33 eggs in all.

Review & Refresh

Question 10.
Color the shapes that have 4 vertices.
Big Ideas Math Answers Grade 1 Chapter 9 Add Two-Digit Numbers 48
Answer:  Square and rectangle

Explanation:
For a square there are 4 vertices and for a rectangle there are 4 vertices. vertices are Nothing but corners or joining of the two lines.

Lesson 9.4 Add Two-Digit Numbers

Explore and Grow

Show how you can use a model to solve.
Big Ideas Math Answer Key Grade 1 Chapter 9 Add Two-Digit Numbers 49

43 + 28 = _________
Big Ideas Math Answer Key Grade 1 Chapter 9 Add Two-Digit Numbers 50
Answer: 43 + 28 = 61

Explanation:
4 tens and 3 ones
2 tens and 8 ones
The figure shows lines represents tens and
circls represent ones

Show and Grow

Question 1.
39 + 45 = ?
Big Ideas Math Answer Key Grade 1 Chapter 9 Add Two-Digit Numbers 51
___________ tens __________ ones
Answer: 7 tens and 14 ones

Explanation:
The figure shows 39 represents 3 tens and 9 ones
and 4 tens 5 ones
3 + 4 = 7 and 9 + 5 = 14.

Apply and Grow: Practice

Question 2.
19 + 35 = ?
Big Ideas Math Answer Key Grade 1 Chapter 9 Add Two-Digit Numbers 52
Answer: 1 + 3 = 4 and 9 + 5 = 14

Explanation:
First figure shows 1 tens and 9 ones
3 tens and 5 ones
The second figure shows 1 + 3 = 4
and 9 + 5 = 14

Question 3.
Big Ideas Math Answer Key Grade 1 Chapter 9 Add Two-Digit Numbers 53
Answer: 60
Explanation:
43 + 17 = 60
In the 4 and 3 are in tens place and 3 and 7 are in ones place
My friend has 43 choclates and me has 17 more of it.
Total choclates is 60.

Question 4.
Big Ideas Math Answer Key Grade 1 Chapter 9 Add Two-Digit Numbers 54
Answer: 81

Explanation:
67 + 41 = 81
my friend jim has 67 lollipops
and rim has 41 lollipops
in total jim and rim has 81 lollipops.

Question 5.
YOU BE THE TEACHER
Is the sum correct? Explain.
Big Ideas Math Answer Key Grade 1 Chapter 9 Add Two-Digit Numbers 55
Answer: 76
Explanation:

58 + 28 = 76
The line represents 10 and circles represents 1
5 tens and 8 ones
2 tens and 8 ones
by adding 8 from the both places we get 16 and there is 10 in it.

Think and Grow: Modeling Real Life

You earn a sticker for every 10 pages you read. You read 34 pages one week and 37 the next. How many stickers do you earn?
Addition problem:
Big Ideas Math Answer Key Grade 1 Chapter 9 Add Two-Digit Numbers 56
Model:
Big Ideas Math Answer Key Grade 1 Chapter 9 Add Two-Digit Numbers 57
Write the missing numbers:
_________ tens _________ one
__________ stickers
Answer: 71

Explanation:
I earn a sticker for every 10 pages
first I read 34 pages and then 37 pages
34 + 37 = 71
so, total 71 pages for every 10 pages 1 sticker 7 stickers

Show and Grow

Question 6.
You earn a coin for every lo cans you recycle. You recycle 18 cans one week and 25 the next. How many coins do you earn?
Big Ideas Math Answer Key Grade 1 Chapter 9 Add Two-Digit Numbers 58
Addition problem:
Big Ideas Math Answer Key Grade 1 Chapter 9 Add Two-Digit Numbers 59
Model:
Big Ideas Math Answer Key Grade 1 Chapter 9 Add Two-Digit Numbers 60
Write the missing numbers:
_________ tens _________ one
__________ coins
Answer:  4 tens and 3 ones 4 coins
Explanation:
If I recycle 10 cans I get 1 coin
in first week i recycled 18 cans and in next week 25 cans
18 + 25 = 43.

Add Two-Digit Numbers Practice 9.4

Question 1.
57 + 15 = ?
Big Ideas Math Answer Key Grade 1 Chapter 9 Add Two-Digit Numbers 61
Answer: 57 + 15 = 72

Explanation:
In 57 5 tens and 7 ones
in 15 1 tens and 5 ones
If we add both we get 7 tens and 2 ones.

Question 2.
Big Ideas Math Answer Key Grade 1 Chapter 9 Add Two-Digit Numbers 62
Answer: 76

Explanation:
In 40 4 tens and 0 ones
in 36 3 tens and 6 ones
if we add both we get 7 tens and 6 ones.

Question 3.
Big Ideas Math Answer Key Grade 1 Chapter 9 Add Two-Digit Numbers 63
Answer: 81
Explanation:
In 29 2 tens and 9 ones
in 52 5 tens and  2 ones
If we add both 29 and 52 is 81
8 tens and 1 ones.

Question 4.
DIG DEEPER!
Do you need to use the make a 10 strategy to find each sum?
28 + 34 = ?      Yes      No
56 + 15 = ?      Yes      No
42 + 21 = ?      Yes      No
68 + 11 = ?      Yes      No
Answer:
28 + 34 = 62      Yes
56 + 15 = 71    Yes
42 + 21 = 63      No
68 + 11 = 79     No
Explanation:
In first sum 28 + 34
if we take 8 and 4 and add we get 12 taken from ones place
we get 10 in it.
In second sum 56 + 15
if we take 6 and 5 and add we get 11 taken from ones place
we get 10 in it.
In third sum 42 + 21
if we take 2 and 1 and add we get 3 taken from ones place
we didn’t get 10 in it
In fourth sum 68 + 11
if we take 8 and 1 and add we get 9 taken from ones place
we didn’t get 10 in it.

Question 5.
You need a box for every 10 muffins you make. You make 33 blueberry muffins and 47 banana muffins. How many boxes do you need?
Big Ideas Math Answer Key Grade 1 Chapter 9 Add Two-Digit Numbers 64

__________ boxes
Answer: 8 boxes
Explanation:
We need a box for 10 muffins that we make
We made 33 blueberry muffins
and 47 banana muffins
if we add we get 33 + 47 = 80
so, I get 8 boxes.

Review & Refresh

Question 6.
Is the equation true or false?
Big Ideas Math Answer Key Grade 1 Chapter 9 Add Two-Digit Numbers 65
Answer: 15 = 16 false

Explanation:
6+ 9 = 15 and 17 – 1= 16
15 is not equal to 16

Lesson 9.5 Practice Addition Strategies

Explore and Grow

Show two ways you can find the sum.

23 + 39 = __________

23 + 39 = __________

Answer:  23 + 39 = 62
Explanation: one method

second method

Show and Grow

Question 1.
47 + 24 = ________
Answer: 72

Explanation:
4 tens and 7 ones
2 tens and 4 ones
tens represents the lines and ones represents the circles

Question 2.
38 + 43 = ________
Answer: 81

Explanation:
3 tens and 8 ones
4 tens and 3 ones
tens represents the lines and ones represents circles

Apply and Grow: Practice

Question 3.
22 + 18 = ________
Answer: 40

Explanation:
2 tens and 2 ones
1 tens and 8 ones
tens represents the lines and ones represents the circles

Question 4.
57 + 34 = __________
Answer: 91

Explanation:
5 tens and 7 ones
3 tens and 4 ones
lines represents the tens and circles represents the ones

Question 5.
73 + 19 = __________
Answer: 92

Explanation:
lines represents the tens and circles represents the ones
7 ten and 3 ones
1 tens and 9 ones

Question 6.
81 + 11 = __________
Answer: 92

Explanation:
the lines represents tens and the circles represents ones
8 tens and 1 ones
1 tens and 1 ones
If we add both we get
92

Question 7.
YOU BE THE TEACHER
Is the sum correct? Explain.
Big Ideas Math Answers 1st Grade 1 Chapter 9 Add Two-Digit Numbers 66
Answer: No

Explanation:
no the sum is not correct.
But figure shown is correct
17 + 26 = 43

Think and Grow: Modeling Real Life

You hove 48 songs. Your friend has 27 more than you. How many songs does your friend have?
Big Ideas Math Answers 1st Grade 1 Chapter 9 Add Two-Digit Numbers 67
Addition equation:

Model:

____________ songs
Answer: 75 songs

Explanation:
Model : songs
I have 48 songs
and my friend had 27 more than me
Total songs that my friend had is 48 + 27 = 75 songs

Show and Grow

Question 8.
Your friend sells 56 candles. You sell 35 more than your friend. How many candles do you sell?
Big Ideas Math Answers 1st Grade 1 Chapter 9 Add Two-Digit Numbers 68
Addition equation:

Model:

____________ candles
Answer: 91 candles

Explanation:
Models: candles
My friend has 56 candles
and me has 35 more than my friend
so, I had 91 candles with me

Practice Addition Strategies Practice 9.5

Question 1.
62 + 29 = _______
Answer: 99

Explanation:
6 tens and 2 ones
2 tens and 9 ones
by adding both the numbers i get is 99

Question 2.
84 + 8 = ________
Answer: 92

Explanation:
8 tens and 4 ones
0 tens and 8 ones
by adding both the numbers i get is 92

Question 3.
75 + 17 = ________
Answer: 92

Explanation:
7 tens and 5 ones
1 tens and 7 ones
by adding both the numbers i get is 92

Question 4.
YOU BE THE TEACHER
Is the sum correct? Explain.
Big Ideas Math Answers 1st Grade 1 Chapter 9 Add Two-Digit Numbers 69
Answer:

Question 5.
Modeling Real Life You collect 12 leaves. Your friend collects 26 more than you. How many leaves does your friend collect?
Big Ideas Math Answers 1st Grade 1 Chapter 9 Add Two-Digit Numbers 70

____________ leaves
Answer: 38 leaves

Explanation:
I collect 12 leaves and my friend collect 26 more than me
total leaves my friend had is
12 + 26 = 38 leaves.

Review & Refresh

Question 6.
Circle the measurable attributes of the table.
Big Ideas Math Answers 1st Grade 1 Chapter 9 Add Two-Digit Numbers 71
Answer: length or height.

Explanation:
For table length or height is the measuring attribute
cubes represents to measure the length and height.

Lesson 9.6 Problem Solving: Addition

Explore and Grow

Model the story.

Newton has 15 dog bones. Descartes gives him 8 more. How many dog bones does Newton have now?

Big Ideas Math Answers Grade 1 Chapter 9 Add Two-Digit Numbers 72
Answer: 23 bones

Explanation:
Newton has 15 bones and Descartes gave him 8 more
The total bones newton had is 23 bones.

Show and Grow

Question 1.
You have 49 toy soldiers. You buy some more. Now you have 84. How many toy soldiers did you buy?
Big Ideas Math Answers Grade 1 Chapter 9 Add Two-Digit Numbers 73
Circle what you know:

Underline what you need to find.

Solve:

_____________ toy soldiers
Answer: 35 toy soldiers

Explanation:
I have 49 toy soldiers
If i buy 35 more
Then i will have 84 in total.

Apply and Grow: Practice

Question 2.
You have 55 pounds of dog food and some cat food. You have 63 pounds of pet food in all. How many pounds of cat food do you have?
Big Ideas Math Answers Grade 1 Chapter 9 Add Two-Digit Numbers 74
Circle what you know.

Underline what you need to find.

Solve:

___________ pounds
Answer:  8 pounds

Explanation:
I have 55 pound of dog food
If i had 8 pounds of cat food
In total the pet food had is 63 pounds

Question 3.
A teacher has 34 erasers. There are 46 fewer erasers than pencils. How many pencils are there?

____________ pencils
Answer:

Question 4.
DIG DEEPER!
You have 25 toys. Your friend has more than you. There are more than 60 toys in all. How many toys can your friend have?
29       33       24      38
Answer: 35 toys

Explanation:
I have 25 toys
and if had 35 then the total toys will be 60
according to the given options it might be less than 35
29, 33 or 24

Think and Grow: Modeling Real Life

You need 60 invitations. You have 36 and buy 36 more. Do you have enough invitations?
Big Ideas Math Answers Grade 1 Chapter 9 Add Two-Digit Numbers 75
Circle what you know.

Underline what you need to find.

Solve:

Compare: ___________ ○ 60        Yes          No
Answer: yes

Explanation:
66 > 60
I had 6 more invitations
Yes they have enough invitations.

Show and Grow

Question 5.
You need 84 bottles of water. You have 48 and buy 32 more. Do you have enough bottles of water?
Big Ideas Math Answers Grade 1 Chapter 9 Add Two-Digit Numbers 76
Circle what you know.

Underline what you need to find.

Solve:

Compare: ___________ ○ 84        Yes          No
Answer:  No

Explanation:
The water bottles I had 80
The water bottles needed is 84
80 < 84
so, there are no enough waterbottles

Problem Solving: Addition Practice 9.6

Question 1.
You have 31 stuffed animals. You and your friend have 60 stuffed animals in all. How many stuffed animals does your friend have?

______________ stuffed animals
Answer:  29

Explanation:
31 + 29 = 60,
There are 29 stuffed animals with my friend.

Question 2.
A store has 56 shirts. There are 28 fewer shirts than pairs of pants. How many pairs of pants are there?

______________ Pairs of pants
Answer: 28

Explanation:
Total shirts is 56 and 56 – 28 = 28,
The pairs of paints are 28.
28 + 28 = 56.

Question 3.
YOU BE THE TEACHER
You have 25 movies. You have 18 more video games than movies. Your friend says you have (43 movies and video games in all. Is your friend correct? Explain.

___________________________________________________

___________________________________________________
Answer: 43:

Explanation
Me having 25 movies and 18 videogames 25 + 18 = 43,
Yes my friend is correct I have 43 movies and  videogames in all.

Question 4.
Modeling Real Life
Newton needs 9o chairs for a party. He has 51. He rents 3 more. Does Newton have enough chairs?

Circle:         Yes          No
Answer: No,

Explanation:
Newton doesn’t have enough chairs.
He needed is 90 he has 51 + 3 = 54.

Review & Refresh

Circle the longer object.

Question 5.
Big Ideas Math Answers Grade 1 Chapter 9 Add Two-Digit Numbers 77
Answer:
The scissor is longer than the brick
circled the scissor

Question 6.
Big Ideas Math Answers Grade 1 Chapter 9 Add Two-Digit Numbers 78
Answer:
The first pencil is longer than the second pencil
First pencil is circled

Add Two-Digit Numbers Performance Task

Question 1.
You play a game. Each red ball you collect is worth 10 points. Each yellow ball you collect is worth 1 point.
Big Ideas Math Answers Grade 1 Chapter 9 Add Two-Digit Numbers 79

a. You collect 3 red balls and 13 yellow balls. How many points do you have?

___________ points
Answer: 30

Explanation
Red ball = 10
yellow = 1.
I have collected 3 red and 13 yellow,
3 × 10 = 30

39 + 13 = 52. Total points are 52.

b. Your teammates score 38 points and 24 points. How many points do your teammates have in all?

___________ points
Answer:
38 + 24 = 62,
62 points our teammates have in all.

c. Your learn wants to have 100 points. Does your team reach its goal?

Yes No
Answer: No,
we didn’t reach the goal.

d. Why do you think a red ball is worth more points?
Answer:
Because it is small in size and easy to handle to kids by seeing in the figure.

Add Two-Digit Numbers Change Practice

Add Tens and Ones Homework & Practice 9.1

Question 1.
56 + 3 = _________
Big Ideas Math Solutions Grade 1 Chapter 9 Add Two-Digit Numbers 80
Answer:
53 + 3 = 56,
5 tens and 6 ones.

Question 2.
22 + 54 = __________
Big Ideas Math Solutions Grade 1 Chapter 9 Add Two-Digit Numbers 81
Answer:
24 + 54 = 78,
7 tens and 8 ones.

Add Tens and Ones Using a Number Line Homework & Practice 9.2

Question 3.
62 + 25 = ___________
Big Ideas Math Solutions Grade 1 Chapter 9 Add Two-Digit Numbers 82
Answer: 62 + 25 = 87

Explanation:
6 tens and 2 ones
2 tens and 5 ones
by adding both we get 87

Question 4.
38 + 51 = ___________
Big Ideas Math Solutions Grade 1 Chapter 9 Add Two-Digit Numbers 82
Answer: 38 + 51 = 89

Explanation:
3 tens and 8 ones
5 tens and 1 ones
by adding both we get 89.

Question 5.
MP Structure
Write an equation that matches the number line.
Big Ideas Math Solutions Grade 1 Chapter 9 Add Two-Digit Numbers 83
Answer: 16 + 52 = 68

Make a 10 to Add Homework & Practice 9.3

Question 6.
42 + 6 = ___________
Big Ideas Math Solutions Grade 1 Chapter 9 Add Two-Digit Numbers 84
Make a 10?       Yes           No
Answer: No

Explanation:
lines represents the tens and circles represents the ones
4 tens and 2 ones
0 tens and 6 ones
By adding ones place from both the sides 2 + 6 = 8
so there is no 10 in it.

Question 7.
27 + 7 = ___________
Big Ideas Math Solutions Grade 1 Chapter 9 Add Two-Digit Numbers 85
Make a 10?       Yes           No
Answer: yes

Explanation:
lines represents the tens and circles represents the ones
2 tens and 7 ones
0 tens and 7 ones
By adding ones place from both the sides 7 + 7 = 14
so there is 10 in it

MP Logic
Complete.
Question 8.
Big Ideas Math Solutions Grade 1 Chapter 9 Add Two-Digit Numbers 86
Answer:
34 + 7 = 41.

Question 9.
Big Ideas Math Solutions Grade 1 Chapter 9 Add Two-Digit Numbers 87
Answer:
59 + 8 = 67

Add Two-Digit Numbers Homework & Practice 9.4

Make quick sketches to find the sum.

Question 10.
Big Ideas Math Solutions Grade 1 Chapter 9 Add Two-Digit Numbers 88
Answer:
28 + 33 = 61, 6 tens and 1 ones.

Question 11.
Big Ideas Math Solutions Grade 1 Chapter 9 Add Two-Digit Numbers 89

____________ tens ___________ one
Answer:
49 + 24 =73, 7 tens and 3 ones.

Question 12.
Modeling Real Life
Your club earns a badge for every 10 trees planted. Your club plants 25 trees in the fall and 25 in the spring. How many badges does your club earn?
Big Ideas Math Solutions Grade 1 Chapter 9 Add Two-Digit Numbers 90

_______________ badges
Answer:
One badge for 10, 25 in spring and 25 in fall.
Total trees planted is 25 + 25 = 50.
So, 5 badges.

Practice Addition Strategies Homework & Practice 9.5

Question 13.
19 + 43 = __________
Answer: 19 + 43 = 62, 6 tens and 2 ones.

Question 14.
66 + 28 = __________
Answer: 66 + 28 = 94, 9 tens and 4 ones.

Problem Solving: Addition Homework & Practice 9.6

Question 15.
Your friend has 5 marbles. You have 23 more than your friend. How many marbles do you have?
Big Ideas Math Solutions Grade 1 Chapter 9 Add Two-Digit Numbers 91

_______________ marbles
Answer:  28 marbles

Explanation
My friend has 5 and me having 23 more
than my friend 23 + 5 = 28.

Question 16.
Modeling Real Life
You need 50 party hats. You have 24. You buy 16 more. Do you have enough party hats?
Big Ideas Math Solutions Grade 1 Chapter 9 Add Two-Digit Numbers 92

Yes           No
Answer: No

Explanation:
Me having 24 caps and bought 16 more
24 + 16 = 40,
No, I don’t have enough caps.

Conclusion:

Download Big Ideas Math Answers Grade 1 Chapter 9 Add Two-Digit Numbers pdf and start your preparation for the exams. Big Ideas Math Answer is the best platform for the students to improve their math skills. If you have any queries about Big Ideas Math Answers Grade 1 Chapters you can post the comments in the below comment box. All the Best!!

Big Ideas Math Answers Grade 2 Chapter 8 Count and Compare Numbers to 1,000

Big Ideas Math Answers Grade 2 Chapter 8

Are you searching for the Big Ideas Math Grade 2 Chapter 8 Count and Compare Numbers to 1,000 Answer Key? If yes, then stay on this page. Have a look at the detailed explanation for all the questions of Big Ideas Math 2nd Grade 8th Chapter Count and Compare Numbers to 1,000 Book. The BIM Grade 2 Answer Key for Chapter 8 Count and Compare Numbers to 1,000 will help you to become a pro in solving questions. Hence Download Big Ideas Math Answers Grade 2 Chapter 8 Count and Compare Numbers to 1,000 PDF and begin the preparation as early as possible.

Big Ideas Math Book Grade 2 Answer Key Chapter 8 Count and Compare Numbers to 1,000

Every student has to prepare all the topics provided in BIM Grade 2 Answers Chapter 8 Count and Compare Numbers to 1,000 to score good marks in the exam. This answer key is also helpful to complete the homework and assignments within the given time. After practicing chapter 8, you can be able to understand counting. The different topics included in Big Ideas Math Book 2nd Grade Answers Chapter 8 Count and Compare Numbers to 1,000 are Count and Compare Numbers to 1,000 Vocabulary, Count to 120 in Different Ways, Count to 1,000 in Different Ways, Place Value Patterns, Find More or Less, Compare Numbers Using Symbols, and Compare Numbers Using a Number Line.

Vocabulary

Lesson 1 Count to 120 in Different Ways

Lesson 2 Count to 1,000 in Different Ways

Lesson 3 Place Value Patterns

Lesson 4 Find More or Less

Lesson 5 Compare Numbers Using Symbols

Lesson 6 Compare Numbers Using a Number Line

Count and Compare Numbers to 1,000 Vocabulary

Big Ideas Math Answers Grade 2 Chapter 8 Count and Compare Numbers to 1,000 1
Organize It
Use the review words to complete the graphic organizer.
Big Ideas Math Answers Grade 2 Chapter 8 Count and Compare Numbers to 1,000 2
Answer:
Big-Ideas-Math-Book-2nd-Grade-Answer-Key-Chapter-8-Count-Compare-Numbers-to-1,000-Count-and-Compare-Numbers-to-1,000-Vocabulary

Define It

Use your vocabulary cards to complete the puzzle.

Big Ideas Math Answer Key Grade 2 Chapter 8 Count and Compare Numbers to 1,000 3

Chapter 8 Vocabulary cards

Big Ideas Math Answer Key Grade 2 Chapter 8 Count and Compare Numbers to 1,000 4
Big Ideas Math Answer Key Grade 2 Chapter 8 Count and Compare Numbers to 1,000 5

Lesson 8.1 Count to 120 in Different Ways

Explore and Grow

Start at 5. Skip count by fives. Circle the numbers you count. Start at 10. Skip count by tens. Color the numbers you count.
Big Ideas Math Answer Key Grade 2 Chapter 8 Count and Compare Numbers to 1,000 6
What patterns do you notice?
____________________
____________________
Answer:
Big-Ideas-Math-Book-2nd-Grade-Answer-Key-Chapter-8 Count-Compare-Numbers-to-1,000-Lesson-8.1-Count-to-120-in Different-Ways-Explore-and-Grow
In column 5 we notice the pattern of count by tens and in column 10 we notice the pattern of count by tens.

Show and Grow

Question 1.
Count by ones.
35, 36, 37, ___, ____, ___, ___, ___
Answer:
35, 36, 37, 38, 39, 40, 41, 42 .

Question 2.
Count by fives.
55, 60, 65, __, ___, ___, ___, ___
Answer:
55, 60, 65, 70, 75, 80, 85, 90

Question 3.
Count by tens.
21, 31, 41, ___, __, ___, ___, ___
Answer:
21, 31, 41, 51, 61, 71, 81, 91 .

Apply and Grow: Practice

Count by ones.

Question 4.
57, 58, 59, __, ___, ___, ___, ___
Answer:
57, 58, 59, 60, 61, 62, 63, 64

Count by fives.

Question 6.
35, 40, 45, __, ___, ___, __, ___
Answer:
35, 40, 45, 50, 55, 60, 65, 70

Question 5.
__, 106, __, 108, __, ___, ___
Answer:
105, 106, 107, 108, 109, 110, 111

Count by fives.

Question 6.
35, 40, 45, __, ___, ___, ___, ___
Answer:
35, 40, 45, 50, 55, 60, 65, 70

Question 7.
__, 80, __, 90, __, __, ___
Answer:
70, 80, 90, 100, 110, 120 .

Count by tens.

Question 8.
12, 22, 32, __, ___, __, ___, __
Anwer:
12, 22, 32, 42, 52, 62, 72, 82 .

Question 9.
__, 50, __, 70, __, __, ___
Answer:
40, 50, 60, 70, 80, 90, 100 .

Question 10.
Number Sense
Newton counts by ones from 47 to 53. Which numbers does he count?
Big Ideas Math Answer Key Grade 2 Chapter 8 Count and Compare Numbers to 1,000 6.1
Answer:
counts by ones from 47 to 53 are :
47, 48, 49, 50, 51, 52, 53 .
numbers does he count are 49 and 52 numbers

Question 11.
Number Sense
Descartes counts by fives from 90 to 115. Which numbers does he count?
Big Ideas Math Answer Key Grade 2 Chapter 8 Count and Compare Numbers to 1,000 7
Answer:
counts by fives from 90 to 115 are :
90, 95, 100, 105, 110, 115 .
numbers does he count are 105 and 110 .

Think and Grow: Modeling Real Life

Newton has 65 points. He captures small aliens worth 5 points. Descartes has 25 points. He captures large aliens worth 10 points.
Big Ideas Math Answer Key Grade 2 Chapter 8 Count and Compare Numbers to 1,000 8
Big Ideas Math Answer Key Grade 2 Chapter 8 Count and Compare Numbers to 1,000 9
Answer:
Number of points Newton have = 65
Number of points for small aliens = 5.
Number of aliens did Newton have = 65 / 5 = 13.

Number of points Descartes have = 25
Number of points for Large aliens = 10.
Number of aliens did Descartes have = 25 / 10 = 2.5
Newton have more aliens than Descartes.so Descartes needs more aliens .

Show and Grow

Question 12.
Newton has 55 points. He collects gold coins worth 10 points. Descartes has 70 points. He collects silver coins worth 5 points. Who needs to collect more coins to reach 100 points?
Big Ideas Math Answer Key Grade 2 Chapter 8 Count and Compare Numbers to 1,000 10
Big Ideas Math Answer Key Grade 2 Chapter 8 Count and Compare Numbers to 1,000 11
Answer:
Number of points Newton have = 55 points
he collects gold coins worth = 10 points each .
for 100 points newton requires 45 points
Number of coins required for 45 points for Newton= 45/ 10 = 4.5 coins.

Number of points Descartes have = 70 points
he collects silver coins worth = 5 points each .
for 100 points Descartes requires 30 points
Number of coins required for 30 points for Descartes= 30/ 5 = 6 coins.
Descartes requires more 6 coins to reach 100 points .

Question 13.
You and your friend count from 30 to 70. You count by fives. Your friend counts by tens. Who says more numbers? Explain.
Answer:
Count by fives:( by me)
30, 35, 40, 45, 50, 55, 60, 65, 70 = total 9 numbers.
Count by tens:( my friend)
30, 40, 50, 60, 70 = total 5 numbers.
I say more numbers than my friend .

Count to 120 in Different Ways Homework & Practice 8.1

Count by ones.

Question 1.
63, 64, 65, __, __, ___, ___, ___
Answer:
63, 64, 65, 66, 67, 68, 69, 70 .

Question 2.
__, 112, __, 114, __, __, ___
Answer:
111, 112, 113, 114, 115, 116, 117 .

Count by fives.

Question 3.
10, 15, 20, __, __, ___, __, ___
Answer:
10, 15, 20, 25, 30, 35, 40, 45 .

Question 4.
__, 95, __, 105, __, __, ___
Answer:
90, 95, 100, 105, 110, 115, 120 .

Count by tens.

Question 5.
44, 54, 64, __, __, __, ___, __
Answer:
44, 54, 64, 74, 84, 94, 104 .

Question 6.
__, 20, __, 40, __, __, __
Answer:
10, 20, 30, 40, 50, 60, 70 .

Question 7.
YOU BE THE TEACHER
Newton counts by fives. Is he correct? Explain.
Big Ideas Math Answer Key Grade 2 Chapter 8 Count and Compare Numbers to 1,000 12
Big Ideas Math Answer Key Grade 2 Chapter 8 Count and Compare Numbers to 1,000 13
Answer:
No, Count by fives means 5, 10, 15, 20, 25, 30 and so on …..
As per the above count 5, 15, 25, 35, 45, 55 they are count by tens as there is 10 differences between the sequence of numbers.

Question 8.
Modeling Real Life
Who needs to make more shots to earn 50 points?
Big Ideas Math Answer Key Grade 2 Chapter 8 Count and Compare Numbers to 1,000 14

Answer:
Newton
Number of points with Newton = 30 .
Newton requires more 20 points to make it 50 points
Points for one shot = 5 .
Number of shots required for Newton = 20 / 5 = 4 shots.

Descartes
Number of points with Descartes = 20 .
Descartes requires more 30 points to make it 50 points
Points for one shot = 10 .
Number of shots required for Descartes = 30 / 10 = 3 shots.

Therefor Newton needs to make more shots than Descartes .

Review & Refresh

Question 9.
34 – 16 = ?
Big Ideas Math Answer Key Grade 2 Chapter 8 Count and Compare Numbers to 1,000 15
Answer:
Big-Ideas-Math-Book-2nd-Grade-Answer-Key-Chapter-8-Count-Compare-Numbers-to-1,000-Count-to-120-Different-Ways-Homework-Practice-8.1-Question-9

Question 10.
75 – 32 = ?
Big Ideas Math Answer Key Grade 2 Chapter 8 Count and Compare Numbers to 1,000 16
Answer:
Big-Ideas-Math-Book-2nd-Grade-Answer-Key-Chapter-8-Count-Compare-Numbers-to-1,000-Count-to-120-Different-Ways-Homework-Practice-8.1-Question-10

Question 11.
93 – 28 = ?
Big Ideas Math Answer Key Grade 2 Chapter 8 Count and Compare Numbers to 1,000 17
Answer:
Big-Ideas-Math-Book-2nd-Grade-Answer-Key-Chapter-8-Count-Compare-Numbers-to-1,000-Count-to-120-Different-Ways-Homework-Practice-8.1-Question-11

Lesson 8.2 Count to 1,000 in Different Ways

Explore and Grow

Count by hundreds to 1,000.
Big Ideas Math Answer Key Grade 2 Chapter 8 Count and Compare Numbers to 1,000 18
Answer:
Big-Ideas-Math-Book-2nd-Grade-Answer-Key-Chapter-8-Count-Compare-Numbers-to-1,000- Lesson-8.2-Count-to-1,000-Different Ways-Explore-Grow-1

Count by tens to 1,000.
Big Ideas Math Answer Key Grade 2 Chapter 8 Count and Compare Numbers to 1,000 19
Answer:
Big-Ideas-Math-Book-2nd-Grade-Answer-Key-Chapter-8-Count-Compare-Numbers-to-1,000- Lesson-8.2-Count-to-1,000-Different Ways-Explore-Grow-2
Count by fives to 1,000.
Big Ideas Math Answer Key Grade 2 Chapter 8 Count and Compare Numbers to 1,000 20
Answer:
Big-Ideas-Math-Book-2nd-Grade-Answer-Key-Chapter-8-Count-Compare-Numbers-to-1,000- Lesson-8.2-Count-to-1,000-Different Ways-Explore-Grow-3
Count by ones to 1,000.
Big Ideas Math Answer Key Grade 2 Chapter 8 Count and Compare Numbers to 1,000 21
Answer:
Big-Ideas-Math-Book-2nd-Grade-Answer-Key-Chapter-8-Count-Compare-Numbers-to-1,000- Lesson-8.2-Count-to-1,000-Different Ways-Explore-Grow-4

Show and Grow

Question 1.
Count by fives.
675, 680, 685, __, __, __, __, ___
Answer:
675, 680, 685, 690, 695, 700, 705, 710 .

Question 2.
Count by tens.
850, 860, 870, __, __, __, __, ___
Answer:
850, 860, 870, 880, 890, 900, 910, 920 .

Question 3.
Count by hundreds.
100, 200, 300, __, __, __, __, ___
Answer:
100, 200, 300, 400, 500, 600, 700, 800 .

Apply and Grow: Practice

Count by fives.

Question 4.
520, 525, 530, __, __, __, __, ___
Answer:
520, 525, 530, 535, 540, 545, 550, 555 .

Question 5.
875, 880, __, __, __, __, ___
Answer:
875, 880, 885, 890, 895, 900, 905 .

Count by tens.

Question 6.
600, 610, 620, __, __, __, __, ___
Answer:
600, 610, 620, 630, 640, 650, 660, 670 .

Question 7.
460, 470, __, __, __, __, ___
Answer:
460, 470, 480, 490, 500, 510, 520 .

Count by hundreds.

Question 8.
200, 300, 400, __, __, __, __, ___
Answer:
200, 300, 400, 500, 600, 700, 800, 900 .

Question 9.
400, 500, __, __, __, __, __
Answer:
400, 500, 600, 700, 800, 900, 1000 .

Question 10.
DIG DEEPER!
Newton counts by hundreds. Find the missing number. Think: How do you know?
Big Ideas Math Answer Key Grade 2 Chapter 8 Count and Compare Numbers to 1,000 22
Answer:
The missing number is 0  as the count start from 0, 100, 200, 300, 400, 500 Each number is 100 more than the previous number.

Question 11.
Structure
Did Descartes count by tens or by hundreds? Think: How do you know?
Big Ideas Math Answer Key Grade 2 Chapter 8 Count and Compare Numbers to 1,000 23
Answer:
It is count by tens as Each number is 10 more than the previous number .

Think and Grow: Modeling Real Life

A summer camp leader has 240 T-shirts. He buys 6 more colors with 10 shirts in each color. How many T-shirts does he have now?
Big Ideas Math Answer Key Grade 2 Chapter 8 Count and Compare Numbers to 1,000 24
Big Ideas Math Answer Key Grade 2 Chapter 8 Count and Compare Numbers to 1,000 25
Answer:
Number of T-Shirts = 240.
Number of T-shirts were bought = 6 X 10 ( 6 different colors ) = 60 T-shirts .
Total Number of T-Shirts = 240 + 60 = 300 .

Show and Grow

Question 12.
You have 100 bracelets. You buy 5 more boxes with 100 bracelets in each box. How many bracelets do you have now?
Big Ideas Math Answer Key Grade 2 Chapter 8 Count and Compare Numbers to 1,000 26
Big Ideas Math Answer Key Grade 2 Chapter 8 Count and Compare Numbers to 1,000 27
Answer:
Number of Bracelets = 100.
Number of Boxes bought = 5 x 100 = 500 bracelets.
Total Number of Bracelets = 100 + 500 = 600 bracelets.

Question 13.
You and your friend count from 370 to 420. You count by tens. Your friend counts by fives. Who says more numbers? Explain.
_________________________
_________________________
Answer:
Count by Tens:( by me)
370, 380, 390, 400, 410, 420 . = total 6 numbers.
Count by fives:( my friend)
370, 375, 380, 385, 390, 395, 400, 405, 410, 415, 420 = total 11 numbers.
My friends say more numbers than me .

Count to 1,000 in Different Ways Homework & Practice 8.2

Count by fives.

Question 1.
445, 450, 455, ___, __, __, __, ___
Answer:
445, 450, 455, 460, 465, 470, 475, 480 .

Question 2.
770, 775, __, __, __, __, ___
Answer:
770, 775, 780, 785, 790, 795, 800,

Count by tens.

Question 3.
520, 530, 540, __, __, __, __, __
Answer:
520, 530, 540, 550, 560, 570, 580, 590 .

Question 4.
660, 670, __, __, __, __, ___
Answer:
660, 670, 680, 690, 700, 710, 720 .

Count by Hundreds.

Question 5.
300, 400, 500, __, __, __, __, ___
Answer:
300, 400, 500, 600, 700, 800, 900, 1000.

Question 6.
200, 300, __, __, __, __, ___
Answer:
200, 300, 400, 500, 600, 700, 800 .

Review & Refresh

Question 7.
DIG DEEPER!
Newton starts at 950 and counts to 1,000 by fives. Complete the number line to show the last 6 numbers he counts.
Big Ideas Math Answer Key Grade 2 Chapter 8 Count and Compare Numbers to 1,000 28
Answer:
Big-Ideas-Math-Book-2nd-Grade-Answer-Key-Chapter-8-Count-Compare-Numbers-to-1,000- Count-to-1,000-Different-Ways-Homework-Practice-8.2-Question-7

Question 8.
Modeling Real Life
A carnival worker has 380 stuffed animals. She buys 6 more boxes with 5 stuffed animals in each box. How many stuffed animals does she have now?
Big Ideas Math Answer Key Grade 2 Chapter 8 Count and Compare Numbers to 1,000 29
Answer:
Number of stuffed animals = 380 .
Number of animals bought = 6 x 5 = 30.
Total Number of stuffed animals = 380 + 30 = 410 Animals .

Question 9.
Modeling Real Life
A water park shop owner has 100 goggles. He buys 4 more colors with 100 goggles in each color. How many goggles does the shop owner have now?
Big Ideas Math Answer Key Grade 2 Chapter 8 Count and Compare Numbers to 1,000 30
Answer:
Number of Goggles = 100.
Number of goggles bought = 4 x 100 = 400
Total Number of Goggles = 100 + 400 = 500 Goggles .

Question 10.
You see 14 geese in a pond. 17 more join them. Then you see 11 more fly to the pond. How many geese do you see in all?
Big Ideas Math Answer Key Grade 2 Chapter 8 Count and Compare Numbers to 1,000 31
Answer:
Number of geese in a pond = 14.
Number of geese joined = 17 .
Total geese’s = 14 + 17 = 31 .
Number of geese’s flew to pond = 11 .
Total Geese’s = 31 + 11 = 42 .

Lesson 8.3 Place Value Patterns

Explore and Grow

What patterns do you see in the shaded row and column? Use the patterns to complete the chart.
Big Ideas Math Answer Key Grade 2 Chapter 8 Count and Compare Numbers to 1,000 32
Answer:
The pattern in Rows = Count by ones.
The pattern in Columns = Count by Tens.
Big-Ideas-Math-Book-2nd-Grade-Answer-Key-Chapter-8-Count-Compare-Numbers-to-1,000-Lesson-8.3-Place-Value-Patterns-Explore-Grow

Show and Grow

Use place value to find the missing numbers.

Question 1.
485, 486, 487, __, __, ___, ___
Answer:
485, 486, 487, 488, 489, 490, 491 .

Question 2.
612, 622, 632, __, __, ___, ___
Answer:
612, 622, 632, 642, 652, 662, 672 .

Question 3.
267, 277, __, 297, ___, 317, __
Answer:
267, 277, 287, 297, 307, 317, 327 .

Question 4.
101, 201, ___, 401, __, __, ___
Answer:
101, 201, 301, 401, 501, 601, 701 .

Apply and Grow: Practice

Use place value to find the missing numbers.

Question 5.
324, 325, ___, 327, __, __, ___
Answer:
324, 325, 326, 327, 328, 329, 330 .

Question 6.
194, 294, __, 494, __, __, ___
Answer:
194, 294, 394, 494, 594, 694, 794 .

Question 7.
463, 473, __, 493, __, __, __
Answer:
463, 472, 483, 493, 503, 513, 523 .

Question 8.
232, 332, __, 532, __, ___, __
Answer:
232, 332, 432, 532, 632, 732, 832 .

Question 9.
985, 986, __, 988, __, __, __
Answer:
985, 986, 987, 988, 989, 990 .

Question 10.
751, 761, __, 781, __, __, __
Answer:
751, 761, 771, 781, 791, 801, 811 .

Question 11.
606, 607, __, 609, __, __, __
Answer:
606, 607, 608, 609, 700, 701, 702 .

Question 12.
Repeated Reasoning
Use place value to describe each pattern.
Big Ideas Math Answer Key Grade 2 Chapter 8 Count and Compare Numbers to 1,000 33
Answer:
Big-Ideas-Math-Book-2nd-Grade-Answer-Key-Chapter-8-Count-Compare-Numbers-to-1,000-Lesson-8.3-Place-Value-Patterns-Apply-Grow-Practice-Question-12

Think and Grow: Modeling Real Life

There are 273 tickets in a bin. Some more are put in the bin. Now there are 973. How many groups of 100 tickets were put in the bin?
Big Ideas Math Answer Key Grade 2 Chapter 8 Count and Compare Numbers to 1,000 34
Big Ideas Math Answer Key Grade 2 Chapter 8 Count and Compare Numbers to 1,000 35
Answer:

Number of tickets in bin = 273.
Number of tickets added in bin = X.
Total Number tickets in bin now = 973 = 273 + X
X = 973 – 273 = 700 .
Number of groups = 700/100 = 7 (Each group contains 100 tickets) .
Therefore 7 groups of 100 tickets were put in bin .

Show and Grow

Question 13.
You have 338 pennies in a jar. You put more in the jar. Now there are 388. How many groups of 10 pennies were put in the jar?
Big Ideas Math Answer Key Grade 2 Chapter 8 Count and Compare Numbers to 1,000 36

Answer:
Number of pennies =338.
Number of pennies added = X.
Total Number of pennies = 388 = 338 + X.
X= 388 – 328 = 50.
Number of groups of 10 pennies were put in the jar = 50 / 10 = 5.

Question 14.
DIG DEEPER!
There are 410 people at a show. 8 more rows of seats get filled. Now there are 490 people. How many people can sit in each row?
Big Ideas Math Answer Key Grade 2 Chapter 8 Count and Compare Numbers to 1,000 37
Explain how you solved.
______________
______________
Answer:
Number of people = 410 .
Number of people added = X.
Total Number of people = 490.
490=410 + X.
X = 490 – 410 = 80 .
Number of people added = 80 .
Number of Rows = 8.
Number of people in each row = 80 /8 = 10.

Place Value Patterns Homework & Practice 8.3

Use place value to find the missing numbers.

Question 1.
710, 711, 712, __, __, __, ___
Answer:
710, 711, 712, 713, 714, 715, 716 .

Question 2.
822, 832, 842, __, __, __, __
Answer:
822, 832, 842, 852, 862, 872, 882 .

Question 3.
325, 425, 525, __, __, __, __
Answer:
325, 425, 525, 625, 725, 825, 925 .

Question 4.
669, 679, __, 699, __, __, ___
Answer:
669, 679, 689, 699, 709, 719, 729 .

Question 5.
534, 535, __, 537, __, __, __
Answer:
534, 535, 536, 537, 538, 539 , 540

Question 6.
368, 468, __, 668, __, __, ___
Answer:
368, 468, 568, 668, 768, 868, 968 .

Question 7.
YOU BE THE TEACHER
Newton says the hundreds digit in the numbers shown increases by 1. Is he correct? Explain.
Big Ideas Math Answer Key Grade 2 Chapter 8 Count and Compare Numbers to 1,000 38

Answer:
No, There is increase in the tens place not in hundreds place .Each Number is 10 more than the previous number so it is count by tens.

Question 8.
Modeling Real Life
A farmer has 467 cornstalks. The farmer grows some more. Now there are 967 cornstalks. How many groups of 100 cornstalks did the farmer add?
Big Ideas Math Answer Key Grade 2 Chapter 8 Count and Compare Numbers to 1,000 39

Answer:
Number of cornstalks = 467 .
Number of cornstalks grown =X
Total Number of cornstalks = 967.
467 + X =967.
X = 967- 467 = 500.
Number of cornstalks grown = 500 .
Number of groups of 100 cornstalks added = 500/100 = 5.

Question 9.
DIG DEEPER!
There are 250 people at a party. 3 more tables get filled. Now there are 280 people. How many people can sit at each table?
Big Ideas Math Answer Key Grade 2 Chapter 8 Count and Compare Numbers to 1,000 40

Answer:
Number of people in the party = 250 .
Number of more people added = X.
Total Number of people in the party = 280 .
250 + X = 280 .
X = 280- 250 = 30 .
Number of more people added = 30 .
Number of tables got filled with added people = 3.
Number of people in each table = 30 / 3 = 10 .

Review & Refresh

Question 10.
8 + 4 = __
Answer:
12.

Question 11.
15 – 8 = __
Answer:
7.

Question 12.
Big Ideas Math Answer Key Grade 2 Chapter 8 Count and Compare Numbers to 1,000 41

Answer:
10.
Big-Ideas-Math-Book-2nd-Grade-Answer-Key-Chapter-8-Count-Compare-Numbers-to-1,000-Place-Value-Patterns-Homework-Practice-8.3-Question-12

Question 13.
Big Ideas Math Answer Key Grade 2 Chapter 8 Count and Compare Numbers to 1,000 42

Answer:
6
Big-Ideas-Math-Book-2nd-Grade-Answer-Key-Chapter-8-Count-Compare-Numbers-to-1,000-Place-Value-Patterns-Homework-Practice-8.3-Question-13

Question 14.
Big Ideas Math Answer Key Grade 2 Chapter 8 Count and Compare Numbers to 1,000 43

Answer:
13.
Big-Ideas-Math-Book-2nd-Grade-Answer-Key-Chapter-8-Count-Compare-Numbers-to-1,000-Place-Value-Patterns-Homework-Practice-8.3-Question-14

Question 15.
Big Ideas Math Answer Key Grade 2 Chapter 8 Count and Compare Numbers to 1,000 44
Answer:
9 .
Big-Ideas-Math-Book-2nd-Grade-Answer-Key-Chapter-8-Count-Compare-Numbers-to-1,000-Place-Value-Patterns-Homework-Practice-8.3-Question-15

Lesson 8.4 Find More or Less

Explore and Grow

Model 253. Use your model to complete the sentences.
Big Ideas Math Answer Key Grade 2 Chapter 8 Count and Compare Numbers to 1,000 45

1 more than 253 is __.
1 less than 253 is __.
10 more than 253 is __.
10 less than 253 is __.
100 more than 253 is __.
100 less than 253 is __.
Answer:
1 more than 253 is 254.
1 less than 253 is 252.
10 more than 253 is 263.
10 less than 253 is 243.
100 more than 253 is 353.
100 less than 253 is 153.

Show and Grow

Question 1.
10 more than 452 is __.
Answer:
10 more than 452 is 462.

Question 2.
10 less than 813 is __.
Answer:
10 less than 813 is 803..

Question 3.
100 less than 729 is __.
Answer:
100 less than 729 is 629.

Question 4.
100 more than 386 is __.
Answer:
100 more than 386 is 486.

Apply and Grow: Practice

Question 5.
10 more than 571 is __.
Answer:
10 more than 571 is 581.

Question 6.
10 less than 333 is __.
Answer:
10 less than 333 is 323.

Question 7.
100 more than 604 is __.
Answer:
100 more than 604 is 704.

Question 8.
100 less than 592 is __.
Answer:
100 less than 592 is 492.

Question 9.
1 more than 934 is __.
Answer:
1 more than 934 is 935.

Question 10.
1 less than 101 is __.
Answer:
1 less than 101 is 100.

Question 11.
10 less than 286 is __.
Answer:
10 less than 286 is 276.

Question 12.
1 more than 467 is __.
Answer:
1 more than 467 is 468.

Question 13.
10 more than 763 is __.
Answer:
10 more than 763 is 773.

Question 14.
100 less than 846 is __.
Answer:
100 less than 846 is 746.

Question 15.
1 less than 999 is __.
Answer:
1 less than 999 is 998.

Question 16.
100 more than 28 is __.
Answer:
100 more than 28 is 128.

Question 17.
100 less than 135 is __.
Answer:
100 less than 135 is 35.

Question 18.
100 more than 900 is __.
Answer:
100 more than 900 is 1000.

Question 19.
Number Sense
Complete each sentence.
Big Ideas Math Answer Key Grade 2 Chapter 8 Count and Compare Numbers to 1,000 46
Answer:
Big-Ideas-Math-Book-2nd-Grade-Answer-Key-Chapter-8-Count-Compare-Numbers-to-1,000-Lesson-8.4-Find-More-Less-Question-19

Think and Grow: Modeling Real Life

An orange tree has 639 oranges. A lemon tree has 100 fewer lemons. How many lemons does the tree have?
Big Ideas Math Answer Key Grade 2 Chapter 8 Count and Compare Numbers to 1,000 47

Answer:
Number of Oranges = 639.
Number of Lemons = 639 – 100 =539.

Show and Grow

Question 20.
A history book has 197 pictures. A science book has 10 more pictures. How many pictures are in the science book?
Big Ideas Math Answer Key Grade 2 Chapter 8 Count and Compare Numbers to 1,000 48
Answer:
Number of pictures in history books = 197 .
Number of pictures in science book = 197 + 10  = 207.

Question 21.
DIG DEEPER!
A boat puzzle has 525 pieces. A bird puzzle has 100 more than the boat puzzle. A space puzzle has 10 fewer than the bird puzzle. How many puzzle pieces does the space puzzle have?
Big Ideas Math Answer Key Grade 2 Chapter 8 Count and Compare Numbers to 1,000 49
Big Ideas Math Answer Key Grade 2 Chapter 8 Count and Compare Numbers to 1,000 50
Answer:
Number of puzzles in boat puzzle = 525 .
Number of puzzles in bird puzzle = 100 + 525 = 625.
Number of puzzles in space puzzle = 625 – 10 = 615 .

Question 22.
DIG DEEPER!
You have 398 points. Newton has 100 fewer than you. Descartes has 10 more than Newton. How many points does Descartes have?
Big Ideas Math Answer Key Grade 2 Chapter 8 Count and Compare Numbers to 1,000 51
Answer:
Number of points = 398 .
Number of points Newton have = 398 – 100 = 298 .
Number of points Descartes have = 298 + 10 = 308 .

Find More or Less Homework & Practice 8.4

Question 1.
10 more than 106 is __.
Answer:
10 more than 106 is 116.

Question 2.
10 less than 467 is __
Answer:
10 less than 467 is 457.

Question 3.
100 more than 321 is __.
Answer:
100 more than 321 is 421.

Question 4.
100 less than 945 is __.
Answer:
100 less than 945 is 845.

Question 5.
1 more than 513 is __.
Answer:
1 more than 513 is 514.

Question 6.
1 less than 899 is __.
Answer:
1 less than 899 is 898.

Question 7.
1 less than 264 is __.
Answer:
1 less than 264 is 263.

Question 8.
100 more than 555 is __.
Answer:
100 more than 555 is 655.

Question 9.
1 more than 852 is __.
Answer:
1 more than 852 is 853.

Question 10.
100 less than 573 is __.
Answer:
100 less than 573 is 473.

Question 11.
10 less than 314 is __
Answer:
10 less than 314 is 304.

Question 12.
10 more than 687 is __
Answer:
10 more than 687 is 697.

Question 13.
Structure
Complete the table.
Big Ideas Math Answer Key Grade 2 Chapter 8 Count and Compare Numbers to 1,000 52

Answer:
Big-Ideas-Math-Book-2nd-Grade-Answer-Key-Chapter-8-Count-Compare-Numbers-to-1,000-Find-More-Less-Homework-Practice-8.4-Question-13

Question 14.
Number Sense
Complete the sentence.
__ is 10 less than 546 and 10 more than __.
Answer:
536 is 10 less than 546 and 10 more than 556.

Question 15.
Modeling Real Life
Your magic book has 163 tricks. Your friend’s magic book has 100 more tricks than yours. How many tricks does your friend’s magic book have?
Big Ideas Math Answers 2nd Grade Chapter 8 Count and Compare Numbers to 1,000 53
Answer:
Number of tricks in my magic books = 163 .
Number of tricks in my friends magic books = 100+163 = 263.

Question 16.
DIG DEEPER!
You have 624 songs. Newton has 100 fewer than you. Descartes has 10 more than Newton. How many songs does Descartes have?
Big Ideas Math Answer Key Grade 2 Chapter 8 Count and Compare Numbers to 1,000 54

Answer:
Number of songs = 624.
Number of songs Newton have = 624 – 100 = 524.
Number of songs Descartes have = 524 + 10  = 534 .

Review & Refresh

Question 17.
A bookcase has 5 shelves. There are 2 stuffed animals on each shelf. How many stuffed animals are there in all?
Big Ideas Math Answer Key Grade 2 Chapter 8 Count and Compare Numbers to 1,000 55
Big Ideas Math Answer Key Grade 2 Chapter 8 Count and Compare Numbers to 1,000 56
Answer:
Number of shelves = 5.
Number of stuffed animals in each shelf = 2
Total Number of stuffed animals in each 5 shelves = 2 added 5 times.
2+2+2+2+2 = 10 animals.

Lesson 8.5 Compare Numbers Using Symbols

Explore and Grow

Make a quick sketch of each number. Circle the greater number.
Big Ideas Math Answer Key Grade 2 Chapter 8 Count and Compare Numbers to 1,000 57
How do you know which number is greater?
____________________
____________________
Answer:

When comparing the values of two numbers, you can use a number line to determine which number is greater. The number on the right is always greater than the number on the left. In the example Above, you can see that 472 is greater than 439 because 472 is to the right of 439 on the number line.

Show and Grow

Question 1.
Compare.
Big Ideas Math Answers 2nd Grade Chapter 8 Count and Compare Numbers to 1,000 58
Answer:
652 > 614.

Apply and Grow: Practice

Compare

Question 2.
Big Ideas Math Answers 2nd Grade Chapter 8 Count and Compare Numbers to 1,000 59
Answer:
324 > 317.

Question 3.
Big Ideas Math Answers 2nd Grade Chapter 8 Count and Compare Numbers to 1,000 60
Answer:
26 < 206 .

Question 4.
Big Ideas Math Answers 2nd Grade Chapter 8 Count and Compare Numbers to 1,000 61
Answer:
546 < 564 .

Question 5.
Big Ideas Math Answers 2nd Grade Chapter 8 Count and Compare Numbers to 1,000 62
Answer:
931 > 842.

Question 6.
Big Ideas Math Answers 2nd Grade Chapter 8 Count and Compare Numbers to 1,000 63
Answer:
700 + 30 + 5 = 735.
735 = 735 .

Question 7.
Big Ideas Math Answer Key Grade 2 Chapter 8 Count and Compare Numbers to 1,000 64
Answer:
400 + 20 = 420.
412 < 420.

Question 8.
Reasoning
Find the number that will make all three comparisons true.
Big Ideas Math Answer Key Grade 2 Chapter 8 Count and Compare Numbers to 1,000 65
Answer:
105.

Question 9.
YOU BE THE TEACHER
Is Newton correct? Explain.
Big Ideas Math Answer Key Grade 2 Chapter 8 Count and Compare Numbers to 1,000 66
Answer:
625 < 631.
The given number is a 3-digit number .In 3-digit number Comparison First check the hundred place both the numbers having 6 in hundred place . so then go to the tens place in 625 we have 2 in tens place and in 631 we have 3 in tens place so 3 is greater than 2. so 631 is greater than 625.

Question 10.
There are 125 kids in a taekwondo club. There are 135 kids in a soccer club. Which club has fewer kids?
Big Ideas Math Answer Key Grade 2 Chapter 8 Count and Compare Numbers to 1,000 67
Answer:
Number of kids in taekwondo club = 125 .
Number of kids in soccer club = 135.
Kids in taekwondo club are 10 fewer than soccer club .

Think and Grow: Modeling Real Life

Newton reads 200 pages on Monday, 70 on Tuesday, and 9 on Wednesday. Descartes reads 297 pages. Who reads more pages in all?
Big Ideas Math Answers 2nd Grade Chapter 8 Count and Compare Numbers to 1,000 68
Big Ideas Math Answers 2nd Grade Chapter 8 Count and Compare Numbers to 1,000 69
Answer:
Number of pages read by Newton on Monday = 200.
Number of pages read by Newton on Tuesday = 70.
Number of pages read by Newton on Wednesday =9.
Total pages read by Newton all 3 days = 200 + 70 + 9 = 279.
Total pages read by Descartes = 297.
Descartes reads more pages.
279 < 297 .

Show and Grow

Question 11.
Newton counts train cars. The train has 100 boxcars, 40 tank cars, and 4 locomotives. Descartes counts a train with 142 cars. Who counts more cars in all?
Big Ideas Math Answer Key Grade 2 Chapter 8 Count and Compare Numbers to 1,000 70
Big Ideas Math Answer Key Grade 2 Chapter 8 Count and Compare Numbers to 1,000 70.1
Answer:
Total cars counted by Newton = 100 + 40 = 140.
Total cars counted by Descartes = 142.
142 > 140 .
Descartes counts more cars .

Question 12.
You have 221 coins in your piggy bank. Your friend has 219 coins. Who has fewer coins?
Big Ideas Math Answer Key Grade 2 Chapter 8 Count and Compare Numbers to 1,000 71
Answer:
Number of coins in my piggy bank = 221.
Number of coins my friend have = 219.
My friend have 2 coins fewer than me .

Question 13.
DIG DEEPER!
652 people go to a play on Friday. 625 people go on Saturday. 655 people go on Sunday. On which day are there fewer than 650 people at the play?
Answer:
People played on Friday = 652.
People played on Saturday =625
People played on Sunday =655.
On Saturday 25 people are fewer than 650 people at the play .

Compare Numbers Using Symbols Homework & Practice 8.5

Compare

Question 1.
Big Ideas Math Answers 2nd Grade Chapter 8 Count and Compare Numbers to 1,000 72
Answer:
923 > 854.

Question 2.
Big Ideas Math Answers 2nd Grade Chapter 8 Count and Compare Numbers to 1,000 73
Answer:
386 < 389.

Question 3.
Big Ideas Math Answers 2nd Grade Chapter 8 Count and Compare Numbers to 1,000 74
Answer:
406 = 406 .

Question 4.
Big Ideas Math Answers 2nd Grade Chapter 8 Count and Compare Numbers to 1,000 75
Answer:
621 > 63.

Question 5.
Big Ideas Math Answers 2nd Grade Chapter 8 Count and Compare Numbers to 1,000 76
Answer:
746 < 752 .

Question 6.
Big Ideas Math Answer Key Grade 2 Chapter 8 Count and Compare Numbers to 1,000 77
Answer:
235 > 130.

Question 7.
Big Ideas Math Answers 2nd Grade Chapter 8 Count and Compare Numbers to 1,000 78
Answer:
500 + 60 + 1 = 561.
562 > 561 .

Question 8.
Big Ideas Math Answers 2nd Grade Chapter 8 Count and Compare Numbers to 1,000 79
Answer:
100 + 10 = 110.
110 = 110.

Question 9.
DIG DEEPER!
What is Descartes’s number?

  • It is less than 300.
  • It is greater than 200.
  • The ones digit is 6 less than 10.
  • The tens digit is 2 more than the ones digit.

Big Ideas Math Answers 2nd Grade Chapter 8 Count and Compare Numbers to 1,000 80
Answer:
lies between 200 – 300.
ones digit = 10 – 6 = 4.
Tens digit = 4 + 2 = 6.
264.

Question 10.
There are 428 pages in a science book. There are 424 pages in a math book. Which book has more pages?
Big Ideas Math Answers 2nd Grade Chapter 8 Count and Compare Numbers to 1,000 81
Answer:
Number of pages in science book = 428
Number of pages in Math book = 424 .
science – 428 > 424 – math
Science book has more pages.

Question 11.
Modeling Real Life
A concession stand sells 300 bags of popcorn on Saturday, 50 on Sunday, and 4 on Monday. They sell 345 drinks. Did they sell more bags of popcorn or drinks?
Big Ideas Math Answers Grade 2 Chapter 8 Count and Compare Numbers to 1,000 82
Big Ideas Math Answers Grade 2 Chapter 8 Count and Compare Numbers to 1,000 83
Total Number of popcorn sold all 3 days = 300 + 50 + 4 = 354.
Number of drinks sold = 345 .
354 > 345 .
popcorn bags are sold more.

Question 12.
DIG DEEPER!
Newton climbs 136 stairs on Friday. He climbs 132 on Saturday. He climbs 128 on Sunday. On which day does he climb more than 134 stairs?
Big Ideas Math Answers Grade 2 Chapter 8 Count and Compare Numbers to 1,000 84
Answer:
Friday = 136.
Saturday = 132.
Sunday = 128 .
On Friday he climbed 136 stairs more than 134 stairs.

Review & Refresh

Find the difference. Use addition to check your answer.

Question 13.
Big Ideas Math Answers Grade 2 Chapter 8 Count and Compare Numbers to 1,000 85

Answer:
Big-Ideas-Math-Book-2nd-Grade-Answer-Key-Chapter-8-Count-Compare-Numbers-to-1,000- Compare-Numbers-Using-Symbols-Homework-Practice-8.5-Question-13

Question 14.
Big Ideas Math Answers Grade 2 Chapter 8 Count and Compare Numbers to 1,000 86
Answer:
Big-Ideas-Math-Book-2nd-Grade-Answer-Key-Chapter-8-Count-Compare-Numbers-to-1,000- Compare-Numbers-Using-Symbols-Homework-Practice-8.5-Question-14

Lesson 8.6 Compare Numbers Using a Number Line

Explore and Grow

Identify a number that is less than 538. Identify a number that is greater than 538. Model the numbers on the number line.
Big Ideas Math Answers Grade 2 Chapter 8 Count and Compare Numbers to 1,000 87

Explain how you know.
____________________
____________________
____________________
Answer:
The number on the right is always greater than the number on the left.
Big-Ideas-Math-Book-2nd-Grade-Answer-Key-Chapter-8-Count-Compare-Numbers-to-1,000-Lesson-8.6-Compare-Numbers-Using-Number-Line-Explore-Grow

Show and Grow

Compare
Big Ideas Math Answers Grade 2 Chapter 8 Count and Compare Numbers to 1,000 88

Question 1.
Big Ideas Math Answers Grade 2 Chapter 8 Count and Compare Numbers to 1,000 89
Answer:
527 > 525.

Question 2.
Big Ideas Math Answers Grade 2 Chapter 8 Count and Compare Numbers to 1,000 90
Answer:
521 < 524 .

Question 3.
Big Ideas Math Answers Grade 2 Chapter 8 Count and Compare Numbers to 1,000 91
Answer:
528 = 528 .

Question 4.
Big Ideas Math Answers Grade 2 Chapter 8 Count and Compare Numbers to 1,000 92
Answer:
530 > 520

Question 5.
Big Ideas Math Answers Grade 2 Chapter 8 Count and Compare Numbers to 1,000 93
Answer:
522 < 523

Question 6.
Big Ideas Math Answers Grade 2 Chapter 8 Count and Compare Numbers to 1,000 94
Answer:
529 > 526

Write a number that makes the statement true.

Question 7.
372 < __
Answer:
372 < 373

Question 8.
195 > __
Answer:
195 > 190

Apply and Grow: Practice

Compare.
Big Ideas Math Answers Grade 2 Chapter 8 Count and Compare Numbers to 1,000 95

Question 9.
Big Ideas Math Answers Grade 2 Chapter 8 Count and Compare Numbers to 1,000 96
Answer:
714 = 714

Question 10.
Big Ideas Math Answers Grade 2 Chapter 8 Count and Compare Numbers to 1,000 97
Answer:
720 > 710

Question 11.
Big Ideas Math Answers Grade 2 Chapter 8 Count and Compare Numbers to 1,000 98
Answer:
718 > 717

Question 12.
Big Ideas Math Answers Grade 2 Chapter 8 Count and Compare Numbers to 1,000 99
Answer:
711 < 713

Write a number that makes the statement true.

Question 13.
736 = __
Answer:
736 = 736 .

Question 14.
461 > __
Answer:
461 > 460

Question 15.
__ < 295
Answer:
290 < 295

Question 16.
__ > 573
Answer:
574> 573

Question 17.
Logic
Is Big Ideas Math Answer Key Grade 2 Chapter 8 Count and Compare Numbers to 1,000 100 greater than or less than Big Ideas Math Answer Key Grade 2 Chapter 8 Count and Compare Numbers to 1,000 101? Explain.
Big Ideas Math Answer Key Grade 2 Chapter 8 Count and Compare Numbers to 1,000 102
Answer:
The Big Ideas Math Answer Key Grade 2 Chapter 8 Count and Compare Numbers to 1,000 101 is greater than Big Ideas Math Answer Key Grade 2 Chapter 8 Count and Compare Numbers to 1,000 100.As the values goes towards 100. The Big Ideas Math Answer Key Grade 2 Chapter 8 Count and Compare Numbers to 1,000 101 is close to 100 than Big Ideas Math Answer Key Grade 2 Chapter 8 Count and Compare Numbers to 1,000 100.so Big Ideas Math Answer Key Grade 2 Chapter 8 Count and Compare Numbers to 1,000 101has higher value than Big Ideas Math Answer Key Grade 2 Chapter 8 Count and Compare Numbers to 1,000 100. Big Ideas Math Answer Key Grade 2 Chapter 8 Count and Compare Numbers to 1,000 101 > Big Ideas Math Answer Key Grade 2 Chapter 8 Count and Compare Numbers to 1,000 100.

Question 18.
DIG DEEPER!
What number might Newton be thinking?
Big Ideas Math Answers Grade 2 Chapter 8 Count and Compare Numbers to 1,000 103
Answer:
342<350<356

Think and Grow: Modeling Real Life

Order the race numbers from least to greatest.
Big Ideas Math Solutions Grade 2 Chapter 8 Count and Compare Numbers to 1,000 104
Model:
Big Ideas Math Solutions Grade 2 Chapter 8 Count and Compare Numbers to 1,000 105
Order from least to greatest: __, __, __, __
Your race number is greater than all of the other numbers but less than 900. What is a possible race number for you?
Big Ideas Math Solutions Grade 2 Chapter 8 Count and Compare Numbers to 1,000 105.1
Answer:
Order from least to greatest : 856 , 865 , 868 , 876 .
Possible race number = 878.

Show and Grow

Question 19.
Order the race times from least to greatest.
Big Ideas Math Solutions Grade 2 Chapter 8 Count and Compare Numbers to 1,000 106
DIG DEEPER!
Your time is less than all of the other times but greater than 320 seconds. What is a possible time for you?
Big Ideas Math Solutions Grade 2 Chapter 8 Count and Compare Numbers to 1,000 107
Answer:
The Race times from least to greatest = 329,  335, 340, 342 .
320<329.
Possible time 325

Compare Numbers Using a Number Line Homework & Practice 8.6

Compare.
Big Ideas Math Solutions Grade 2 Chapter 8 Count and Compare Numbers to 1,000 108

Question 1.
Big Ideas Math Solutions Grade 2 Chapter 8 Count and Compare Numbers to 1,000 109
Answer:
450 < 460

Question 2.
Big Ideas Math Solutions Grade 2 Chapter 8 Count and Compare Numbers to 1,000 110
Answer:
459 > 457

Question 3.
Big Ideas Math Solutions Grade 2 Chapter 8 Count and Compare Numbers to 1,000 111
Answer:
455 > 451

Question 4.
Big Ideas Math Solutions Grade 2 Chapter 8 Count and Compare Numbers to 1,000 112
Answer:
456 = 456

Question 5.
Big Ideas Math Solutions Grade 2 Chapter 8 Count and Compare Numbers to 1,000 113
Answer:
455 > 454

Question 6.
Big Ideas Math Solutions Grade 2 Chapter 8 Count and Compare Numbers to 1,000 114
Answer:
452 < 453

Write a number that makes the statement true.

Question 7.
529 > __
Answer:
529 > 528

Question 8.
815 < __
Answer:
815 < 820

Question 9.
__ < 142
Answer:
140 < 142

Question 10.
__ = 364
Answer:
364 = 364

Question 11.
YOU BE THE TEACHER
Is Descartes correct? Explain.
______________
______________
______________
Big Ideas Math Answers Grade 2 Chapter 8 Count and Compare Numbers to 1,000 115
Answer:
986 < 987 .
The numbers which move to the right of the number line will increase in value .
The number on the right is always greater than the number on the left.
so the number 987 is greater than 986 .

Question 12.
DIG DEEPER!
I am not greater than 243. I am not less than 243. What number am I? Explain how you know.
Answer:
243

Question 13.
Modeling Real Life
Order the numbers from least to greatest.
Big Ideas Math Solutions Grade 2 Chapter 8 Count and Compare Numbers to 1,000 116
Answer:
The numbers from least to greatest = 675 , 679, 689, 698 .

DIG DEEPER!
Your car’s number is greater than all of the others but less than 705. What is a possible number for your car?

Review & Refresh

Question 14.
Circle the shapes with flat surfaces that are circles.
Big Ideas Math Solutions Grade 2 Chapter 8 Count and Compare Numbers to 1,000 117

Answer:
Big-Ideas-Math-Book-2nd-Grade-Answer-Key-Chapter-8-Count-Compare-Numbers-to-1,000-Compare-Numbers-Using-Number-Line-Homework-Practice 8.6-Question-14

Count and Compare Numbers to 1,000 Performance Task

The table shows the number of each type of fish in a tank.

Question 1.
Complete the table.
Big Ideas Math Solutions Grade 2 Chapter 8 Count and Compare Numbers to 1,000 118
Answer:
Big-Ideas-Math-Book-2nd-Grade-Answer-Key-Chapter-8-Count-Compare-Numbers-to-1,000-Count-Compare-Numbers-1,000-Performance-Task-Question-1
Question 2.
Compare the numbers of fish.
Big Ideas Math Solutions Grade 2 Chapter 8 Count and Compare Numbers to 1,000 119

Answer:
Big-Ideas-Math-Book-2nd-Grade-Answer-Key-Chapter-8-Count-Compare-Numbers-to-1,000-Count-Compare-Numbers-1,000-Performance-Task-Question-2

Question 3.
All of the purple, green, and pink fish are moved to a new exhibit. How many fish are left in the tank?
Big Ideas Math Answer Key Grade 2 Chapter 8 Count and Compare Numbers to 1,000 120

Total Number of fishes = 351 + 529 + 312 + 458 + 312 = 1962.
Number of fishes moved to new exhibit = 312 + 529 + 351 = 1192
Number of fishes left = 1962 – 1192 = 770.

Question 4.
A school of 24 fish swim in an array. Draw an array for the fish.
Answer:
Big-Ideas-Math-Book-2nd-Grade-Answer-Key-Chapter-8-Count-Compare-Numbers-to-1,000-Count-Compare-Numbers-1,000-Performance-Task-Question-4

Count and Compare Numbers to 1,000 Activity

Number Boss

To Play: Place Number Cards 0–9 in a pile. Each player flips 3 cards and makes a three-digit number. Compare the numbers. The player with the greater number takes both sets of cards. If the numbers are equal, flip cards again. The person with the greater number takes all of the cards. Repeat until all of the cards have been used.

Big Ideas Math Answer Key Grade 2 Chapter 8 Count and Compare Numbers to 1,000 121

Count and Compare Numbers to 1,000 Chapter Practice

8.1 Count to 120 in Different Ways

Question 1.
Count by ones.
113, 114, 115, __, __, __, __, ___
Answer:
113, 114, 115, 116, 117, 118, 119, 120

Question 2.
Count by fives.
25, 30, 35, __, __, __, __, ___
Answer:
25, 30, 35, 40, 45, 50, 55, 60

Question 3.
Count by tens.
33, 43, 53, __, __, __, __, __
Answer:
33, 43, 53, 63, 73, 83, 93, 103

8.2 Count to 1,000 in Different Ways

Question 4.
Count by fives.
210, 215, 220, ___, __, __, __, ___
Answer:
210, 215, 220, 225, 230, 235, 240, 245

Question 5.
Count by tens.
740, 750, 760, __, __, __, __, __, ___
Answer:
740, 750, 760, 770, 780, 790, 800, 810

Question 6.
Count by hundreds.
300, 400, 500, __, __, __, __, __
Answer:
300 , 400, 500, 600, 700, 800, 900, 1000

8.3 Place Value Patterns

Use place value to find the missing numbers.

Question 7.
854, 855, 856, __, __, __, __
Answer:
854, 855, 856, 857, 858, 859, 860

Question 8.
940, 950, 960, __, __, __, __
Answer:
940, 950, 960, 970, 980, 990, 1000

Question 9.
275, 375, 475, ___, __, __, __
Answer:
275, 375, 475, 575, 675, 775, 875

Question 10.
Repeated Reasoning
Use place value to describe each pattern.
Big Ideas Math Answer Key Grade 2 Chapter 8 Count and Compare Numbers to 1,000 122

Answer:
Big-Ideas-Math-Book-2nd-Grade-Answer-Key-Chapter-8-Count-Compare-Numbers-to-1,000-8.3-Place-Value-Patterns-Question-10

8.4 Find More or Less

Question 11.
10 more than 813 is ___
Answer:
10 more than 813 is 823.

Question 12.
10 less than 976 is __
Answer:
10 less than 976 is 966.

Question 13.
100 more than 254 is __.
Answer:
100 more than 254 is 354.

Question 14.
100 less than 531 is __.
Answer:
100 less than 531 is 431.

Question 15.
1 more than 444 is ___
Answer:
1 more than 444 is 445.

Question 16.
1 less than 622 is __.
Answer:
1 less than 622 is 621

Question 17.
Modeling Real Life
Your craft book has 110 ideas. Your friend’s craft book has 10 fewer ideas than yours. How many ideas does your friend’s craft book have?
Big Ideas Math Answer Key Grade 2 Chapter 8 Count and Compare Numbers to 1,000 123
Answer:
Number of Ideas in my craft book = 110 .
Number of ideas in my friends craft book =110 – 10 =100 .

Question 18.
Modeling Real Life
You have 324 beads. Newton has 100 more than you. Descartes has10 fewer than Newton. How many beads does Descartes have?
Big Ideas Math Answer Key Grade 2 Chapter 8 Count and Compare Numbers to 1,000 124
Answer:
Number of beads = 324.
Number of beads newton have = 324 + 100 = 424 .
Number of beads Descartes have = 424 – 10 = 414 .

8.5 Compare Numbers Using Symbols

Compare

Question 19.
Big Ideas Math Answer Key Grade 2 Chapter 8 Count and Compare Numbers to 1,000 125
Answer:
583 = 583

Question 20.
Big Ideas Math Answer Key Grade 2 Chapter 8 Count and Compare Numbers to 1,000 126
Answer:
626 < 725

Question 21.
Big Ideas Math Answer Key Grade 2 Chapter 8 Count and Compare Numbers to 1,000 127
Answer:
932 > 910

Question 22.
Big Ideas Math Answer Key Grade 2 Chapter 8 Count and Compare Numbers to 1,000 128
Answer:
49 < 411

Question 23.
Big Ideas Math Answer Key Grade 2 Chapter 8 Count and Compare Numbers to 1,000 129
Answer:
300 + 40 + 6 = 346
328 <346

Question 24.
Big Ideas Math Answer Key Grade 2 Chapter 8 Count and Compare Numbers to 1,000 130
Answer:
200 + 10 + 8 = 218
280 > 218

Question 25.
There are 318 kids in a gymnastics club. There are 219 kids in a swim club. Which club has fewer kids?
Big Ideas Math Answer Key Grade 2 Chapter 8 Count and Compare Numbers to 1,000 131
Answer:
Number of kids in Gymnastics club = 318.
Number of kids in Swim club =219.
Swim club has 99 fewer kids than Gymnastics club .

8.6 Compare Numbers Using a Number Line

Compare
Big Ideas Math Answer Key Grade 2 Chapter 8 Count and Compare Numbers to 1,000 132

Question 26.
Big Ideas Math Answer Key Grade 2 Chapter 8 Count and Compare Numbers to 1,000 133
Answer:
683 < 687

Question 27.
Big Ideas Math Answer Key Grade 2 Chapter 8 Count and Compare Numbers to 1,000 134
Answer:
689 > 688

Question 28.
Big Ideas Math Answer Key Grade 2 Chapter 8 Count and Compare Numbers to 1,000 135
Answer:
681 = 681

Question 29.
Big Ideas Math Answer Key Grade 2 Chapter 8 Count and Compare Numbers to 1,000 136
Answer:
690 > 680

Write a number that makes the statement true.

Question 30.
324 < __
Answer:
324 < 325

Question 31.
136 > __
Answer:
136 > 133

Question 32.
__ = 750
Answer:
750 = 750

Question 33.
__ < 871
Answer:
771 < 871

Question 34.
Number Sense
What number might Descartes be thinking?
Big Ideas Math Answer Key Grade 2 Chapter 8 Count and Compare Numbers to 1,000 137
Answer:
238<___>325.
238<315>325

Count and Compare Numbers to 1,000 Cumulative Practice 1 – 8

Question 1.
Which equation can you use to check your answer to 32 − 18?
Big Ideas Math Answer Key Grade 2 Chapter 8 Count and Compare Numbers to 1,000 138
Answer:
32 – 18 =14.
Equation
18 + 14 = 32

Question 2.
Which number does not belong?
Big Ideas Math Answer Key Grade 2 Chapter 8 Count and Compare Numbers to 1,000 139

Answer:
324, 334, 344, 354, 364, 374
345 does not belong

Question 3.
Find the missing digits.
Big Ideas Math Answer Key Grade 2 Chapter 8 Count and Compare Numbers to 1,000 140

Answer:
Big-Ideas-Math-Book-2nd-Grade-Answer-Key-Chapter-8-Count-Compare-Numbers-to-1,000-Count-Compare-Numbers-1,000-Cumulative-Practice-1 - 8-Question-3

Question 4.
Use the number cards to decompose to subtract.
Big Ideas Math Answer Key Grade 2 Chapter 8 Count and Compare Numbers to 1,000 141
Answer:
Big-Ideas-Math-Book-2nd-Grade-Answer-Key-Chapter-8-Count-Compare-Numbers-to-1,000-Count-Compare-Numbers-1,000-Cumulative-Practice-1 - 8-Question-4

Question 5.
Which choice does not show 124?
Big Ideas Math Answer Key Grade 2 Chapter 8 Count and Compare Numbers to 1,000 142

Answer:
Big-Ideas-Math-Book-2nd-Grade-Answer-Key-Chapter-8-Count-Compare-Numbers-to-1,000-Count-Compare-Numbers-1,000-Cumulative-Practice-1 - 8-Question-5

Question 6.
Which quick sketch shows 43 − 25?
Big Ideas Math Answer Key Grade 2 Chapter 8 Count and Compare Numbers to 1,000 143
Answer:
43 – 25 = 18
Big-Ideas-Math-Book-2nd-Grade-Answer-Key-Chapter-8-Count-Compare-Numbers-to-1,000-Count-Compare-Numbers-1,000-Cumulative-Practice-1 - 8-Question-6

Question 7.
You have 4 fewer gel pens than your friend. You have 6 gel pens. Which picture shows how many gel pens your friend has?
Big Ideas Math Answer Key Grade 2 Chapter 8 Count and Compare Numbers to 1,000 144

Answer:
Number of gel pens with me = 6.
Number of gel pens with my friend = 6 + 4 = 10.Big-Ideas-Math-Book-2nd-Grade-Answer-Key-Chapter-8-Count-Compare-Numbers-to-1,000-Count-Compare-Numbers-1,000-Cumulative-Practice-1 - 8-Question-7

Question 8.
Which number does not belong?
Big Ideas Math Answer Key Grade 2 Chapter 8 Count and Compare Numbers to 1,000 145
Answer:
563<_____<567 . It can be 564, 565, 566
Big-Ideas-Math-Book-2nd-Grade-Answer-Key-Chapter-8-Count-Compare-Numbers-to-1,000-Count-Compare-Numbers-1,000-Cumulative-Practice-1 - 8-Question-8

Question 9.
Your friend uses compensation to add. Complete the equation to show what numbers he added after using compensation.
Big Ideas Math Answer Key Grade 2 Chapter 8 Count and Compare Numbers to 1,000 146

Answer:
Big-Ideas-Math-Book-2nd-Grade-Answer-Key-Chapter-8-Count-Compare-Numbers-to-1,000-Count-Compare-Numbers-1,000-Cumulative-Practice-1 - 8-Question-9

Question 10.
Which choices show 238?
Big Ideas Math Answer Key Grade 2 Chapter 8 Count and Compare Numbers to 1,000 147
Answer:
Big-Ideas-Math-Book-2nd-Grade-Answer-Key-Chapter-8-Count-Compare-Numbers-to-1,000-Count-Compare-Numbers-1,000-Cumulative-Practice-1 - 8-Question-10

Question 11.
Which picture shows 2 groups of 3?
Big Ideas Math Answer Key Grade 2 Chapter 8 Count and Compare Numbers to 1,000 148
Answer:
Big-Ideas-Math-Book-2nd-Grade-Answer-Key-Chapter-8-Count-Compare-Numbers-to-1,000-Count-Compare-Numbers-1,000-Cumulative-Practice-1 - 8-Question-11

Question 12.
Descartes wants to use addition to subtract 51 − 25. Help him complete the number line and equations.
Big Ideas Math Answer Key Grade 2 Chapter 8 Count and Compare Numbers to 1,000 149

Answer:
Big-Ideas-Math-Book-2nd-Grade-Answer-Key-Chapter-8-Count-Compare-Numbers-to-1,000-Count-Compare-Numbers-1,000-Cumulative-Practice-1 - 8-Question-12

Conclusion:

The above mentioned data about Big Ideas Math Answers Grade 2 Chapter 8 Count and Compare Numbers to 1,000 is helpful for the students. Make use of the given pdf and begin your preparation. If you have any queries regarding BIM Book 2nd Grade 8th Chapter Count and Compare Numbers to 1,000, then please write a comment below.

Big Ideas Math Answers Grade 3 Chapter 4 Division Facts and Strategies

Big Ideas Math Answers Grade 3 Chapter 4

Big Ideas Math Grade 3 Chapter 4 Division Facts and Strategies Solutions in the pdf format is provided here. Students have to be perfect in the basics of mathematics. So, everyone must download free Big ideas Math Book Grade 3 Chapter 4 Division Facts and Strategies Answer Key PDFand start their preparation. This handy and user-friendly answer key provides the answer and detailed solution for each and every problem in BIM 3rd Grade 4th Chapter Textbook. It will help the students while doing assignments and homework.

Big Ideas Math 3rd Grade Chapter 4 Division Facts and Strategies Answer Key

By solving the questions from Big Ideas Math Book Grade 3 Chapter 4 Division Facts and Strategies, you will know how to divide two numbers and different methods to solve it. The different topics covered in the Division Facts and Strategies chapter are Use Arrays to Divide, Relate Multiplication and Division, Divide by 2, 5, or 10, Divide by 3 or 4, Divide by 6 or 7, Divide by 8 or 9, and Divide by 0 or 1.

After solving the division problems from all these lessons, you have practice strategies where you can check your skills. The success criteria of Big Ideas Math Answers Grade 3 Chapter 4 Divison Facts and Strategies is able to compare multiplication and divison, know what is dividend, divisor, and quotient and can explain divison equation in an array. For the perfect preparation, you can tap on the links and download Answers for Big Ideas Math Book Grade 3 Chapter 4 Division Facts and Strategies.

Lesson – 1: Use Arrays to Divide

Lesson – 2: Relate Multiplication and Division

Lesson – 3: Divide by 2, 5, or 10

Lesson – 4: Divide by 3 or 4

Lesson – 5: Divide by 6 or 7

Lesson – 6: Divide by 8 or 9

Lesson – 7: Divide by 0 or 1

Lesson – 8: Practice Division Strategies

Lesson – 9: Problem Solving: Division

Performance Task

Lesson 4.1 Use Arrays to Divide

Explore and Grow

Build an array to model 12. Draw the array.
Number of rows: _______
Number in each row: ______
Answer:
Number of rows: 4
Number in each row: 3
Big Ideas Math Answers Grade 3 Chapter 4 Division Facts & Strategies img_1

Structure
Compare your array with your partner’s array. How are they the same? How are they different?
Answer: The number of counters is the same but the rows and columns are different.

Think and Grow: Division and Arrays

Example
There are 40 counters. The counters are in5 equal rows. How many counters are in each row?
Big Ideas Math Answer Key Grade 3 Chapter 4 Division Facts and Strategies 4.1 1
There are ______ counters in each row.

Answer: 8

Explanation:
Number of rows = 5
Total number of counters = 40
So, the division equation is 40 ÷ 5 = 8
There are 8 counters in each row.

Show and Grow

Find the quotient.
Question 1.
Big Ideas Math Answer Key Grade 3 Chapter 4 Division Facts and Strategies 4.1 2
14 ÷ 2 = _____
Answer: 7

Explanation:
Number of counters = 14
Number of rows = 2
To find the quotient you have to divide the number of counters by the number of rows.
14 ÷ 2 = 7
Thus the number of columns in each row = 7

Question 2.
Big Ideas Math Answer Key Grade 3 Chapter 4 Division Facts and Strategies 4.1 3
12 ÷ 3 = ______
Answer: 4

Explanation:
Number of counters = 12
Number of rows = 3
To find the quotient you have to divide the number of counters by the number of rows.
12 ÷ 3 = 4
Thus the number of columns in each row = 4

Question 3.
There are 20 counters. The counters are in 4 equal rows. How many counters are in each row?
4 rows of _____
20 ÷ 4 = _____
Answer: 5

Explanation:
Given,
There are 20 counters. The counters are in 4 equal rows.
To find the quotient you have to divide the number of counters by the number of rows.
20 ÷ 4 = 5
4 rows of 5
There are 5 counters in each row.

Question 4.
You have 21 counters. You arrange them with 7 counters in each row. How many rows of counters do you make?
____ rows of 7
21 ÷ 7 = _____
Answer: 3

Explanation:
Given,
You have 21 counters. You arrange them with 7 counters in each row.
21 ÷ 7 = 3
3 rows 7
Thus you make 3 rows of 7 counters.

Apply and Grow: Practice

Question 5.
There are 25 counters. The counters are in 5 equal rows. How many counters are in each row?
5 rows of _____
25 ÷ 5 = _____
Answer: 5

Explanation:
Given,
There are 25 counters.
The counters are in 5 equal rows.
To find the quotient you have to divide the number of counters by the number of rows.
25 ÷ 5 = 5
There are 5 counters in each row.

Question 6.
There are 48 counters. The counters are in 8 equal rows. How many counters are in each row?
8 rows of _____
48 ÷ 8 = _____
Answer: 6

Explanation:
Given,
There are 48 counters. The counters are in 8 equal rows.
To find the quotient you have to divide the number of counters by the number of rows.
48 ÷ 8 = 6
8 rows of 6 counters
There are 6 counters in each row.

Question 7.
You have 42 counters. You arrange them with 6 counters in each row. How many rows of counters do you make?
____ rows of 6
42 ÷ 6 = _____
Answer: 7

Explanation:
Given,
You have 42 counters. You arrange them with 6 counters in each row.
7 rows of 6
42 ÷ 6 = 7
There are 7 rows of counters.

Question 8.
You have 27 counters. You arrange them with 9 counters in each row. How many rows of counters do you make?
____ rows of 9
27 ÷ 9 = _____
Answer: 3

Explanation:
Given that,
You have 27 counters. You arrange them with 9 counters in each row.
27 ÷ 9 = 3
3 rows of 9.
There are 3 rows of counters.

Write a division equation for the array.
Question 9.
Big Ideas Math Answer Key Grade 3 Chapter 4 Division Facts and Strategies 4.1 4
_____ ÷ _____ = _____
Answer: 4 ÷ 2 = 2

Explanation:
There are 4 rows and 2 counters.
The division equation for the array is 4 ÷ 2 = 2

Question 10.
Big Ideas Math Answer Key Grade 3 Chapter 4 Division Facts and Strategies 4.1 5
____ ÷ _____ = _____
Answer: 40 ÷ 4 = 10

Explanation:
There are 4 rows and 40 counters.
The division equation for the array is 40 ÷ 4 = 10

Question 11.
YOU BE THE TEACHER
Your friend has 63 counters in 7 equal rows. Your friend says that finding 7 ÷ 63 will give the number of columns. Is your friend correct? Explain.
Answer: No your friend is incorrect.

Explanation:
Given,
Your friend has 63 counters in 7 equal rows.
Your friend says that finding 7 ÷ 63 will give the number of columns.
Your friend is incorrect because you have to divide the number of counters by a number of equal rows.
63 ÷ 7 = 9
There are 9 columns.

Think and Grow: Modeling Real Life

Two groups of students are playing a flip and find game. Your group arranges 48 cards in 8 equal rows. The other group arranges 28 cards in 4 equal rows. Which group has rows with more cards?
Draw:
Division equations:
_______ has rows with more cards.

Explanation:
Given,
Two groups of students are playing a flip and find game.
Your group arranges 48 cards in 8 equal rows.
48 ÷ 8 = 6 in each row
The other group arranges 28 cards in 4 equal rows.
28 ÷ 4 = 7 in each row.
The second group has 7 rows with more cards.

Show and Grow

Question 12.
Newton arranges 12 magnets in 6 equal rows. Descartes arranges 15 magnets in 3 equal rows. Who has rows with more magnets?
Big Ideas Math Answer Key Grade 3 Chapter 4 Division Facts and Strategies 4.1 6
Answer:
Given,
Newton arranges 12 magnets in 6 equal rows.
12 ÷ 6 = 2
Descartes arranges 15 magnets in 3 equal rows.
15 ÷ 3 = 5
Thus Descartes has 5 rows with more magnets.

Question 13.
You have 18 jars of green slime and 12 jars of purple slime at a party. Each guest takes 3 jars. There are none left. How many guests are at the party?
Big Ideas Math Answer Key Grade 3 Chapter 4 Division Facts and Strategies 4.1 7
Answer:
Given that,
You have 18 jars of green slime and 12 jars of purple slime at a party.
18 + 12 = 30 jars
Each guest takes 3 jars. There are none left.
30 ÷ 3 = 10
Thus there are 10 guests at the party.

Use Arrays to Divide Homework & Practice 4.1

Find the quotient.
Question 1.
Big Ideas Math Answer Key Grade 3 Chapter 4 Division Facts and Strategies 4.1 8
10 ÷ 2 = _____
Answer: 5

Explanation:
Number of counters = 10
Number of rows = 2
To find the quotient you have to divide the number of counters by the number of rows.
10 ÷ 2 = 5
Thus there are 5 counters in each row.

Question 2.
Big Ideas Math Answer Key Grade 3 Chapter 4 Division Facts and Strategies 4.1 9
35 ÷ 5 = _____
Answer: 7

Explanation:
Number of counters = 35
Number of rows = 5
To find the quotient you have to divide the number of counters by the number of rows.
35 ÷ 5 = 7
Thus there are 7 counters in each row.

Question 3.
There are 9 counters. The counters are in 3 equal rows. How many counters are in each row?
3 rows of ____
9 ÷ 3 = ____
Answer: 3

Explanation:
Given that,
There are 9 counters. The counters are in 3 equal rows.
To find the quotient you have to divide the number of counters by the number of rows.
9 ÷ 3 = 3
3 rows of 3
Thus there are 3 counters in each row.

Question 4.
There are 60 counters. The counters are in 6 equal rows. How many counters are in each row?
6 rows of _____
60 ÷ 6 = ____
Answer: 10.

Explanation:
Given,
There are 60 counters.
The counters are in 6 equal rows.
To find the number of counters in each row you have to divide the total number of counters by the number of rows.
60 ÷ 6 = 10
6 rows of 10 counters
Thus there are 10 counters in each row.

Question 5.
You have 72 counters. You arrange them with 8 counters in each row. How many rows of counters do you make?
____ rows of 8
72 ÷ 8 = ____
Answer: 9

Explanation:
Given,
You have 72 counters. You arrange them with 8 counters in each row.
To find the number of counters in each row you have to divide the total number of counters by the number of rows.
72 ÷ 8 = 9
9 rows of 8.
Thus there are 9 counters in each row.

Question 6.
You have 24 counters. You arrange them with 6 counters in each row. How many rows of counters do you make?
____ rows of 6
24 ÷ 6 = ____
Answer: 4

Explanation:
Given,
You have 24 counters. You arrange them with 6 counters in each row.
24 ÷ 6 = 4
4 rows of 6
Thus there are 4 counters in each row.

Question 7.
Writing
How can you use an array to find 35 ÷ 7?
Answer: 5

Explanation:
Number of counters = 35
Number of rows = 7
To find the number of counters in each row you have to divide the total number of counters by the number of rows.
35 ÷ 7 = 5

Question 8.
Precision
Label the parts of the division problem using quotient, dividend and divisor.
Big Ideas Math Answer Key Grade 3 Chapter 4 Division Facts and Strategies 4.1 10
Answer:
12 is dividend
6 is divisor and
2 is quotient

Question 9.
Modeling Real Life
Your apartment building has 40 mailboxes in 5 equal rows. Your friend’s apartment building has 40 mailboxes in 4 equal rows. Which apartment building has rows with more mailboxes?
Answer: Your friend’s apartment building 10 rows with more mailboxes

Explanation:
Given,
Your apartment building has 40 mailboxes in 5 equal rows.
40 ÷ 5 = 8
Your friend’s apartment building has 40 mailboxes in 4 equal rows.
40 ÷ 4 = 10
Thus Your friend’s apartment building 10 rows with more mailboxes.

Question 10.
Modeling Real Life
There are 8 snack bags of pretzels and 10 snack bags of popcorn. The bags are divided equally between 9 friends. How many snack bags does each friend get?
Big Ideas Math Answer Key Grade 3 Chapter 4 Division Facts and Strategies 4.1 11
Answer:
Given,
There are 8 snack bags of pretzels and 10 snack bags of popcorn.
8 + 10 = 18 snack bags
The bags are divided equally between 9 friends.
18 ÷ 9 = 2
Thus each friend gets 2 snack bags.

Review & Refresh

Find the missing factor
Question 11.
____ × 2 = 20
Answer: 10

Explanation:
Let the missing factor be a.
a × 2 = 20
a = 20/2
a = 10
Thus the missing factor is 10.

Question 12.
1 × ____ = 5
Answer: 5

Explanation:
Let the missing factor be b.
1 × b = 5
b = 5/1
b = 5
Thus the missing factor is 5.

Question 13.
____ × 6 = 0
Answer: 0

Explanation:
Let the missing factor be c.
c × 6 = 0
c = 0/6
c = 0
Thus the missing factor is 0.

Question 14.
1 × ___ = 0
Answer: 0

Explanation:
Let the missing factor be d.
1 × d = 0
d = 0/1
d = 0
Thus the missing factor is 0.

Question 15.
____ × 4 = 40
Answer: 10

Explanation:
Let the missing factor be e.
e × 4 = 40
e = 40/4
e = 10
Thus the missing factor is 10.

Question 16.
7 × ____ = 7
Answer: 1

Explanation:
Let the missing factor be f.
7 × f = 7
f = 7/7
f = 1
Thus the missing factor is 1.

Lesson 4.2 Relate Multiplication and Division

Explore and Grow

Use 24 counters to make an array. Draw the array. Write a multiplication equation and a division equation for the array.
____ × ____ = _____
_____ ÷ ____ = ____
Answer: 6 × 4 = 24
24 ÷ 6 = 4

Explanation:
Big Ideas Math Answers Grade 3 Chapter 4 Division Facts & Strategies img_2
The division equation is 24 ÷ 6 = 4 or 24 ÷ 4 = 6

Structure
Compare your equations to your partner’s equations. How are they the same? How are they different?
Answer:
Both the equations are the same but the rows and columns are different.

Think and Grow: Multiplication and Division

A fact family is a group of related facts that uses the same numbers.
Big Ideas Math Answers 3rd Grade Chapter 4 Division Facts and Strategies 4.2 1

Answer:
Multiplication:
5 rows of 6 counters
5 × 6 = 30
30 counters
Division:
30 counters in 5 equal rows
30 ÷ 5 = 6
60 counters in each row.
Fact family for 5, 6 and 30:
5 × 6 = 30
6 × 5 = 30
30 ÷ 5 = 6
30 ÷ 6 = 5

Show and Grow

Use the array to complete the equations.
Question 1.
Big Ideas Math Answers 3rd Grade Chapter 4 Division Facts and Strategies 4.2 2
3 × ____ = 24
24 ÷ 3 = ____
Answer: 8

Explanation:
Number of rows = 3
Number of counters = 24
To find the number of counters in each row you have to divide the total number of counters by the number of rows.
24 ÷ 3 = 8
3 × x = 24
x = 24/3
x = 8

Question 2.
Big Ideas Math Answers 3rd Grade Chapter 4 Division Facts and Strategies 4.2 3
4 × ____ = 20
20 ÷ 4 = _____
Answer: 5

Explanation:
Number of rows = 4
Number of counters = 20
To find the number of counters in each row you have to divide the total number of counters by the number of rows.
4 × x = 20
x = 20/4
x = 5
20 ÷ 4 = 5

Question 3.
Draw an array to find 2 × 7. Write the other facts in the fact family.
2 × 7 = _____
___________
___________
___________
Answer: 14

Explanation:
BIM 3rd Grade Answers Chapter 4 Division Facts & Strategies img_3
The fact family for the numbers 2, 7 and 14
2 × 7 = 14
7 × 2 = 14
14 ÷ 2 = 7
14 ÷ 7 = 2

Apply and Grow: Practice

Question 4.
Draw an array to find 3 × 6.
Write the other 3 facts in the fact family.
3 × 6 = ______
____________
_____________
____________
Answer: 18

Explanation:
Big Ideas Math Answer Key Grade 3 Chapter 4 Division Facts & Strategies img_4
The fact family for 3, 6 and 18 are
3 × 6 = 18
6 × 3 = 18
18 ÷ 3 = 6
18 ÷ 6 = 3

Complete the fact family.
Question 5.
7 × ____ = 70
____ × 7 = 70
70 ÷ 10 = ____
70 ÷ ___ = 10
Answer:
The fact family for 7, 10 and 70 are:
7 × 10 = 70
10 × 7 = 70
70 ÷ 10 = 7
70 ÷ 7 = 10

Question 6.
5 × ____ = 40
____ × 5 = 40
40 ÷ 8 = _____
40 ÷ ____ = 8
Answer:
The fact family for 5, 8 and 40 are:
5 × 8 = 40
8 × 5 = 40
40 ÷ 8 = 5
40 ÷ 5 = 8

Write the fact family for the numbers.
Question 7.
2, 5, 10
Answer:
The fact family for 2, 5 and 10 are:
2 × 5 = 10
5 × 2 = 10
10 ÷ 2 = 5
10 ÷ 5 = 2

Question 8.
4, 3, 12
Answer:
The fact family for 3, 4 and 12 are:
4 × 3 = 12
3 × 4 = 12
12 ÷ 3 = 4
12 ÷ 4 = 3

Question 9.
Structure
Find each product. Then match the multiplication fact with the related division fact.
Big Ideas Math Answers 3rd Grade Chapter 4 Division Facts and Strategies 4.2 4
Answer:
Big-Ideas-Math-Answers-3rd-Grade-Chapter-4-Division-Facts-and-Strategies-4.2-4

Question 10.
DIG DEEPER!
Is 4 × 6 = 24 part of the fact family for 3 × 8 = 24? Explain.
Answer: No. Because the rows and columns are different.

Think and Grow: Modeling Real Life

Your teacher divides the items shown equally among 9 students. Write two equations that you can use to show how many straws each student gets.
Big Ideas Math Answers 3rd Grade Chapter 4 Division Facts and Strategies 4.2 5
Division equation:
Multiplication equation:

Answer:
Your teacher divides the items shown equally among 9 students.
Straws – 54
54 ÷ 9 = 6
Thus each student gets 6 straws.
Division Equation is 54 ÷ 9 = 6
Multiplication equation 9 × 6 = 54

Show and Grow

Question 11.
Use the table above to write two equations that you can use to show how many containers of clay each student gets.
Answer:
Your teacher divides the items shown equally among 9 students.
Containers of the day – 27
27 ÷ 9 = 3
Division Equation is 27 ÷ 9 = 3
Multiplication equation is 3 × 9 = 27

Question 12.
Use the table above to find how many more toothpicks students will get than straws.
Big Ideas Math Answers 3rd Grade Chapter 4 Division Facts and Strategies 4.2 6
Answer:
Your teacher divides the items shown equally among 9 students.
Toothpicks – 72
72 ÷ 9 = 8
Each student gets 8 toothpicks
Straws – 54
54 ÷ 9 = 6
Thus each student gets 6 straws.
8 – 6 = 2
Thus Each student will get 2 toothpicks more than the straws.

Question 13.
Explain how a multiplication fact can help you solve 30 ÷ 3 = _____.
Answer: 10 × 3 = 30

Explanation:
The multiplication fact to help you to solve 30 ÷ 3 is 10 × 3.
By using the multiplication fact you can get the solution for 30 ÷ 3 = 10

Relate Multiplication and Division Homework & Practice 4.2

Use the array to complete the equations
Question 1.
Big Ideas Math Answers 3rd Grade Chapter 4 Division Facts and Strategies 4.2 7
2 × ___ = 8
8 ÷ 2 = _____
Answer: 4

Explanation:
Number of counters = 8
Number of rows = 2
8 ÷ 2 = 4
2 × x = 8
x = 8/2
x = 4

Question 2.
Big Ideas Math Answers 3rd Grade Chapter 4 Division Facts and Strategies 4.2 8
3 × ____ = 9
9 ÷ 3 = ____
Answer: 3

Explanation:
Number of counters = 9
Number of rows  = 3
3 × x = 9
x = 9/3
x = 3
9 ÷ 3 = 3

Question 3.
Draw an array to find 5 × 7. Write the other 3 facts in the fact family.
5 × 7 = _____
___________
___________
___________
Answer: 35

Explanation:
The other 3 facts are
5 × 7 = 35
7 × 5 = 35
35 ÷ 5 = 7
35 ÷ 7 = 5

Complete the fact family
Question 4.
Big Ideas Math Answers 3rd Grade Chapter 4 Division Facts and Strategies 4.2 9
Answer: 1, 9

Explanation:
9 × 1 = 9
9 ÷ 1 = 9
1 × 9 = 9
9 ÷ 9 = 1

Question 5.
Big Ideas Math Answers 3rd Grade Chapter 4 Division Facts and Strategies 4.2 10
Answer: 7, 6

Explanation:
6 × 7 = 42
42 ÷ 7 = 6
7 × 6 = 42
42 ÷ 6 = 7

Write the fact family for the numbers.
Question 6.
4, 8, 32
Answer: 8 × 4 = 32, 32 ÷ 4 = 8, 32 ÷ 8 = 4

Explanation:
The facts family for the numbers are
4 × 8 = 32
8 × 4 = 32
32 ÷ 8 = 4
32 ÷ 4 = 8

Question 7.
6, 7, 42
Answer: 6 × 7 = 42, 42 ÷ 6 = 7, 42 ÷ 7 = 6

Explanation:
The facts family for the numbers are
6 × 7 = 42
7 × 6 = 42
42 ÷ 7 = 6
42 ÷ 6 = 7

Question 8.
Which One Doesn’t Belong?
Which equation does not belong with the other three?
3 × 7 = 21
7 × 3 = 21
21 ÷ 7 = 3
7 + 3 = 10
Answer: 7 + 3 = 10

Explanation:
The equation that does not belong to other three equation is 7 + 3 = 10.

Question 9.
DIG DEEPER!
Newton has 16 pennies. He wants to put them in stacks that are the same height. How many stacks does he make?
Big Ideas Math Answers 3rd Grade Chapter 4 Division Facts and Strategies 4.2 11
Answer: 2

Explanation:
Given,
Newton has 16 pennies. He wants to put them in stacks that are the same height.
16 ÷ 8 = 2
Thus he makes 2 stacks.

Question 10.
Modeling Real Life
Your art teacher divides the items shown equally among 6 students. Write two equations that you can use to show how many pieces of paper each student gets.
Big Ideas Math Answers 3rd Grade Chapter 4 Division Facts and Strategies 4.2 12
Answer:
Given
Your art teacher divides the items shown equally among 6 students.
48 ÷ 6 = 8 paintbrushes
42 ÷ 6 = 7 pieces of paper
Thus each student gets 7 pieces of paper.

Question 11.
Modeling Real Life
Use the table above to find how many more paintbrushes students will get than paint trays.
Answer:
Your art teacher divides the items shown equally among 6 students.
18 ÷ 6 = 3 paint trays
48 ÷ 6 = 8 paintbrushes
8 – 3 = 5
Thus the students will get 5 more paintbrushes than paint trays.

Review & Refresh

Find the missing factor.
Question 12.
5 × ____ = 45
Answer: 9

Explanation:

Let the missing factor be g.
5 × g = 45
g = 45/5
g = 9
Thus the missing factor is 9.

Question 13.
2 × ____ = 16
Answer: 8

Explanation:
Let the missing factor be h.
2 × h = 16
h = 16/2
h = 8
Thus the missing factor is 8.

Question 14.
_____ × 2 = 4
Answer: 2

Explanation:
Let the missing factor be i.
i × 2 = 4
i = 4/2
i = 2
Thus the missing factor is 2.

Question 15.
____ × 3 = 15
Answer: 5

Explanation:
Let the missing factor be j.
j × 3 = 15
j = 15/3
j = 5
Thus the missing factor is 5.

Question 16.
5 × ____ = 10
Answer: 2

Explanation:
Let the missing factor be k.
5 × k = 10
k = 10/5
k = 2
Thus the missing factor is 2.

Question 17.
5 × ____ = 5
Answer: 1

Explanation:
Let the missing factor be l.
5 × l = 5
l = 5/5
l = 1
Thus the missing factor is 1.

Lesson 4.3 Divide by 2, 5, or 10

Explore and Grow

Use the number line to model 10 ÷ 2.
Big Ideas Math Answers Grade 3 Chapter 4 Division Facts and Strategies 4.3 1
10 ÷ 2 = _____
Answer: 5

Structure
In your number line model, does 2 represent the number of equal groups or the size of the groups? Explain.
Answer: Yes, the number line model 2 represents the number of equal groups or the size of the groups.

Think and Grow: Divide by 2, 5, or 10

Example
Find 16 ÷ 2.
Big Ideas Math Answers Grade 3 Chapter 4 Division Facts and Strategies 4.3 2

Answer:
Number of rows = 2
Number of counters = 16
Division equation is 16 ÷ 2 = 8
Multiplication equation is 2 × 8 = 16

Example
Find 20 ÷ 5.
Think: 5 times what number is 20?
5 × ____ = 20
Big Ideas Math Answers Grade 3 Chapter 4 Division Facts and Strategies 4.3 3

Answer:
Number of rows = 5
Number of counters = 20
Division equation is 20 ÷ 5 = 4
Multiplication equation is 5 × 4 = 20

Example
Find 30 ÷ 10.
Think: 10 times what number is 30?
10 × _____ = 30
Big Ideas Math Answers Grade 3 Chapter 4 Division Facts and Strategies 4.3 4

Answer:
Number of rows = 10
Number of counters = 30
Division equation is 30 ÷ 10 = 3
Multiplication equation is 10 × 3 = 30

Show and Grow

Write the related multiplication fact. Then find the quotient
Question 1.
Find 60 ÷ 10.
10 × ____ = 60
60 ÷ 10 = ____
Answer: 6, 6

Explanation:
The related multiplication fact for 60 and 10 is
10 × x = 60
x = 60/10
x = 6
60 ÷ 10 = 6
Thus the quotient is 6.

Question 2.
Find 14 ÷ 2
2 × ____ = 14
14 ÷ 2 = _____
Answer: 7, 7

Explanation:
The related multiplication fact for 14 and 2 is
2 × x = 14
x = 14/2
x = 7
14 ÷ 2 = 7
Thus the quotient is 7.

Question 3.
Find 35 ÷ 5
5 × ___ = 35
35 ÷ 5 = ____
Answer: 7, 7

Explanation:
The related multiplication fact for 5 and 35 is
5 × x = 35
x = 35/5
x = 7
35 ÷ 5 = 7
Thus the quotient is 7.

Write the related multiplication fact. Then find the quotient
Question 4.
Find 4 ÷ 2.
2 × ____ = 4
4 ÷ 2 = ____
Answer: 2, 2

Explanation:
The related multiplication fact for 2 and 4 is
2 × x = 4
x = 4/2
x = 2
4 ÷ 2 = 2
Thus the quotient is 2.

Question 5.
Find 15 ÷ 5.
5 × ____ = 15
15 ÷ 5 = ____
Answer: 3, 3

Explanation:
The related multiplication fact for 5 and 15 is
5 × x = 15
x = 15/5
x = 3
15 ÷ 5 = 3
Thus the quotient is 3.

Question 6.
Find 10 ÷ 10.
10 × ____ = 10
10 ÷ 10 = ____
Answer: 1, 1

Explanation:
The related multiplication fact for 10 and 10 is
10 × x = 10
x = 10/10
x = 1
10 ÷ 10 = 1
Thus the quotient is 1.

Find the quotient
Question 7.
70 ÷ 10 = _____
Answer: 7
Divide both the numbers 70 and 10
70/10 = 7
Thus 7 is the quotient.

Question 8.
25 ÷ 5 = _____
Answer: 5
Divide both the numbers 25 and 5.
25/5 = 5
Thus 5 is the quotient.

Question 9.
18 ÷ 2 = ____
Answer: 6
Divide both the numbers 18 and 2.
18/2 = 9
Thus 9 is the quotient.

Question 10.
Big Ideas Math Answers Grade 3 Chapter 4 Division Facts and Strategies 4.3 5
Answer: 6
Divide both the numbers 30 and 5.
30/5 = 6
Thus 6 is the quotient.

Question 11.
Big Ideas Math Answers Grade 3 Chapter 4 Division Facts and Strategies 4.3 6
Answer: 8
Divide both the numbers 16 and 2.
16/2 = 8
Thus 8 is the quotient.

Question 12.
Big Ideas Math Answers Grade 3 Chapter 4 Division Facts and Strategies 4.3 7
Answer: 9
Divide both the numbers 10 and 90.
90/10 = 9
Thus 9 is the quotient.

Question 13.
Divide 60 by 10.
Answer: 6
Divide both the numbers 60 and 10.
60/10 = 6
Thus 6 is the quotient.

Question 14.
Divide 14 by 2.
Answer: 7
Divide both the numbers 2 and 14.
14/2 = 7
Thus 7 is the quotient.

Question 15.
Divide 45 by 5.
Answer: 9
Divide both the numbers 45 and 5.
45/5 = 9
Thus 9 is the quotient.

Find the missing divisor.
Question 16.
12 ÷ ____ = 6
Answer: 2

Explanation:
Let the missing divisor is x
12 ÷ x = 6
12/x = 6
12 = 6 × x
x = 12/6
x = 2
Thus the missing divisor is 2.

Question 17.
10 ÷ ____ = 2
Answer: 5

Explanation:
Let the missing divisor is y
10 ÷ y = 2
10/y = 2
10 = 2 × y
y = 10/2
y = 5
Thus the missing divisor is 5.

Question 18.
50 ÷ ____ = 5
Answer: 10

Explanation:
Let the missing divisor is z.
50 ÷ z = 5
50/z = 5
50 = 5 × z
z = 50/5
z = 10
Thus the missing divisor is 10.

Question 19.
You make 2 batches of pancakes. You use 6 cups of flour. How many cups of flour are in 1 batch?
Big Ideas Math Answers Grade 3 Chapter 4 Division Facts and Strategies 4.3 8
Answer: 3

Explanation:
Given that,
You make 2 batches of pancakes.
You use 6 cups of flour.
6/2 = 3 cups of flour
There are 3 cups of flour in 1 batch.

Question 20.
DIG DEEPER!
I am an even number. If you multiply me by 5, then divide the product by 10, the quotient is 2. What number am I?
Answer: 20

Explanation:
I am an even number. If you multiply me by 5, then divide the product by 10, the quotient is 2.
Let 4 be the even number.
4 × 5 = 20
20/10 = 2
Thus the number is 4.

Think and Grow: Modeling Real Life

Fourteen students say a dog is their favorite pet. How many symbols should you draw to complete the picture graph?
Big Ideas Math Answers Grade 3 Chapter 4 Division Facts and Strategies 4.3 9
Division equation:
You should draw _____ symbols

Answer:
Given,
Fourteen students say a dog is their favorite pet.
Big-Ideas-Math-Answers-Grade-3-Chapter-4-Division-Facts-and-Strategies-4.3-9
🙂 = 2 students
14 ÷ 2 = 7 pictures

Show and Grow

Question 21.
Twenty-five students say riding a bike is their favorite summer activity. How many symbols should you draw to complete the picture graph?
Big Ideas Math Answers Grade 3 Chapter 4 Division Facts and Strategies 4.3 10
Answer:
Given,
Twenty-five students say riding a bike is their favorite summer activity.
🙂 = 5 students
25 = 🙂 🙂🙂🙂🙂
25 ÷ 5 = 5

Question 22.
Erasers cost 10¢ each. How many erasers can you buy with 70¢?
Answer:
Given that,
Erasers cost 10¢ each.
You need to buy with 70¢
10 × x = 70
x = 70/10
x = 7
Therefore you can buy 7 erasers for 70¢.

Question 23.
You have 26 red linking cubes and 24 blue linking cubes. You use all of the linking cubes to make towers with 10 linking cubes each. How many towers do you make?
Answer: 5

Explanation:
Given that,
You have 26 red linking cubes and 24 blue linking cubes.
26 + 24 = 50 linking cubes
You use all of the linking cubes to make towers with 10 linking cubes each.
50 ÷ 10 = 5 towers
Thus you can make 5 towers.

Divide by 2, 5, or 10 Homework & Practice 4.3

Write the related multiplication fact. Then find the quotient.
Question 1.
Find 20 ÷ 10
10 × ____ = 20
20 ÷ 10 = ____
Answer: 2, 2

Explanation:
Let x be the unknown factor.
10 × x  = 20
x = 20/10
x = 2
First, we need to check whether the divisor or dividend is the related multiplication fact or not.
Next check whether the divisor or the dividend the product in the related multiplication fact or not.
If both are the same then the quotient is the unknown factor.
20/10 = 2
Thus the quotient is 2.

Question 2.
Find 8 ÷ 2.
2 × ___ = 8
8 ÷ 2 = ____
Answer: 4, 4

Explanation:
Let y be the unknown factor.
First, we need to check whether the divisor or dividend is the related multiplication fact or not.
Next check whether the divisor or the dividend the product in the related multiplication fact or not.
If both are the same then the quotient is the unknown factor.
2 × y = 8
y = 8/2
y = 4
Thus the quotient is 4.

Question 3.
Find 50 ÷ 5
5 × ___ = 50
50 ÷ 5 = ____
Answer: 10, 10

Explanation:
Let z be the unknown factor.
First, we need to check whether the divisor or dividend is the related multiplication fact or not.
Next check whether the divisor or the dividend the product in the related multiplication fact or not.
If both are the same then the quotient is the unknown factor.
5 × z = 50
z = 50/5
z = 10
Thus the quotient is 10.

Find the quotient
Question 4.
100 ÷ 10 = _____
Answer: 10

Explanation:
100/10 = 10
The quotient is 10.

Question 5.
45 ÷ 5 = _____
Answer: 9

Explanation:
45/5 = 9
The quotient is 9.

Question 6.
14 ÷ 2 = _____
Answer: 7

Explanation:
14/2 = 7
The quotient is 7.

Question 7.
Big Ideas Math Answers Grade 3 Chapter 4 Division Facts and Strategies 4.3 11
Answer: 6

Explanation:
12/2 = 6
The quotient is 6.

Question 8.
Big Ideas Math Answers Grade 3 Chapter 4 Division Facts and Strategies 4.3 12
Answer: 6

Explanation:
60/10 = 6
The quotient is 6.

Question 9.
Big Ideas Math Answers Grade 3 Chapter 4 Division Facts and Strategies 4.3 13
Answer: 2

Explanation:
10/5 = 2
The quotient is 2.

Find the missing divisor
Question 10.
20 ÷ ___ = 5
Answer: 4

Explanation:
Let x be the missing divisor
20 ÷ x = 5
20/x = 5
20 = x × 5
x = 20/5
x = 4
Thus the missing divisor is 4.

Question 11.
40 ÷ ____ = 10
Answer: 4

Explanation:
Let x be the missing divisor.
40 ÷ x = 10
40/x = 10
40 = 10 × x
x = 40/10
x = 4
Thus the missing divisor is 4.

Question 12.
4 ÷ ____ = 2
Answer: 2

Explanation:
Let x be the missing divisor.
4 ÷ x = 2
4/x = 2
4 = 2 × x
x = 4/2
x = 2
Thus the missing divisor is 2.

Question 13.
Number Sense
The American flag has 50 stars. It has 10 times as many stars as the Chinese flag. How many stars are on the Chinese flag?
Answer:
Given,
The American flag has 50 stars. It has 10 times as many stars as the Chinese flag.
50/10 = 5
Thus there are 5 stars on the Chinese flag.

Question 14.
Open-Ended Write a division equation for each description.
Big Ideas Math Answers Grade 3 Chapter 4 Division Facts and Strategies 4.3 14
Answer: 10 ÷ 2 = 5
In this equation 2 is the divisor, 5 is the quotient and 10 is the dividend.

Question 15.
Repeated Reasoning
Complete the table.
Big Ideas Math Answers Grade 3 Chapter 4 Division Facts and Strategies 4.3 15
Answer:
Big-Ideas-Math-Answers-Grade-3-Chapter-4-Division-Facts-and-Strategies-4.3-15

Question 16.
You buy 20 flowers. You want an equal number of flowers in each of the 5 pots. How many flowers do you put in each pot?
Big Ideas Math Answers Grade 3 Chapter 4 Division Facts and Strategies 4.3 16
Answer:
Given,
You buy 20 flowers. You want an equal number of flowers in each of the 5 pots.
20/5 = 4
Thus there are 4 flowers in each pot.

Question 17.
Modeling Real Life
Fifty students say they have traveled on an airplane. How many symbols should you draw to complete the picture graph?
Big Ideas Math Answers Grade 3 Chapter 4 Division Facts and Strategies 4.3 17
Answer:
Given that,
Fifty students say they have traveled on an airplane.
🙂 = 10 students
50 = 10 × 5
= 🙂 × 5
Big-Ideas-Math-Answers-Grade-3-Chapter-4-Division-Facts-and-Strategies-4.3-17

Question 18.
Modeling Real Life
A jeweler has 17 gold rings and 18 silver rings. She puts them in a ring tray in 5 rows. How many rings are in each row?
Answer:  7 rings

Explanation:
Given that,
A jeweler has 17 gold rings and 18 silver rings.
17 + 18 = 35
She puts them in a ring tray in 5 rows.
35/5 = 7
Therefore there are 7 rings in each row.

Review & Refresh

Compare.
Question 19.
Big Ideas Math Answers Grade 3 Chapter 4 Division Facts and Strategies 4.3 18
Answer: >

Explanation:
8 × 3 = 24
7 × 3 = 21
24 > 21
So, 8 × 3 > 7 × 3

Question 20.
Big Ideas Math Answers Grade 3 Chapter 4 Division Facts and Strategies 4.3 19
Answer: =

Explanation:
5 × 3 = 15
15 = 15
So, 15 = 5 × 3

Question 21.
Big Ideas Math Answers Grade 3 Chapter 4 Division Facts and Strategies 4.3 20
Answer: <

Explanation:
3 × 3 = 9
4 × 3 = 12
9 < 12
So, 3 × 3 < 4 × 3

Lesson 4.4 Divide by 3 or 4

Explore and Grow

Put 12 counters into 3 equal groups. Draw to show your groups.
Use your equal groups to help you find the quotient.
12 ÷ 3 = ____
Answer: 4
BIM Grade 3 Chapter 4 Division Facts & Strategies img_4

Structure
Put 12 counters into 4 equal groups. Draw to show your groups. Write a division equation to match. What do you notice?
Answer: 3
Bigideas Math Answers Grade 3 Chapter 4 Division Facts & Strategies img_5

Think and Grow: Divide by 3 or 4

Example
Find 18 ÷ 3
Big Ideas Math Solutions Grade 3 Chapter 4 Division Facts and Strategies 4.4 1

Answer: 6

Explanation:
Let the unknown number be x.
18 ÷ 3 = x
18/3 = x
x = 6
Thus 18 is 3 times 6.

Example
Find 32 ÷ 4.
Big Ideas Math Solutions Grade 3 Chapter 4 Division Facts and Strategies 4.4 2

Answer: 8

Explanation:
Let the unknown number be x.
32 ÷ 4 = x
32/4 = x
x = 8
Thus 32 is 4 times 8.
4 × 8 = 32

Show and Grow

Complete the model and find the quotient.
Question 1.
Find 28 ÷ 4.
Big Ideas Math Solutions Grade 3 Chapter 4 Division Facts and Strategies 4.4 3
28 ÷ 4 = _____
Answer: 7

Explanation:
Number of rows = 4
Number of counters = 28
Divide the number of counters by the number of rows
28/4 = 7
Thus there are 7 columns in each row.

Question 2.
Find 9 ÷ 3.
Big Ideas Math Solutions Grade 3 Chapter 4 Division Facts and Strategies 4.4 4
9 ÷ 3 = _____
Answer: 3

Explanation:

Number of rows = 3
Number of counters = 9
Divide the number of counters by the number of rows
9/3 = 3
Thus there are 3 columns in each row.

Apply and Grow: Practice

Complete the model and find the quotient.
Question 3.
Find 8 ÷ 4.
Big Ideas Math Solutions Grade 3 Chapter 4 Division Facts and Strategies 4.4 5
8 ÷ 4 = _____
Answer: 2

Explanation:

Number of rows = 4
Number of counters = 8
Divide the number of counters by the number of rows
8/4 = 2
Thus there are 2 columns in each row.

Question 4.
Find 24 ÷ 3.
Big Ideas Math Solutions Grade 3 Chapter 4 Division Facts and Strategies 4.4 6
24 ÷ 3 = _____
Answer: 8

Explanation:

Number of rows = 3
Number of counters = 24
Divide the number of counters by the number of rows
24/3 = 8
Thus there are 8 columns in each row.

Find the quotient
Question 5.
12 ÷ 3 = _____
Answer: 4

Explanation:
Divide the two numbers 12 and 3.
12/3 =4
Thus the quotient is 4

Question 6.
20 ÷ 4 = _____
Answer: 5

Explanation:
Divide the two numbers 20 and 4.
20/4 = 5
Thus the quotient is 5.

Question 7.
15 ÷ 3 = _____
Answer: 5

Explanation:
Divide the two numbers 15 and 3
15/3 = 5
Thus the quotient is 5.

Question 8.
Big Ideas Math Solutions Grade 3 Chapter 4 Division Facts and Strategies 4.4 7
Answer: 8

Explanation:
Divide the two numbers 32 and 4
32/4 = 8
Thus the quotient is 8.

Question 9.
Big Ideas Math Solutions Grade 3 Chapter 4 Division Facts and Strategies 4.4 8
Answer: 6

Explanation:
Divide the two numbers 3 and 18
18/3 = 6
Thus the quotient is 6.

Question 10.
Big Ideas Math Solutions Grade 3 Chapter 4 Division Facts and Strategies 4.4 9
Answer: 9

Explanation:
Divide the two numbers 36 and 4
36/4 = 9
Thus the quotient is 9.

Question 11.
Divide 9 by 3.
Answer: 3

Explanation:
Divide the two numbers 3 and 9
9/3 = 3
Thus the quotient is 3.

Question 12.
Divide 12 by 4.
Answer: 3

Explanation:
Divide the two numbers 12 and 4.
12/4 = 3
Thus the quotient is 3.

Question 13.
Divide 21 by 3.
Answer: 7

Explanation:
Divide the two numbers 21 by 3
21/3 = 7
Thus the quotient is 7.

Compare
Question 14.
Big Ideas Math Solutions Grade 3 Chapter 4 Division Facts and Strategies 4.4 10
Answer: >

Explanation:
Divide the two numbers.
27 ÷ 3 = 9
28 ÷ 4 = 7
9 > 7
27 ÷ 3 > 28 ÷ 4

Question 15.
Big Ideas Math Solutions Grade 3 Chapter 4 Division Facts and Strategies 4.4 11
Answer: >

Explanation:
Divide the two numbers
30 ÷ 3 =10
24 ÷ 4 = 6
10 > 6
30 ÷ 3 > 24 ÷ 4

Question 16.
Big Ideas Math Solutions Grade 3 Chapter 4 Division Facts and Strategies 4.4 12
Answer: <

Explanation:
Divide the two numbers
6 ÷ 3 = 2
16 ÷ 4 = 4
2 < 4
6 ÷ 3 < 16 ÷ 4

Question 17.
There are 36 water bottles in a package. The bottles are in 4 rows. How many water bottles are in each row?
Answer: 9

Explanation:
Given,
There are 36 water bottles in a package.
The bottles are in 4 rows.
36/4 = 9
Thus 9 water bottles are in each row.

Question 18.
DIG DEEPER!
Can you divide 20 students into 3 equal groups? Explain.
Answer: No
20 is not the multiple of 3. So, 20 students cannot be divided into 3 equal groups.

Think and Grow: Modeling Real Life

You arrange 36 chairs in 4 equal rows. You arrange 21 music stands in 3 equal rows. How many more chairs are in each row than music stands?
Big Ideas Math Solutions Grade 3 Chapter 4 Division Facts and Strategies 4.4 13
Division equations:
There are ______ more chairs in each row.

Answer:
Given,
You arrange 36 chairs in 4 equal rows.
36/4 = 9 chairs in each row
You arrange 21 music stands in 3 equal rows.
21/3 = 7 chairs in each row
Thus there are 2 more chairs in each row.

Show and Grow

Question 19.
You arrange 30 cups of fruit punch in 3 equal rows. You arrange 24 cups of lemonade in 4 equal rows. How many more cups of fruit punch are in each row than cups of lemonade?
Answer:
Given,
You arrange 30 cups of fruit punch in 3 equal rows.
30/3 = 10 cups of fruit punch
You arrange 24 cups of lemonade in 4 equal rows.
24/4 = 6 cups of lemonade
10 > 6
Thus 4 more cups of fruit punch are in each row than cups of lemonade.

Question 20.
You have a bag of 25 carrot sticks. You eat 5 of them and equally share the rest with 4 friends. How many carrot sticks does each friend get?
Big Ideas Math Solutions Grade 3 Chapter 4 Division Facts and Strategies 4.4 14
Answer: 5

Explanation:
Given that,
You have a bag of 25 carrot sticks. You eat 5 of them and equally share the rest with 4 friends.
25 – 5 = 20 carrot sticks
20/4 = 5 carrot sticks
Thus each friend gets 5 carrot sticks.

Question 21.
Your teacher has 18 yellow pencils and 18 red pencils. She puts 3 pencils on each desk in the class. How many desks are in the class?
Answer: 12

Explanation:
Given that,
Your teacher has 18 yellow pencils and 18 red pencils.
18 + 18 = 36 pencils
She puts 3 pencils on each desk in the class.
36/3 = 12
Thus there are 12 desks in the class.

Divide by 3 or 4 Homework & Practice 4.4

Complete the model and find the quotient
Question 1.
Find 27 ÷ 3
Big Ideas Math Solutions Grade 3 Chapter 4 Division Facts and Strategies 4.4 15
27 ÷ 3 = ______
Answer: 9

Explanation:
Number of rows = 3
Number of counters = 27
Divide the number of counters by the number of rows
27/3 = 9
Thus there are 9 counters in each row.

Question 2.
Find 20 ÷ 4.
Big Ideas Math Solutions Grade 3 Chapter 4 Division Facts and Strategies 4.4 16
20 ÷ 4 _____
Answer: 5

Explanation:

Number of rows = 4
Number of counters = 20
Divide the number of counters by the number of rows
20/4 = 5
Thus there are 5 counters in each row.

Find the quotient
Question 3.
6 ÷ 3 = _____
Answer: 2

Explanation:
Divide the two numbers 3 and 6.
6/3 = 2
Thus the quotient is 2.

Question 4.
28 ÷ 4 = _____
Answer: 7

Explanation:
Divide the two numbers 28 and 4.
28/4 = 7
Thus the quotient is 7.

Question 5.
18 ÷ 3 = _____
Answer: 6

Explanation:
Divide the two numbers 18 and 3.
18/3 = 6
Thus the quotient is 6.

Question 6.
Big Ideas Math Solutions Grade 3 Chapter 4 Division Facts and Strategies 4.4 17
Answer: 2

Explanation:
Divide the two numbers 4 and 8.
8/4 = 2
Thus the quotient is 2.

Question 7.
Big Ideas Math Solutions Grade 3 Chapter 4 Division Facts and Strategies 4.4 18
Answer: 8

Explanation:
Divide the two numbers 24 and 3
24/3 = 8
Thus the quotient is 8.

Question 8.
Big Ideas Math Solutions Grade 3 Chapter 4 Division Facts and Strategies 4.4 19
Answer: 4

Explanation:
Divide the two numbers 16 and 4.
16/4 = 4
Thus the quotient is 4.

Compare
Question 9.
Big Ideas Math Solutions Grade 3 Chapter 4 Division Facts and Strategies 4.4 20
Answer: <

Explanation:
Divide the two numbers.
9 ÷ 3 = 3
15 ÷ 3 = 5
3 < 5
So, 9 ÷ 3 < 15 ÷ 3

Question 10.
Big Ideas Math Solutions Grade 3 Chapter 4 Division Facts and Strategies 4.4 21
Answer: <

Explanation:
Divide the two numbers.
12 ÷ 3 = 4
12 ÷ 4 = 3
4 > 3
12 ÷ 3 > 12 ÷ 4

Question 11.
Big Ideas Math Solutions Grade 3 Chapter 4 Division Facts and Strategies 4.4 22
Answer: <

Explanation:
Divide the two numbers.
32 ÷ 4 = 8
36 ÷ 4 = 9
8 < 9
32 ÷ 4 < 36 ÷ 4

Question 12.
Number Sense
Tissue boxes are sold in packs of 3. A doctor’s office needs 21 boxes. How many packs should the office buy?
Answer: 7

Explanation:
Given that,
Tissue boxes are sold in packs of 3.
A doctor’s office needs 21 boxes.
21/3 = 7 packs
Therefore the office should buy 7 packs.

Question 13.
Structure
Write the division equation represented by the number line.
Big Ideas Math Solutions Grade 3 Chapter 4 Division Facts and Strategies 4.4 23
Answer: 32 ÷ 4 = 8

Explanation:
The count starts from 0.
The count jumps for every 4s.
There are 8 jumps.
Thus the division equation is 32 ÷ 4 = 8

Question 14.
YOU BE THE TEACHER
Your friend says she can use 3 × 6 = 18 to help find 6 ÷ 3. Is your friend correct? Explain.
Answer: Your friend is incorrect

Explanation:
Your friend says she can use 3 × 6 = 18 to help find 6 ÷ 3.
6 ÷ 3 = 2
The multiplication equation is different from the division equation.
By this, you can say that your friend is incorrect.

Question 15.
Modeling Real Life
A food vending machine has 40 snacks in rows of 4. A drink vending machine has 21 drinks in rows of 3. How many more rows of snacks are there than rows of drinks?
Answer:
Given,
A food vending machine has 40 snacks in rows of 4.
40/4 = 10
A drink vending machine has 21 drinks in rows of 3.
21/3 = 7
10 – 7 = 3
Thus 3 more rows of snacks are there than rows of drinks.

Question 16.
Modeling Real Life
A delivery person has 13 large packages and 14 small packages. He delivers 3 packages to each house. There are none left. How many houses does he deliver to?
Big Ideas Math Solutions Grade 3 Chapter 4 Division Facts and Strategies 4.4 24
Answer: 9

Explanation:
Given,
A delivery person has 13 large packages and 14 small packages.
13 + 14 = 27
He delivers 3 packages to each house. There are none left.
27/3 = 9
Thus he delivers 9 houses.

Review & Refresh

Find the product
Question 17.
Big Ideas Math Solutions Grade 3 Chapter 4 Division Facts and Strategies 4.4 25
Answer: 48

Explanation:
Multiply the two numbers 8 and 6.
8 × 6 = 48

Question 18.
Big Ideas Math Solutions Grade 3 Chapter 4 Division Facts and Strategies 4.4 26
Answer: 0

Explanation:
Multiply the two numbers 6 and 0.
Any number multiplied by 0 will be always 0.
6 × 0 = 0

Question 19.
Big Ideas Math Solutions Grade 3 Chapter 4 Division Facts and Strategies 4.4 27
Answer: 12

Explanation:
Multiply the two numbers 6 and 2.
6 × 2 = 12

Question 20.
Big Ideas Math Solutions Grade 3 Chapter 4 Division Facts and Strategies 4.4 28
Answer: 60

Explanation:
Multiply the two numbers 10 and 6.
10 × 6 = 60

Question 21.
Big Ideas Math Solutions Grade 3 Chapter 4 Division Facts and Strategies 4.4 29
Answer: 42

Explanation:
Multiply the two numbers 7 and 6.
7 × 6 = 42

Lesson 4.5 Divide by 6 or 7

Explore and Grow

Complete the statements and the models.
Big Ideas Math Answer Key Grade 3 Chapter 4 Division Facts and Strategies 4.5 1
Answer: 42 ÷ 6 = 7
Big-Ideas-Math-Answer-Key-Grade-3-Chapter-4-Division-Facts-and-Strategies-4.5-1

Reasoning
Without solving, which quotient is greater? Explain how you know.
Answer: The second figure is having a greater quotient.
42 ÷ 6 = 7
42 ÷ 7 = 6

Think and Grow: Divide by 6 or 7

Example
Find 48 ÷ 6.
Think: 6 times what number is 48?
6 × _____ = 48
Big Ideas Math Answer Key Grade 3 Chapter 4 Division Facts and Strategies 4.5 2

Answer: 8

Explanation:
Number of rows = 6
Number of counters = 48
Divide the number of counters by the number of rows.
48/6 = 8
There are 8 columns.
6 × 8 = 48

Example
Find 49 ÷ 7.
Think: 7 times what number is 49?
7 × ____ = 49
Big Ideas Math Answer Key Grade 3 Chapter 4 Division Facts and Strategies 4.5 3

Answer: 7

Explanation:
Number of rows = 7
Number of counters = 49
Divide the number of counters by the number of rows
49/7 = 7
There are 7 columns.
7 × 7 = 49

Show and Grow

Complete the model and find the quotient
Question 1.
Find 28 ÷ 7.
Big Ideas Math Answer Key Grade 3 Chapter 4 Division Facts and Strategies 4.5 4
28 ÷ 7 = _____
Answer: 4

Explanation:
Number of rows = 7
Number of counters = 28
Divide the number of counters by the number of rows
28/7 = 4
There are 4 columns.

Question 2.
Find 54 ÷ 6.
Big Ideas Math Answer Key Grade 3 Chapter 4 Division Facts and Strategies 4.5 5
54 ÷ 6 = _____
Answer: 9

Explanation:
Number of rows = 6
Number of counters = 54
Divide the number of counters by the number of rows
54/6 = 9
There are 9 columns.

Apply and Grow: Practice

Complete the model and find the quotient
Question 3.
Find 36 ÷ 6.
Big Ideas Math Answer Key Grade 3 Chapter 4 Division Facts and Strategies 4.5 6
36 ÷ 6 = _____
Answer: 6

Explanation:
Number of rows = 6
Number of counters = 36
Divide the number of counters by the number of rows
36/6 = 6
There are 6 columns.

Question 4.
Find 14 ÷ 7.
Big Ideas Math Answer Key Grade 3 Chapter 4 Division Facts and Strategies 4.5 7
14 ÷ 7 = _____
Answer: 2

Explanation:
Number of rows = 7
Number of counters = 14
Divide the number of counters by the number of rows
14/7 = 2
There are 2 columns.

Find the quotient
Question 5.
60 ÷ 6 = _____
Answer: 10

Explanation:
Divide the two numbers 60 and 6.
60/6 = 10
Thus the quotient is 10.

Question 6.
35 ÷ 7 = _____
Answer: 5

Exp7lanation:
Divide the two numbers 35 and 7.
35/7 = 5
Thus the quotient is 5.

Question 7.
24 ÷ 6 = _____
Answer: 4

Explanation:
Divide the two numbers 24 and 6.
24/6 = 4
Thus the quotient is 4.

Question 8.
Big Ideas Math Answer Key Grade 3 Chapter 4 Division Facts and Strategies 4.5 8
Answer: 2

Explanation:
Divide the two numbers 12 and 6.
12/6 = 2
Thus the quotient is 2.

Question 9.
Big Ideas Math Answer Key Grade 3 Chapter 4 Division Facts and Strategies 4.5 9
Answer: 6

Explanation:
Divide the two numbers 42 and 7.
42/7 = 6
Thus the quotient is 6.

Question 10.
Big Ideas Math Answer Key Grade 3 Chapter 4 Division Facts and Strategies 4.5 10
Answer: 9

Explanation:
Divide the two numbers 63 and 7
63/7 = 9
Thus the quotient is 9.

Find the missing divisor
Question 11.
28 ÷ ____ = 7
Answer: 4

Explanation:
Let the missing divisor be a.
28 ÷ a = 7
28/a = 7
28 = 7a
a = 28/7
a = 4
Thus the missing divisor is 4.

Question 12.
30 ÷ ____ = 6
Answer: 5

Explanation:
Let the missing divisor be b.
30 ÷ b = 6
30/b = 6
30 = 6 × b
b = 30/6
b = 5
Thus the missing divisor is 5.

Question 13.
70 ÷ _____ = 7
Answer: 10

Explanation:
Let the missing divisor be c.
70 ÷ c = 7
70/c = 7
70 = 7 × c
c = 70/7
c = 10
Thus the missing divisor is 10.

Question 14.
You have 24 stones for the game mancala. There are 6 holes on the board. Each hole gets an equal number of stones. How many stones do you put in each hole?
Answer: 4

Explanation:
Given that,
You have 24 stones for the game mancala.
There are 6 holes on the board.
Each hole gets an equal number of stones.
24/6 = 4
Thus you put 4 stones in each hole.

Question 15.
Number Sense
Write the correct symbol to make each equation true
Big Ideas Math Answer Key Grade 3 Chapter 4 Division Facts and Strategies 4.5 11
Answer:
Big-Ideas-Math-Answer-Key-Grade-3-Chapter-4-Division-Facts-and-Strategies-4.5-11

Think and Grow: Modeling Real Life

You have 54 craft sticks. You use all of the sticks to make hexagons. How many hexagons can you make?
Big Ideas Math Answer Key Grade 3 Chapter 4 Division Facts and Strategies 4.5 12.1
Division equation:
You can make _____ hexagons.

Answer:
Given,
You have 54 craft sticks. You use all of the sticks to make hexagons.
hexagon – 6
54/6 = 9
You can make 9 hexagons.

Show and Grow

Question 16.
You use 35 craft sticks to make 7 polygons. You use the same number of craft sticks for each polygon. How many craft sticks do you use for each polygon?
Answer: 5

Explanation:
Given,
You use 35 craft sticks to make 7 polygons. You use the same number of craft sticks for each polygon.
35/7 = 5
Thus you use 5 craft sticks for each polygon.

Question 17.
There are 42 students in gym class. The teacher divides the students into 7 teams. How many more students would be on each team if the teacher divides the students into 6 teams?
Answer:
Given that,
There are 42 students in gym class. The teacher divides the students into 7 teams.
42/7 = 6 students
If the teacher divides the students into 6 teams then,
42/6 = 7 students

Question 18.
You have a tray of 12 oatmeal bars. You keep 6 of them. How many bars can you give to each of your 6 friends?
Answer:
Given that,
You have a tray of 12 oatmeal bars. You keep 6 of them.
12/6 = 2
Thus you can give 2 bars to each of your 6 friends.

Divide by 6 or 7 Homework & Practice 4.5

Complete the model and find the quotient
Question 1.
Find 18 ÷ 6
Big Ideas Math Answer Key Grade 3 Chapter 4 Division Facts and Strategies 4.5 12
18 ÷ 6 = _____
Answer: 3

Explanation:
Number of rows = 6
Number of counters = 18
Divide the number of counters by the number of rows
18/6 = 3
Thus there are 3 counters in each row

Question 2.
Find 35 ÷ 7
Big Ideas Math Answer Key Grade 3 Chapter 4 Division Facts and Strategies 4.5 13
35 ÷ 7 = _____
Answer: 5

Explanation:
Number of rows = 7
Number of counters = 35
Divide the number of counters by the number of rows
35/7 = 5
Thus there are 5 counters in each row

Find the quotient
Question 3.
42 ÷ 6 = _____
Answer: 7

Explanation:
Divide the two numbers 42 and 6.
42/6 = 7
Thus the quotient is 7.

Question 4.
28 ÷ 7 = _____
Answer: 4

Explanation:
Divide the two numbers 28 and 7
28/7 = 4
Thus the quotient is 4.

Question 5.
54 ÷ 6 = _____
Answer: 9

Explanation:
Divide the two numbers 54 and 6.
54/6 = 9
Thus the quotient is 9.

Question 6.
Big Ideas Math Answer Key Grade 3 Chapter 4 Division Facts and Strategies 4.5 14
Answer: 7

Explanation:
Divide the two numbers 49 and 7.
49/7 = 7
Thus the quotient is 7.

Question 7.
Big Ideas Math Answer Key Grade 3 Chapter 4 Division Facts and Strategies 4.5 15
Answer: 10

Explanation:
Divide the two numbers 7 and 70
70/7 = 10
Thus the quotient is 10.

Question 8.
Big Ideas Math Answer Key Grade 3 Chapter 4 Division Facts and Strategies 4.5 16
Answer: 5

Explanation:
Divide the two numbers 30 and 6.
30/6 = 5
Thus the quotient is 5.

Find the missing divisor.
Question 9.
36 ÷ ____ = 6
Answer: 9

Explanation:
Let the missing divisor be x.
36 ÷ x = 6
36/x = 6
36 = 6 × x
x = 36/6
x = 6
Thus the missing divisor is 6.

Question 10.
14 ÷ ____ = 7
Answer: 2

Explanation:
Let the missing divisor be y.
14 ÷ y = 7
14/y = 7
14 = 7 × y
y = 14/7
y = 2
Thus the missing divisor is 2.

Question 11.
60 ÷ _____ = 6
Answer: 10

Explanation:
Let the missing divisor be z.
60 ÷ z = 6
60/z = 6
60 = 6 × z
z = 60/6
z = 10
Thus the missing divisor is 10.

Question 12.
Number Sense
There are 7 continents. A scientist has 63 days to spend studying on all the continents. She wants to spend an equal number of days on each one. How many days can she spend on each continent?
Answer: 9

Explanation:
Given,
There are 7 continents. A scientist has 63 days to spend studying on all the continents.
She wants to spend an equal number of days on each one.
63/7 = 9 days
Thus she can spend 9 days on each continent.

Question 13.
Logic
I am an odd number. When you multiply me by 6, then divide the product by 3, the quotient is 10. What number am I?
Answer: 5

Explanation:
Let us consider 5 to be the odd number.
Now multiply by 6.
5 × 6 = 30
Now divide by 3.
30/3 = 10
So, the number is 5.

Question 14.
DIG DEEPER!
You deal 52 cards to 7 players. Does each player get the same number of cards? Explain.
Big Ideas Math Answer Key Grade 3 Chapter 4 Division Facts and Strategies 4.5 17
Answer: No. Because 52 is not the multiple of 7.
Thus each player will not get the same number of cards.

Question 15.
Modeling Real Life
There are 70 students at a summer camp. A counselor divides the students into teams of 10. How many more teams would the counselor make if he divides the students into teams of 7?
Answer:
Given,
There are 70 students at a summer camp. A counselor divides the students into teams of 10.
If he divides into 7 teams then there will be 10 students in each team.
70/7 = 10

Question 16.
Modeling Real Life
There are 31 students in your class. Seven students are called to the nurse’s office to get their hearing checked. Your teacher divides the rest of the students into groups of 6. How many groups are there?
Answer:
Given that,
There are 31 students in your class. Seven students are called to the nurse’s office to get their hearing checked.
31 – 7 = 24
Your teacher divides the rest of the students into groups of 6.
24/6 = 4 groups
Thus there are 4 groups.

Review & Refresh

Find the product
Question 17.
Big Ideas Math Answer Key Grade 3 Chapter 4 Division Facts and Strategies 4.5 18
Answer: 48

Explanation:
Multiply the two numbers 8 and 6.
8 × 6 = 48

Question 18.
Big Ideas Math Answer Key Grade 3 Chapter 4 Division Facts and Strategies 4.5 19
Answer: 72

Explanation:
Multiply the two numbers 9 and 8.
9 × 8 = 72

Question 19.
Big Ideas Math Answer Key Grade 3 Chapter 4 Division Facts and Strategies 4.5 20
Answer: 80

Explanation:
Multiply the two numbers 10 and 8.
10 × 8 = 80

Question 20.
Big Ideas Math Answer Key Grade 3 Chapter 4 Division Facts and Strategies 4.5 21
Answer: 0

Explanation:
Multiply the two numbers 0 and 8.
Any number multiplied by 0 will be always 0.
Thus 8 × 0 = 0

Question 21.
Big Ideas Math Answer Key Grade 3 Chapter 4 Division Facts and Strategies 4.5 22
Answer: 8

Explanation:
Multiply the two numbers 1 and 8.
Any number multiplied by 1 will be always the same number.
So, 1 × 8 = 8

Lesson 4.6 Divide by 8 or 9

Explore and Grow

Use repeated subtraction to find 48 ÷ 8.
Answer:
48 – 8 = 40
40 – 8 = 32
32 – 8 = 24
24 – 8 = 16
16 – 8 = 8
8 – 8 = 0
At sixth number we got 0.
So, 48/8 in repeated subtraction is 6.

Reasoning
How many times did you subtract 8 from 48? Does the quotient represent the number of groups or the size of the groups? Explain.
Answer: I subtracted 8 from 48 six times. Yes, the quotient represents the number of groups or size of the groups.

Think and Grow: Divide 8 or 9

Example
Find 40 ÷ 8.
Think: 8 times what number is 40?
8 × ____= 40
Big Ideas Math Answers 3rd Grade Chapter 4 Division Facts and Strategies 4.6 1

Answer: 5

Explanation:
Number of rows = 8
Number of counters = 40
Divide the number of counters by the number of rows
40/8 = 5
Thus there are 5 columns.
8 times 5 is 40.

Example
Find 54 ÷ 9.
Think: 9 times what number is 54?
9 × ____ = 54
Big Ideas Math Answers 3rd Grade Chapter 4 Division Facts and Strategies 4.6 2

Answer: 6

Explanation:
Number of rows = 9
Number of counters = 54
Divide the number of counters by the number of rows
54/9 = 6
Thus there are 6 columns.
9 times 6 is 54.

Show and Grow

Complete the model and find the quotient
Question 1.
24 ÷ 8 = ____
Big Ideas Math Answers 3rd Grade Chapter 4 Division Facts and Strategies 4.6 3
24 ÷ 8 = ____
Answer: 3

Explanation:
Number of rows = 8
Number of counters = 24
Divide the number of counters by the number of rows
24/8 = 3
There are 3 columns
8 times 3 is 24.

Question 2.
18 ÷ 9 = ____
Big Ideas Math Answers 3rd Grade Chapter 4 Division Facts and Strategies 4.6 4
18 ÷ 9 = ____
Answer: 2

Explanation:
Number of rows = 9
Number of counters = 18
Divide the number of counters by the number of rows
18/9 = 2
There are 2 columns
2 times 9 is 18.

Apply and Grow: Practice

Complete the model and find the quotient
Question 3.
Find 36 ÷ 9.
Big Ideas Math Answers 3rd Grade Chapter 4 Division Facts and Strategies 4.6 5
36 ÷ 9 = _____
Answer: 4

Explanation:
Number of rows = 9
Number of counters = 36
Divide the number of counters by the number of rows
36/9 = 4
There are 4 columns
4 times 9 is 36.

Question 4.
Find 32 ÷ 8.
Big Ideas Math Answers 3rd Grade Chapter 4 Division Facts and Strategies 4.6 6
32 ÷ 8 = ____
Answer: 4

Explanation:
Number of rows = 8
Number of counters = 32
Divide the number of counters by the number of rows
32/8 = 4
There are 4 columns
8 times 4 is 32.

Find the quotient
Question 5.
45 ÷ 9 = _____
Answer: 5

Explanation:
Divide the two numbers 45 and 9
45/9 = 5
Thus the quotient is 5.

Question 6.
56 ÷ 8 = _____
Answer: 7

Explanation:
Divide the two numbers 56 and 8.
56/8 = 7
Thus the quotient is 7.

Question 7.
90 ÷ 9 = ____
Answer: 10

Explanation:
Divide the two numbers 90 and 9.
90/9 = 10
Thus the quotient is 10.

Question 8.
Big Ideas Math Answers 3rd Grade Chapter 4 Division Facts and Strategies 4.6 7
Answer: 2

Explanation:
Divide the two numbers 9 and 18
18/9 = 2
Thus the quotient is 2.

Question 9.
Big Ideas Math Answers 3rd Grade Chapter 4 Division Facts and Strategies 4.6 8
Answer: 6

Explanation:
Divide the two numbers 8 and 48
48/8 = 6
Thus the quotient is 6.

Question 10.
Big Ideas Math Answers 3rd Grade Chapter 4 Division Facts and Strategies 4.6 9
Answer: 2

Explanation:
Divide the two numbers 16 and 8.
16/8 = 2
Thus the quotient is 2.

Question 11.
Divide 54 by 9.
Answer: 6

Explanation:
Divide the two numbers 54 and 9
54/9 = 6
Thus the quotient is 6.

Question 12.
Divide 64 by 8.
Answer: 8

Explanation:
Divide the two numbers 64 and 8.
64/8 = 8
Thus the quotient is 8.

Question 13.
Divide 63 by 9
Answer: 7

Explanation:
Divide the two numbers 63 and 9
63/9 = 7
Thus the quotient is 7.

Compare
Question 14.
Big Ideas Math Answers 3rd Grade Chapter 4 Division Facts and Strategies 4.6 10
Answer: =

Explanation:
Divide the two numbers
81 ÷ 9 = 9
72 ÷ 8 = 9
9 = 9
So, 81 ÷ 9 = 72 ­÷ 8

Question 15.
Big Ideas Math Answers 3rd Grade Chapter 4 Division Facts and Strategies 4.6 11
Answer: >

Explanation:
Divide the two numbers
72 ÷ 9 = 8
40 ÷ 8 = 5
8 > 5
So, 72 ÷ 9 > 40 ÷ 8

Question 16.
Big Ideas Math Answers 3rd Grade Chapter 4 Division Facts and Strategies 4.6 12
Answer: <

Explanation:
Divide the two numbers
16 ÷ 8 = 2
27 ÷ 9 = 3
2 < 3
So, 16 ÷ 8 < 27 ÷ 9

Question 17.
A comic book has 63 pages. You read 9 pages each night. How many nights will it take to read the entire book?
Answer: 7

Explanation:
Given that,
A comic book has 63 pages. You read 9 pages each night.
63/9 = 7
Thus it will take 7 nights to read the entire book.

Question 18.
Logic
Find the missing number.
Big Ideas Math Answers 3rd Grade Chapter 4 Division Facts and Strategies 4.6 13
Answer: 8

Explanation:
8 times 8 is 64.
Thus the missing number is 8.

Think and Grow: Modeling Real Life

There are 9 innings in a baseball game. The table shows how many innings Newton played each season. How many games has Newton played in all?
Big Ideas Math Answers 3rd Grade Chapter 4 Division Facts and Strategies 4.6 14
Division equation:
Newton played ______ games in all.

Answer:
Given,
There are 9 innings in a baseball game.
The table shows how many innings Newton played each season.
Season 1: 81/9 = 9 games
Season 2: 63/9 = 7 games
9 + 7 = 16 games
Therefore Newton played 16 games in all.

Show and Grow

Question 19.
The third-grade and fourth-grade classes are going on a field trip. A van can carry 8 students. How many vans are needed in all?
Big Ideas Math Answers 3rd Grade Chapter 4 Division Facts and Strategies 4.6 15
Answer:
Given that,
The third-grade and fourth-grade classes are going on a field trip.
A van can carry 8 students.
Grade 3 – 48 students
Grade 4 – 56 students
third grade – 48/8 = 6 vans
fourth grade – 56/8 = 7 vans
6 + 7 = 13 vans
Thus 13 vans are needed in all.

Question 20.
Seventy-two ballet dancers are arranged into an array with 9 columns. How many dancers are in each column?
Answer: 8 dancers

Explanation:
Given that,
Seventy-two ballet dancers are arranged into an array with 9 columns.
72/9 = 8
Thus 8 dancers are in each column.

Question 21
You make 15 paper elephants and 17 paper lions. You give all of the animals away to 8 friends. Each friend gets the same number of animals. How many animals does each friend get?
Big Ideas Math Answers 3rd Grade Chapter 4 Division Facts and Strategies 4.6 16
Answer: 4

Explanation:
Given that,
You make 15 paper elephants and 17 paper lions.
15 + 17 = 32
You give all of the animals away to 8 friends.
Each friend gets the same number of animals.
32/8 = 4
Thus each friend gets 4 animals.

Divide by 8 or 9 Homework & Practice 4.6

Complete the model and find the quotient
Question 1.
Find 16 ÷ 8.
Big Ideas Math Answers 3rd Grade Chapter 4 Division Facts and Strategies 4.6 17
16 ÷ 8 = ____
Answer: 2

Explanation:
Number of rows = 8
Number of counters = 16
Divide the number of counters by the number of rows
Division equation is 16 ÷ 8 = 2
Thus the number of columns is 2.

Question 2.
Find 45 ÷ 9.
Big Ideas Math Answers 3rd Grade Chapter 4 Division Facts and Strategies 4.6 18
45 ÷ 9 = _____
Answer: 5

Explanation:
Number of rows = 9
Number of counters = 45
Divide the number of counters by the number of rows
Division equation is 45 ÷ 9 = 5
Thus the number of columns is 5.

Find the quotient
Question 3.
48 ÷ 8. = _____
Answer: 6

Explanation:
Divide the two numbers 48 and 8.
48/8 = 6
Thus the quotient is 6.

Question 4.
63 ÷ 9 = ______
Answer: 7

Explanation:
Divide the two numbers 63 and 9.
63/9 = 7
Thus the quotient is 7.

Question 5.
54 ÷ 9 = _____
Answer: 6

Explanation:
Divide the two numbers 54 and 9.
54/9 = 6
Thus the quotient is 6.

Question 6.
Big Ideas Math Answers 3rd Grade Chapter 4 Division Facts and Strategies 4.6 19
Answer: 4

Explanation:
Divide the two numbers 32 and 8.
32/8 = 4
Thus the quotient is 4.

Question 7.
Big Ideas Math Answers 3rd Grade Chapter 4 Division Facts and Strategies 4.6 20
Answer: 9

Explanation:
Divide the two numbers 72 and 8.
72/8 = 9
Thus the quotient is 9.

Question 8.
Big Ideas Math Answers 3rd Grade Chapter 4 Division Facts and Strategies 4.6 21
Answer: 2

Explanation:
Divide the two numbers 18 and 9.
18/9 = 2
Thus the quotient is 2.

Compare
Question 9.
Big Ideas Math Answers 3rd Grade Chapter 4 Division Facts and Strategies 4.6 22
Answer: =

Explanation:
Divide the two numbers
90/9 = 10
80/8 = 10
10 = 10
90 ÷ 9 = 80 ÷ 8

Question 10.
Big Ideas Math Answers 3rd Grade Chapter 4 Division Facts and Strategies 4.6 23
Answer:  <

Explanation:
Divide the two numbers
27/9 = 3
56/8 = 7
3 < 7
27 ÷ 9 < 56 ÷ 8

Question 11.
Big Ideas Math Answers 3rd Grade Chapter 4 Division Facts and Strategies 4.6 24
Answer: >

Explanation:
Divide the two numbers
72 ÷ 9 = 8
40 ÷ 8 = 5
8 > 5
72 ÷ 9 > 40 ÷ 8

Question 12.
A food truck owner needs 64 whole wheat pitas. The pitas come in packages of 8. How many packages should she buy?
Answer: 8

Explanation:
Given that,
A food truck owner needs 64 whole-wheat pitas. The pitas come in packages of 8.
64 ÷ 8 = 8
Thus she should buy 8 packages.

Question 13.
Patterns
Complete the division tables.
Big Ideas Math Answers 3rd Grade Chapter 4 Division Facts and Strategies 4.6 25
Answer:
Big-Ideas-Math-Answers-3rd-Grade-Chapter-4-Division-Facts-and-Strategies-4.6-25

Question 14.
Structure
Describe two different ways to find 54 ÷ 9.
Answer: 6
The different ways to find 54 ÷ 9 is a division equation and multiplication equation.
54/9 = 6
6 × 9 = 54

Question 15.
Modeling Real Life
A youth group leader is preparing for a remote-control car race. Batteries are sold in packs of 8. How many packs of batteries should he buy?
Big Ideas Math Answers 3rd Grade Chapter 4 Division Facts and Strategies 4.6 26
Answer:
Given,
A youth group leader is preparing for a remote-control car race.
Batteries are sold in packs of 8.
cars – 48
48 ÷ 8 = 6 batteries for cars
24 ÷ 8 = 3 batteries for remote

Question 16.
Modeling Real Life
Newton divides 32 treats equally between 8 friends. Descartes divides 27 treats equally between 9 friends. Whose friends get more treats? Explain.
Answer:
Given that,
Newton divides 32 treats equally between 8 friends.
32 ÷ 8 = 4
Descartes divides 27 treats equally between 9 friends.
27 ÷ 9 = 3
Thus Newton’s friends get more treats.

Review & Refresh

Find the missing factor.
Question 17.
____ × 4 = 0
Answer: 0

Explanation:
Let the missing factor be x.
x × 4 = 0
x = 0/4
x = 0
Thus the missing factor is 0.

Question 18.
1 × ____ = 2
Answer: 2

Explanation:
Let the missing factor be y.
1 × y = 2
y = 2/1
y = 2
Thus the missing factor is 2.

Question 19.
9 × ____ = 9
Answer: 1

Explanation:
Let the missing factor be z.
9 × z = 9
z = 9/9
z = 1
Thus the missing factor is 1.

Lesson 4.7 Divide by 0 or 1

Explore and Grow

Find the quotients
Big Ideas Math Answers Grade 3 Chapter 4 Division Facts and Strategies 4.7 1
Answer:
Big-Ideas-Math-Answers-Grade-3-Chapter-4-Division-Facts-and-Strategies-4.7-1

Structure
What patterns do you notice? Use the patterns to find each quotient.
52 ÷ 52 = _____
0 ÷ 52 = _____
52 ÷ 1 = ______
Answer:
52 ÷ 52 = 1
Any number divided by the same number will be always 1.
0 ÷ 52 = 0
Any number divided by 0 will be always 0.
52 ÷ 1 = 52
Any number divided by 1 will be the same number.

Think and Grow: Divide by 0 or 1

Dividing a number by 1 or itself:
• Any number divided by1 is itself.
• Any number (except 0) divided by itself is 1.
Example
Find 7 ÷ 1.
Think: 1 time what number is 7?
1 × ____ = 7
Write: 7 × 1 = _____

Answer:
Any number divided by 1 will be the same number.
7 × 1 = 7

Example
Find 5 ÷ 5.
Think: 5 times what number is 5?
5 × ____ = 5
Write: 5 ÷ 5 = _____

Dividing with 0:
• 0 divided by any number (except 0) is 0.
• You cannot divide by 0.

Answer:
5 × 1 = 5
5 times 1 is 5.
5 ÷ 5 = 1

Example
Find 0 ÷ 8.
Think: 8 times what number is 0?
8 × ____ = 0
Write: 0 × 8 = _____

Answer:
0 times 8 is 0.
Any number multiplied by 0 is always 0.
0 × 8 = 0

Example
Find 9 ÷ 0.
Think: 0 times what number is 9?
There is no such number. So, you cannot divide by 0.

Show and Grow

Write the related multiplication fact. Then find the quotient.
Question 1.
2 ÷ 2 = _____
Answer: 1

Explanation:
The related multiplication fact for 2, 1 is
2 × 1 = 2
1 × 2 = 2
2 ÷ 2 = 1
Thus the quotient is 1.

Question 2.
0 ÷ 3 = _____
Answer: 0

Explanation:
The related multiplication fact for 0, 3 is
0 × 3 = 0
3 × 0 = 0
Thus the quotient is 0.

Question 3.
10 ÷ 1 =_____
Answer: 10

Explanation:
The related multiplication fact for 1, 10
1 × 10 = 10
10 × 1 = 10
10 ÷ 1 = 10
Thus the quotient is 10.

Question 4.
25 ÷ 25 = _____
Answer: 1

Explanation:
The related multiplication fact for 25, 1
1 × 25 = 25
25 × 1 = 25
25 ÷ 25 = 1
Thus the quotient is 1.

Apply and Grow: Practice

Write the related multiplication fact. Then find the quotient.
Question 5.
6 ÷ 1 = ______
Answer: 6

Explanation:
The related multiplication fact for 6,1
1 × 6 =6
6 × 1 = 6
6 ÷ 1 = 6
Thus the quotient is 6.

Question 6.
0 ÷ 2 = _____
Answer: 0

Explanation:
The related multiplication fact for 0,2
Any number divided by 0 will be always 0.
0 ÷ 2 = 0
Thus the quotient is 0.

Question 7.
9 ÷ 9 = ____
Answer: 1

Explanation:
The related multiplication fact for 1, 9
1 × 9 = 9
9 × 1 = 9
9 ÷ 9 = 1
Thus the quotient is 1.

Question 8.
8 ÷ 1 = _____
Answer: 8

Explanation:
The related multiplication fact for 1, 8
1 × 8 = 8
8 × 1 =8
8 ÷ 1 = 8
Thus the quotient is 8.

Find the quotient
Question 9.
5 ÷ 1 = _____
Answer: 5

Explanation:
Any number divided by 1 will be the same number.
5/1 = 5
Thus the quotient is 5

Question 10.
0 ÷ 7 = _____
Answer: 0

Explanation:
Any number divided by 0 will be always 0.
0/7 = 0
Thus the quotient is 0.

Question 11.
4 ÷ 4 = _____
Answer: 1

Explanation:
4/4 = 1
Thus the quotient is 1.

Question 12.
Big Ideas Math Answers Grade 3 Chapter 4 Division Facts and Strategies 4.7 2.1
Answer: 15

Explanation:
Any number divided by 1 will be the same number.
15/1 = 1
Thus the quotient is 15.

Question 13.
Big Ideas Math Answers Grade 3 Chapter 4 Division Facts and Strategies 4.7 2
Answer: 0

Explanation:
Any number divided by 0 will be always 0.
0/6 = 0
Thus the quotient is 0.

Question 14.
Big Ideas Math Answers Grade 3 Chapter 4 Division Facts and Strategies 4.7 3
Answer: 1

Explanation:
24/24 = 1
Thus the quotient is 1.

Find the missing dividend or divisor.
Question 15.
2 ÷ ____ = 2
Answer: 1

Explanation:
Let the missing divisor be x.
2 ÷ x = 2
2/x = 2
2 = 2 × x
x = 2/2
x = 1
Thus the missing divisor is 1.

Question 16.
____ ÷ 12 = 0
Answer: 0

Explanation:
Let the missing divisor be y.
y ÷ 12 = 0
y/12 = 0
y = 0 × 12
y = 0
Thus the missing divisor is 0.

Question 17.
8 ÷ ____ = 1
Answer: 8

Explanation:
Let the missing divisor be z.
8 ÷ z = 1
8/z = 1
8 = 1 × z
z = 8
Thus the missing divisor is 8.

Compare
Question 18.
Big Ideas Math Answers Grade 3 Chapter 4 Division Facts and Strategies 4.7 4
Answer: >

Explanation:
Divide the two numbers
4 ÷ 1 = 4
3 ÷ 1 = 3
4 > 3
So, 4 ÷ 1 > 3 ÷ 1

Question 19.
Big Ideas Math Answers Grade 3 Chapter 4 Division Facts and Strategies 4.7 5
Answer: =

Explanation:
Any number divided by 0 will be always 0.
0 ÷ 6 = 0
0 ÷ 9 = 0
So, 0 ÷ 6 = 0 ÷ 9

Question 20.
Big Ideas Math Answers Grade 3 Chapter 4 Division Facts and Strategies 4.7 6
Answer: <

Explanation:
Any number divided by 0 will be always 0.
0 ÷ 5 = 0
7 ÷ 1 = 7
0 < 7
0 ÷ 5 < 7 ÷ 1

Question 21.
There are 2 cheese blocks. Each mousetrap has 1 cheese block on it. How many mousetraps are there?
Big Ideas Math Answers Grade 3 Chapter 4 Division Facts and Strategies 4.7 7
Answer: 2

Explanation:
Given that,
There are 2 cheese blocks. Each mousetrap has 1 cheese block on it.
2 ÷ 1 = 2
Thus there are 2 mousetraps.

Question 22.
Reasoning
Your friend says 14 ÷ 0 and 0 ÷ 14 both equal 0. Is your friend correct? Explain.
Answer: Incorrect

Explanation:
Your friend is incorrect because 0 divided by any number except 0 will be always 0.
0/14 = 0
but
14/0 =  infinity

Think and Grow: Modeling Real Life

A clown shares 10 balloons equally with 10 children. How many balloons does each child receive?
Big Ideas Math Answers Grade 3 Chapter 4 Division Facts and Strategies 4.7 8
Division equation:
Each child receives a _____ balloon.

Answer: 1

Explanation:
Given that,
A clown shares 10 balloons equally with 10 children.
10/10 = 1
Therefore each child receives 1 balloon.

Show and Grow

Question 23.
You have 9 tokens. You need 1 token to play an arcade game. How many games can you play?
Answer: 9

Explanation:
Given that,
You have 9 tokens. You need 1 token to play an arcade game.
9/1 = 9
Thus you can play 9 games.

Question 24.
You have 9 quarters. You put 5 of them in your backpack. You divide the other quarters equally among 4 friends. How many quarters does each friend get?
Answer: 1

Explanation:
Given,
You have 9 quarters. You put 5 of them in your backpack.
9 – 5 = 4
You divide the other quarters equally among 4 friends.
4/4 = 1
Thus each friend gets 1 quarter.

Question 25.
You ask your friend the question below.
“What is 475 divided by475?”
Your friend immediately says 1. How does your friend solve the problem so quickly? Explain.
Answer:
475/475 = 1
Your friend solved the problem very quickly because he used the logic ” Any number divided by the same number will be always 1″.

Divide by 0 or 1 Homework & Practice 4.7

Write the related multiplication fact. Then find the quotient.
Question 1.
3 ÷ 1 = _____
Answer: 3

Explanation:
The related multiplication fact for 1,3
1 × 3 = 3
3 × 1 = 3
Divide the two numbers
3 ÷ 1 = 3
Thus the quotient is

Question 2.
0 ÷ 7 = _____
Answer: 0

Explanation:
The related multiplication fact for 0, 7
0 × 7 = 0
7 × 0 = 0
Divide the two numbers 0, 7
0/7 = 0
Thus the quotient is 0

Find the quotient.
Question 3.
8 ÷ 1 = _____
Answer: 8

Explanation:
The related multiplication fact for 1, 8
8 × 1 = 8
1 × 8 = 8
Divide the two numbers 8, 1
8/1 = 8
Thus the quotient is 8

Question 4.
0 ÷ 2 = _____
Answer: 0

Explanation:
The related multiplication fact for 0, 2
0 × 2 = 0
2 × 0 = 0
Divide the two numbers
0/2 = 0
Thus the quotient is 0

Question 5.
7 ÷ 7 = _____
Answer: 1

Explanation:
The related multiplication fact for 7, 1
1 × 7 = 7
7 × 1 = 7
Divide the two numbers
7/7 = 1
Thus the quotient is 1

Question 6.
Big Ideas Math Answers Grade 3 Chapter 4 Division Facts and Strategies 4.7 9
Answer: 1

Explanation:
The related multiplication fact for 1, 12
12 × 1 = 12
1 × 12 = 12
Divide the two numbers
12/12 = 1
Thus the quotient is 1

Question 7.
Big Ideas Math Answers Grade 3 Chapter 4 Division Facts and Strategies 4.7 10
Answer: 4

Explanation:
The related multiplication fact for 1, 4
1 × 4 = 4
4 × 1 = 4
Divide the two numbers
4/1 = 4
Thus the quotient is 4

Question 8.
Big Ideas Math Answers Grade 3 Chapter 4 Division Facts and Strategies 4.7 11
Answer: 0

Explanation:
The related multiplication fact for 0, 10
0 × 10 = 0
10 × 0 = 0
Divide the two numbers
0/10 = 0
Thus the quotient is 0

Find the missing dividend or divisor
Question 9.
____ ÷ 3 = 0
Answer: 0

Explanation:
Let the missing dividend be x
x ÷ 3 = 0
x/3 = 0
x = 0 × 3
x = 0
Thus the missing dividend is 0.

Question 10.
10 ÷ ____ = 10
Answer: 1

Explanation:
Let the missing divisor be y.
10 ÷ y = 10
10/y = 10
10 = y × 10
y = 10/10
y = 1
Thus the missing divisor is 1.

Question 11.
____ ÷ 4 = 0
Answer: 0

Explanation:
Let the missing dividend be z.
z ÷ 4 = 0
z/4 = 0
z = 0 × 4
z = 0
Thus the missing dividend is 0.

Compare
Question 12.
Big Ideas Math Answers Grade 3 Chapter 4 Division Facts and Strategies 4.7 12
Answer: =

Explanation:
Divide the two numbers
0 ÷ 7 = 0
0 ÷ 1 = 0
0 = 0
0 ÷ 7 = 0 ÷ 1

Question 13.
Big Ideas Math Answers Grade 3 Chapter 4 Division Facts and Strategies 4.7 13
Answer: <

Explanation:
Divide the two numbers
0 ÷ 9 = 0
2 ÷ 1 = 2
0 < 2
0 ÷ 9 < 2 ­÷ 1

Question 14.
Big Ideas Math Answers Grade 3 Chapter 4 Division Facts and Strategies 4.7 14
Answer: >

Explanation:
Divide the two numbers
6 ÷ 1 = 6
6 ÷ 6 = 1
6 > 1
6 ÷ 1 > 6 ÷ 6

Question 15.
Reasoning
How are the multiplication rules for 0 and 1 similar to the division rules for 0 and 1? How are they different?
Answer:
Multiplication and division are closely related, given that division is the inverse operation of multiplication. When we divide, we look to separate into equal groups, while multiplication involves joining equal groups.
multiplication rules for 0 and 1 are not similar to division rules for 0 and 1.
Any number multiplied by 0 will be zero.
But any number divided by 0 will be infinity.

Question 16.
Number Sense
Which has the greatest quotient?
6 ÷ 6
4 ÷ 1
0 ÷ 8
3 ÷ 1
Answer: 4 ÷ 1

Explanation:
6 ÷ 6 = 1
4 ÷ 1 = 4
0 ÷ 8 = 0
3 ÷ 1 = 3
Thus the correct answer is option b.

Question 17.
DIG DEEPER!
Use the numbers 0, 1, 5, and 10 to make the number sentence true.
____ ÷ _____ < _____ ÷ ______
Answer:
You need to write the equation by using the above numbers.
0 ÷ 1 < 5 ÷ 10

Question 18.
Modeling Real Life
There are 5 bird eggs. An equal number of eggs hatch each day for 5 days. How many eggs hatch each day?
Big Ideas Math Answers Grade 3 Chapter 4 Division Facts and Strategies 4.7 15
Answer:
Given that,
There are 5 bird eggs. An equal number of eggs hatch each day for 5 days.
5/5 = 1
Therefore 1 egg hatch each day.

Question 19.
Modeling Real Life
A hotel orders twenty-four 48-inch televisions and twenty-six 60-inch televisions. The hotel puts 1 television in each room. How many rooms can get a new television?
Answer:
Given,
A hotel orders twenty-four 48-inch televisions and twenty-six 60-inch televisions.
The hotel puts 1 television in each room.
24 + 26 = 50
Thus 50 rooms can get a new television.

Review & Refresh

Find the product.
Question 20.
2 × 4 × 1 = _____
Answer: 8

Explanation:
First multiply 2 × 4
2 × 4 = 8
8 × 1 = 8
So, 2 × 4 × 1 = 8

Question 21.
2 × 3 × 5 = _____
Answer: 30

Explanation:
Multiply 2 and 3
2 × 3 = 6
6 × 5 = 30
So, 2 × 3 × 5 = 30

Question 22.
2 × 3 × 7 = _____
Answer: 42

Explanation:
Multiply 2 and 3
2 × 3 = 6
6 × 7 = 42
So, 2 × 3 × 7 = 42

Question 23.
5 × (2 × 9) = _____
Answer: 90

Explanation:
First, multiply the numbers in the bracket.
2 × 9 = 18
18 × 5 = 90

Question 24.
2 × (3 × 3) = _____
Answer: 18

Explanation:
First, multiply the numbers in the bracket.
3 × 3 = 9
9 × 2 = 18

Question 25.
2 × (8 × 1) = _____
Answer: 16

Explanation:
First, multiply the numbers in the bracket.
8 × 1 = 8
2 × 8 = 16

Lesson 4.8 Practice Division Strategies

Explore and Grow

Use and Strategies to find the quotient
36 ÷ 4 = _____
Answer: 9

Explanation:
You can find the quotient by using the number line.

Construct Arguments
What other strategies can you use to solve? Explain the strategy to your partner.
Answer:
You can also find the quotient by using the repeated subtraction.
36 – 4 = 32
32 – 4 = 28
28 – 4 = 24
24 – 4 = 20
20 – 4 = 16
16 – 4 = 12
12 – 4 = 8
8 – 4 = 4
4 – 4 = 0
Count the number of times subtraction repeated.
The subtraction repeated 9 times.
Thus the quotient is 9.

Think and Grow: Practice Division Strategies

Example
Find 45 ÷ 5.
One Way:
Use a multiplication fact. Draw an array to help.
Big Ideas Math Solutions Grade 3 Chapter 4 Division Facts and Strategies 4.8 1

Another Way:
Use a number line. Start at 45. Count back by 5s until you reach 0.
Big Ideas Math Solutions Grade 3 Chapter 4 Division Facts and Strategies 4.8 2

Show and Grow

Use any strategy to find the quotient.
Question 1.
30 ÷ 6 = _____
Answer: 5

Explanation:

Big Ideas Math Answers 3rd Grade Chapter 4 Division Facts and Strategies 4.2 1

Multiplication:
5 rows of 6 counters
5 × 6 = 30
30 counters
Division:
30 counters in 5 equal rows
30 ÷ 5 = 6
60 counters in each row.
Fact family for 5, 6 and 30:
5 × 6 = 30
6 × 5 = 30
30 ÷ 5 = 6
30 ÷ 6 = 5

Question 2.
56 ÷ 8 = _____
Answer: 7

Explanation:
Number of rows = 8
Number of counters = 56
Divide the number of rows by the number of counters.
56/8 = 7
Thus the quotient is 7.

Apply and Grow: Practice

Use any strategy to find the quotient.
Question 3.
40 ÷ 10 = _____
Answer: 4

Explanation:
You can find the quotient by using the repeated subtraction.
40 – 10 = 30
30 – 10 = 20
20 – 10 = 10
10 – 10 = 0
Thus the quotient is 4.

Question 4.
18 ÷ 3 = _____
Answer: 6

Explanation:
You can find the quotient by using the repeated subtraction.
18 – 3 = 15
15 – 3 = 12
12 – 3 = 9
9 – 3 = 6
6 – 3 = 3
3 – 3 = 0
Thus the quotient is 6.

Question 5.
49 ÷ 7 = ____
Answer: 7

Explanation:
You can find the quotient by using the repeated subtraction.
49 – 7 = 42
42 – 7 = 35
35 – 7 = 28
28 – 7 = 21
21 – 7 = 14
14 – 7 = 7
7 – 7 = 0
Thus the quotient is 7.

Question 6.
____ = 36 ÷ 9
Answer: 4

Explanation:
You can find the quotient by using the repeated subtraction.
36 – 9 = 27
27 – 9 = 18
18 – 9 = 9
9 – 9 = 0
Thus the quotient is 4.

Question 7.
____ = 4 ÷ 4
Answer: 1

Explanation:
You can find the quotient by using the repeated subtraction.
4 – 4 = 0
Thus the quotient is 1.

Question 8.
______ = 18 ÷ 2
Answer: 9

Explanation:
You can find the quotient by using the repeated subtraction.
18 – 2 = 16
16 – 2 = 14
14 – 2 = 12
12 – 2 = 10
10 – 2 = 8
8 – 2 = 6
6 – 2 = 4
4 – 2 = 2
2 – 2 = 0
Thus the quotient is 0.

Question 9.
Big Ideas Math Solutions Grade 3 Chapter 4 Division Facts and Strategies 4.8 3
Answer: 12

Explanation:
You can find the quotient by using the repeated subtraction.
12 – 1 = 11
11 – 1 = 10
10 – 1 = 9
9 – 1 = 8
8 – 1 = 7
7 – 1 = 6
6 – 1 = 5
5 – 1 = 4
4 – 1 = 3
3 – 1 = 2
2 – 1 = 1
1 – 1 = 0
Thus the quotient is 12.

Question 10.
Big Ideas Math Solutions Grade 3 Chapter 4 Division Facts and Strategies 4.8 4
Answer: 8

Explanation:
Divide the two numbers 5 and 40
40/5 = 8
Thus the quotient is 8.

Question 11.
Big Ideas Math Solutions Grade 3 Chapter 4 Division Facts and Strategies 4.8 5
Answer: 6

Explanation:
Divide the two numbers 8 and 48
48/8 = 6
Thus the quotient is 6.

Question 12.
Big Ideas Math Solutions Grade 3 Chapter 4 Division Facts and Strategies 4.8 6
Answer: 4

Explanation:
Divide the two numbers 6 and 24
24/6 = 4
Thus the quotient is 4.

Question 13.
Big Ideas Math Solutions Grade 3 Chapter 4 Division Facts and Strategies 4.8 7
Answer: 5

Explanation:
Divide the two numbers 4 and 20.
20/4 = 5
Thus the quotient is 5.

Question 14.
Big Ideas Math Solutions Grade 3 Chapter 4 Division Facts and Strategies 4.8 8
Answer: 3

Explanation:
Divide the two numbers 7 and 21
21/7 = 3
Thus the quotient is 3.

Question 15.
Big Ideas Math Solutions Grade 3 Chapter 4 Division Facts and Strategies 4.8 9
Answer: 10

Explanation:
Divide the two numbers 2 and 20.
20/2 = 10
Thus the quotient is 10.

Question 16.
Big Ideas Math Solutions Grade 3 Chapter 4 Division Facts and Strategies 4.8 10
Answer: 0

Explanation:
Divide the two numbers 0 and 7
0/7 = 0
Thus the quotient is 0.

Question 17.
Divide 54 by 6.
Answer: 9

Explanation:
Divide the two numbers 54 and 6.
54/6 = 9
Thus the quotient is 9.

Question 18.
Divide 27 by 9.
Answer: 3

Explanation:
Divide the two numbers 9 and 27.
27/9 = 3
Thus the quotient is 3.

Question 19.
Divide 70 by 10.
Answer: 7

Explanation:
Divide the two numbers 10 and 70
70/10 = 7
Thus the quotient is 7.

Question 20.
A construction worker wants to build a 32-foot-long wall in 8 hours. How many feet should he complete each hour so that it is built on time?
Big Ideas Math Solutions Grade 3 Chapter 4 Division Facts and Strategies 4.8 11
Answer: 4

Explanation:
Given that,
A construction worker wants to build a 32-foot-long wall in 8 hours.
32/4 = 4
Thus he should complete 4 feet each hour.

Question 21.
Writing
How can you use multiplication to help find 42 ÷ 7? Explain.
Answer:
42 = x × 7
x = 42/7
x = 6
So, 42 ÷ 7 = 6

Question 22.
YOU BE THE TEACHER
Your friend says she only needs to write two equations for the 4, 4, 16 fact family. Is your friend correct? Explain.
Answer:
4 × 4 = 16
16 ÷ 4 = 4
Yes, your friend is correct.

Think and Grow: Modeling Real Life

There are 72 oranges at a grocery store arranged into an array with 9 columns. There are 80 lemons arranged into an array with 8 columns. Which fruits as more rows?
Big Ideas Math Solutions Grade 3 Chapter 4 Division Facts and Strategies 4.8 12
Models:
Division equations:
The ______ have more rows.

Answer:
Given that,
There are 72 oranges at a grocery store arranged into an array with 9 columns.
72 ÷ 9 = 8
There are 80 lemons arranged into an array with 8 columns.
80 ÷ 8 = 10
Thus lemons have more rows.

Show and Grow

Question 23.
There are 63 peaches arranged into an array with 7 rows. There are 70 pears arranged into an array with 10 rows. Which fruit has more columns?
Big Ideas Math Solutions Grade 3 Chapter 4 Division Facts and Strategies 4.8 13
Answer:
Given,
There are 63 peaches arranged into an array with 7 rows.
63 ÷ 7 = 9
There are 70 pears arranged into an array with 10 rows.
70 ÷ 10 = 7
Thus peaches have more columns.

Question 24.
Newton has 20 quarters. He wants to trade all of them for bills. How many $1 bills would he get? How many $5 bills would he get?
Answer:
Given that,
Newton has 20 quarters. He wants to trade all of them for bills
20/5 = 4
Thus he will get four $5 bills.

Question 25.
A box of fruit snacks costs $5. Each box has 8 bags. Descartes spends $20 on fruit snacks. How many bags of fruit snacks does he get?
Answer:
Given that,
A box of fruit snacks costs $5. Each box has 8 bags.
$5 × 8 = $40
Descartes spends $20 on fruit snacks.
40 – 20 = $20
Thus Descartes get 4 bags of fruit snacks.

Practice Division Strategies Homework & Practice 4.8

Use any strategy to find the quotient
Question 1.
8 ÷ 1 = ____
Answer: 8

Explanation:
Divide the two numbers 8 and 1.
8/1 = 8
Thus the quotient is 8.

Question 2.
25 ÷ 5 = _____
Answer: 5

Explanation:
Divide the two numbers 5 and 25.
25/5 = 5
Thus the quotient is 5.

Question 3.
72 ÷ 8 = _____
Answer: 9

Explanation:
Divide the two numbers 8 and 72.
72/8 = 9
Thus the quotient is 9.

Question 4.
____ = 45 ÷ 9
Answer: 5

Explanation:
Divide the two numbers 9 and 45
45/9 = 5
Thus the quotient is 5.

Question 5.
____ = 0 ÷ 3.
Answer: 0

Explanation:
Divide the two numbers 0 and 3.
0 divided by any number is zero.
0/3 = 0
Thus the quotient is 0.

Question 6.
____ = 18 ÷ 2
Answer: 9

Explanation:
Divide the two numbers 18 and 2.
18/2 = 9
Thus the quotient is 9.

Question 7.
Big Ideas Math Solutions Grade 3 Chapter 4 Division Facts and Strategies 4.8 14
Answer: 3

Explanation:
Divide the two numbers 9 and 27.
27/9 = 3
Thus the quotient is 3.

Question 8.
Big Ideas Math Solutions Grade 3 Chapter 4 Division Facts and Strategies 4.8 15
Answer: 2

Explanation:
Divide the two numbers 14 and 7.
14/7 = 2
Thus the quotient is 2.

Question 9.
Big Ideas Math Solutions Grade 3 Chapter 4 Division Facts and Strategies 4.8 16
Answer: 10

Explanation:
Divide the two numbers 2 and 20.
20/2 = 10
Thus the quotient is 10.

Question 10.
Big Ideas Math Solutions Grade 3 Chapter 4 Division Facts and Strategies 4.8 17
Answer: 4

Explanation:
Divide the two numbers 3 and 12
12/3 = 4
Thus the quotient is 4.

Question 11.
Big Ideas Math Solutions Grade 3 Chapter 4 Division Facts and Strategies 4.8 18
Answer: 1

Explanation:
Divide the two numbers 16 and 16
16/16 = 1
Thus the quotient is 1.

Question 12.
Big Ideas Math Solutions Grade 3 Chapter 4 Division Facts and Strategies 4.8 19
Answer: 7

Explanation:
Divide the two numbers 10 and 70.
70/10 = 7
Thus the quotient is 7.

Question 13.
Big Ideas Math Solutions Grade 3 Chapter 4 Division Facts and Strategies 4.8 20
Answer: 6

Explanation:
Divide the two numbers 5 and 30.
30/5 = 6
Thus the quotient is 6.

Question 14.
Big Ideas Math Solutions Grade 3 Chapter 4 Division Facts and Strategies 4.8 21
Answer: 8

Explanation:
Divide the two numbers 8 and 64.
64/8 = 8
Thus the quotient is 8.

Question 15.
Divide 60 by 6.
Answer: 10

Explanation:
Divide the two numbers 6 and 60
60/6 = 10
Thus the quotient is 10.

Question 16.
Divide 9 by 3.
Answer: 3

Explanation:
Divide the two numbers 3 and 9.
9/3 = 3
Thus the quotient is 3.

Question 17.
Divide 63 by 9.
Answer: 7

Explanation:
Divide the two numbers 9 and 63.
63/9 = 7
Thus the quotient is 7.

Question 18.
DIG DEEPER!
Without dividing, how can you tell which quotient is smaller, 30 ÷ 5 or 30 ÷ 6? Explain.
Answer: The quotient of 30 ÷ 6  is smaller.

Explanation:
The largest divisor will have the smallest quotient.
30 ÷ 5 = 6
30 ÷ 6 = 5

Question 19.
Number Sense
Which expressions have a quotient of 7?
Big Ideas Math Solutions Grade 3 Chapter 4 Division Facts and Strategies 4.8 23
Answer:
Big-Ideas-Math-Solutions-Grade-3-Chapter-4-Division-Facts-and-Strategies-4.8-23

Explanation:
70/10 = 7
64/8 = 8
35 ÷ 5 = 7
42 ÷ 7 = 6
21 ÷ 3 = 7
24 ÷ 8 = 3
7 ÷ 1 = 7

Question 20.
Reasoning
Use +, -, ×, or to complete the equations.
Big Ideas Math Solutions Grade 3 Chapter 4 Division Facts and Strategies 4.8 22
Answer:
Big-Ideas-Math-Answer-Key-Grade-3-Chapter-4-Division-Facts-and-Strategies-4.5-11

Question 21.
Modeling Real Life
Newton has 60 dimes. He trades all of them for $1 bills. How many bills does he get?
Big Ideas Math Solutions Grade 3 Chapter 4 Division Facts and Strategies 4.8 24
Answer: 6

Explanation:
Newton has 60 dimes. He trades all of them for $1 bills.
Convert from dime to dollar.
10 dime = 1 dollar
60 dimes = $6

Question 22.
Modeling Real Life
Two girls are getting braids in their hair. The first girl wants a total of 30 beads with 3 beads on each braid. The second girl wants a total of 45 beads with 5 beads on each braid. Which girl has more braids?
Answer:
Given that,
Two girls are getting braids in their hair.
The first girl wants a total of 30 beads with 3 beads on each braid.
30/3 = 10
The second girl wants a total of 45 beads with 5 beads on each braid.
45/5 = 9
The first girl has more braids.

Review & Refresh

Find the sum.
Question 23.
Big Ideas Math Solutions Grade 3 Chapter 4 Division Facts and Strategies 4.8 25
Answer: 597

460
+137
597

Question 24.
Big Ideas Math Solutions Grade 3 Chapter 4 Division Facts and Strategies 4.8 26
Answer: 932
625
+297
932

Question 25.
Big Ideas Math Solutions Grade 3 Chapter 4 Division Facts and Strategies 4.8 27
Answer: 750
386
+ 364
750

Lesson 4.9 Problem Solving: Division

Explore and Grow

Use any strategy to solve.
There are 27 cups for a cup-stacking game. There are 3 players. Each player gets an equal number of cups. How many cups does each player get?
Big Ideas Math Answer Key Grade 3 Chapter 4 Division Facts and Strategies 4.9 1
Answer:
Given,
There are 27 cups for a cup-stacking game.
There are 3 players.
27/3 = 9
Each player gets an equal number of cups.
Thus each player gets 9 cups.

Reasoning
Compare your strategy to your partner’s strategy. How are they the same? How are they different?

Think and Grow: Using the Problem-Solving Plan

Example
Newton buys a box of flower vases. The box has 2 rows with 4 vases in each row. The box costs $48. How much does each vase cost?
Understand the Problem
What do you need to find?
• You need to find how much each ______ costs.

Answer: You need to find each vase cost.

What do you know?
• The box has ______ rows of vases.
• Each row has ______ vases.
• The box costs _____.

Answer:
• The box has 2 rows of vases.
• Each row has 4 vases.
• The box costs $48.

Make a Plan

How will you solve?
• Multiply _____ by ______ to find how many _______ are in the box.
• Then divide _______ by the product.

Answer:
• Multiply 2 by 4 to find how many vases are in the box.
• Then divide cost by the product.

Solve
Big Ideas Math Answer Key Grade 3 Chapter 4 Division Facts and Strategies 4.9 2
Each vase costs _______.

Answer:
2 × 4 = 8
48/8 = 6
Thus each vase cost $6.

Show and Grow

Question 1.
Descartes buys 3 packs of colored pencils and a backpack. He spends $33. The backpack costs $12. Each pack of colored pencils costs the same amount. How much is each pack of colored pencils?
Answer:
Given,
Descartes buys 3 packs of colored pencils and a backpack.
He spends $33. The backpack costs $12.
33 – 12 = $21
Each pack of colored pencils costs the same amount.
21/3 = $7
Thus each pack of colored pencils is $7.

Apply and Grow: Practice

Question 2.
You have 4 pages of homework and 2 chores to complete before you get free time. Each task takes 7 minutes to complete. You have60 minutes until dinner. What information do you know that will help you find how much free time you will have before dinner?
Answer:
Given that,
You have 4 pages of homework and 2 chores to complete before you get free time.
Each task takes 7 minutes to complete.
4 + 2 = 6
6 × 7 = 42 minutes
You have 60 minutes until dinner.
60 – 42 = 18 minutes
You will have 18 minutes of free time before dinner.

Question 3.
You are using dots to create a picture. You use 99 dots in all. Forty-six are blue. Thirty-five are purple. You use an equal number of yellow and green dots for the rest of the picture. How many green dots are in your picture?
Big Ideas Math Answer Key Grade 3 Chapter 4 Division Facts and Strategies 4.9 3
Answer:
Given,
You are using dots to create a picture. You use 99 dots in all. Forty-six are blue.
Thirty-five are purple.
35 + 46 = 81
99 – 81 = 18
You use an equal number of yellow and green dots for the rest of the picture.
18/2 = 9
You use 9 yellow dots and 9 green dots.

Question 4.
You are making oatmeal for each of your 4 family members. You have 36 raisins and 20 blueberries. You want everyone to get an equal number of each. How many raisins and how many blueberries should you put in each bowl?
Big Ideas Math Answer Key Grade 3 Chapter 4 Division Facts and Strategies 4.9 4
Big Ideas Math Answer Key Grade 3 Chapter 4 Division Facts and Strategies 4.9 5
Answer:
Given that,
You are making oatmeal for each of your 4 family members.
You have 36 raisins and 20 blueberries.
36 + 20 = 56
You want everyone to get an equal number of each.
56/4 = 14
Therefore 14 raisins and blueberries should put in each bowl.

Think and Grow: Modeling Real Life

You have 2 boxes of crayons. One box has 24 crayons. The other box has 48 crayons. Eighteen of the crayons are broken. You put the crayons that are not broken, into 6 equal groups. How many crayons are in each group?
Big Ideas Math Answer Key Grade 3 Chapter 4 Division Facts and Strategies 4.9 6
Understand the problem:
Make a plan:
Solve:
There are _____ crayons in each group.

Answer:
You have 2 boxes of crayons. One box has 24 crayons. The other box has 48 crayons.
24 + 48 = 72 crayons
Eighteen of the crayons are broken.
72 – 18 = 54 crayons
You put the crayons that are not broken, into 6 equal groups.
54/6 = 9 crayons.
Therefore there are 9 crayons in each group.

Show and Grow

Question 5.
You have 2 bags of beach glass. One bag has 17 pieces. The other bag has 18 pieces. You give 5 pieces to your friend. You sort the rest of the pieces into 3 equal groups. How many pieces are in each group?
Big Ideas Math Answer Key Grade 3 Chapter 4 Division Facts and Strategies 4.9 7
Explain how you can check your answer.
Answer:
Given,
You have 2 bags of beach glass. One bag has 17 pieces. The other bag has 18 pieces.
17 + 18 = 35 pieces
You give 5 pieces to your friend.
35 – 5 = 30 pieces
You sort the rest of the pieces into 3 equal groups.
30/3 = 10
Thus there are 10 pieces in each group.

Problem Solving: Division Homework & Practice 4.9

Question 1.
A truck carries 4 chairs and a table. The table weighs 35 pounds. The total weight of the chairs and table is 63 pounds. How much does each chair weigh?
Answer: 7 pounds

Explanation:
Given that,
A truck carries 4 chairs and a table. The table weighs 35 pounds.
The total weight of the chairs and table is 63 pounds.
63 pounds – 35 pounds = 28 pounds
There are 4 chairs.
Divide 28 by 4 to find the weight of each chair.
28/4 = 7
Therefore each chair weighs 7 pounds.

Question 2.
You volunteer at a nursing home for 3 hours a day. You volunteered 6 hours last week and 12 hours this week. How many days did you volunteer?
Answer: 6 days

Explanation:
Given that,
You volunteer at a nursing home for 3 hours a day.
You volunteered 6 hours last week and 12 hours this week.
6 hours – 2 days
12 hours – 4 days
4 + 2 = 6 days
Thus you volunteer for 6 days.

Question 3.
Writing Write and solve your own word problem involving division.
Answer:
You work for 49 hours a week. How many hours do you work in a day?
Explanation:
You work for 49 hours a week.
Convert from weeks to days
1 week = 7 days
49/7 = 7 hours
Thus you work 7 hours a day.

Question 4.
Modeling Real Life
You have 2 trays of flavored ice cubes. One tray has 16 ice cubes. The other tray has 12 ice cubes. Eight of the ice cubes are not completely frozen yet. You divide the frozen ice cubes equally into 5 glasses. How many ice cubes are in each glass?
Answer:
Given that,
You have 2 trays of flavored ice cubes. One tray has 16 ice cubes.
16 × 2 = 32 ice cubes
The other tray has 12 ice cubes. Eight of the ice cubes are not completely frozen yet.
12 – 8 = 4
32 + 4 = 36 ice cubes
You divide the frozen ice cubes equally into 5 glasses.
36/5 = 7(approx)
Thus you divide 7 frozen ice cubes equally into 5 glasses.

Question 5.
Modeling Real Life
You have 2 bags of seashells. One bag has 13 seashells. The other bag has 25 seashells. You give6 seashells to your friend. You sort the rest of the seashells into 4 equal groups. How many seashells are in each group?
Big Ideas Math Answer Key Grade 3 Chapter 4 Division Facts and Strategies 4.9 8
Answer:
Given that,
You have 2 bags of seashells. One bag has 13 seashells.
2 × 13 = 26 seashells
The other bag has 25 seashells. You give 6 seashells to your friend.
25 – 6 = 19
You sort the rest of the seashells into 4 equal groups.
26 + 19 = 45 seashells
45/4 = 11 (approx)
Thus there are 11 seashells in 4 equal groups.

Review & Refresh

Question 6.
Big Ideas Math Answer Key Grade 3 Chapter 4 Division Facts and Strategies 4.9 9
Answer:
i. 5 sides
ii. 5 vertices
iii. pentagon

Explanation:
The above figure consists of 5 sides and 5 vertices. The polygon with 5 sides is called a pentagon.

Question 7.
Big Ideas Math Answer Key Grade 3 Chapter 4 Division Facts and Strategies 4.9 10
Answer:
i. 4 sides
ii. 4 vertices
iii. Quadrilateral

Explanation:
The above figure consists of 4 sides and 4 vertices. The polygon with 4 sides is called a Quadrilateral.

Division Facts and Strategies Performance Task

You, your friend, and your cousin run for class president. Three teachers and 60 students are gathered in the gym for the election.
Question 1.
There is one teacher in each row. Each row has the same number of students. Find the number of people sitting in each row.
Big Ideas Math Answers 3rd Grade Chapter 4 Division Facts and Strategies 1
Answer:
Given that,
You, your friend, and your cousin run for class president.
Three teachers and 60 students are gathered in the gym for the election.
There is one teacher in each row. Each row has the same number of students.
60/3 = 20 students
Thus 20 people are sitting in each row.

Question 2.
Sixty students vote in the election. Thirty students vote for you. Twenty students vote for your friend. Five students vote for your cousin.
a.How many students still need to vote?

Answer:
Sixty students vote in the election. Thirty students vote for you.
60 – 30 = 30
Twenty students vote for your friend.
30 – 20 = 10
Five students vote for your cousin.
10 – 5 = 5
Thus still 5 students need to vote.

b.The remaining students all vote for your cousin. Complete the picture graph.
Big Ideas Math Answers 3rd Grade Chapter 4 Division Facts and Strategies 2
Answer:
Big-Ideas-Math-Answers-3rd-Grade-Chapter-4-Division-Facts-and-Strategies-2

Question 3.
As class president, you raise $95 to buy new toys for the school playground. Each ball costs $8 and each flying disc costs $6. You want to buy some balls and 3 flying discs. How many balls can you buy? How much money do you have left?
Answer:
Given that,
As class president, you raise $95 to buy new toys for the school playground.
Each ball costs $8 and each flying disc costs $6.
You want to buy some balls and 3 flying discs.
3 × $6 = $18
$95 – $18 = $77
We need to how many balls can you buy for $77.
72/8 = 9
77 – 72 = 5
Thus $5 left.

Division Facts and Strategies Activity

Four in a Row Blastoff!
Directions:
1. Players take turns rolling the die.
2. On your turn, choose and solve any division expression in the row of your roll. If you find the correct answer, place a counter on the space.
3. The first player to create 4 counters in a row, horizontally, vertically, or diagonally, wins! A spaceship blasts off! Cross out one spaceship and play again until all spaceships have blasted off!
Big Ideas Math Answers 3rd Grade Chapter 4 Division Facts and Strategies 3

Answer:
I choose 3, 9 ÷ 3 = 3
I choose 4, 16 ÷ 4 = 4
I choose 5, 25 ÷ 5 = 5
I choose 6, 36 ÷ 6 = 6

Division Facts and Strategies Chapter Practice

4.1 Use Arrays to Divide

Question 1.
There are 20 counters. The counters are in 4 equal rows. How many counters are in each row?
4 rows of ____
20 ÷ 4 = _____
Answer:
Given that,
There are 20 counters. The counters are in 4 equal rows.
20/4 = 5
Thus there are 4 rows of 5 counters
20 ÷ 4 = 5
Thus there are 5 counters in each row.

Question 2.
You have 32 counters. You arrange them with 8 counters in each row. How many rows of counters do you make?
____ rows of 8
32 ÷ 8 = _____
Answer:
Given,
You have 32 counters. You arrange them with 8 counters in each row.
32/8 = 4
4 rows of 8.
32 ÷ 8 = 4
Thus there are 4 rows of counters.

Question 3.
Precision
Label the parts of the division problem using quotient, dividend, and divisor
Big Ideas Math Answers Grade 3 Chapter 4 Division Facts and Strategies chp 3
Answer:
The label for the above problem is
21 is dividend
3 is quotient
7 is divisor

4.2 Relate Multiplication and Division

Write the fact family for the numbers.
Question 4.
1, 5, 5
Answer:
The fact family for the numbers is
Multiplication Equation:
1 × 5 = 5
5 × 1 = 5
Division Equation:
5 ÷ 5 = 1
5 ÷ 1 = 5

Question 5.
2, 7, 14
Answer:
The fact family for the numbers is
Multiplication Equation:
2 × 7 = 14
7 × 2 = 14
Division Equation:
14 ÷ 2 = 7
14 ­÷ 7 = 2

Question 6.
9, 5, 45
Answer:
The fact family for the numbers is
Multiplication Equation:
9 × 5 = 45
5 × 9 = 45
Division Equation:
45 ÷ 5 = 9
45 ÷ 9 = 5

Question 7.
8, 6, 48
Answer:
The fact family for the numbers is
Multiplication Equation:
8 × 6 = 48
6 × 8 = 48
Division Equation:
48 ÷ 6 = 8
48 ÷ 8 = 6

4.3 Divide 2, 5, or 10

Write the related multiplication fact. The find the quotient.
Question 8.
Find 15 ÷ 5
5 × ____ = 15
15 ÷ 5 = ____
Answer:
Let the unknown number be x.
5 × x = 15
x = 15/5
x = 3
15 ÷ 5 = 3

Question 9.
Find 14 ÷ 2
2 × ____ = 14
14 ÷ 2 = ____
Answer:
Let the unknown number be x.
2 × x = 14
x = 14/2
x = 7
14 ÷ 2 = 7

Question 10.
Find 40 ÷ 10
10 × ____ = 40
40 ÷ 10 = ____
Answer:
Let the unknown number be x.
10 × x = 40
x = 40/10
x = 4
40 ÷ 10 = 4

Find the quotient.
Question 11.
60 ÷ 10 = _____
Answer: 6

Explanation:
Divide the two numbers 60 and 10.
60/10 = 6
Thus the quotient is 6.

Question 12.
20 ÷ 5 = _____
Answer: 4

Explanation:
Divide the two numbers 20 and 5.
20/5 = 4
Thus the quotient is 4.

Question 13.
20 ÷ 2 = _____
Answer: 10

Explanation:
Divide the two numbers 20 and 2.
20/2 = 10
Thus the quotient is 10.

Question 14.
Big Ideas Math Answers Grade 3 Chapter 4 Division Facts and Strategies chp 14
Answer: 5

Explanation:
Divide the two numbers 25 and 5.
25/5 = 5
Thus the quotient is 5.

Question 15.
Big Ideas Math Answers Grade 3 Chapter 4 Division Facts and Strategies chp 15
Answer: 9

Explanation:
Divide the two numbers 18 and 2.
18/2 = 9
Thus the quotient is 9.

Question 16.
Big Ideas Math Answers Grade 3 Chapter 4 Division Facts and Strategies chp 16

Answer: 3

Explanation:
Divide the two numbers 30 and 10.
30/10 = 3
Thus the quotient is 3.

4.4 Divide by 3 or 4

Find the quotient.
Question 17.
18 ÷ 3 = _____
Answer: 6

Explanation:
Divide the two numbers 18 and 3.
18/3 = 6
Thus the quotient is 6.

Question 18.
40 ÷ 4 = ____
Answer: 10

Explanation:
Divide the two numbers 40 and 4.
40/4 = 10
Thus the quotient is 10.

Question 19.
24 ÷ 4 = _____
Answer: 6

Explanation:
Divide the two numbers 24 and 4.
24/4 = 6
Thus the quotient is 6.

Question 20.
Big Ideas Math Answers Grade 3 Chapter 4 Division Facts and Strategies chp 20
Answer: 9

Explanation:
Divide the two numbers 3 and 27.
27/3 = 9
Thus the quotient is 9.

Question 21.
Big Ideas Math Answers Grade 3 Chapter 4 Division Facts and Strategies chp 21
Answer: 3

Explanation:
Divide the two numbers 12 and 4.
12/4 = 3
Thus the quotient is 3.

Question 22.
Big Ideas Math Answers Grade 3 Chapter 4 Division Facts and Strategies chp 22
Answer: 8

Explanation:
Divide the two numbers 24 and 3.
24/3 = 8
Thus the quotient is 8.

Question 23.
Structure
Write the division equation represented by the number line.
Big Ideas Math Answers Grade 3 Chapter 4 Division Facts and Strategies chp 23
____ ÷ ____ = ____
Answer: 21 ÷ 3 = 7
Explanation:
The count starts from 0.
The count jumps for every 3s.
The division equation is 21 ÷ 3 = 7

4.5 Divide by 6 or 7

Find the quotient
Question 24.
42 ÷ 6 = _____
Answer: 7

Explanation:
Divide the two numbers 42 and 6.
42/6 = 7
Thus the quotient is 7.

Question 25.
54 ÷ 6 = _____
Answer: 9

Explanation:
Divide the two numbers 54 and 6.
54/6 = 9
Thus the quotient is 9.

Question 26.
28 ÷ 7 = ____
Answer: 4

Explanation:
Divide the two numbers 28 and 7.
28/7 = 4
Thus the quotient is 4.

Question 27.
Big Ideas Math Answers Grade 3 Chapter 4 Division Facts and Strategies chp 27
Answer: 3

Explanation:
Divide the two numbers 21 and 7.
21/7 = 3
Thus the quotient is 3.

Question 28.
Big Ideas Math Answers Grade 3 Chapter 4 Division Facts and Strategies chp 28
Answer: 6

Explanation:
Divide the two numbers 7 and 42.
42/7 = 6
Thus the quotient is 6.

Question 29
Big Ideas Math Answers Grade 3 Chapter 4 Division Facts and Strategies chp 29
Answer: 5

Explanation:
Divide the two numbers 30 and 6.
30/6 = 5
Thus the quotient is 5.

Find the missing divisor
Question 30.
18 ÷ ____ = 6
Answer: 3

Explanation:
Let the missing divisor be u.
18 ÷ u = 6
18/u = 6
18 = 6 × u
u = 18/6
u = 3
Thus the missing divisor is 3.

Question 31.
35 ÷ ____ = 7
Answer: 5

Explanation:
Let the missing divisor be v.
35 ÷ v = 7
35/v = 7
35 = 7 × v
v = 35/7
v = 5
Thus the missing divisor is 5.

Question 32.
36 ÷ ____ = 6
Answer: 6

Explanation:
Let the missing divisor be w.
36 ÷ w = 6
36/w = 6
36 = 6 × w
w = 36/6
w = 6
Thus the missing divisor is 6.

4.6 Divide by 8 or 9

Find the quotient
Question 33.
36 ÷ 9 = _____
Answer: 4

Explanation:
Divide the two numbers 36 and 9
36/9 = 4
Thus the quotient is 4.

Question 34.
64 ÷ 8 = ____
Answer: 8

Explanation:
Divide the two numbers 8 and 64
64/8 = 8
Thus the quotient is 8.

Question 35.
90 ÷ 9 = _____
Answer: 10

Explanation:
Divide the two numbers 90 and 9.
90/9 = 10
Thus the quotient is 10.

Question 36.
Big Ideas Math Answers Grade 3 Chapter 4 Division Facts and Strategies chp 36
Answer: 4

Explanation:
Divide the two numbers 32 and 8.
32/8 = 4
Thus the quotient is 4.

Question 37.
Big Ideas Math Answers Grade 3 Chapter 4 Division Facts and Strategies chp 37
Answer: 5

Explanation:
Divide the two numbers 45 and 9.
45/9 = 5
Thus the quotient is 5.

Question 38.
Big Ideas Math Answers Grade 3 Chapter 4 Division Facts and Strategies chp 38
Answer: 7

Explanation:
Divide the two numbers 56 and 8
56/8 = 7
Thus the quotient is 7.

Compare
Question 39.
Big Ideas Math Answers Grade 3 Chapter 4 Division Facts and Strategies chp 39
Answer: >

Explanation:
Divide the two numbers.
63 ÷ 9 = 7
48 ÷ 8 = 6
7 > 6
So, 63 ÷ 9 > 48 ÷ 8

Question 40.
Big Ideas Math Answers Grade 3 Chapter 4 Division Facts and Strategies chp 40
Answer: =

Explanation:
Divide the two numbers
32 ÷ 8 = 4
36 ÷ 9 = 4
4 = 4
So, 32 ÷ 8 = 36 ÷ 9

Question 41.
Big Ideas Math Answers Grade 3 Chapter 4 Division Facts and Strategies chp 41
Answer: <

Explanation:
Divide the two numbers
54 ÷ 9 = 6
80 ÷ 8 = 10
6 < 10
So, 54 ÷ 9 < 80 ÷ 10

Question 42.
A crayon box has 48 crayons. There are 8 crayons in each row. How many rows of crayons are there?
Big Ideas Math Answers Grade 3 Chapter 4 Division Facts and Strategies chp 42
Answer: 6

Explanation:
Given,
A crayon box has 48 crayons. There are 8 crayons in each row.
48/8 = 6
Thus there are 6 rows of crayons.

4.7 Divide with 0 or 1

Find the quotient
Question 43.
13 ÷ 13 = _____
Answer: 1

Explanation:
Divide the two numbers 13 and 13.
13/13 = 1
Thus the quotient is 1.

Question 44.
0 ÷ 9 = _____
Answer: 0

Explanation:
Divide the two numbers 0 and 9
Any number divided by 0 will be always 0.
0/9 = 0
Thus the quotient is 0.

Question 45.
3 ÷ 1 = _____
Answer: 3

Explanation:
Divide the two numbers 3 and 1.
3/1 = 3
Thus the quotient is 3.

Question 46.
Big Ideas Math Answers Grade 3 Chapter 4 Division Facts and Strategies chp 46
Answer: 2

Explanation:
Divide the two numbers 1 and 2.
2/1 = 2
Thus the quotient is 2.

Question 47.
Big Ideas Math Answers Grade 3 Chapter 4 Division Facts and Strategies chp 47
Answer: 0

Explanation:
Divide the two numbers 0 and 5.
Any number divided by 0 will be always 0.
Thus the quotient is 0.

Question 48.
Big Ideas Math Answers Grade 3 Chapter 4 Division Facts and Strategies chp 48
Answer: 1

Explanation:
Divide the two numbers 10 and 10.
10/10 = 1
Thus the quotient is 1.

Question 49.
Number Sense
Which has the greatest quotient?
15 ÷ 15
0 ÷ 10
6 ÷ 1
8 ÷ 1
Answer: 8 ÷ 1

Explanation:
Check all the quotient
15 ÷ 15 = 1
0 ÷ 10 = 0
6 ÷ 1 = 6
8 ÷ 1 = 8
Thus the correct answer is option D.

4.8 Practice Division Strategies

Use any Strategy to find the quotient
Question 50.
16 ÷ 4 = _____
Answer: 4

Explanation:
Divide the two numbers 4 and 16.
16/4 = 4
Thus the quotient is 4.

Question 51.
72 ÷ 9 = ____
Answer: 8

Explanation:
Divide the two numbers 9 and 72.
72/9 = 8
Thus the quotient is 8.

Question 52.
20 ÷ 10 = _____
Answer: 2

Explanation:
Divide the two numbers 20 and 10.
20/10 = 2
Thus the quotient is 2.

Question 53.
Divide 18 by 6.
Answer: 3

Explanation:
Divide the two numbers 18 and 6.
18/6 = 3
Thus the quotient is 3.

Question 54.
Divide 49 by 7.
Answer: 7

Explanation:
Divide the two numbers 49 and 7.
49/7 = 7
Thus the quotient is 7.

Question 55.
Divide 30 by 6.
Answer: 5

Explanation:
Divide the two numbers 30 and 6.
30/6 = 5
Thus the quotient is 5.

4.9 Problem Solving: Division

Question 56.
Modeling Real Life
A group of 13 servicemen and 11 servicewomen are being honored at an event. They stand in 3 equal rows. How many servicemen and servicewomen are in each row?
Big Ideas Math Answers Grade 3 Chapter 4 Division Facts and Strategies chp 56
Answer: 9

Explanation:
Given,
A group of 13 servicemen and 11 servicewomen are being honored at an event.
13 + 14 = 27
They stand in 3 equal rows.
27/3 = 9
Thus there are 9 servicemen and servicewomen in each row.

Division Facts and Strategies Cumulative Practice

Question 1.
Which expression is the same as 6 × 3?
A. 3 × 6
B. 3 + 3 + 3 + 3 + 3 + 3
C. 6 + 6 + 6
D. 6 × 6 × 6
Answer: A. 3 × 6

Explanation:
Given the expression 6 × 3
By using the multiplication equation
we can say that 3 × 6 is the same as 6 × 3
Thus the correct answer is option A.

Question 2.
Use the clues to find the number.
• The number is more than 10.
• The number is less than 20.
• When the number is divided by 3, the quotient is even.
• When the number is divided by 4, the quotient is odd.
What is the number?
A. 15
B. 12
C. 24
D. 16
Answer: B. 12

Explanation:
Let the number be 12
12 ÷ 3 = 4 (even)
12 ÷ 4 = 3 (odd)
Thus the correct answer is option B.

Question 3.
Which expression is equal to 6 × 7?
A. (7 × 8) – 2
B. (5 × 7) + (1 × 7)
C. (5 × 1) + (5 × 2)
D. (5 × 1) × 7
Answer: B. (5 × 7) + (1 × 7)

Explanation:
A. (7 × 8) – 2
56 – 2 = 54
B. (5 × 7) + (1 × 7)
35 + 7 = 42
C. (5 × 1) + (5 × 2)
5 + 10 = 15
D. (5 × 1) × 7
5 × 7 = 35
6 × 7 = 42
Thus the correct answer is option B.

Question 4.
Which number makes the statement true?
Big Ideas Math Solutions Grade 3 Chapter 4 Division Facts and Strategies cp 4
A. 4
B. 8
C. 16
D. 2
Answer: A. 4

Explanation:
2 × 4 = x × 2
8 = 2x
x = 8/2
x = 4
Thus the correct answer is option A.

Question 5.
Which equation is true?
Big Ideas Math Solutions Grade 3 Chapter 4 Division Facts and Strategies cp 5
Answer: C

Explanation:
A. (5 × 4) × 1 = (5 × 4) + 1
20 ≠ 21
B. (5 × 4) + 1 = (5 × 4) × 1
21 ≠ 20
C. (5 × 4) × 1 = (5 × 4) × 1
20 = 20
D. (5 × 4) × 1 = 5 + (4 × 1)
20 ≠ 9
Thus the correct answer is option C.

Question 6.
You earn 10 points for each level you complete in a video game. When you have to restart a level, you lose 2 points. You complete 9 levels but have to restart 6 levels. How many points do you have?
Big Ideas Math Solutions Grade 3 Chapter 4 Division Facts and Strategies cp 6
A. 78 points
B. 102 points
C. 90 points
D. 42 points
Answer: 78 points

Explanation:
Given,
You earn 10 points for each level you complete in a video game.
When you have to restart a level, you lose 2 points.
You complete 9 levels but have to restart 6 levels.
9 × 10 = 90 points
6 × 2 = 12 points
90 – 12 = 78 points
Thus the correct answer is option A.

Question 7.
The table shows the numbers of items at the finish line of a race.
Big Ideas Math Solutions Grade 3 Chapter 4 Division Facts and Strategies cp 7
Part A How many medals are there?

Answer:
Medals:
Number of packages = 7
Number in each package = 9
7 × 9 = 63 medals

PartB Sixty students are running in the race. Is there enough of each item for all of the students? Explain.
Answer:
Given,
Sixty students are running in the race.
Water bottles:
Number of packages = 9
Number in each package = 8
9 × 8 = 72
Medals:
Number of packages = 7
Number in each package = 9
7 × 9 = 63 medals
Bananas:
Number of packages = 9
Number in each package = 6
9 × 6 = 54
Thus there are not enough bananas for all 60 students.

Question 8.
Which numbers are multiples of 2, 5, and 10?
Big Ideas Math Solutions Grade 3 Chapter 4 Division Facts and Strategies cp 8
Answer: 20

Explanation:
The fact family for 2, 5 and 10.
2 × 10 = 20
Thus the correct answer is option A.

Question 9.
Newton and Descartes share 18 toys equally. Which equation shows how many toys each has?
A. 18 ÷ 2 = 9
B. 18 – 2 = 16
C. 18 × 2 = 36
D. 18 ÷ 1 = 18
Answer: 18 ÷ 2 = 9

Explanation:
Given,
Newton and Descartes share 18 toys equally.
18/2 = 9
Thus the correct answer is option A.

Question 10.
Which multiplication fact can Newton use to check his answer?
72 ÷ 8 = 9
Big Ideas Math Solutions Grade 3 Chapter 4 Division Facts and Strategies cp 10
Answer: 9 × 8

Explanation:
72 ÷ 8 = 9
The multiplication fact for 9, 8 and 72 is
9 × 8 = 72
Thus the correct answer is option D.

Question 11.
Thirty dollars is divided evenly among 6 children. How much money (in dollars) does each child receive?
Big Ideas Math Solutions Grade 3 Chapter 4 Division Facts and Strategies cp 11
Answer:
Given,
Thirty dollars is divided evenly among 6 children
30/6 = 5
Thus each child receives 5 dollars.

Question 12.
Which expressions have a product of 16?
Big Ideas Math Solutions Grade 3 Chapter 4 Division Facts and Strategies cp 12
Answer: A. 4 × 4

Explanation:
The product of 16 is 4 × 4.
Thus the correct answer is option A.

Question 13.
Which equation shows the Multiplication Property of Zero?
A. 8 + 0 = 8
B. 8  – 0 = 8
C. 8 × 1 = 8
D. 8 × 0 = 0
Answer: D. 8 × 0 = 0

Explanation:
Any number multiplied by 0 is always 0.|
8 × 0 = 0
Thus the correct answer is option D.

Question 14.
One ticket to an amusement park costs $8. How much does it cost for a family of 4?
Big Ideas Math Solutions Grade 3 Chapter 4 Division Facts and Strategies cp 14.1
Big Ideas Math Solutions Grade 3 Chapter 4 Division Facts and Strategies cp 14.2
Answer: A. $32

Explanation:
Given,
One ticket to an amusement park costs $8.
8 × 4 = $32
Thus the correct answer is option A.

Question 15.
Which expression is represented by this number line?
Big Ideas Math Solutions Grade 3 Chapter 4 Division Facts and Strategies cp 15
A. 21 + 3
B. 3 × 7
C. 21 – 3
D. 7 + 3
Answer: B. 3 × 7

Explanation:
The count starts at 0.
It takes 3 jumps of 7.
3 × 7 = 21
Thus the correct answer is option B.

Question 16.
Which equations are in the fact family for 2, 10, and 20?
Big Ideas Math Solutions Grade 3 Chapter 4 Division Facts and Strategies cp 16
Answer: A, B, D, E

Explanation:
The fact family for 2, 10, 20 is:
2 × 10 = 20
10 × 2 = 20
20 ÷ 2 = 10
20 ÷ 10 = 2
Thus the correct answer is option A, B, D, E.

Division Facts and Strategies STEAM Performance Task

Big Ideas Math Solutions Grade 3 Chapter 4 Division Facts and Strategies sp 1

Dogs grow just like humans. They start as puppies, become adults, and then enter old age. But dogs age faster than humans. In general, to find the age of a small dog in dog years, multiply its age in human years by 5. To find the age of a large dog in dog years, multiply its age in human years by 7.

Question 1.
A small dog and a large dog are both 9 years old in human years. How old is each dog in dog years?
Answer:
Given,
A small dog and a large dog are both 9 years old in human years.
To find the age of a small dog in dog years, multiply its age in human years by 5.
5 × 9 = 45 years
Thus the age of a small dog is 45 years
To find the age of a large dog in dog years, multiply its age in human years by 7.
9 × 7 = 63 years
Thus the age of a large dog is 63 years.

Question 2.
The table shows the usual life spans of 6 dog breeds in dog years. Find each life span in human years. Organize the data in a bar graph.
Big Ideas Math Solutions Grade 3 Chapter 4 Division Facts and Strategies sp 2
Big Ideas Math Solutions Grade 3 Chapter 4 Division Facts and Strategies sp 3
Answer:
Big-Ideas-Math-Solutions-Grade-3-Chapter-4-Division-Facts-and-Strategies-sp-3

Question 3.
Write and answer a question about your graph.
Answer:
Draw the graph for the above table.
Bigideas math answers grade 3 chapter 4 division facts and strategies img_6

Question 4.
Cats also age faster than humans. The table shows the age of a cat for each human year.
Big Ideas Math Solutions Grade 3 Chapter 4 Division Facts and Strategies sp 4
a. A cat is 6 years old in human years. If the pattern in the table continues, how old is the cat in cat years?
Big Ideas Math Solutions Grade 3 Chapter 4 Division Facts and Strategies sp 5

Answer:
Given that,
A cat is 6 years old in human years.
By using the above table we can find the age of the cat in cat years.
6 years – 40 years

b. A cat is 10 years old in human years. How old is the cat in cat years?

Answer:
Given that,
A cat is 10 years old in human years.
By using the above table we can find the age of the cat in cat years.
1 year (human) = 4 years of cat
10 years = 56 years
Thus the age of cat in cat years is 56 years.

c. A small dog, a large dog, and a cat are each 6 years old (in human years). Order the animals by their ages (in animal years) from least to greatest.
Big Ideas Math Solutions Grade 3 Chapter 4 Division Facts and Strategies sp 6
Answer:
A small dog, a large dog, and a cat are each 6 years old (in human years).
Small dog in dog years, multiply its age in human years by 5.
6 × 5 = 30 years
To find the age of a large dog in dog years, multiply its age in human years by 7.
7 × 7 = 49 years
A cat is 6 years old in human years.
6 years – 40 years
Now arrange them from least to the greatest.
small dog, cat, and a large dog.

Question 5.
Use the Internet or another resource to answer each question.
a. What is another dog breed that is not listed in the table?

Answer: Bull Dog

b. About how long, how tall, and how heavy is one of these dogs?
Length: _____ inches
Height: _____ inches
Weight: ______ pounds

Answer:
The length, height, and weight of the bulldog is given below
length – 12 to 15 inches
height – 12 to 15 inches
weight – 40 to 43 pounds

c. What is one interesting fact about this dog breed?

Answer:
The interesting fact about bulldog is
1. These are independent
2. The dogs were bred for their wrinkles.

d. About how long would you expect this breed to live?
Answer: The life span of the bulldog is 8 to 10 years.

Conclusion:

I hope that the Big Ideas Math Answers Grade 3 Chapter 4 Division Facts and Strategies Solution Key is useful for the students who are preparing for the exams. You can get the free download link of Big Ideas Math Book Grade 3 Chapter 4 Division Facts and Strategies Answer Key PDF. Stay in touch with us to find the solutions for other chapters of 3rd grade.

Big Ideas Math Answers Grade 7 Accelerated | Big Ideas Math Book 7th Grade Accelerated Answer Key

Big Ideas Math Answers Grade 7 Accelerated

Trying to provide your kid the necessary practice for Big Ideas Math Grade 7 Accelerated Topics as a part of their preparation? Don’t bother as we have listed the best preparation resources all in one place. Make the most out of the quick links available here for 7th Grade Accelerated Big Ideas Math Answers and score better grades in exams. Achieve your learning targets easily by availing the handy study material for Big Ideas Math 7th Grade Accelerated Answer Key and clear your exams with flying colors.

Big Ideas Math Book 7th Grade Accelerated Answer Key | 7th Grade Accelerated Big Ideas Math Solutions

We have provided the 7th Grade Accelerated BIM Textbook Answers for all the Chapters in detail here. Refer to them as a part of your preparation and resolve your queries at the moment. All the BIM Book Grade 7 Accelerated Solutions given here are as per the latest textbooks. Questions covered in the Big Ideas Math Modeling Real Life Grade 7 Accelerated Answers are from Exercises, Chapter Tests, Review Tests, Cumulative Practice, Assessment Tests, etc.

Importance of High School BIM Grade 7 Accelerated Solution Key

There are numerous perks of studying from the Big Ideas Math Grade 7 Accelerated Solutions. They are in the below fashion

  • You can assess your preparation standards taking the help of the Big Ideas Math 7th Grade Accelerated Solutions.
  • Gain a deeper understanding of the concepts taking the help of the BIM Book Grade 7 Accelerated Answer Key.
  • Big Ideas Math Grade 7 Accelerated Textbook Solutions provided covers questions from Performance Tests, Review Tests, Chapter Tests, Exercises, Cumulative Practice, Assessment Tests, etc.
  • All the 7th Grade Accelerated Big Ideas Math Answer Key PDF is given adhering to the latest Common Core State Standards Curriculum.
  • Engaging Activities, Thought Provoking Exercises provided builds problem-solving skills and enhances math proficiency.

FAQs on BigIdeas Math Grade 7 Accelerated Textbook Solution Key

1. Where do I get the Grade 7 Accelerated BIM Answers for all Chapters?

You can find the Grade 7 Accelerated Big Ideas Math Solutions for all Chapters on our page.

2. Can I Score better grades in exams taking the help of the Big Ideas Math Grade 7 Accelerated Textbook Solutions?

Yes, taking the help of these Big Ideas Math Grade 7 Accelerated Answer Key you can score better grades in exams. Alongside complement your preparation with the right textbooks and clear the exam with utmost confidence.

3. How to access Modeling Real Life 7th Grade Big Ideas Math Answer Key?

All you have to do is just tap on the quick links and avail the respective chapter in the Big Ideas Math 7th Grade Accelerated and begin your preparation right away.

Big Ideas Math Geometry Answers Chapter 5 Congruent Triangles

Big Ideas Math Geometry Answers Chapter 5

If you are looking across the web for better preparation resources regarding the Big Ideas Math Geometry Ch 5 Congruent Triangles then this is the one stop destination for all your needs. For better understanding of the concepts we have compiled all the Big Ideas Math Geometry Answers Ch 5 Congruent Triangles in a simple and easy to understand language. Boost your math skills and have a deeper understanding of concepts taking help from the Congruent Triangles BIM Book Geometry Solution Key.

Big Ideas Math Book Geometry Answer Key Chapter 5 Congruent Triangles

Prepare using the Big Ideas Math Book Geometry chapter 5 Congruent Triangles Answer Key and get a good hold of the entire concepts. Clarify all your doubts taking help of the BIM Book Geometry Ch 5 Congruent Triangles Solutions provided. Simply tap on the BIM Geometry Chapter 5 Congruent Triangles Answers and prepare the corresponding topic in no time. Big Ideas Math Geometry Congruent Triangles Solution Key covers questions from Lessons 5.1 to 5.8, Practice Tests, Assessment Tests, Chapter Tests, etc. Attempt the exam with confidence and score better grades in exams.

Congruent Triangles Maintaining Mathematical Proficiency

Find the coordinates of the midpoint M of the segment with the given endpoints. Then find the distance between the two points.

Question 1.
P(- 4, 1) and Q(0, 7)
Answer:
The given points are:
P (-4, 1), Q (0, 7)
We know that,
The midpoint M of the segment with the 2 endpoints is:
( \(\frac{x1 + x2}{2}\), \(\frac{y1 + y2}{2}\) )
Let the give points are:
(x1, y1) and (x2, y2)
So,
By comparing the given poits,
We will get
x1 = -4, x2 = 0, y1 = 1, y2 = 7
Hence,
The midpoint M = ( \(\frac{-4 + 0}{2}\), \(\frac{1 + 7}{2}\) )
= ( \(\frac{-4}{2}\), \(\frac{8}{2}\) )
= (-2, 4)
Hence, from the above,
We can conclude that the midpoint M of the segment with the given endpoints is: (-2, 4)

Question 2.
G(3, 6) and H(9, – 2)
Answer:
The given points are:
G (3, 6), H (9, -2)
We know that,
The midpoint M of the segment with the 2 endpoints is:
( \(\frac{x1 + x2}{2}\), \(\frac{y1 + y2}{2}\) )
Let the give points are:
(x1, y1) and (x2, y2)
So,
By comparing the given poits,
We will get
x1 = 3, x2 = 9, y1 = 6, y2 = -2
Hence,
The midpoint M = ( \(\frac{3 + 9}{2}\), \(\frac{6 – 2}{2}\) )
= ( \(\frac{12}{2}\), \(\frac{4}{2}\) )
= (6, 2)
Hence, from the above,
We can conclude that the midpoint M of the segment with the given endpoints is: (6, 2)

Question 3.
U(- 1, – 2) and V(8, 0)
Answer:
The given points are:
U (-1, -2), V (8, 0)
We know that,
The midpoint M of the segment with the 2 endpoints is:
( \(\frac{x1 + x2}{2}\), \(\frac{y1 + y2}{2}\) )
Let the give points are:
(x1, y1) and (x2, y2)
So,
By comparing the given poits,
We will get
x1 = -1, x2 = 8, y1 = -2, y2 = 0
Hence,
The midpoint M = ( \(\frac{-1 + 8}{2}\), \(\frac{-2 + 0}{2}\) )
= ( \(\frac{7}{2}\), \(\frac{-2}{2}\) )
= ( \(\frac{7}{2}\), -1 )
Hence, from the above,
We can conclude that the midpoint M of the segment with the given endpoints is: ( \(\frac{7}{2}\), -1 )

Solve the equation.

Question 4.
7x + 12 = 3x
Answer:
The given equation is:
7x + 12 = 3x
So,
7x – 3x = 12
4x = 12
x = \(\frac{12}{4}\)
x = 3
Hence, from the above,
We can conclude that the value of x is: 3

Question 5.
14 – 6t = t
Answer:
The given equation is:
14 – 6t = t
So,
14 = 6t + t
7t = 14
t = \(\frac{14}{7}\)
t = 2
Hence, from the above,
We can conclude that the value of t is: 2

Question 6.
5p + 10 = 8p + 1
Answer:
The given equation is:
5p + 10 = 8p + 1
So,
5p – 8p = 1 – 10
-3p = -9
3p = 9
p = \(\frac{9}{3}\)
p = 3
Hence, from the above,
We can conclude that the value of p is: 3

Question 7.
w + 13 = 11w – 7
Answer:
The given equation is:
w + 13 = 11w – 7
So,
w – 11w = -7 – 13
-10w = -20
10w = 20
w = \(\frac{20}{10}\)
w = 2
Hence, from the above,
We can conclude that the value of w is: 2

Question 8.
4x + 1 = 3 – 2x
Answer:
The given equation is:
4x + 1 = 3 – 2x
So,
4x + 2x = 3 – 1
6x = 2
x = \(\frac{2}{6}\)
x = \(\frac{1}{3}\)
Hence, from the above,
We can conclude that the value of x is: \(\frac{1}{3}\)

Question 9.
z – 2 = 4 + 9z
Answer:
The given equation is:
z – 2 = 4 + 9z
So,
z – 9z = 4 + 2
-8z = 6
z = –\(\frac{6}{8}\)
z = –\(\frac{3}{4}\)
Hence, from the above,
We can conclude that the value of z is: –\(\frac{3}{4}\)

Question 10.
ABSTRACT REASONING
Is it possible to find the length of a segment in a coordinate plane without using the Distance Formula? Explain your reasoning.
Answer:
Yes, it is possible to find the length of a segment in a coordinate plane without using the distance formula
Since the segment is a portion of a line, we can use the graph to calculate the distance of a segment even though it would not provide accurate results.
Hence,
We use the distance formula to find the length of a segment in a coordinate plane

Congruent Triangles Mathematical Practices

Monitoring Progress

Classify each statement as a definition, a postulate, or a theorem. Explain your reasoning.

Question 1.
In a coordinate plane, two non-vertical lines are perpendicular if and only if the product of their slopes is – 1.
Answer:
The given statement is:
In a coordinate plane, two non-vertical lines are perpendicular if and only if the product of their slopes is – 1.
We know that,
According to the “parallel and perpendicular lines theorem”, two non-vertical lines are perpendicular if and only if the product of their slopes is -1
Hence, from the above,
We can conclude that the given statement is a Theorem

Question 2.
If two lines intersect to form a linear pair of congruent angles, then the lines are perpendicular.
Answer:
The given statement is:
If two lines intersect to form a linear pair of congruent angles, then the lines are perpendicular.
We know that,
According to the “Linear pair perpendicular theorem”,
When two straight lines intersect at a point and form a linear pair of congruent angles, then the lines are perpendicular
Hence, from the above,
We can conclude that the given statement is a Theorem

Question 3.
If two lines intersect to form a right angle. then the lines are perpendicular.
Answer:
The given statement is:
If two lines intersect to form a right angle. then the lines are perpendicular.
We know that,
According to the “Perpendicular lines theorem”,
When two lines intersect to form a right angle, the lines are perpendicular
Hence, from the above,
We can conclude that the given statement is a Theorem

Question 4.
Through any two points, there exists exactly one line.
Answer:
The given statement is:
Through any two points, there exists exactly one line
We know that,
Between two points, only one line can be drawn and we don’t need any proof to prove the above statement
We know that,
The statement that is true without proof to prove is called “Postulate”
Hence, from the above,
We can conclude that the given statement is a Postulate

5.1 Angles of Triangles

Exploration 1

Writing a Conjecture

Work with a partner.

Big Ideas Math Geometry Answers Chapter 5 Congruent Triangles 1

a. Use dynamic geometry software to draw any triangle and label it ∆ABC.
Answer:
By using the dynamic geometry software, the triangle drawn is:

b. Find the measures of the interior angles of the triangle.
Answer:
From part (a),
We can observe that the vertices of the triangle are: A, B, and C
Let the interior angles of the vertices A, B, and C be α, β, and γ respectively
Hence,
The measures of the given triangle are:

Hence, from the above,
The measures of the interior angles are:
α = 62.1°, β = 64.1°, and γ = 53.8°

c. Find the sum of the interior angle measures.
Answer:
From part (b),
The measures of the interior angles are:
α = 62.1°, β = 64.1°, and γ = 53.8°
Hence,
The sum of the interior angles = 62.1° + 64.1° + 53.1° = 180°
Hence, from the above,
We can conclude that the sum of the interior angle measures is: 180°

d. Repeat parts (a)-(c) with several other triangles. Then write a conjecture about the sum of the measures of the interior angles of a triangle.
Answer:
The representation of the 3 different triangles and their internal angle measures is:

Hence, from the above,
We can conclude that the conjecture about the sum of the measures of the interior angles of a triangle is:
The sum of the internal angle measures of a triangle is always: 180°

CONSTRUCTING VIABLE ARGUMENTS
To be proficient in math, you need to reason inductively about data and write conjectures.
Answer:
Inductive reasoning:
Inductive reasoning is the process of arriving at a conclusion based on a set of observations.
Inductive reasoning is used in geometry in a similar way.
Conjecture:
A statement you believe to be true based on inductive reasoning.

Exploration 2

Writing a Conjecture

Big Ideas Math Geometry Answers Chapter 5 Congruent Triangles 2

Work With a partner.

a. Use dynamic geometry software to draw any triangle and label it ∆ABC.
Answer:
The triangle drawn by using the dynamic geometry software is:

Hence, from the above,
We can conclude that the vertices of the triangle are: A, B, and C

b. Draw an exterior angle at any vertex and find its measure.
Answer:
From part (a),
The vertices of the triangle are: A, B, and C
Let the external angle measures of the triangle are: α, β, and γ
Hence,
The representation of the external angle measures of the triangle are:

Hence,
From the above,
We can conclude that
The external angle measures of the triangle are:
α = 310.7°, β = 299.3°, and γ = 290°

c. Find the measures of the two nonadjacent interior angles of the triangle.
Answer:
From part (b),
The external angle measures of the triangle are:
α = 310.7°, β = 299.3°, and γ = 290°
Hence,
The representation of the non-adjacent interior angles and the external angle measures of the triangle are:

Hence, from the above,
The angle measures of two non-adjacent sides are:
α = 70°, β = 60.7°, and γ = 49.3°

d. Find the sum of the measures of the two nonadjacent interior angles. Compare this sum to the measure of the exterior angle.
Answer:
From part (b),
The external angle measures of the triangle are:
α = 310.7°, β = 299.3°, and γ = 290°
From part (c),
The measures of the two non-adjacent interior angles are:
α = 70°, β = 60.7°, and γ = 49.3°
Now,
The sum of the measures of the external angles of the triangle are:
α + β + γ = 310.7° + 299.3°+ 290°
= 900.0°
The sum of the measures of the two non-adjacent interior angles is:
α + β + γ = 70° + 60.7° + 49.3°
= 180.0
Hence, from the above,
We can conclude that the sum of the measures of the external angles is 5 times the sum of the measures of the two non-adjacent interior angles

e. Repeat parts (a)-(d) with several other triangles. Then write a conjecture that compares the measure of an exterior angle with the sum of the measures of the two nonadjacent interior angles.
Answer:

Hence, from the above,
We can conclude that
The external angle measure of a vertex for a given triangle = 360° – (Internal angle measure of a vertex that we are finding the external angle measure)
The sum of the internal angle measures of the triangle is: 180°

Communicate Your Answer

Question 3.
How are the angle measures of a triangle related?
Answer:
The angle measures of a triangle are related as shown below:
The external angle measure of a vertex for a given triangle = 360° – (Internal angle measure of a vertex that we are finding the external angle measure)
The sum of the internal angle measures of the triangle is: 180°

Question 4.
An exterior angle of a triangle measures 32° What do you know about the measures of the interior angles? Explain your reasoning.
Answer:
It is given that an exterior angle of a triangle measures 32°
We know that,
The external angle measure of a vertex for a given triangle = 360° – (Internal angle measure of a vertex that we are finding the external angle measure)
So,
32° = 360° – (The internal angle measure of 32°)
The internal angle measure of 32° = 360° – 32°
The interior angle measure of 32° = 328°
Hence, from the above,
We can conclude that the interior angle measure of a triangle for an external angle measure of 32° is: 328°

Lesson 5.1 Angles of Triangles

Monitoring Progress

Question 1.
Draw an obtuse isosceles triangle and an acute scalene triangle.
Answer:
The figures of an obtuse isosceles triangle and an acute triangle are as follows:

Question 2.
∆ABC has vertices A(0, 0), B(3, 3), and C(- 3, 3), Classify the triangle by its sides. Then determine whether it is a right triangle.
Answer:
The given points are:
A (0, 0), B (3, 3), and C (-3, 3)
and the triangle is ΔABC
We know that,
To find whether the given triangle is a right-angled triangle or not,
We have to prove,
AC² = AB² + BC²
Where,
AC is the distance between A and C points
AB is the distance between A and B points
BC is the distance between B and C points
We know that,
The distance between 2 points = √(x2 – x1)² + (y2 – y1)²
Now,
Let the given points be considered as A(x1, y1), B(x2, y2), and C( x3, y3)
So,
AB = √(3 – 0)² + (3 – 0)² = √3² + 3²
= √9 + 9 = √18
BC = √(-3 – 3)² + (3 – 3)²
= √(-6)² + 0²
= √6² = 6
AC = √(-3 – 0)² + (3 – 0)²
= √(-3)² + 3²
= √9 + 9 = √18
Now,
AC² = AB² + BC²
(√18)² = (√18)² + 6²
18 = 18 + 36
18 ≠54
Hence, from the above,
We can conclude that the given triangle is not a right-angled triangle

Question 3.
Find the measure of ∠1
Big Ideas Math Geometry Answers Chapter 5 Congruent Triangles 3
Answer:
The given figure is:
Big Ideas Math Geometry Answers Chapter 5 Congruent Triangles 3
We know that,
The measure of an exterior angle of a triangle is equal to the sum of the measures of the two non-adjacent interior angles
From the given triangle,
The exterior angle is: (5x – 10)°
The interior angles are: 40°, 3x°, ∠1
So,
(5x – 10)° = 40° + 3x°
5x° – 3x° = 40° + 10°
2x° = 50°
x = 50° ÷ 2
x = 25°
So,
The interior angles are 40°, 3 (25)°, ∠1
= 40°, 75°, ∠1
We know that,
The sum of the interior angles of a triangle is: 180°
So,
40° + 75° + ∠1 = 180°
115° + ∠1 = 180°
∠1 = 180° – 115°
∠1 = 65°
Hence, from the above,
We can conclude that the value of ∠1 is: 65°

Question 4.
Find the measure of each acute angle.
Big Ideas Math Geometry Answers Chapter 5 Congruent Triangles 36
Answer:
The given figure is:
Big Ideas Math Geometry Answers Chapter 5 Congruent Triangles 36
We know that,
The sum of the interior angles in a triangle is: 180°
From the given figure,
The interior angles of the right-angled triangle are: 90°, 2x°, and (x – 6)°
So,
90° + 2x° + (x – 6)° = 180°
84°+ 3x° = 180°
3x° = 180° – 84°
3x° = 96°
x = 96° ÷ 3°
x = 32°
So,
The measure of each acute angle is 90°, 2x°, (x – 6)°
= 90°, 2(32)°, (32 – 6)°
= 90°, 64°, 26°
Hence, from the above,
We can conclude that,
The measure of each acute angle is 90°, 64°, and 26°

Exercise 5.1 Angles of Triangles

Vocabulary and Core Concept Check

Question 1.
WRITING
Can a right triangle also be obtuse? Explain our reasoning.
Answer:
Big Ideas Math Answer Key Geometry Chapter 5 Congruent Triangles 5.1 a 1

Question 2.
COMPLETE THE SENTENCE
The measure of an exterior angle of a triangle is equal to the sum of the measures of the two ____________ interior angles.
Answer:
The given statement is:
The measure of an exterior angle of a triangle is equal to the sum of the measures of the two ____________ interior angles.
Hence,
The completed form of the given statement is:
The measure of an exterior angle of a triangle is equal to the sum of the measures of the two non-adjacent interior angles.

Monitoring Progress and Modeling with Mathematics

In Exercises 3-6, classify the triangle by its sides and by measuring its angles.

Question 3.
Big Ideas Math Geometry Answers Chapter 5 Congruent Triangles 4
Answer:
Big Ideas Math Answer Key Geometry Chapter 5 Congruent Triangles 5.1 a 3

Question 4.
Big Ideas Math Geometry Answers Chapter 5 Congruent Triangles 5
Answer:
The given figure is:
Big Ideas Math Geometry Answers Chapter 5 Congruent Triangles 5
We know that,
“|” represents the “Congruent” or “Equal” in geometry
So,
From the given figure,
We can observe that all three sides of the given triangle are equal
We know that,
If a triangle has all the sides equal, then the triangle is called an “Equilateral triangle”
Hence, from the above,
We can conclude that the ΔLMN is an “Equilateral triangle”

Question 5.
Big Ideas Math Geometry Answers Chapter 5 Congruent Triangles 6
Answer:
Big Ideas Math Answer Key Geometry Chapter 5 Congruent Triangles 5.1 a 5

Question 6.
Big Ideas Math Geometry Answers Chapter 5 Congruent Triangles 7
Answer:
The given figure is:
Big Ideas Math Geometry Answers Chapter 5 Congruent Triangles 7
We know that,
If any side is not equal to each other in the triangle, then the triangle is called a “Scalene triangle”
The angle greater than 90° is called as “Obtuse angle”
An angle less than 90° is called an “Acute angle”
Hence, from the above,
We can conclude that ΔABC is an “Acute scalene triangle”

In Exercises 7-10, classify ∆ABC by its sides. Then determine whether it is a right triangle.

Question 7.
A(2, 3), B(6, 3), (2, 7)
Answer:
Big Ideas Math Answer Key Geometry Chapter 5 Congruent Triangles 5.1 a 7

Question 8.
A(3, 3), B(6, 9), (6, – 3)
Answer:
The given points are:
A (3, 3), B(6, 9), and C (6, -3)
We know that,
To find whether the given triangle is a right angle or not,
We have to prove,
AC² = AB² + BC²
Where,
AC is the distance between points A and C
AB is the distance between points A and B
BC is the distance between points B and C
The slope of any one side must be equal to -1
Now,
Let the given points be
A (x1, y1), B(x2, y2), and C (x3, y3)
So,
A (x1, y1)= (3, 3), B (x2, y2) = (6, 9), and C (x3, y3) = (6, -3)
We know that,
The distance between 2 points = √(x2 – x1)² + (y2 – y1)²
So,
AB = √(6 – 3)² + (9 – 3)²
= √3² + 6²
= √9 + 36 = √45
BC = √6 – 6)² + (-3 – 9)²
= √0 + 12²
= √12² = 12
AC = √(6 – 3)² + (-3 – 3)²
= √(3)² + (-6)²
= √9 + 36 = √45
So,
From the length of the sides,
We can say that the given triangle is an Isosceles triangle,
We know that,
Slope (m) = \(\frac{y2 – y1} {x2 – x1}\)
So,
Slope of AB = \(\frac{9 – 3} {6 – 3}\)
= \(\frac{6} {3}\)
= 2
Slope of BC = \(\frac{-9 – 3} {6 – 6}\)
= \(\frac{-12} {0}\)
= Undefined
Slope of AC = \(\frac{-3 – 3} {6 – 3}\)
= \(\frac{-6} {3}\)
= -2
Hence, from the above,
We can conclude that the given triangle is not a right triangle

Question 9.
A(1, 9), B(4, 8), C(2, 5)
Answer:
Big Ideas Math Answer Key Geometry Chapter 5 Congruent Triangles 5.1 a 9

Question 10.
A(- 2, 3), B(0, – 3), C(3, – 2)
Answer:
The given points are:
A (-2, 3), B(0, -3), and C (3, -2)
We know that,
To find whether the given triangle is a right angle or not,
We have to prove,
AC² = AB² + BC²
Where,
AC is the distance between points A and C
AB is the distance between points A and B
BC is the distance between points B and C
The slope of any one side must be equal to -1
Now,
Let the given points be
A (x1, y1), B(x2, y2), and C (x3, y3)
So,
A (x1, y1)= (-2, 3), B (x2, y2) = (0, -3), and C (x3, y3) = (3, -2)
We know that,
The distance between 2 points = √(x2 – x1)² + (y2 – y1)²
So,
AB = √(0 – [-2])² + (3 – 3)²
= √2² + 0²
= √4 + 0 = 2
BC = √3 – 0)² + (-2 -[-3] )²
= √9 + 1²
= √10
AC = √(3 – [-2])² + (-2 – 3)²
= √(5)² + (-5)²
= √25 + 25 = √50
Now,
AC² = AB² + BC²
50 = 10 + 4
50 ≠ 14
So,
From the length of the sides,
We can say that the given triangle is a scalene triangle since all the lengths of the sides are different
We know that,
Slope (m) = \(\frac{y2 – y1} {x2 – x1}\)
So,
Slope of AB = \(\frac{9 – 3} {6 – 3}\)
= \(\frac{6} {3}\)
= 2
Slope of BC = \(\frac{-9 – 3} {6 – 6}\)
= \(\frac{-12} {0}\)
= Undefined
Slope of AC = \(\frac{-3 – 3} {6 – 3}\)
= \(\frac{-6} {3}\)
= -2
Hence, from the above,
We can conclude that the given triangle is not a right triangle

In Exercises 11 – 14. find m∠1. Then classify the triangle by its angles

Question 11.
Big Ideas Math Geometry Answers Chapter 5 Congruent Triangles 8
Answer:
Big Ideas Math Answer Key Geometry Chapter 5 Congruent Triangles 5.1 a 11

Question 12.
Big Ideas Math Geometry Answers Chapter 5 Congruent Triangles 9
Answer:
The given figure is:
Big Ideas Math Geometry Answers Chapter 5 Congruent Triangles 9
We know that,
The sum of interior angles in a triangle is: 180°
So,
From the above,
The interior angles of the given triangle are: 40°,  30°,  ∠1
Now,
40° + 30° + ∠1 = 180°
70 + ∠1 = 180°
∠1 = 180° – 70°
∠1 = 110°
We know that,
The angle greater than 90° is called an “Obtuse angle”
Hence, from the above,
We can conclude that the given triangle is an “Obtuse angled triangle”

Question 13.
Big Ideas Math Geometry Answers Chapter 5 Congruent Triangles 10
Answer:
Big Ideas Math Answer Key Geometry Chapter 5 Congruent Triangles 5.1 a 13

Question 14.
Big Ideas Math Geometry Answers Chapter 5 Congruent Triangles 11
Answer:
The given figure is:
Big Ideas Math Geometry Answers Chapter 5 Congruent Triangles 11
We know that,
The sum of interior angles in a triangle is: 180°
So,
From the above,
The interior angles of the given triangle are: 60°,  60°,  ∠1
Now,
60° + 60° + ∠1 = 180°
120 + ∠1 = 180°
∠1 = 180° – 120°
∠1 = 60°
We know that,
An angle less than 90° is called an “Acute angle”
The triangle that all the angles 60° is called an “Equilateral triangle”
Hence, from the above,
We can conclude that the given triangle is an “Equilateral triangle”

In Exercises 15-18, find the measure of the exterior angle.

Question 15.
Big Ideas Math Geometry Answers Chapter 5 Congruent Triangles 12
Answer:
Big Ideas Math Answer Key Geometry Chapter 5 Congruent Triangles 5.1 a 15

Question 16.
Big Ideas Math Geometry Answers Chapter 5 Congruent Triangles 13
Answer:
The given figure is:
Big Ideas Math Geometry Answers Chapter 5 Congruent Triangles 13
We know that,
An exterior angle is equal to the sum of the two non-adjacent interior angles in a triangle
So,
(2x – 2)° = x° + 45°
2x° – x° = 45° + 2°
x = 47°
Hence,
The measure of the exterior angle is: (2x – 2)°
= (2 (47) – 2)°
= (94 – 2)°
= 92°
Hence, from the above,
We can conclude that the measure of the exterior angle is: 92°

Question 17.
Big Ideas Math Geometry Answers Chapter 5 Congruent Triangles 14
Answer:
Big Ideas Math Answer Key Geometry Chapter 5 Congruent Triangles 5.1 a 17

Question 18.
Big Ideas Math Geometry Answers Chapter 5 Congruent Triangles 15
Answer:
The given figure is:
Big Ideas Math Geometry Answers Chapter 5 Congruent Triangles 15
We know that,
An exterior angle is equal to the sum of the two non-adjacent interior angles in a triangle
So,
(7x – 16)° = (x + 8)° + 4x°
7x° – 5x° = 16° + 8°
2x = 24°
x = 24° ÷ 2
x = 12°
Hence,
The measure of the exterior angle is: (7x – 16)°
= (7 (12) – 16)°
= (84 – 16)°
= 68°
Hence, from the above,
We can conclude that the measure of the exterior angle is: 68°

In Exercises 19-22, find the measure of each acute angle.

Question 19.
Big Ideas Math Geometry Answers Chapter 5 Congruent Triangles 16
Answer:
Big Ideas Math Answer Key Geometry Chapter 5 Congruent Triangles 5.1 a 19

Question 20.
Big Ideas Math Geometry Answers Chapter 5 Congruent Triangles 17
Answer:
The given figure is:
Big Ideas Math Geometry Answers Chapter 5 Congruent Triangles 17
From the given figure,
We can observe that one angle is 90° and the 2 sides are perpendicular
So,
We can say that the given triangle is a right-angled triangle
We know that,
The sum of interior angles of a triangle is: 180°
So,
x° + (3x + 2)° + 90° = 180°
4x° + 2° + 90° = 180°
4x° = 180° – 90° – 2°
4x° = 88°
x = 88° ÷ 4°
x = 22°
So,
The 2 acute angle measures are: x° and (3x + 2)°
= 22° and (3(22) + 2)°
= 22° and (66 + 2)°
= 22° and 68°
Hence, from the above,
We can conclude that the 2 acute angle measures are: 22° and 68°

Question 21.
Big Ideas Math Geometry Answers Chapter 5 Congruent Triangles 18
Answer:
Big Ideas Math Answer Key Geometry Chapter 5 Congruent Triangles 5.1 a 21

Question 22.
Big Ideas Math Geometry Answers Chapter 5 Congruent Triangles 19
Answer:
The given figure is:
Big Ideas Math Geometry Answers Chapter 5 Congruent Triangles 19
From the given figure,
We can observe that one angle is 90° and the 2 sides are perpendicular
So,
We can say that the given triangle is a right-angled triangle
We know that,
The sum of interior angles of a triangle is: 180°
So,
(19x – 1)° + (13x – 5)° + 90° = 180°
32x° – 6° + 90° = 180°
32x° = 180° – 90° – 6°
4x° = 84°
x = 84° ÷ 4°
x = 21°
So,
The 2 acute angle measures are: (19x – 1)° and (13x – 5)°
= (19 (21) – 1)° and (13(21) – 5)°
= 398° and (273 – 5)°
= 398° and 268°
Hence, from the above,
We can conclude that the 2 acute angle measures are: 398° and 268°

In Exercises 23-26. find the measure of each acute angle in the right triangle.

Question 23.
The measure of one acute angle is 5 times the measure of the other acute angle.
Answer:
Big Ideas Math Answer Key Geometry Chapter 5 Congruent Triangles 5.1 a 23

Question 24.
The measure of one acute angle is times the measure of the other acute angle.
Answer:

Question 25.
The measure of one acute angle is 3 times the sum of the measure of the other acute angle and 8.
Answer:
Big Ideas Math Answer Key Geometry Chapter 5 Congruent Triangles 5.1 a 25

Question 26.
The measure of one acute angle is twice the difference of the measure of the other acute angle and 12.
Answer:
The given statement is:
The measure of one acute angle is twice the difference of the measure of the other acute angle and 12.
So,
x° + [2 (x – 12)]° = 90°
x° + 2x° – 2(12)° = 90°
3x° – 24° = 90°
3x° = 90° + 24°
3x° = 114°
x = 114° ÷ 3
x = 38°
So,
The 2 acute angle measures are: x°, 2 (x – 12)°
= 38°, 2 (38 – 12)°
= 38°, 2(26)°
= 38° , 52°
Hence, from the above,
We can conclude that the acute angle measures are: 38°, 52°

ERROR ANALYSIS
In Exercises 27 and 28, describe and correct the error in finding m∠1.

Question 27.
Big Ideas Math Geometry Answers Chapter 5 Congruent Triangles 20
Answer:
Big Ideas Math Answer Key Geometry Chapter 5 Congruent Triangles 5.1 a 27

Question 28.
Big Ideas Math Geometry Answers Chapter 5 Congruent Triangles 21
Answer:
We know that,
The exterior angle of a triangle is equal to the sum of the non-adjacent interior angles of a triangle
So,
From the figure,
The external angle is: ∠1
The interior angles are 80°, 50°
So,
∠1 = 80° + 50°
∠1 = 130°
Now,
The interior angle measure of ∠1= 180° – (External angle measure of 130°)
= 180° – 130°
= 50°
Hence, from the above,
The internal angle measure of ∠1 is: 50°

In Exercises 29-36, find the measure of the numbered angle.

Big Ideas Math Geometry Answers Chapter 5 Congruent Triangles 22

Question 29.
∠1
Answer:
Big Ideas Math Answer Key Geometry Chapter 5 Congruent Triangles 5.1 a 29

Question 30.
∠2
Answer:
We know that,
The external angle measure is equal to the sum of the non-adjacent interior angles
So,
∠2 = 90° + 40°
∠2 = 130°

Question 31.
∠3
Answer:
Big Ideas Math Answer Key Geometry Chapter 5 Congruent Triangles 5.1 a 31

Question 32.
∠3
Answer:
From the above figure,
∠2 = ∠4
Hence, from the above,
We can conclude that
∠2 = ∠4 = 130°

Question 33.
∠5
Answer:
Big Ideas Math Answer Key Geometry Chapter 5 Congruent Triangles 5.1 a 33

Question 34.
∠6
Answer:
The external angle measure is equal to the sum of the non-adjacent interior angles
So,
∠6 = 90° + ∠3
∠6 = 90° + 50°
∠6 = 140°

Question 35.
∠7
Answer:
Big Ideas Math Answer Key Geometry Chapter 5 Congruent Triangles 5.1 a 35

Question 36.
∠8
Answer:
The external angle measure is equal to the sum of the non-adjacent interior angles
So,
∠8 = 90° + ∠1
∠6 = 90° + 50°
∠6 = 140°

Question 37.
USING TOOLS
Three people are standing on a stage. The distances between the three people are shown in the diagram. Classify the triangle by its sides and by measuring its angles.
Big Ideas Math Geometry Answers Chapter 5 Congruent Triangles 23
Answer:
Big Ideas Math Answer Key Geometry Chapter 5 Congruent Triangles 5.1 a 37

Question 38.
USING STRUCTURE
Which of the following sets of angle measures could form a triangle? Select all that apply.
(A) 100°, 50°, 40°
Answer:
The given angles are: 100°, 50°, 40°
We know that,
The sum of the angles of a triangle should be equal to 180°
So,
The sum of the given angles = 100° + 50° + 40°
= 100° + 90°
= 190°
Hence, from the above,
We can conclude that the given angles do not form a triangle

(B) 96°, 74°, 10°
Answer:
The given angles are: 96°, 74°, 10°
We know that,
The sum of the angles of a triangle should be equal to 180°
So,
The sum of the given angles = 96° + 74° + 10°
= 96° + 84°
= 180°
Hence, from the above,
We can conclude that the given angles forms a triangle

(C) 165°, 113°, 82°
Answer:
The given angles are: 165°, 113°, 82°
We know that,
The sum of the angles of a triangle should be equal to 180°
So,
The sum of the given angles = 165° + 113° + 82°
= 165° + 195°
= 360°
But,
We know that,
The sum of exterior angles of a triangle is: 360°
Hence, from the above,
We can conclude that the given angles forms a triangle

(D) 101°, 41°, 38°
Answer:
The given angles are: 101°, 41°, 38°
We know that,
The sum of the angles of a triangle should be equal to 180°
So,
The sum of the given angles = 101° + 38° + 41°
= 101° + 79°
= 180°
Hence, from the above,
We can conclude that the given angles forms a triangle

(E) 90°, 45°, 45°
Answer:
The given angles are: 90°, 45°, 45°
We know that,
The sum of the angles of a triangle should be equal to 180°
So,
The sum of the given angles = 90° + 45° + 45°
= 90° + 90°
= 180°
Hence, from the above,
We can conclude that the given angles forms a triangle

(F) 84°, 62°, 34°
Answer:
The given angles are: 84°, 62°, 34°
We know that,
The sum of the angles of a triangle should be equal to 180°
So,
The sum of the given angles = 84° + 62° + 34°
= 84° + 96°
= 180°
Hence, from the above,
We can conclude that the given angles forms a triangle

Question 39.
MODELING WITH MATHEMATICS
You are bending a strip of metal into an isosceles triangle for a sculpture. The strip of metal is 20 inches long. The first bend is made 6 inches from one end. Describe two ways you could complete the triangle.
Answer:
Big Ideas Math Answer Key Geometry Chapter 5 Congruent Triangles 5.1 a 39

Question 40.
THOUGHT-PROVOKING
Find and draw an object (or part of an object) that can be modeled by a triangle and an exterior angle. Describe the relationship between the interior angles of the triangle and the exterior angle in terms of the object.
Answer:

From the above figure,
We can say that
The sum of the interior angles of a given triangle is: 180°
The sum of the exterior angles of a given triangle is: 360°
The relation between the interior angles and the exterior angles is:
The exterior angle measure = Sum of the two non-adjacent interior angles

Question 41.
PROVING A COROLLARY
Prove the Corollary to the Triangle Sum Theorem (Corollary 5. 1).
Given ∆ABC is a right triangle
Prove ∠A and ∠B are complementary
Big Ideas Math Geometry Answers Chapter 5 Congruent Triangles 24
Answer:
Big Ideas Math Answer Key Geometry Chapter 5 Congruent Triangles 5.1 a 41

Question 42.
PROVING A THEOREM
Prove the Exterior Angle Theorem (Theorem 5.2).
Given ∆ABC, exterior ∠ACD
Prove m∠A + m∠B = m∠ACD

Answer:
It is given that
In ΔABC, the exterior angle is ∠ACD
We have to prove that
m∠A + m∠B = m∠ACD
Proof:

Hence, from the above,
We can conclude that
m∠A + m∠B = m∠ACD is proven

Question 43.
CRITICAL THINKING
Is it possible to draw an obtuse isosceles triangle? obtuse equilateral triangle? If so, provide examples. If not, explain why it is not possible.
Answer:
Big Ideas Math Answer Key Geometry Chapter 5 Congruent Triangles 5.1 a 43

Question 44.
CRITICAL THINKING
Is it possible to draw a right isosceles triangle? right equilateral triangle? If so, provide an example. If not, explain why it is not possible.
Answer:
It is possible to draw a right isosceles triangle but it is not possible to draw a right equilateral triangle
We know that,
In a triangle, if the length of the 2 sides are equal and one angle is a right-angle, then, it is called an “Right Isosceles triangle”
In a triangle, if the length of all the sides are equal and each angle is 60°, then it is an “Equilateral triangle”
Hence,
From the above definitions,
We can observe that it is possible to draw right isosceles triangle but it is not possible to dran a right equilateral triangle

Question 45.
MATHEMATICAL CONNECTIONS
∆ABC is isosceles.
AB = x, and BC = 2x – 4.
a. Find two possible values for x when the perimeter of ∆ABC is 32.
b. How many possible values are there for x when the perimeter of ∆ABC is 12?
Answer:
Big Ideas Math Answer Key Geometry Chapter 5 Congruent Triangles 5.1 a 45

Question 46.
HOW DO YOU SEE IT?
Classify the triangles, in as many ways as possible. without finding any measurements.
a. Big Ideas Math Geometry Answers Chapter 5 Congruent Triangles 26
Answer:
The given figure is:
Big Ideas Math Geometry Answers Chapter 5 Congruent Triangles 26
From the figure,
We can observe that all the length of the sides of the triangle are equal
We know that,
The triangle that has the length of all the sides equal is called an “Equilateral triangle”
Hence, from the above,
We can conclude that the given triangle is an “Equilateral triangle”

b. Big Ideas Math Geometry Answers Chapter 5 Congruent Triangles 27
Answer:
The given figure is:
Big Ideas Math Geometry Answers Chapter 5 Congruent Triangles 27
From the figure,
We can observe that the lengths of all the 3 sides are different
We know that,
The triangle that has all the different side lengths is called a “Scalene triangle”
Hence, from the above,
We can conclude that the given triangle is called a “Scalene triangle”

c. Big Ideas Math Geometry Answers Chapter 5 Congruent Triangles 28
Answer:
The given figure is:
Big Ideas Math Geometry Answers Chapter 5 Congruent Triangles 28
From the figure,
We can observe that the length of all the 3 sides are different and 1 angle is obtuse i.e., greater than 90°
We know that,
The triangle that has any angle obtuse is called an “Obtuse angled triangle”
Hence, from the above,
We can conclude that the given triangle is an “Obtuse angled scalene triangle”

d. Big Ideas Math Geometry Answers Chapter 5 Congruent Triangles 29
Answer:
The given figure is:
Big Ideas Math Geometry Answers Chapter 5 Congruent Triangles 29
From the figure,
We can observe that 1 angle is 90° and the 2 sides are perpendicular to each other
We know that,
The triangle that has an angle of 90° and the slope -1 is called a “Right-angled triangle”
Hence, from the above,
We can conclude that the given triangle is called a “Right-angled triangle”

Question 47.
ANALYZING RELATIONSHIPS
Which of the following could represent the measures of an exterior angle and two interior angles of a triangle? Select all that apply.
A) 100°, 62°, 38°
(B) 81°, 57°, 24°
(C) 119°, 68°, 49°
(D) 95°, 85°, 28°
(E) 92°, 78°, 68°
(F) 149°, 101°, 48°
Answer:
Big Ideas Math Answer Key Geometry Chapter 5 Congruent Triangles 5.1 a 47

Question 48.
MAKING AN ARGUMENT
Your friend claims the measure of an exterior angle will always be greater than the sum of the nonadjacent interior angle measures. Is your friend correct? Explain your reasoning.
Answer:
No, your friend is not correct

Explanation:
We know that,
According to the exterior angle theorem,
The external angle measure is always equal to the sum of the non-adjacent internal angle measures
But,
According to your friend,
The external angle measure will always be greater than the sum of the non-adjacent interior angle measures
Hence, from the above,
We can conclude that your friend is not correct

MATHEMATICAL CONNECTIONS
In Exercises 49-52, find the values of x and y.

Question 49.
Big Ideas Math Geometry Answers Chapter 5 Congruent Triangles 30
Answer:
Big Ideas Math Answer Key Geometry Chapter 5 Congruent Triangles 5.1 a 49

Question 50.
Big Ideas Math Geometry Answers Chapter 5 Congruent Triangles 31
Answer:
The given figure is:
Big Ideas Math Geometry Answers Chapter 5 Congruent Triangles 31
From the figure,
We have to obtain the values of x and y
Now,
By using the alternate angles theorem,
x = 118°
Now,
By using the exterior angle theorem,
x = y + 22°
y = x – 22°
y = 118° – 22°
y = 96°
Hence, from the above,
We can conclude that the values of x and y are: 118° and 96° respectively

Question 51.
Big Ideas Math Geometry Answers Chapter 5 Congruent Triangles 32
Answer:
Big Ideas Math Answer Key Geometry Chapter 5 Congruent Triangles 5.1 a 51

Question 52.
Big Ideas Math Geometry Answers Chapter 5 Congruent Triangles 33
Answer:
The given figure is:
Big Ideas Math Geometry Answers Chapter 5 Congruent Triangles 33
From the above figure,
We have to find the values of x and y
Now,
By using the sum of interior angle measures,
x° + 64° + 90° = 180°
x° + 154° = 180°
x° = 180° – 154°
x° = 26°
Now,
By using the exterior angle theorem,
y° = x° + 64°
y° = 26° + 64°
y° = 90°
Hence, from the above,
We can conclude that the values of x and y are: 26° and 90° respectively

Question 53.
PROVING A THEOREM
Use the diagram to write a proof of the Triangle Sum Theorem (Theorem 5. 1). Your proof should be different from the proof of the Triangle Sum Theorem shown in this lesson.
Big Ideas Math Geometry Answers Chapter 5 Congruent Triangles 34
Answer:
Big Ideas Math Answer Key Geometry Chapter 5 Congruent Triangles 5.1 a 53.1
Big Ideas Math Answer Key Geometry Chapter 5 Congruent Triangles 5.1 a 53.2

Maintaining Mathematical Proficiency

Use the diagram to find the measure of the segment or angle.

Big Ideas Math Geometry Answers Chapter 5 Congruent Triangles 35

Question 54.
m∠KHL
Answer:
From the given figures,
We can observe that
∠ABC = ∠GHK
∠KHL = ∠GHK / 2
So,
(6x + 2)° = (3x + 1)° + (5x – 27)°
6x – 3x – 5x = 1 – 27 – 2
6x – 8x = -27 – 1
-2x = -28
2x = 28
x = 28 ÷ 2
x = 14
So,
∠KHL = ∠GHK / 2
= [(3 (14) + 1)° + (5 (14) – 27)°] / 2
= [43° + 43°] / 2
= 86° / 2
= 43°
Hence, from the above,
We can conclude that
∠KHL = 43°

Question 55.
m∠ABC
Answer:
Big Ideas Math Answer Key Geometry Chapter 5 Congruent Triangles 5.1 a 55

Question 56.
GH
Answer:
From the given figures,
We can observe that
AB = GH
So,
3y = 5y – 8
3y – 5y = -8
-2y = -8
2y = 8
y = 8 ÷ 2
y = 4
So,
The value of GH = 3y = 3 (4) = 12
Hence, from the above,
We can conclude that the value of GH is: 12

Question 57.
BC
Answer:
Big Ideas Math Answer Key Geometry Chapter 5 Congruent Triangles 5.1 a 57

5.2 Congruent Polygons

Exploration 1

Describing Rigid Motions

Work with a partner: of the four transformations you studied in Chapter 4, which are rigid motions? Under a rigid motion. why is the image of a triangle always congruent to the original triangle? Explain your reasoning.
Big Ideas Math Answers Geometry Chapter 5 Congruent Triangles 37
Answer:
Rigid motion occurs in geometry when an object moves but maintains its shape and size, which is unlike non-rigid motions, such as dilations, in which the object’s size changes. All rigid motion starts with the original object, called the pre-image, and results in the transformed object, called the image.
There are 4 types of rigid motion. They are:
a. Translation
b. Rotation
c. Reflection
d. Glide reflection
We know that,
Rotation only occurs in terms of 90° or 180°
Now,
The given transformations are:
Big Ideas Math Answers Geometry Chapter 5 Congruent Triangles 37
So,
From the above figure,
The first figure and the second figure are different
The second figure and the third figure are the same in shape
The first figure and the fourth figure are the same in shape
So,
We can say that the first and the fourth figures are rigid motions
W can say that the second and the third figures are rigid motions
In the second and the third figures,
The “Rotation” takes place i.e., the second figure is rotated 180° keeping the original shape
In the first and the fourth figures,
The “Reflection” takes place i.e., the first figure is reflected keeping the original shape
Now,
The image of the triangle is always congruent to the original triangle because of the “Translation” i.e., the original triangle and the image of the triangle have the same sides and the same angles but not in the same position.

Exploration 2

Finding a Composition of Rigid Motions

Work with a partner. Describe a composition of rigid motions that maps ∆ABC to ∆DEF. Use dynamic geometry software to verify your answer.
LOOKING FOR STRUCTURE
To be proficient in math, you need to look closely to discern a pattern or structure.

a. ∆ABC ≅ ∆DEF
Big Ideas Math Answers Geometry Chapter 5 Congruent Triangles 38
Answer:

b. ∆ABC ≅ ∆DEF
Big Ideas Math Answers Geometry Chapter 5 Congruent Triangles 39
Answer:

c. ∆ABC ≅ ∆DEF
Big Ideas Math Answers Geometry Chapter 5 Congruent Triangles 40
Answer:

d. ∆ABC ≅ ∆DEF
Big Ideas Math Answers Geometry Chapter 5 Congruent Triangles 41
Answer:

Communicate Your Answer

Question 3.
Given two congruent triangles. how can you use rigid motions to map one triangle to the other triangle?
Answer:

Question 4.
The vertices of ∆ABC are A(1, 1), B(3, 2), and C(4, 4). The vertices of ∆DEF are D(2, – 1), E(0, 0), and F(- 1, 2). Describe a composition of rigid motions that maps ∆ABC to ∆DEF.
Answer:

Lesson 5.2 Congruent Polygons

Monitoring Progress

In the diagram, ABGH ≅ CDEF.

Big Ideas Math Answers Geometry Chapter 5 Congruent Triangles 42

Question 1.
Identify all pairs of congruent corresponding parts.
Answer:

Question 2.
Find the value of x.
Answer:

Question 3.
In the diagram at the left. show that ∆PTS ≅ ∆RTQ.
Big Ideas Math Answers Geometry Chapter 5 Congruent Triangles 43
Answer:

Use the diagram.

Big Ideas Math Answers Geometry Chapter 5 Congruent Triangles 44

Question 4.
Find m∠DCN.
Answer:

Question 5.
What additional information is needed to conclude that ∆NDC ≅ ∆NSR?
Answer:

Exercise 5.2 Congruent Polygons

Question 1.
WRITING
Based on this lesson. what information do you need to prove that two triangles are congruent? Explain your reasoning.
Answer:
Big Ideas Math Answers Geometry Chapter 5 Congruent Triangles 5.2 a 1

Question 2.
DIFFERENT WORDS, SAME QUESTION
Which is different? Find “both” answers.
Big Ideas Math Answers Geometry Chapter 5 Congruent Triangles 45

Is ∆ABC ≅ ∆RST?
Answer:

Is ∆KJL ≅ ∆SRT?
Answer:

Is ∆JLK ≅ ∆STR?
Answer:

Is ∆LKJ ≅ ∆TSR?
Answer:

Monitoring Progress and Modeling with Mathematics

In Exercises 3 and 4. identify all pairs of congruent corresponding parts. Then write another congruence statement for the polygons.

Question 3.
∆ABC ≅ ∆DEF
Big Ideas Math Answers Geometry Chapter 5 Congruent Triangles 46
Answer:
Big Ideas Math Answers Geometry Chapter 5 Congruent Triangles 5.2 a 3

Question 4.
GHJK ≅ ∆QRST
Big Ideas Math Answers Geometry Chapter 5 Congruent Triangles 47
Answer:

In Exercises 5-8, ∆XYZ ≅ ∆MNL. Copy and complete the statement.

Big Ideas Math Answers Geometry Chapter 5 Congruent Triangles 48

Question 5.
m∠Y = ______
Answer:
Big Ideas Math Answers Geometry Chapter 5 Congruent Triangles 5.2 a 5

Question 6.
m∠M = ______
Answer:

Question 7.
m∠Z = _______
Answer:
Big Ideas Math Answers Geometry Chapter 5 Congruent Triangles 5.2 a 7

Question 8.
XY= _______
Answer:

In Exercises 9 and 10. find the values of x and y.

Question 9.
ABCD ≅ EFGH
Big Ideas Math Answers Geometry Chapter 5 Congruent Triangles 49
Answer:
Big Ideas Math Answers Geometry Chapter 5 Congruent Triangles 5.2 a 9

Question 10.
∆MNP ≅ ∆TUS
Big Ideas Math Answers Geometry Chapter 5 Congruent Triangles 50
Answer:

In Exercises 11 and 12. show that the polygons are congruent. Explain your reasoning.

Question 11.
Big Ideas Math Answers Geometry Chapter 5 Congruent Triangles 51
Answer:
Big Ideas Math Answers Geometry Chapter 5 Congruent Triangles 5.2 a 11

Question 12.
Big Ideas Math Answers Geometry Chapter 5 Congruent Triangles 52
Answer:

In Exercises 13 and 14, find m∠1.

Question 13.
Big Ideas Math Answers Geometry Chapter 5 Congruent Triangles 53
Answer:
Big Ideas Math Answers Geometry Chapter 5 Congruent Triangles 5.2 a 13

Question 14.
Big Ideas Math Answers Geometry Chapter 5 Congruent Triangles 54
Answer:

Question 15.
PROOF
Triangular postage stamps, like the ones shown, are highly valued by stamp collectors. Prove that ∆AEB ≅ ∆CED.
Big Ideas Math Answers Geometry Chapter 5 Congruent Triangles 55
Given \(\overline{A B}\) || \(\overline{D C}\), \(\overline{A B}\) ≅ \(\overline{D C}\) is the midpoint of \(\overline{A C}\) and \(\overline{B D}\)
Prove ∆AEB ≅ ∆CED
Answer:
Big Ideas Math Answers Geometry Chapter 5 Congruent Triangles 5.2 a 15

Question 16.
PROOF
Use the information in the figure to prove that ∆ABG ≅ ∆DCF
Big Ideas Math Answers Geometry Chapter 5 Congruent Triangles 56
Answer:

ERROR ANALYSIS
In Exercises 17 and 18, describe and correct the error.

Question 17.
Big Ideas Math Answers Geometry Chapter 5 Congruent Triangles 57
Answer:
Big Ideas Math Answers Geometry Chapter 5 Congruent Triangles 5.2 a 17

Question 18.
Big Ideas Math Answers Geometry Chapter 5 Congruent Triangles 58
Answer:

Question 19.
PROVING A THEOREM
Prove the Third Angles Theorem (Theorem 5.4) by using the Triangle Sum Theorem (Theorem 5. 1).
Answer:
Big Ideas Math Answers Geometry Chapter 5 Congruent Triangles 5.2 a 19.1
Big Ideas Math Answers Geometry Chapter 5 Congruent Triangles 5.2 a 19.2

Question 20.
THOUGHT PROVOKING
Draw a triangle. Copy the triangle multiple times to create a rug design made of congruent triangles. Which property guarantees that all the triangles are congruent?
Answer:

Question 21.
REASONING
∆JKL is congruent to ∆XYZ Identify all pairs of congruent corresponding parts.
Answer:
Big Ideas Math Answers Geometry Chapter 5 Congruent Triangles 5.2 a 21

Question 22.
HOW DO YOU SEE IT?
In the diagram, ABEF ≅ CDEF
Big Ideas Math Answers Geometry Chapter 5 Congruent Triangles 59
a. Explain how you know that \(\overline{B E}\) ≅ \(\overline{D E}\) and ∠ABE ≅∠CDE.
Answer:

b. Explain how you know that ∠GBE ≅ ∠GDE.
Answer:

c. Explain how you know that ∠GEB ≅ ∠GED.
Answer:

d. Do you have enough information to prove that ∠BEG ≅ ∠DEG? Explain.
Answer:

MATHEMATICAL CONNECTIONS
In Exercises 23 and 24, use the given information to write and solve a system of linear equations to find the values of x and y.

Question 23.
∆LMN ≅ ∆PQR. m∠L = 40°, m∠M = 90° m∠P = (17x – y)°. m∠R (2x + 4y)°
Answer:
Big Ideas Math Answers Geometry Chapter 5 Congruent Triangles 5.2 a 23

Question 24.
∆STL ≅ ∆XYZ, m∠T = 28°, m∠U = (4x + y)°, m∠X = 130°, m∠Y = (8x – 6y)°
Answer:

Question 25.
PROOF
Prove that the criteria for congruent triangles in this lesson is equivalent to the definition of congruence in terms of rigid motions.
Answer:
Big Ideas Math Answers Geometry Chapter 5 Congruent Triangles 5.2 a 25

Maintaining Mathematical Proficiency

What can you conclude from the diagram?

Question 26.
Big Ideas Math Answers Geometry Chapter 5 Congruent Triangles 60
Answer:

Question 27.
Big Ideas Math Answers Geometry Chapter 5 Congruent Triangles 61
Answer:
Big Ideas Math Answers Geometry Chapter 5 Congruent Triangles 5.2 a 27

Question 28.
Big Ideas Math Answers Geometry Chapter 5 Congruent Triangles 62
Answer:

Question 29.
Big Ideas Math Answers Geometry Chapter 5 Congruent Triangles 63
Answer:
Big Ideas Math Answers Geometry Chapter 5 Congruent Triangles 5.2 a 29

5.3 Proving Triangle Congruence by SAS

Exploration 1

Drawing Triangles

Big Ideas Math Geometry Answer Key Chapter 5 Congruent Triangles 64

Big Ideas Math Geometry Answer Key Chapter 5 Congruent Triangles 65

Big Ideas Math Geometry Answer Key Chapter 5 Congruent Triangles 66

Work with a partner.

Use dynamic geometry software.
a. Construct circles with radii of 2 units and 3 units centered at the origin. Construct a 40° angle with its vertex at the origin. Label the vertex A.
Answer:

b. Locate the point where one ray of the angle intersects the smaller circle and label this point B. Locate the point where the other ray of the angle intersects the larger circle and label this point C. Then draw ∆ABC.
Answer:

c. Find BC, m∠B, and m∠C.
Answer:

d. Repeat parts (a)-(c) several times. redrawing the angle indifferent positions. Keep track of your results by copying and completing the table below. What can you conclude?
USING TOOLS STRATEGICALLY
To be proficient in math, you need to use technology to help visualize the results of varying assumptions, explore consequences, and compare predictions with data.
Answer:

Communicate Your Answer

Question 2.
What can you conclude about two triangles when you know that two pairs of corresponding sides and the corresponding included angles are congruent?
Answer:

Question 3.
How would you prove your conclusion in Exploration 1(d)?
Answer:

Lesson 5.3 Proving Triangle Congruence by SAS

Monitoring Progress

In the diagram, ABCD is a square with four congruent sides and four right
angles. R, S, T, and U are the midpoints of the sides of ABCD. Also, \(\overline{R T}\) ⊥ \(\overline{S U}\) and \(\overline{S V}\) ≅ \(\overline{V U}\).
Big Ideas Math Geometry Answer Key Chapter 5 Congruent Triangles 67
Question 1.
Prove that ∆SVR ≅ ∆UVR.
Answer:

Question 2.
Prove that ∆BSR ≅ ∆DUT.
Answer:

Question 3.
You are designing the window shown in the photo. You want to make ∆DRA congruent to ∆DRG. You design the window so that \(\overline{D A}\) ≅ \(\overline{D G}\) and ∠ADR ≅ ∠GDR. Use the SAS Congruence Theorem to prove ∆DRA ≅ ∆DRG.
Big Ideas Math Geometry Answer Key Chapter 5 Congruent Triangles 68
Answer:

Exercise 5.3 Proving Triangle Congruence by SAS

vocabulary and core concept check

Question 1.
WRITING
What is an included angle?
Answer:
Big Ideas Math Geometry Answer Key Chapter 5 Congruent Triangles 5.3 a 1

Question 2.
COMPLETE THE SENTENCE
If two sides and the included angle of one triangle are congruent to two sides and the included angle of a second triangle, then __________ .
Answer:

Monitoring progress and Modeling with Mathematics

In Exercises 3-8, name the included an1e between the pair of sides given.

Big Ideas Math Geometry Answer Key Chapter 5 Congruent Triangles 69

Question 3.
\(\overline{J K}\) and \(\overline{K L}\)
Answer:
Big Ideas Math Geometry Answer Key Chapter 5 Congruent Triangles 5.3 a 3

Question 4.
\(\overline{P K}\) and \(\overline{L K}\)
Answer:

Question 5.
\(\overline{L P}\) and \(\overline{L K}\)
Answer:
Big Ideas Math Geometry Answer Key Chapter 5 Congruent Triangles 5.3 a 5

Question 6.
\(\overline{J L}\) and \(\overline{J K}\)
Answer:

Question 7.
\(\overline{K L}\) and \(\overline{J L}\)
Answer:
Big Ideas Math Geometry Answer Key Chapter 5 Congruent Triangles 5.3 a 7

Question 8.
\(\overline{K P}\) and \(\overline{P L}\)
Answer:

In Exercises 9-14, decide whether enough information is given to prove that the triangles are congruent using the SAS Congruence Theorem (Theorem 5.5). Explain.

Question 9.
∆ABD, ∆CDB
Big Ideas Math Geometry Answer Key Chapter 5 Congruent Triangles 70
Answer:
Big Ideas Math Geometry Answer Key Chapter 5 Congruent Triangles 5.3 a 9

Question 10.
∆LMN, ∆NQP
Big Ideas Math Geometry Answer Key Chapter 5 Congruent Triangles 71
Answer:

Question 11.
∆YXZ, ∆WXZ
Big Ideas Math Geometry Answer Key Chapter 5 Congruent Triangles 72
Answer:
Big Ideas Math Geometry Answer Key Chapter 5 Congruent Triangles 5.3 a 11

Question 12.
∆QRV, ∆TSU
Big Ideas Math Geometry Answer Key Chapter 5 Congruent Triangles 73
Answer:

Question 13.
∆EFH, ∆GHF
Big Ideas Math Geometry Answer Key Chapter 5 Congruent Triangles 74
Answer:
Big Ideas Math Geometry Answer Key Chapter 5 Congruent Triangles 5.3 a 13

Question 14.
∆KLM, ∆MNK
Big Ideas Math Geometry Answer Key Chapter 5 Congruent Triangles 75
Answer:

In Exercises 15 – 18, write a proof.

Question 15.
Given \(\overline{P Q}\) bisects ∠SPT, \(\overline{S P}\) ≅ \(\overline{T P}\)
Prove ∆SPQ ≅ ∆TPQ
Big Ideas Math Geometry Answer Key Chapter 5 Congruent Triangles 76
Answer:
Big Ideas Math Geometry Answer Key Chapter 5 Congruent Triangles 5.3 a 15

Question 16.
Given \(\overline{A B}\) ≅ \(\overline{C D}\), \(\overline{A B}\) || \(\overline{C D}\)
Prove ∆ABC ≅ ∆CDA
Big Ideas Math Geometry Answer Key Chapter 5 Congruent Triangles 77
Answer:

Question 17.
Given C is the midpoint of \(\overline{A E}\) and \(\overline{B D}\)
Prove ∆ABC ≅ ∆EDC
Big Ideas Math Geometry Answer Key Chapter 5 Congruent Triangles 78
Answer:
Big Ideas Math Geometry Answer Key Chapter 5 Congruent Triangles 5.3 a 17

Question 18.
Given \(\overline{P T}\) ≅ \(\overline{R T}\), \(\overline{Q T}\) ≅ \(\overline{S T}\)
Prove ∆PQT ≅ ∆RST
Big Ideas Math Geometry Answer Key Chapter 5 Congruent Triangles 79
Answer:

In Exercises 19-22, use the given information to name two triangles that are congruent. Explain your reasoning.

Question 19.
∠SRT ≅ ∠URT, and R is the center of the circle.
Big Ideas Math Geometry Answer Key Chapter 5 Congruent Triangles 80
Answer:

Big Ideas Math Geometry Answer Key Chapter 5 Congruent Triangles 5.3 a 19

Question 20.
ABCD is a square with four congruent sides and four congruent angles.
Big Ideas Math Geometry Answer Key Chapter 5 Congruent Triangles 81
Answer:

Question 21.
RSTUV is a regular pentagon.
Big Ideas Math Geometry Answer Key Chapter 5 Congruent Triangles 82
Answer:
Big Ideas Math Geometry Answer Key Chapter 5 Congruent Triangles 5.3 a 21

Question 22.
\(\overline{M K}\) ⊥ \(\overline{M N}\), \(\overline{K L}\) ⊥ \(\overline{N L}\), and M and L are centers of circles.
Big Ideas Math Geometry Answer Key Chapter 5 Congruent Triangles 83
Answer:

CONSTRUCTION
In Exercises 23 and 24, construct a triangle that is congruent to ∆ABC using the SAS Congruence Theorem (Theorem 5.5).

Question 23.
Big Ideas Math Geometry Answer Key Chapter 5 Congruent Triangles 84
Answer:
Big Ideas Math Geometry Answer Key Chapter 5 Congruent Triangles 5.3 a 23

Question 24.
Big Ideas Math Geometry Answer Key Chapter 5 Congruent Triangles 85
Answer:

Question 25.
ERROR ANALYSIS
Describe and correct the error in finding the value of x.
Big Ideas Math Geometry Answer Key Chapter 5 Congruent Triangles 86
Answer:
Big Ideas Math Geometry Answer Key Chapter 5 Congruent Triangles 5.3 a 25

Question 26.
HOW DO YOU SEE IT?
What additional information do you need to prove that ∆ABC ≅ ∆DBC?
Big Ideas Math Geometry Answer Key Chapter 5 Congruent Triangles 87
Answer:

Question 27.
PROOF
The Navajo rug is made of isosceles triangles. You know ∠B ≅∠D. Use the SAS Congruence Theorem (Theorem 5.5 to show that ∆ABC ≅ ∆CDE. (See Example 3.)
Big Ideas Math Geometry Answer Key Chapter 5 Congruent Triangles 88
Answer:
Big Ideas Math Geometry Answer Key Chapter 5 Congruent Triangles 5.3 a 27

Question 28.
THOUGHT PROVOKING
There are six possible subsets of three sides or angles of a triangle: SSS, SAS, SSA, AAA, ASA, and AAS. Which of these correspond to congruence theorems? For those that do not, give a counterexample.
Answer:

Question 29.
MATHEMATICAL CONNECTIONS
Prove that
∆ABC ≅ ∆DEC
Then find the values of x and y.
Big Ideas Math Geometry Answer Key Chapter 5 Congruent Triangles 89
Answer:
Big Ideas Math Geometry Answer Key Chapter 5 Congruent Triangles 5.3 a 29

Question 30.
MAKING AN ARGUMENT
Your friend claims it is possible to Construct a triangle congruent to ∆ABC by first constructing \(\overline{A B}\) and \(\overline{A C}\), and then copying ∠C. Is your friend correct? Explain your reasoning.
Big Ideas Math Geometry Answer Key Chapter 5 Congruent Triangles 90
Answer:

Question 31.
PROVING A THEOREM
Prove the Reflections in Intersecting Lines Theorem (Theorem 4.3).
Answer:
Big Ideas Math Geometry Answer Key Chapter 5 Congruent Triangles 5.3 a 31.1
Big Ideas Math Geometry Answer Key Chapter 5 Congruent Triangles 5.3 a 31.2
Big Ideas Math Geometry Answer Key Chapter 5 Congruent Triangles 5.3 a 31.3
Big Ideas Math Geometry Answer Key Chapter 5 Congruent Triangles 5.3 a 31.4

Maintaining Mathematical Proficiency

Classify the triangle by its sides and by measuring its angles.

Question 32.
Big Ideas Math Geometry Answer Key Chapter 5 Congruent Triangles 91
Answer:

Question 33.
Big Ideas Math Geometry Answer Key Chapter 5 Congruent Triangles 92
Answer:
Big Ideas Math Geometry Answer Key Chapter 5 Congruent Triangles 5.3 a 33

Question 34.
Big Ideas Math Geometry Answer Key Chapter 5 Congruent Triangles 93
Answer:

Question 35.
Big Ideas Math Geometry Answer Key Chapter 5 Congruent Triangles 94
Answer:
Big Ideas Math Geometry Answer Key Chapter 5 Congruent Triangles 5.3 a 35

5.4 Equilateral and Isosceles Triangles

Exploration 1

Writing a Conjecture about Isosceles Triangles

Big Ideas Math Geometry Solutions Chapter 5 Congruent Triangles 95

Big Ideas Math Geometry Solutions Chapter 5 Congruent Triangles 96

Work with a partner: Use dynamic geometry software.

a. Construct a circle with a radius of 3 units centered at the origin.
Answer:

b. Construct ∆ABC so that B and C are on the circle and A is at the origin.
Answer:

c. Recall that a triangle is isosceles if it has at least two congruent sides. Explain why ∆ABC is an isosceles triangle.
Answer:

d. What do you observe about the angles of ∆ABC?
Answer:

e. Repeat parts (a)-(d) with several other isosceles triangles using circles of different radii. Keep track of your observations by copying and completing the table below. Then write a conjecture about the angle measures of an isosceles triangle.
CONSTRUCTING VIABLE ARGUMENTS
To be proficient in math, you need to make conjectures and build a logical progression of statements to explore the truth of your conjectures.
Answer:

f. Write the converse of the conjecture you wrote in part (e). Is the converse true?
Answer:

Communicate Your Answer

Question 2.
What conjectures can you make about the side lengths and angle measures of an
isosceles triangle?
Answer:

Question 3.
How would you prove your conclusion in Exploration 1 (e)? in Exploration 1(f)?
Answer:

Lesson 5.4 Equilateral and Isosceles Triangles

Monitoring Progress

Copy and complete the statement.

Big Ideas Math Geometry Solutions Chapter 5 Congruent Triangles 97

Question 1.
If \(\overline{H G}\) ≅ \(\overline{H K}\), then ∠ _______ ≅ ∠ _______ .
Answer:

Question 2.
If ∠KHJ ≅∠KJH, then ______ ≅ ______ .
Answer:

Question 3.
Find the length of \(\overline{S T}\) of the triangle at the left.
Answer:

Question 4.
Find the value of x and y in the diagram.
Big Ideas Math Geometry Solutions Chapter 5 Congruent Triangles 98
Answer:

Question 5.
In Example 4, show that ∆PTS ≅ ∆QTR
Answer:

Exercise 5.4 Equilateral and Isosceles Triangles

Vocabulary and Core Concept Check

Question 1.
VOCABULARY
Describe how to identify the vertex angle of an isosceles triangle.
Answer:
Big Ideas Math Geometry Answers Chapter 5 Congruent Triangles 5.4 a 1

Question 2.
WRITING
What is the relationship between the base angles of an isosceles triangle? Explain.
Answer:

Monitoring Progress and Modeling with Mathematics

In Exercises 3-6. copy and complete the statement. State which theorem you used.

Big Ideas Math Geometry Solutions Chapter 5 Congruent Triangles 99

Question 3.
If \(\overline{A E}\) ≅ \(\overline{D E}\) then ∠_____ ≅ ∠_____ .
Answer:
Big Ideas Math Geometry Answers Chapter 5 Congruent Triangles 5.4 a 3

Question 4.
If \(\overline{A B}\) ≅ \(\overline{E B}\) then ∠_____ ≅ ∠_____ .
Answer:

Question 5.
If ∠D ≅ ∠CED, then _______ ≅ _______ .
Answer:
Big Ideas Math Geometry Answers Chapter 5 Congruent Triangles 5.4 a 5

Question 6.
If ∠EBC ≅ ∠ECB, then _______ ≅ _______ .
Answer:

In Exercises 7-10. find the value of x.

Question 7.
Big Ideas Math Geometry Solutions Chapter 5 Congruent Triangles 100
Answer:
Big Ideas Math Geometry Answers Chapter 5 Congruent Triangles 5.4 a 7

Question 8.
Big Ideas Math Geometry Solutions Chapter 5 Congruent Triangles 101
Answer:

Question 9.
Big Ideas Math Geometry Solutions Chapter 5 Congruent Triangles 102
Answer:
Big Ideas Math Geometry Answers Chapter 5 Congruent Triangles 5.4 a 9

Question 10.
Big Ideas Math Geometry Solutions Chapter 5 Congruent Triangles 103
Answer:

Question 11.
MODELING WITH MATHEMATICS
The dimensions of a sports pennant are given in the diagram. Find the values of x and y.
Big Ideas Math Geometry Solutions Chapter 5 Congruent Triangles 104
Answer:
Big Ideas Math Geometry Answers Chapter 5 Congruent Triangles 5.4 a 11

Question 12.
MODELING WITH MATHEMATICS
A logo in an advertisement is an equilateral triangle with a side length of 7 centimeters. Sketch the logo and give the measure of each side.
Answer:

In Exercises 13-16, find the values of x and y.

Question 13.
Big Ideas Math Geometry Solutions Chapter 5 Congruent Triangles 105
Answer:
Big Ideas Math Geometry Answers Chapter 5 Congruent Triangles 5.4 a 13

Question 14.
Big Ideas Math Geometry Solutions Chapter 5 Congruent Triangles 106
Answer:

Question 15.
Big Ideas Math Geometry Solutions Chapter 5 Congruent Triangles 107
Answer:
Big Ideas Math Geometry Answers Chapter 5 Congruent Triangles 5.4 a 15

Question 16.
Big Ideas Math Geometry Solutions Chapter 5 Congruent Triangles 108
Answer:

CONSTRUCTION
In Exercises 17 and 18, construct an equilateral triangle whose sides are the given length.

Question 17.
3 inches
Answer:
Big Ideas Math Geometry Answers Chapter 5 Congruent Triangles 5.4 a 17

Question 18.
1.25 inches
Answer:

Question 19.
ERROR ANALYSIS
Describe and correct the error in finding the length of \(\overline{B C}\).
Big Ideas Math Geometry Solutions Chapter 5 Congruent Triangles 109
Answer:
Big Ideas Math Geometry Answers Chapter 5 Congruent Triangles 5.4 a 19

Question 20.
PROBLEM SOLVING
The diagram represents part of the exterior of the Bow Tower in Calgary. Alberta, Canada, In the diagram. ∆ABD and ∆CBD arc congruent equilateral triangles.
Big Ideas Math Geometry Solutions Chapter 5 Congruent Triangles 110

a. Explain why ∆ABC is isosceles.
Answer:

b. Explain ∠BAE ≅ ∠BCE.
Answer:

c. Show that ∆ABE and ∆CBE arc congruent.
Answer:

d. Find the measure of ∠BAE.
Answer:

Question 21.
FINDING A PATTERN
In the pattern shown. each small triangle is an equilateral triangle with an area of 1 square unit.
Big Ideas Math Geometry Solutions Chapter 5 Congruent Triangles 111
a. Explain how you know that an triangle made out of equilateral triangles is equilateral.
b. Find the areas of the first four triangles in the pattern.
c. Describe any patterns in the areas. Predict the area of the seventh triangle in the pattern. Explain your reasoning.
Answer:
Big Ideas Math Geometry Answers Chapter 5 Congruent Triangles 5.4 a 21

Question 22.
REASONING
The base of isosceles ∆XYZ is \(\overline{Y Z}\). What
can you prove? Select all that apply.
(A) \(\overline{X Y}\) ≅ \(\overline{X Z}\)
(B) ∠X ≅ ∠Y
(C) ∠Y ≅ ∠Z
(D) \(\overline{Y Z}\) ≅ \(\overline{Z X}\)
Answer:

In Exercises 23 and 24, find the perimeter of the triangle.

Question 23.
Big Ideas Math Geometry Solutions Chapter 5 Congruent Triangles 112
Answer:
Big Ideas Math Geometry Answers Chapter 5 Congruent Triangles 5.4 a 23

Question 24.
Big Ideas Math Geometry Solutions Chapter 5 Congruent Triangles 113
Answer:

MODELING WITH MATHEMATICS
In Exercises 25 – 28. use the diagram based on the color wheel. The 12 triangles in the diagram are isosceles triangles with congruent vertex angles.
Big Ideas Math Geometry Solutions Chapter 5 Congruent Triangles 114

Question 25.
Complementary colors lie directly opposite each other on the color wheel. Explain how you know that the yellow triangle is congruent to the purple triangle.
Answer:
Big Ideas Math Geometry Answers Chapter 5 Congruent Triangles 5.4 a 25

Question 26.
The measure of the vertex angle of the yellow triangle is 30°. Find the measures of the base angles.
Answer:

Question 27.
Trace the color wheel. Then form a triangle whose vertices are the midpoints of the bases of the red. yellow. and blue triangles. (These colors are the primary colors.) What type of triangle is this?
Answer:
Big Ideas Math Geometry Answers Chapter 5 Congruent Triangles 5.4 a 27

Question 28.
Other triangles can be brined on the color wheel that are congruent to the triangle in Exercise 27. The colors on the vertices of these triangles are called triads. What are the possible triads?
Answer:

Question 29.
CRITICAL THINKING
Are isosceles triangles always acute triangles? Explain your reasoning.
Answer:
Big Ideas Math Geometry Answers Chapter 5 Congruent Triangles 5.4 a 29

Question 30.
CRITICAL THINKING
Is it possible for an equilateral triangle to have an angle measure other than 60°? Explain your reasoning.
Answer:

Question 31.
MATHEMATICAL CONNECTIONS
The lengths of the sides of a triangle are 3t, 5t – 12, and t + 20. Find the values of t that make the triangle isosceles. Explain your reasoning.
Answer:
Big Ideas Math Geometry Answers Chapter 5 Congruent Triangles 5.4 a 31

Question 32.
MATHEMATICAL CONNECTIONS
The measure of an exterior angle of an isosceles triangle is x°. Write expressions representing the possible angle measures of the triangle in terms of x.
Answer:

Question 33.
WRITING
Explain why the measure of the vertex angle of an isosceles triangle must be an even number of degrees when the measures of all the angles of the triangle are whole numbers.
Answer:
Big Ideas Math Geometry Answers Chapter 5 Congruent Triangles 5.4 a 33

Question 34.
PROBLEM SOLVING
The triangular faces of the peaks on a roof arc congruent isosceles triangles with vertex angles U and V.
Big Ideas Math Geometry Solutions Chapter 5 Congruent Triangles 115
a. Name two angles congruent to ∠WUX. Explain your reasoning.
b. Find the distance between points U and V.
Answer:

Question 35.
PROBLEM SOLVING
A boat is traveling parallel to the shore along \(\vec{R}\)T. When the boat is at point R, the captain measures the angle to the lighthouse as 35°. After the boat has traveled 2.1 miles, the captain measures the angle to the lighthouse to be 70°.
Big Ideas Math Geometry Solutions Chapter 5 Congruent Triangles 116
a. Find SL. Explain your reasoning.
b. Explain how to find the distance between the boat and the shoreline.
Answer:
Big Ideas Math Geometry Answers Chapter 5 Congruent Triangles 5.4 a 35

Question 36.
THOUGHT PROVOKING
The postulates and theorems in this book represent Euclidean geometry. In spherical geometry, all points are points on the surface of a sphere. A line is a circle on the sphere whose diameter is equal to the diameter of the sphere. In spherical geometry, do all equiangular triangles have the same angle measures? Justify your answer.
Answer:

Question 37.
PROVING A COROLLARY
Prove that the Corollary to the Base Angles Theorem (Corollary 5.2) follows from the Base Angles Theorem (Theorem 5.6).
Answer:
Big Ideas Math Geometry Answers Chapter 5 Congruent Triangles 5.4 a 37

Question 38.
HOW DO YOU SEE IT?
You are designing fabric purses to sell at the school fair.
Big Ideas Math Geometry Solutions Chapter 5 Congruent Triangles 117
a. Explain why ∆ABE ≅ ∆DCE.
b. Name the isosceles triangles in the purse.
c. Name three angles that are congruent to ∠EAD.
Answer:

Question 39.
PROVING A COROLLARY
Prove that the Corollary to the Converse of the Base Angles Theorem (Corollary 5.3) follows from the Converse of the Base Angles Theorem (Theorem 5.7)
Answer:
Big Ideas Math Geometry Answers Chapter 5 Congruent Triangles 5.4 a 39

Question 40.
MAKING AN ARGUMENT
The coordinates of two points are T(0, 6) and U(6, 0) Your friend claims that points T, U, and V will always be the vertices of an isosceles triangle when V is any point on the line y = x. Is your friend correct? Explain your reasoning.
Answer:

Question 41.
PROOF
Use the diagram to prove that ∆DEF is equilateral.
Big Ideas Math Geometry Solutions Chapter 5 Congruent Triangles 118
Given ∆ABC is equilateral
∠CAD ≅ ∠ABE ≅ ∠BCF
Prove ∆DEF is equilateral
Answer:
Big Ideas Math Geometry Answers Chapter 5 Congruent Triangles 5.4 a 41.1
Big Ideas Math Geometry Answers Chapter 5 Congruent Triangles 5.4 a 41.2

Maintaining Mathematical Proficiency

Use the given property to complete the statement.

Question 42.
Reflexive Property of Congruence (Theorem 2. 1): ________ ≅ \(\overline{S E}\)
Answer:

Question 43.
Symmetric Property of Congruence (Theorem 2.1): If ________ ≅ ________, then \(\overline{R S}\) ≅ \(\overline{J K}\)
Answer:
Big Ideas Math Geometry Answers Chapter 5 Congruent Triangles 5.4 a 43

Question 44.
Transitive Property of Congruence (Theorem 2.1): If \(\overline{E F}\) ≅ \(\overline{P Q}\), and \(\overline{P Q}\) ≅ \(\overline{U V}\) ________ ≅ ________.
Answer:

5.1 to 5.4 Quiz

Find the measure of the exterior angle.

Question 1.
Big Ideas Math Geometry Solutions Chapter 5 Congruent Triangles 119
Answer:

Question 2.
Big Ideas Math Geometry Solutions Chapter 5 Congruent Triangles 120
Answer:

Question 3.
Big Ideas Math Geometry Solutions Chapter 5 Congruent Triangles 121
Answer:

Identify all pairs of congruent corresponding parts. Then write another congruence statement for the polygons.

Question 4.
∆ABC ≅ ∆DEF
Big Ideas Math Geometry Solutions Chapter 5 Congruent Triangles 122
Answer:

Question 5.
QRST ≅ WXYZ
Big Ideas Math Geometry Solutions Chapter 5 Congruent Triangles 123
Answer:

Decide whether enough information is given to prove that the triangles are congruent using the SAS Congruence Theorem (Thm 5.5). If so, write a proof. If not, explain why.

Question 6.
∆CAD, ∆CBD
Big Ideas Math Geometry Solutions Chapter 5 Congruent Triangles 124
Answer:

Question 7.
∆GHF, ∆KHJ
Big Ideas Math Geometry Solutions Chapter 5 Congruent Triangles 125
Answer:

Question 8.
∆LWP, ∆NMP
Big Ideas Math Geometry Solutions Chapter 5 Congruent Triangles 126
Answer:

Copy and complete the statement. State which theorem you used.

Big Ideas Math Geometry Solutions Chapter 5 Congruent Triangles 127

Question 9.
If VW ≅ WX, then ∠______ ≅ ∠ ________.
Answer:

Question 10.
If XZ ≅ XY. then∠______ ≅ ∠ ________.
Answer:

Question 11.
If ∠ZVX ≅∠ZXV, then ∠______ ≅ ∠ ________.
Answer:

Question 12.
If ∠XYZ ≅∠ZXY, then ∠______ ≅ ∠ ________.
Answer:

Find the values of x and y.

Question 13.
∆DEF ≅ ∆QRS
Big Ideas Math Geometry Solutions Chapter 5 Congruent Triangles 128
Answer:

Question 14.
Big Ideas Math Geometry Solutions Chapter 5 Congruent Triangles 129
Answer:

Question 15.
In a right triangle, the measure of one acute angle is 4 times the difference of the measure of the other acute angle and 5. Find the measure ol each acute angle in the triangle. (Section 5.1)
Answer:

Question 16.
The figure shows a stained glass window. (Section 5.1 and Section 5.3)

Big Ideas Math Geometry Solutions Chapter 5 Congruent Triangles 130

a. Classify triangles 1 – 4 by their angles.
Answer:

b. Classify triangles 4 – 6 by their sides.
Answer:

c. Is there enough information given to prove that ∆7 ≅ ∆8? If so, label the vertices
and write a proof. If not, determine what additional information is needed.
Answer:

5.5 Proving Triangle Congruence by SSS

Exploration 1

Drawing Triangles

Work with a partner.
Use dynamic geometry software.

Big Ideas Math Answer Key Geometry Chapter 5 Congruent Triangles 131

Big Ideas Math Answer Key Geometry Chapter 5 Congruent Triangles 132

Big Ideas Math Answer Key Geometry Chapter 5 Congruent Triangles 133

a. Construct circles with radii of 2 units and 3 units centered at the origin. Label the origin A. Then draw \(\overline{B C}\) of length 4 units.
Answer:

b. Move \(\overline{B C}\) so that B is on the smaller circle and C is on the larger circle. Then draw ∆ABC.
Answer:

c. Explain why the side lengths of ∆ABC are 2, 3, and 4 units.
Answer:

d. Find m∠A, m∠B, and m∠C.
Answer:

e. Repeat parts (b)and (d) several times, moving \(\overline{B C}\) to different locations. Keep track of ‘our results by copying and completing the table below. What can you conclude?
USING TOOLS STRATEGICALLY
To be proficient in math, you need to use technology to help visualize the results of varying assumptions, explore consequences, and compare predictions with data.
Answer:

Communicate Your Answer

Question 2.
What can you conclude about two triangles when you know the corresponding sides are congruent?
Answer:

Question 3.
How would you prove your conclusion in Exploration 1(e)?
Answer:

Lesson 5.5 Proving Triangle Congruence by SSS

Monitoring Progress

Decide whether the congruence statement is true. Explain your reasoning.

Question 1.
∆DFG ≅ ∆HJK
Big Ideas Math Answer Key Geometry Chapter 5 Congruent Triangles 134
Answer:

Question 2.
∆ACB ≅ ∆CAD
Big Ideas Math Answer Key Geometry Chapter 5 Congruent Triangles 135
Answer:

Question 3.
∆QPT ≅ ∆RST
Big Ideas Math Answer Key Geometry Chapter 5 Congruent Triangles 136
Answer:

Determine whether the figure is stable. Explain your reasoning.

Question 4.
Big Ideas Math Answer Key Geometry Chapter 5 Congruent Triangles 137
Answer:

Question 5.
Big Ideas Math Answer Key Geometry Chapter 5 Congruent Triangles 138
Answer:

Question 6.
Big Ideas Math Answer Key Geometry Chapter 5 Congruent Triangles 139
Answer:

Use the diagram.

Big Ideas Math Answer Key Geometry Chapter 5 Congruent Triangles 140

Question 7.
Redraw ∆ABC and ∆DCB side by side with corresponding parts in the same position.
Answer:

Question 8.
Use the information in the diagram to prove that ∆ABC ≅ ∆DCB.
Answer:

Exercise 5.5 Proving Triangle Congruence by SSS

Vocabulary and Core Concept Check

Question 1.
COMPLETE THE SENTENCE
The side opposite the right angle is called the __________of the right triangle.
Answer:
Big Ideas Math Geometry Solutions Chapter 5 Congruent Triangles 5.5 a 1

Question 2.
WHICH ONE DOESNT BELONG?
Which triangles legs do not belong with the other three? Explain your reasoning.
Big Ideas Math Answer Key Geometry Chapter 5 Congruent Triangles 142
Answer:

Monitoring Progress and Modeling with Mathematics

In Exercises 3 and 4, decide whether enough information is given to prove that the triangles are congruent using the SSS Congruence Theorem (Theorem 5.8). Explain.

Question 3.
∆ABC, ∆DBE
Big Ideas Math Answer Key Geometry Chapter 5 Congruent Triangles 141
Answer:
Big Ideas Math Geometry Solutions Chapter 5 Congruent Triangles 5.5 a 3

Question 4.
∆PQS, ∆RQS
Big Ideas Math Answer Key Geometry Chapter 5 Congruent Triangles 143
Answer:

In Exercises 5 and 6, decide whether enough information is given to prove that the triangles are congruent using the HL Congruence Theorem (Theorem 5.9). Explain.

Question 5.
∆ABC, ∆FED
Big Ideas Math Answer Key Geometry Chapter 5 Congruent Triangles 144
Answer:
Big Ideas Math Geometry Solutions Chapter 5 Congruent Triangles 5.5 a 5

Question 6.
∆PQT, ∆SRT
Big Ideas Math Answer Key Geometry Chapter 5 Congruent Triangles 145
Answer:

In Exercises 7-10. decide whether the congruence statement is true. Explain your reasoning.

Question 7.
∆RST ≅ ∆TQP
Big Ideas Math Answer Key Geometry Chapter 5 Congruent Triangles 146
Answer:
Big Ideas Math Geometry Solutions Chapter 5 Congruent Triangles 5.5 a 7

Question 8.
∆ABD ≅ ∆CDB
Big Ideas Math Answer Key Geometry Chapter 5 Congruent Triangles 147
Answer:

Question 9.
∆DEF ≅ ∆DGF
Big Ideas Math Answer Key Geometry Chapter 5 Congruent Triangles 148
Answer:
Big Ideas Math Geometry Solutions Chapter 5 Congruent Triangles 5.5 a 9

Question 10.
∆JKL ≅ ∆LJM
Big Ideas Math Answer Key Geometry Chapter 5 Congruent Triangles 149
Answer:

In Exercises 11 and 12, determine whether the figure is stable. Explain your reasoning.

Question 11.
Big Ideas Math Answer Key Geometry Chapter 5 Congruent Triangles 150
Answer:
Big Ideas Math Geometry Solutions Chapter 5 Congruent Triangles 5.5 a 11

Question 12.
Big Ideas Math Answer Key Geometry Chapter 5 Congruent Triangles 151
Answer:

In Exercises 13 and 14, redraw the triangles so they are side by side with corresponding parts in the same position. Then write a proof.

Question 13.
Given \(\overline{A C}\) ≅ \(\overline{B D}\)
\(\overline{A B}\) ⊥ \(\overline{A D}\)
\(\overline{C D}\) ⊥ \(\overline{A D}\)
Prove ∆BAD ≅ ∆CDA
Big Ideas Math Answer Key Geometry Chapter 5 Congruent Triangles 152
Answer:
Big Ideas Math Geometry Solutions Chapter 5 Congruent Triangles 5.5 a 13

Question 14.
Given G is the midpoint of \(\overline{E H}\), \(\overline{F G}\) ≅ \(\overline{G I}\), ∠E and ∠H are right angles.
Prove ∆EFG ≅ ∆HIG
Big Ideas Math Answer Key Geometry Chapter 5 Congruent Triangles 153
Answer:

In Exercises 15 and 16. write a proof.

Question 15.
Given \(\overline{L M}\) ≅ \(\overline{J K}\), \(\overline{M J}\) ≅ \(\overline{K L}\)
Prove ∆LMJ ≅ ∆JKL
Big Ideas Math Answer Key Geometry Chapter 5 Congruent Triangles 154
Answer:
Big Ideas Math Geometry Solutions Chapter 5 Congruent Triangles 5.5 a 15

Question 16.
Given \(\overline{W X}\) ≅ \(\overline{V Z}\), \(\overline{W Y}\) ≅ \(\overline{V Y}\), \(\overline{Y Z}\) ≅ \(\overline{Y X}\)
Prove ∆VWX ≅ ∆WVZ
Big Ideas Math Answer Key Geometry Chapter 5 Congruent Triangles 155
Answer:

CONSTRUCTION
In Exercises 17 and 18, construct a triangle that is congruent to ∆QRS using the SSS Congruence Theorem Theorem 5.8).

Question 17.
Big Ideas Math Answer Key Geometry Chapter 5 Congruent Triangles 156
Answer:
Big Ideas Math Geometry Solutions Chapter 5 Congruent Triangles 5.5 a 17

Question 18.
Big Ideas Math Answer Key Geometry Chapter 5 Congruent Triangles 157
Answer:

Question 19.
ERROR ANALYSIS
Describe and correct the error in identifying congruent triangles.
Big Ideas Math Answer Key Geometry Chapter 5 Congruent Triangles 158
Answer:
Big Ideas Math Geometry Solutions Chapter 5 Congruent Triangles 5.5 a 19

Question 20.
ERROR ANALYSIS
Describe and correct the error in determining the value of x that makes the triangles congruent.
Big Ideas Math Answer Key Geometry Chapter 5 Congruent Triangles 159
Answer:

Question 21.
MAKING AN ARGUMENT
Your friend claims that in order to use the SSS Congruence Theorem (Theorem 5.8) Lo prove that two triangles are congruent, both triangles must be equilateral triangles. Is your friend correct? Explain your reasoning.
Answer:
Big Ideas Math Geometry Solutions Chapter 5 Congruent Triangles 5.5 a 21

Question 22.
MODELING WITH MATHEMATICS
The distances between consecutive bases on a softball field are the same. The distance from home plate to second base is the same as the distance from first base to third base. The angles created at each base are 90°. Prove
∆HFS ≅ ∆FST ≅ ∆STH
Big Ideas Math Answer Key Geometry Chapter 5 Congruent Triangles 160
Answer:

Question 23.
REASONING
To support a tree you attach wires from the trunk of the tree to stakes in the ground, as shown in the diagram.
Big Ideas Math Answer Key Geometry Chapter 5 Congruent Triangles 161
a. What additional information do you need to use the HL Congruence Theorem (Theorem 5.9) to prove that ∆JKL ≅ ∆MKL?
b. Suppose K is the midpoint of JM. Name a theorem you could use to prove that ∆JKL ≅ ∆MKL. Explain your reasoning.
Answer:
Big Ideas Math Geometry Solutions Chapter 5 Congruent Triangles 5.5 a 23

Question 24.
REASONING
Use the photo of the Navajo rug, where \(\overline{B C}\) ≅ \(\overline{D E}\) and \(\overline{A C}\) ≅ \(\overline{C E}\)
Big Ideas Math Answer Key Geometry Chapter 5 Congruent Triangles 162
a. What additional intormation do you need to use the SSS Congruence Theorem (Theorem 5.8) to prove that ∆ABC ≅ ∆CDE?
b. What additional information do you need to use the HL Congruence Theorem (Theorem 5.9) to prove that ∆ABC ≅ ∆CDE?
Answer:

In Exercises 25-28. use the given coordinates to determine whether ∆ABC ≅ ∆DEF.

Question 25.
A(- 2, – 2), B(4, – 2), C(4, 6), D(5, 7), E(5, 1), F(13, 1)
Answer:
Big Ideas Math Geometry Solutions Chapter 5 Congruent Triangles 5.5 a 25

Question 26.
A(- 2, 1), B(3, – 3), C(7, 5), D(3, 6), E(S, 2), F( 10, 11)
Answer:

Question 27.
A(0, 0), B(6, 5), C(9, 0), D(0, – 1), E(6, – 6), F(9, – 1)
Answer:
Big Ideas Math Geometry Solutions Chapter 5 Congruent Triangles 5.5 a 27

Question 28.
A(- 5, 7), B(- 5, 2), C(0, 2), D(0, 6), E(o, 1), F(4, 1)
Answer:

Question 29.
CRITICAL THINKING
You notice two triangles in the tile floor of a hotel lobby. You want to determine whether the triangles are congruent. but you only have a piece of string. Can you determine whether the triangles are congruent? Explain.
Answer:
Big Ideas Math Geometry Solutions Chapter 5 Congruent Triangles 5.5 a 29

Question 30.
HOW DO YOU SEE IT?
There are several theorems you can use to show that the triangles in the “square” pattern are congruent. Name two of them.
Big Ideas Math Answer Key Geometry Chapter 5 Congruent Triangles 163
Answer:

Question 31.
MAKING AN ARGUMENT
Your cousin says that ∆JKL is congruent to ∆LMJ by the SSS Congruence Theorem (Thm. 5.8). Your friend says that ∆JKL is congruent to ∆LMJ by the HL Congruence Theorem (Thm. 5.9). Who is correct? Explain your reasoning.
Big Ideas Math Answer Key Geometry Chapter 5 Congruent Triangles 164
Answer:
Big Ideas Math Geometry Solutions Chapter 5 Congruent Triangles 5.5 a 31

Question 32.
THOUGHT PROVOKING
The postulates and theorems in this book represent Euclidean geometry. In spherical geometry. all points are points on the surface of a sphere. A line is a circle on the sphere whose diameter is equal to the diameter of the sphere. In spherical geometry. do you think that two triangles are congruent if their corresponding sides are congruent? Justify your answer.
Answer:

USING TOOLS
In Exercises 33 and 34, use the given information to sketch ∆LMN and ∆STU. Mark the triangles with the given information.

Question 33.
Big Ideas Math Answer Key Geometry Chapter 5 Congruent Triangles 165
Answer:
Big Ideas Math Geometry Solutions Chapter 5 Congruent Triangles 5.5 a 33

Question 34.
Big Ideas Math Answer Key Geometry Chapter 5 Congruent Triangles 166
Answer:

Question 35.
CRITICAL THINKING
The diagram shows the light created by two spotlights, Both spotlights are the same distance from the stage.
Big Ideas Math Answer Key Geometry Chapter 5 Congruent Triangles 167
Answer:
a. Show that ∆ABD ≅ ∆CBD. State which theorem or postulate you used and explain your reasoning.
b. Are all four right triangles shown in the diagram Congruent? Explain your reasoning.
Answer:
Big Ideas Math Geometry Solutions Chapter 5 Congruent Triangles 5.5 a 35

Question 36.
MATHEMATICAL CONNECTIONS
Find all values of x that make the triangles congruent. Explain.
Big Ideas Math Answer Key Geometry Chapter 5 Congruent Triangles 168
Answer:

Maintaining Mathematical proficiency

Use the congruent triangles.

Big Ideas Math Answer Key Geometry Chapter 5 Congruent Triangles 169

Question 37.
Name the Segment in ∆DEF that is congruent to \(\overline{A C}\).
Answer:
Big Ideas Math Geometry Solutions Chapter 5 Congruent Triangles 5.5 a 37

Question 38.
Name the segment in ∆ABC that is congruent to \(\overline{E F}\).
Answer:

Question 39.
Name the angle in ∆DEF that is congruent to ∠B.
Answer:
Big Ideas Math Geometry Solutions Chapter 5 Congruent Triangles 5.5 a 39

Question 40.
Name the angle in ∆ABC that is congruent to ∠F.
Answer:

5.6 Proving Triangle Congruence by ASA and AAS

Exploration 1

Determining Whether SSA Is Sufficient

Work with a partner.
a. Use dynamic geometry software to construct ∆ABC. Construct the triangle so that vertex B is at the origin. \(\overline{A B}\) has a length of 3 units. and \(\overline{B C}\) has a length of 2 units.
Answer:

b. Construct a circle with a radius of 2 units centered at the origin. Locate point D where the circle intersects \(\overline{A C}\). Draw \(\overline{B D}\).
Big Ideas Math Geometry Answers Chapter 5 Congruent Triangles 170
Answer:

c. ∆ABC and ∆ABD have two congruent sides and a non included congruent angle.
Name them.
Answer:

d. Is ∆ABC ≅ ∆ABD? Explain your reasoning.
Answer:

e. Is SSA sufficient to determine whether two triangles are congruent? Explain your reasoning.
Answer:

Exploration 2

Determining Valid Congruence Theorems

Work with a partner. Use dynamic geometry software to determine which of the following are valid triangle congruence theorems. For those that are not valid. write a counter example. Explain your reasoning.
CONSTRUCTING VIABLE ARGUMENTS
To be proficient in math, you need to recognize and use counterexamples.

Possible Congruence Theorem Valid or not valid?
SSS
SSA
SAS
AAS
ASA
AAA

Answer:

Communicate Your Answer

Question 3.
What information is sufficient to determine whether two triangles are congruent?
Answer:

Question 4.
Is it possible to show that two triangles are congruent using more than one congruence theorem? If so, give an example.
Answer:

Lesson 5.6 Proving Triangle Congruence by ASA and AAS

Monitoring Progress

Question 1.
Can the triangles be proven congruent with the information given in the diagram? If so, state the theorem you would use.
Big Ideas Math Geometry Answers Chapter 5 Congruent Triangles 171
Answer:

Question 2.
In the diagram, \(\overline{A B}\) ⊥ \(\overline{A D}\), \(\overline{D E}\) ⊥ \(\overline{A D}\), and \(\overline{A C}\) ≅ \(\overline{D C}\) . Prove ∆ABC ≅ ∆DEF.
Big Ideas Math Geometry Answers Chapter 5 Congruent Triangles 172
Answer:

Question 3.
In the diagram, ∠S ≅ ∠U and \(\overline{B D}\)\(\overline{B D}\) . Prove that ∆RST ≅ ∆VYT
Big Ideas Math Geometry Answers Chapter 5 Congruent Triangles 173
Answer:

Lesson 5.6 Proving Triangle Congruence by ASA and AAS

Vocabulary and Core Concept Check

Question 1.
WRITING
How arc the AAS Congruence Theorem (Theorem 5. 11) and the ASA Congruence
Theorem (Theorem 5.10) similar? How are they different?
Answer:
Big Ideas Math Answer Key Geometry Chapter 5 Congruent Triangles 5.6 a 1

Question 2.
WRITING
You know that a pair of triangles has two pairs of congruent corresponding angles. What other information do you need to show that the triangles are congruent?
Answer:

Monitoring Progress and Modeling with Mathematics

In Exercises 3-6, decide whether enough information is given to prove that the triangles are congruent. If so, state the theorem you would use.

Question 3.
∆ABC, ∆QRS
Big Ideas Math Geometry Answers Chapter 5 Congruent Triangles 174
Answer:
Big Ideas Math Answer Key Geometry Chapter 5 Congruent Triangles 5.6 a 3

Question 4.
∆ABC, ∆DBC
Big Ideas Math Geometry Answers Chapter 5 Congruent Triangles 175
Answer:

Question 5.
∆XYZ, ∆JKL
Big Ideas Math Geometry Answers Chapter 5 Congruent Triangles 176
Answer:
Big Ideas Math Answer Key Geometry Chapter 5 Congruent Triangles 5.6 a 5

Question 6.
∆RSV, ∆UTV
Big Ideas Math Geometry Answers Chapter 5 Congruent Triangles 177
Answer:

In Exercises 7 and 8, state the third congruence statement that is needed to prove that ∆FGH ≅ ∆LMN the given theorem.

Big Ideas Math Geometry Answers Chapter 5 Congruent Triangles 178
Question 7.
Given \(\overline{G H}\) ≅ \(\overline{M N}\), ∠G ≅ ∠M, _______ = ________
Use the AAS Congruence Theorem (Thm. 5.11).
Answer:
Big Ideas Math Answer Key Geometry Chapter 5 Congruent Triangles 5.6 a 7

Question 8.
Given \(\overline{F G}\) ≅ \(\overline{L M}\), ∠G ≅ ∠M, _______ = ________
Use the ASA Congruence Theorem (Thm. 5.10).
Answer:

In Exercises 9 – 12. decide whether you can use the given information to prove that ∆ABC ≅ ∆DEF Explain your reasoning.

Question 9.
∠A ≅ ∠G, ∠C ≅∠F, \(\overline{A C}\) ≅ \(\overline{D F}\)
Answer:
Big Ideas Math Answer Key Geometry Chapter 5 Congruent Triangles 5.6 a 9

Question 10.
∠C ≅ ∠F, \(\overline{A B}\) ≅ \(\overline{D E}\), \(\overline{B C}\) ≅ \(\overline{E F}\)
Answer:

Question 11.
∠B ≅ ∠E, ∠C ≅∠F, \(\overline{A C}\) ≅ \(\overline{D E}\)
Answer:
Big Ideas Math Answer Key Geometry Chapter 5 Congruent Triangles 5.6 a 11

Question 12.
∠A ≅ ∠D, ∠B ≅∠E, \(\overline{B C}\) ≅ \(\overline{E F}\)
Answer:

CONSTRUCTION
In Exercises 13 and 14, construct a triangle that is congruent to the given triangle using the ASA Congruence Theorem (Theorem 5.10). Use a compass and straightedge.

Question 13.
Big Ideas Math Geometry Answers Chapter 5 Congruent Triangles 179
Answer:
Big Ideas Math Answer Key Geometry Chapter 5 Congruent Triangles 5.6 a 13

Question 14.
Big Ideas Math Geometry Answers Chapter 5 Congruent Triangles 180
Answer:

ERROR ANALYSIS
In Exercises 15 and 16, describe and correct the error.

Question 15.
Big Ideas Math Geometry Answers Chapter 5 Congruent Triangles 181
Answer:
Big Ideas Math Answer Key Geometry Chapter 5 Congruent Triangles 5.6 a 15

Question 16.
Big Ideas Math Geometry Answers Chapter 5 Congruent Triangles 182
Answer:

PROOF
In Exercises 17 and 18, prove that the triangles are congruent using the ASA Congruence Theorem (Theorem 5.10).

Question 17.
Given M is the midpoint of \(\overline{N L}\).
Big Ideas Math Geometry Answers Chapter 5 Congruent Triangles 183
Prove ∆NQM ≅ ∆MPL
Big Ideas Math Geometry Answers Chapter 5 Congruent Triangles 184
Answer:
Big Ideas Math Answer Key Geometry Chapter 5 Congruent Triangles 5.6 a 17.1
Big Ideas Math Answer Key Geometry Chapter 5 Congruent Triangles 5.6 a 17.2

Question 18.
Given \(\overline{A J}\) ≅ \(\overline{K C}\) ∠BJK ≅ ∠BKJ, ∠A ≅ ∠C
Prove ∆ABK ≅ ∆CBJ
Big Ideas Math Geometry Answers Chapter 5 Congruent Triangles 185
Answer:

PROOF
In Exercises 19 and 20, prove that the triangles are congruent using the AAS Congruence Theorem (Theorem 5.11).

Question 19.
Given \(\overline{V W}\) ≅ \(\overline{U W}\), ∠X ≅ ∠Z
Prove ∆XWV ≅ ∆ZWU
Big Ideas Math Geometry Answers Chapter 5 Congruent Triangles 186
Answer:
Big Ideas Math Answer Key Geometry Chapter 5 Congruent Triangles 5.6 a 19

Question 20.
Given ∠NKM ≅∠LMK, ∠L ≅∠N
Prove ∆NMK ≅ ∆LKM
Big Ideas Math Geometry Answers Chapter 5 Congruent Triangles 187
Answer:

PROOF
In Exercises 21-23, write a paragraph proof for the theorem about right triangles.

Question 21.
Hypotenuse-Angle (HA) Congruence Theorem
If an angle and the hypotenuse of a right triangle are congruent to an angle and the hypotenuse of a second right triangle, then the triangles are congruent.
Answer:
Big Ideas Math Answer Key Geometry Chapter 5 Congruent Triangles 5.6 a 21

Question 22.
Leg-Leg (LL) Congruence Theorem
If the legs of a right triangle are congruent to the legs of a second right triangle, then the triangles are congruent.
Answer:

Question 23.
Angle-Leg (AL) Congruence Theorem
If an angle and a leg of a right triangle are congruent to an angle and a leg of a second right triangle, then the triangles are Congruent.
Answer:
Big Ideas Math Answer Key Geometry Chapter 5 Congruent Triangles 5.6 a 23

Question 24.
REASONING
What additional in information do you need to prove ∆JKL ≅ ∆MNL by the ASA Congruence Theorem (Theorem 5. 10)?
Big Ideas Math Geometry Answers Chapter 5 Congruent Triangles 188
(A) \(\overline{K M}\) ≅ \(\overline{K J}\)
(B) \(\overline{K H}\) ≅ \(\overline{N H}\)
(C) ∠M ≅ ∠J
(D) ∠LKJ ≅ ∠LNM
Answer:

Question 25.
MATHEMATICAL CONNECTIONS
This toy contains △ABC and △DBC. Can you conclude that △ABC ≅ △DBC from the given angle measures? Explain
Big Ideas Math Geometry Answers Chapter 5 Congruent Triangles 189
m∠ABC = (8x – 32)°
m∠DBC = (4y – 24)°
m∠BCA = (5x + 10)°
m∠BCD = (3y + 2)°
m∠CAB = (2x – 8)°
m∠CDB = (y – 6)°
Answer:
Big Ideas Math Answer Key Geometry Chapter 5 Congruent Triangles 5.6 a 25.1
Big Ideas Math Answer Key Geometry Chapter 5 Congruent Triangles 5.6 a 25.2

Question 26.
REASONING
Which of the following congruence statements are true? Select all that apply.
Big Ideas Math Geometry Answers Chapter 5 Congruent Triangles 190
(A) \(\overline{B D}\) ≅ \(\overline{B D}\)
(B) ∆STV ≅ ∆XVW
(C) ∆TVS ≅ ∆VWU
(D) ∆VST ≅ ∆VUW
Answer:

Question 27.
PROVING A THEOREM
Prove the Converse of the Base Angles Theorem (Theorem 5.7). (Hint: Draw an auxiliary line inside the triangle.)
Answer:
Big Ideas Math Answer Key Geometry Chapter 5 Congruent Triangles 5.6 a 27.1
Big Ideas Math Answer Key Geometry Chapter 5 Congruent Triangles 5.6 a 27.2

Question 28.
MAKING AN ARGUMENT
Your friend claims to be able Lo rewrite any proof that uses the AAS Congruence Theorem (Thin. 5. 11) as a proof that uses the ASA Congruence Theorem (Thin. 5.10). Is this possible? Explain our reasoning.
Answer:

Question 29.
MODELING WITH MATHEMATICS
When a light ray from an object meets a mirror, it is reflected back to your eye. For example, in the diagram, a light ray from point C is reflected at point D and travels back to point A. The law of reflection states that the angle of incidence, ∠CDB. is congruent to the angle of reflection. ∠ADB.
Big Ideas Math Geometry Answers Chapter 5 Congruent Triangles 191
a. Prove that ∆ABD is Congruent to ∆CBD.
Given ∠CBD ≅∠ABD
DB ⊥ AC
Prove ∆ABD ≅ ∆CBD
b. Verify that ∆ACD is isosceles.
c. Does moving away from the mirror have an effect on the amount of his or her reflection a person sees? Explain.
Answer:
Big Ideas Math Answer Key Geometry Chapter 5 Congruent Triangles 5.6 a 29.1
Big Ideas Math Answer Key Geometry Chapter 5 Congruent Triangles 5.6 a 29.2

Question 30.
HOW DO YOU SEE IT?
Name as man pairs of congruent triangles as you can from the diagram. Explain how you know that each pair of triangles is congruent.
Big Ideas Math Geometry Answers Chapter 5 Congruent Triangles 192
Answer:

Question 31.
CONSTRUCTION
Construct a triangle. Show that there is no AAA congruence rule by constructing a second triangle that has the same angle measures but is not congruent.
Answer:
Big Ideas Math Answer Key Geometry Chapter 5 Congruent Triangles 5.6 a 31

Question 32.
THOUGHT PROVOKING
Graph theory is a branch of mathematics that studies vertices and the way they are connected. In graph theory. two polygons are isomorphic if there is a one-to-one mapping from one polygon’s vertices to the other polygon’s vertices that preserves adjacent vertices. In graph theory, are any two triangles isomorphic? Explain your reasoning. second triangle that has the same angle measures but is not congruent.
Answer:

Question 33.
Mathematical Connections
Six statements are given about ∆TUV and ∆XYZ
Big Ideas Math Geometry Answers Chapter 5 Congruent Triangles 193
a. List all combinations of three given statements that could provide enough information to prove that ∆TUV is congruent to ∆XYZ.
b. You choose three statements at random. What is the probability that the statements you choose provide enough information to prove that the triangles are congruent?
Answer:
Big Ideas Math Answer Key Geometry Chapter 5 Congruent Triangles 5.6 a 33.1
Big Ideas Math Answer Key Geometry Chapter 5 Congruent Triangles 5.6 a 33.2

Maintaining Mathematical proficiency

Find the coordinates of the midpoint of the line segment with the given endpoints.

Question 34.
C(1, 0) and D(5, 4)
Answer:

Question 35.
J(- 2, 3) and K(4, – 1)
Answer:
Big Ideas Math Answer Key Geometry Chapter 5 Congruent Triangles 5.6 a 35

Question 36.
R(- 5, – 7) and S(2, – 4)
Answer:

Copy and angle using a compass and straightedge.

Question 37.
Big Ideas Math Geometry Answers Chapter 5 Congruent Triangles 194
Answer:
Big Ideas Math Answer Key Geometry Chapter 5 Congruent Triangles 5.6 a 37

Question 38.
Big Ideas Math Geometry Answers Chapter 5 Congruent Triangles 195
Answer:

5.7 Using Congruent Triangles

Exploration 1

Measuring the Width of a River

Big Ideas Math Geometry Answers Chapter 5 Congruent Triangles 219

Work with a partner:
The figure shows how a surveyor can measure the width of a river by making measurements on only one side of the river.

a. Study the figure. Then explain how the surveyor can find the width of the river.
Answer:

b. Write a proof to verify that the method you described in part (a) is valid.
Given ∠A is a right angle, ∠D is a right angle, \(\overline{A C}\) ≅ \(\overline{C D}\)
Answer:

c. Exchange Proofs with your partner and discuss the reasoning used.
CRITIQUING THE REASONING OF OTHERS
To be proficient in math, you need to listen to or read the arguments of others, decide whether they make sense, and ask useful questions to clarify or improve the arguments.
Answer:

Exploration 2

Measuring the Width of a River

Big Ideas Math Geometry Answers Chapter 5 Congruent Triangles 196

Work with a partner. It was reported that one of Napoleon’s offers estimated the width of a river as follows. The officer stood on the hank of the river and lowered the visor on his cap until the farthest thin visible was the edge of the bank on the other side. He then turned and rioted the point on his side that was in line with the tip of his visor and his eye. The officer then paced the distance to this point and concluded that distance was the width of the river.

a. Study the figure. Then explain how the officer concluded that the width of the river is EG.
Answer:

b. Write a proof to verify that the conclusion the officer made is correct.
Given ∠DEG is a right angle, ∠DEF is a right angle, ∠EDG ≅ ∠EDF
Answer:

c. Exchange proofs with your partner and discuss the reasoning used.
Answer:

Communicate Your Answer

Question 3.
How can you use congruent triangles to make an indirect measurement?
Answer:

Question 4.
Why do you think the types of measurements described in Explorations 1 and 2 are called indirect measurements?
Answer:

Lesson 5.7 Using Congruent Triangles

Monitoring Progress

Question 1.
Explain how you can prove that ∠A ≅ ∠C.
Big Ideas Math Geometry Answers Chapter 5 Congruent Triangles 197
Answer:

Question 2.
In Example 2, does it mailer how far from point N you place a stake at point K? Explain.
Answer:

Question 3.
Write a plan to prove that ∆PTU ≅ ∆UQP.
Big Ideas Math Geometry Answers Chapter 5 Congruent Triangles 198
Answer:

Question 4.
Use the construction of an angle bisector on page 42. What segments can you assume are congruent?
Answer:

Exercise 5.7 Using Congruent Triangles

Vocabulary and core concept check

Question 1.
COMPLETE THE SENTENCE
_____________ parts of congruent triangle are congruent.
Answer:
Big Ideas Math Answers Geometry Chapter 5 Congruent Triangles 5.7 a 1

Question 2.
WRITING
Describe a situation in which you might choose to use indirect measurement with
congruent triangles to find a measure rather than measuring directly.
Answer:

Monitoring Progress and Modeling With Mathematics

In Exercise 3-8, explain how to prove that the statement is true.

Question 3.
∠A ≅ ∠D
Big Ideas Math Geometry Answers Chapter 5 Congruent Triangles 199
Answer:
Big Ideas Math Answers Geometry Chapter 5 Congruent Triangles 5.7 a 3

Question 4.
∠Q ≅∠T
Big Ideas Math Geometry Answers Chapter 5 Congruent Triangles 200
Answer:

Question 5.
\(\overline{J M}\) ≅ \(\overline{L M}\)
Big Ideas Math Geometry Answers Chapter 5 Congruent Triangles 201
Answer:
Big Ideas Math Answers Geometry Chapter 5 Congruent Triangles 5.7 a 5

Question 6.
\(\overline{A C}\) ≅ \(\overline{D B}\)
Big Ideas Math Geometry Answers Chapter 5 Congruent Triangles 202
Answer:

Question 7.
\(\overline{G K}\) ≅ \(\overline{H J}\)
Big Ideas Math Geometry Answers Chapter 5 Congruent Triangles 203
Answer:
Big Ideas Math Answers Geometry Chapter 5 Congruent Triangles 5.7 a 7

Question 8.
\(\overline{Q W}\) ≅ \(\overline{V T}\)
Big Ideas Math Geometry Answers Chapter 5 Congruent Triangles 204
Answer:

In Exercises 9-12, write a plan to prove that ∠1 ≅∠2.

Question 9.
Big Ideas Math Geometry Answers Chapter 5 Congruent Triangles 205
Answer:
Big Ideas Math Answers Geometry Chapter 5 Congruent Triangles 5.7 a 9

Question 10.
Big Ideas Math Geometry Answers Chapter 5 Congruent Triangles 206
Answer:

Question 11.
Big Ideas Math Geometry Answers Chapter 5 Congruent Triangles 207
Answer:
Big Ideas Math Answers Geometry Chapter 5 Congruent Triangles 5.7 a 11

Question 12.
Big Ideas Math Geometry Answers Chapter 5 Congruent Triangles 208
Answer:

In Exercises 13 and 14. write a proof to verify that the construction is valid.

Question 13.
Line perpendicular to a line through a point not on the line
Big Ideas Math Geometry Answers Chapter 5 Congruent Triangles 209
Plan for proof ∆APQ ≅ ∆BPQ by the congruence Theorem (Theorem 5.8). Then show the ∆APM ≅ ∆BPM using the SAS Congruence Theorem (Theorem 5.5). Use corresponding parts of congruent triangles to show that ∠AMP and ∠BMP are right angles.
Answer:
Big Ideas Math Answers Geometry Chapter 5 Congruent Triangles 5.7 a 13.1
Big Ideas Math Answers Geometry Chapter 5 Congruent Triangles 5.7 a 13.2

Question 14.
Line perpendicular to a line through a p0int on the line
Big Ideas Math Geometry Answers Chapter 5 Congruent Triangles 210
Plan for Proof Show that ∆APQ ≅ ∆BPQ by the SSS Congruence Theorem (Theorem 5.8) Use corresponding parts of congruent triangles to show that ∠QPA and ∠QPB are right angles.
Answer:

In Exercises 15 and 16, use the information given in the diagram to write a proof.

Question 15.
Prove \(\overline{F L}\) ≅ \(\overline{H N}\)
Big Ideas Math Geometry Answers Chapter 5 Congruent Triangles 211
Answer:
Big Ideas Math Answers Geometry Chapter 5 Congruent Triangles 5.7 a 15.1
Big Ideas Math Answers Geometry Chapter 5 Congruent Triangles 5.7 a 15.2

Question 16.
Prove ∆PUX ≅ ∆QSY
Big Ideas Math Geometry Answers Chapter 5 Congruent Triangles 212
Answer:

Question 17.
MODELING WITH MATHEMATICS
Explain how to find the distance across the canyon.
Big Ideas Math Geometry Answers Chapter 5 Congruent Triangles 213
Answer:
Big Ideas Math Answers Geometry Chapter 5 Congruent Triangles 5.7 a 17

Question 18.
HOW DO YOU SEE IT?
Use the tangram puzzle.
Answer:

Big Ideas Math Geometry Answers Chapter 5 Congruent Triangles 214

a. Which triangle(s) have an area that is twice the area of the purple triangle?
b. How man times greater is the area of the orange triangle than the area of the purple triangle?
Answer:

Question 19.
PROOF
Prove that the green triangles in the Jamaican flag congruent if \(\overline{A D}\) || \(\overline{B C}\) and E is the midpoint of \(\overline{A C}\).
Big Ideas Math Geometry Answers Chapter 5 Congruent Triangles 215
Answer:
Big Ideas Math Answers Geometry Chapter 5 Congruent Triangles 5.7 a 19.1
Big Ideas Math Answers Geometry Chapter 5 Congruent Triangles 5.7 a 19.2

Question 20.
THOUGHT PROVOKING
The Bermuda Triangle is a region in the Atlantic Ocean in which many ships and planes have mysteriously disappeared. The vertices are Miami. San Juan. and Bermuda. Use the Internet or some other resource to find the side lengths. the perimeter, and the area of this triangle (in miles). Then create a congruent triangle on land using cities as vertices.
Big Ideas Math Geometry Answers Chapter 5 Congruent Triangles 216
Answer:

Question 21.
MAKING AN ARGUMENT
Your friend claims that ∆WZY can be proven congruent to ∆YXW using the HL Congruence Theorem (Thm. 5.9). Is your friend correct? Explain your reasoning.
Big Ideas Math Geometry Answers Chapter 5 Congruent Triangles 217
Answer:
Big Ideas Math Answers Geometry Chapter 5 Congruent Triangles 5.7 a 21

Question 22.
CRITICAL THINKING
Determine whether each conditional statement is true or false. If the statement is false, rewrite it as a true statement using the converse, inverse, or contrapositive.
a. If two triangles have the same perimeter, then they are congruent.
b. If two triangles are congruent. then they have the same area.
Answer:

Question 23.
ATTENDING TO PRECISION
Which triangles are congruent to ∆ABC? Select all that apply.
Big Ideas Math Geometry Answers Chapter 5 Congruent Triangles 218
Answer:
Big Ideas Math Answers Geometry Chapter 5 Congruent Triangles 5.7 a 23

Maintaining Mathematical Proficiency

Find the perimeter of the polygon with the given vertices.

Question 24.
A(- 1, 1), B(4, 1), C(4, – 2), D(- 1, – 2)
Answer:

Question 25.
J(- 5, 3), K(- 2, 1), L(3, 4)
Answer:
Big Ideas Math Answers Geometry Chapter 5 Congruent Triangles 5.7 a 25

5.8 Coordinate Proofs

Exploration 1

Writing a coordinate Proof

Big Ideas Math Geometry Answer Key Chapter 5 Congruent Triangles 218

Work with a partner.

a. Use dynamic geometry software to draw \(\overline{A B}\) with endpoints A(0, 0) and B(6, 0).
Answer:

b. Draw the vertical line x = 3.
Answer:

c. Draw ∆ABC so that C lies on the line x = 3.
Answer:

d. Use your drawing to prove that ∆ABC is an isosceles triangle.
Answer:

Exploration 2

Writing a Coordinate proof

Work with a partner.

a. Use dynamic geometry software to draw \(\overline{A B}\) with endpoints A(0, 0) and B(6, 0).

b. Draw the vertical line x = 3.

c. Plot the point C(3, 3) and draw ∆ABC. Then use your drawing to prove that ∆ABC is an isosceles right triangle.
Big Ideas Math Geometry Answer Key Chapter 5 Congruent Triangles 219

d. Change the coordinates of C so that C lies below the x-axis and ∆ABC is an isosceles right triangle.
Answer:

e. Write a coordinate proof to show that if C lies on the line x = 3 and ∆ABC is an isosceles right triangle. then C must be the point (3, 3) or the point found in part (d).
CRITIQUING THE REASONING OF OTHERS
To be proficient in math, you need to understand and use stated assumptions, definitions, and previously established results.
Answer:

Communicate Your Answer

Question 3.
How can you use a coordinate plane to write a proof?
Answer:

Question 4.
Write a coordinate proof to prove that ∆ABC with vertices A(0, 0), 8(6, 0), and C(3, 3√3) is an equilateral triangle.
Answer:

Lesson 5.8 Coordinate Proofs

Monitoring Progress

Question 1.
Show another way to place the rectangle in Example 1 part (a) that is convenient
for finding side lengths. Assign new coordinates.
Answer:

Question 2.
A square has vertices (0, 0), (m, 0), and (0, m), Find the fourth vertex.
Answer:

Question 3.
Write a plan for the proof.
Given \(\vec{G}\)J bisects ∠OGH.
Proof ∆GJO ≅ ∆GJH
Big Ideas Math Geometry Answer Key Chapter 5 Congruent Triangles 220
Answer:

Question 4.
Graph the points 0(0, 0), H(m, n), and J(m, 0). Is ∆OHJ a right triangle? Find the side lengths and the coordinates of the midpoint of each side.
Answer:

Question 5.
Write a coordinate proof.
Big Ideas Math Geometry Answer Key Chapter 5 Congruent Triangles 221
Given Coordinates of vertices of ∆NPO and ∆NMO
Prove ∆NPO ≅ ∆NMO
Answer:

Exercise 5.8 Coordinate Proofs

Vocabulary and Core Concept Check

Question 1.
VOCABULARY
How is a coordinate proof different from other types of proofs you have studied?
How is it the same?
Answer:
Big Ideas Math Geometry Answer Key Chapter 5 Congruent Triangles 5.8 a 1

Question 2.
WRITING
Explain why it is convenient to place a right triangle on the grid as shown when writing a coordinate proof.
Big Ideas Math Geometry Answer Key Chapter 5 Congruent Triangles 222
Answer:

Maintaining Progress and Modeling with Mathematics

In Exercises 3-6, place (he figure in a coordinate plane in a convenient way. Assign coordinates to each vertex. Explain the advantages of your placement.

Question 3.
a right triangle with leg lengths of 3 units and 2 units
Answer:
Big Ideas Math Geometry Answer Key Chapter 5 Congruent Triangles 5.8 a 3

Question 4.
a square with a side length of 3 units
Answer:

Question 5.
an isosceles right triangle with leg length p
Answer:
Big Ideas Math Geometry Answer Key Chapter 5 Congruent Triangles 5.8 a 5

Question 6.
a scalene triangle with one side length of 2m
Answer:

In Exercises 7 and 8, write a plan for the proof.

Question 7.
Given Coordinates of vertices of ∆OPM and ∆ONM Prove ∆OPM and ∆ONM are isosceles triangles.
Big Ideas Math Geometry Answer Key Chapter 5 Congruent Triangles 223
Answer:
Big Ideas Math Geometry Answer Key Chapter 5 Congruent Triangles 5.8 a 7

Question 8.
Given G is the midpoint of \(\overline{H F}\).
Prove ∆GHJ ≅ ∆GFO
Big Ideas Math Geometry Answer Key Chapter 5 Congruent Triangles 224
Answer:

In Exercises 9-12, place the figure in a coordinate plane and find the indicated length.

Question 9.
a right triangle with leg lengths of 7 and 9 units; Find the length of the hypotenuse.
Answer:
Big Ideas Math Geometry Answer Key Chapter 5 Congruent Triangles 5.8 a 9

Question 10.
an isosceles triangle with a base length of 60 units and a height of 50 units: Find the length of one of the legs.
Answer:

Question 11.
a rectangle with a length o! 5 units and a width of 4 units: Find the length of the diagonal.
Answer:
Big Ideas Math Geometry Answer Key Chapter 5 Congruent Triangles 5.8 a 11

Question 12.
a square with side length n: Find the length of the diagonal.
Answer:

In Exercises 13 and 14, graph the triangle with the given vertices. Find the length and the slope of each side of the triangle. Then find the coordinates of the midpoint of each side. Is the triangle a right triangle? isosceles? Explain. Assume all variables are positive and in m ≠ n.)

Question 13.
A(0, 0), B(h, h), C(2h, 0)
Answer:
Big Ideas Math Geometry Answer Key Chapter 5 Congruent Triangles 5.8 a 13.1
Big Ideas Math Geometry Answer Key Chapter 5 Congruent Triangles 5.8 a 13.2

Question 14.
D(0, n), E(m, n), F(m, 0)
Answer:

In Exercises 15 and 16, find the coordinates of any unlabeled vertices. Then find the indicated length(s).

Question 15.
Find ON and MN.
Big Ideas Math Geometry Answer Key Chapter 5 Congruent Triangles 225
Answer:
Big Ideas Math Geometry Answer Key Chapter 5 Congruent Triangles 5.8 a 15

Question 16.
Find OT.
Big Ideas Math Geometry Answer Key Chapter 5 Congruent Triangles 226
Answer:

PROOF
In Exercises 17 and 18, rite a coordinate proof.

Question 17.
Given Coordinates of vertices of ∆DEC and ∆BOC
Prove ∆DEC ≅ ∆BOC
Big Ideas Math Geometry Answer Key Chapter 5 Congruent Triangles 227
Answer:
Big Ideas Math Geometry Answer Key Chapter 5 Congruent Triangles 5.8 a 17

Question 18.
Given Coordinates of ∆DEA, H is the midpoint of \(\overline{D A}\), G is the mid point of \(\overline{E A}\)
Prove \(\overline{D G}\) ≅ \(\overline{E H}\)
Big Ideas Math Geometry Answer Key Chapter 5 Congruent Triangles 228
Answer:

Question 19.
MODELING WITH MATHEMATICS
You and your cousin are camping in the woods. You hike to a point that is 500 meters cast and 1200 meters north of the Campsite. Your cousin hikes to a point that is 1000 meters cast of the campsite. Use a coordinate proof to prove that the triangle formed by your Position, your Cousin’s position. and the campsite is isosceles. (See Example 5.)
Big Ideas Math Geometry Answer Key Chapter 5 Congruent Triangles 229
Answer:
Big Ideas Math Geometry Answer Key Chapter 5 Congruent Triangles 5.8 a 19.1
Big Ideas Math Geometry Answer Key Chapter 5 Congruent Triangles 5.8 a 19.2

Question 20.
MAKING AN ARGUMENT
Two friends see a drawing of quadrilateral PQRS with vertices P(0, 2), Q(3, – 4), R(1, – 5), and S(- 2, 1). One friend says the quadrilateral is a parallelogram but not a rectangle. The other friend says the quadrilateral is a rectangle. Which friend is correct? Use a coordinate proof to support your answer.
Answer:

Question 21.
MATHEMATICAL CONNECTIONS
Write an algebraic expression for the coordinates of each endpoint of a line segment whose midpoint is the origin.
Answer:
Big Ideas Math Geometry Answer Key Chapter 5 Congruent Triangles 5.8 a 21

Question 22.
REASONING
The vertices of a parallelogram are (w, 0), (o, v), (- w, 0), and (0, – v). What is the midpoint of the side in Quadrant III?
(a) \(\left(\frac{w}{2}, \frac{v}{2}\right)\)
(b) \(\left(-\frac{w}{2},-\frac{v}{2}\right)\)
(c) \(\left(-\frac{w}{2}, \frac{v}{2}\right)\)
(d) \(\left(\frac{w}{2},-\frac{v}{2}\right)\)
Answer:

Question 23.
REASONING
A rectangle with a length of 3h and a width of k has a vertex at (- h, k), Which point cannot be a vertex of the rectangle?
(A) (h, k)
(B) (- h, 0)
(c) (2h, 0)
(D) (2h, k)
Answer:
Big Ideas Math Geometry Answer Key Chapter 5 Congruent Triangles 5.8 a 23

Question 24.
THOUGHT PROVOKING
Choose one of the theorems you have encountered up to this point that you think would be easier to prove with a coordinate proof than with another type of proof. Explain your reasoning. Then write a coordinate proof.
Answer:

Question 25.
CRITICAL THINKING
The coordinates of a triangle are (5d – 5d), (0, – 5d), and (5d, 0). How sh
would the coordinates be changed to make a coordinate proof easier to complete?
Answer:
Big Ideas Math Geometry Answer Key Chapter 5 Congruent Triangles 5.8 a 25

Question 26.
HOW DO YOU SEE IT?
without performing any calculations, how do you know that the diagonals of square TUVW are perpendicular to each oilier? How can you use a similar diagram to show that the diagonals of any square are perpendicular to each other?
Big Ideas Math Geometry Answer Key Chapter 5 Congruent Triangles 230
Answer:

Question 27.
PROOF
Write a coordinate proof for each statement.
a. The midpoint o! the hypotenuse of a right triangle is the same distance from each vertex of the triangle.
b. Any two congruent right isosceles triangles can be combined to form a single isosceles triangle.
Answer:
Big Ideas Math Geometry Answer Key Chapter 5 Congruent Triangles 5.8 a 27.1
Big Ideas Math Geometry Answer Key Chapter 5 Congruent Triangles 5.8 a 27.2

Maintaining Mathematical proficiency

\(\vec{Y}\)W bisects ∠XYZ such that m∠XYW = (3x – 7)° and m∠WYZ = (2x + 1)°.

Question 28.
Find the value of x.
Answer:

Question 29.
Find m∠XYZ
Answer:
Big Ideas Math Geometry Answer Key Chapter 5 Congruent Triangles 5.8 a 29

Congruent Triangles Chapter Review

5.1 Angles of Triangles

Question 1.
Classify the triangle at the right by its sides and by measuring its angles.
Big Ideas Math Geometry Solutions Chapter 5 Congruent Triangles 231
Answer:

Find the measure of the exterior angle.

Question 2.
Big Ideas Math Geometry Solutions Chapter 5 Congruent Triangles 232
Answer:

Question 3.
Big Ideas Math Geometry Solutions Chapter 5 Congruent Triangles 234
Answer:

Find the measure of each acute angle.

Question 4.
Big Ideas Math Geometry Solutions Chapter 5 Congruent Triangles 235
Answer:

Question 5.
Big Ideas Math Geometry Solutions Chapter 5 Congruent Triangles 236
Answer:

5.2 Congruent Polygons

Question 6.
In the diagram. GHJK ≅ LMNP. Identify all pairs of congruent corresponding parts. Then write another congruence statement for the quadrilaterals.
Big Ideas Math Geometry Solutions Chapter 5 Congruent Triangles 237
Answer:

Question 7.
Find m ∠ V.
Big Ideas Math Geometry Solutions Chapter 5 Congruent Triangles 238
Answer:

5.3 Proving Triangle Congruence by SAS

Decide whether enough information is given to prove that ∆WXZ ≅ ∆YZX using the SAS Congruence Theorem (Theorem 5.5). If so, write a proof. If not, explain why.

Question 8.
Big Ideas Math Geometry Solutions Chapter 5 Congruent Triangles 239
Answer:

Question 9.
Big Ideas Math Geometry Solutions Chapter 5 Congruent Triangles 240
Answer:

5.4 Equilateral and Isosceles Triangles

Copy and Complete the statement.

Big Ideas Math Geometry Solutions Chapter 5 Congruent Triangles 241

Question 10.
If \(\overline{Q P}\) ≅ \(\overline{Q R}\), then ∠ ______ ≅ ∠ ______ .
Answer:

Question 11.
If ∠TRV ≅ ∠TVR, then ______ ≅ ______ .
Answer:

Question 12.
If \(\overline{R Q} \cong \overline{R S}\), then ∠ ______ ≅ ∠ ______ .
Answer:

Question 13.
If ∠SRV ≅ ∠SVR, then ______ ≅ ______ .
Answer:

Question 14.
Find the values of x and y in the diagram.
Big Ideas Math Geometry Solutions Chapter 5 Congruent Triangles 242
Answer:

5.5 Proving Triangle Congruence by SSS

Question 15.
Decide whether enough information is given to prose that ∆LMP ≅ ∆NPM using the SSS Congruence Theorem (Thin. 5.8). If so, write a proof. If not, explain why.
Big Ideas Math Geometry Solutions Chapter 5 Congruent Triangles 243
Answer:

Question 16.
Decide whether enough information is given to prove that ∆WXZ ≅ ∆YZX using the HL Congruence Theorem (Thm. 5.9). If so, write a proof. If not, explain why.
Big Ideas Math Geometry Solutions Chapter 5 Congruent Triangles 244
Answer:

5.6 Proving Triangle Congruence by ASA and AAS

Question 17.
∆EFG, ∆HJK
Big Ideas Math Geometry Solutions Chapter 5 Congruent Triangles 245
Answer:

Question 18.
∆TUS, ∆QRS
Big Ideas Math Geometry Solutions Chapter 5 Congruent Triangles 246
Answer:

Decide whether enough information is given to prove that the triangles are congruent using the ASA Congruence Theorem (Thm. 5.10). If so, write a proof, If not, explain why.

Question 19.
∆LPN, ∆LMN
Big Ideas Math Geometry Solutions Chapter 5 Congruent Triangles 247
Answer:

Question 20.
∆WXZ, ∆YZX
Big Ideas Math Geometry Solutions Chapter 5 Congruent Triangles 248
Answer:

5.7 Using Congruent Triangles

Question 21.
Explain how to prove that ∠K ≅∠N.

Answer:

Question 22.
Write a plan to prkove that ∠1 ≅ ∠2

Answer:

5.8 Coordinate Proofs

Question 23.
Write a coordinate proof.
Given Coordinates of vertices of quadrilateral OPQR
Prove ∆OPQ ≅ ∆QRO
Big Ideas Math Geometry Solutions Chapter 5 Congruent Triangles 251
Answer:

Question 24.
Place an isosceles triangle in a coordinate plane in a way that is convenient for finding side lengths. Assign coordinates to each vertex.
Answer:

Question 25.
A rectangle has vertices (0, 0), (2k, 0), and (0, k), Find the fourth vertex.
Answer:

Congruent Triangles Test

Write a Proof.

Question 1.
Given \(\overline{C A} \cong \overline{C B} \cong \overline{C D} \cong \overline{C E}\)
Prove ∆ABC ≅ ∆EDC
Big Ideas Math Geometry Solutions Chapter 5 Congruent Triangles 252
Answer:

Question 2.
Given \(\overline{J K}\|\overline{M L}, \overline{M J}\| \overline{K L}\)
Prove ∆MJK ≅ ∆KLM
Big Ideas Math Geometry Solutions Chapter 5 Congruent Triangles 253
Answer:

Question 3.
Gven \overline{Q R} \cong \overrightarrow{R S}\(\), ∠P ≅ ∠T
Prove ∆SRP ≅ ∆QRT
Big Ideas Math Geometry Solutions Chapter 5 Congruent Triangles 254
Answer:

Question 4.
Find the measure of each acute angle in the figure at the right.
Big Ideas Math Geometry Solutions Chapter 5 Congruent Triangles 255
Answer:

Question 5.
Is it possible to draw an equilateral triangle that is not equiangular? If so, provide an example. If not, explain why.
Answer:

Question 6.
Can you use the Third Angles Theorem (Theorem 5.4) to prove that two triangles are congruent? Explain your reasoning.
Answer:

Write a plan through that ∠1 ≅∠2

Question 7.
Big Ideas Math Geometry Solutions Chapter 5 Congruent Triangles 256
Answer:

Question 8.
Big Ideas Math Geometry Solutions Chapter 5 Congruent Triangles 257
Answer:

Question 9.
Is there more than one theorem that could be used to prove that ∆ABD ≅ ∆CDB? If so, list all possible theorems.
Big Ideas Math Geometry Solutions Chapter 5 Congruent Triangles 258
Answer:

Question 10.
Write a coordinate proof t0 show that the triangles created b the keyboard stand are congruent.
Big Ideas Math Geometry Solutions Chapter 5 Congruent Triangles 259
Answer:

Question 11.
The picture shows the Pyramid of Cestius. which is located in Rome, Italy. The measure of the base for the triangle shown is 100 Roman feet. The measures of the other two sides of the triangle are both 144 Roman feet.
Big Ideas Math Geometry Solutions Chapter 5 Congruent Triangles 260
a. Classify the triangle shown by its sides.
Answer:

b. The measure of ∠3 is 40° What are the measures of ∠1 and ∠2? Explain your reasoning.
Answer:

Congruent Triangles Cumulative Assessment

Question 1.
Your friend claims that the Exterior Angle Theorem (Theorem 5.2) can be used to prove the Triangle Sum Theorem (Theorem 5, 1). Is your friend correct? Explain your reasoning.
Answer:

Question 2.
Use the steps in the construction to explain how you know that the line through point P is parallel to line m.
Big Ideas Math Answer Key Geometry Chapter 5 Congruent Triangles 261
Answer:

Question 3.
The coordinate plane shows ∆JKL and ∆XYZ
Big Ideas Math Answer Key Geometry Chapter 5 Congruent Triangles 262
a. Write a composition of transformations that maps ∆JKL to ∆XYZ
Answer:

b. Is the composition a congruence transformation? If so, identify all congruent corresponding parts.
Answer:

Question 4.
The directed line segment RS is shown. Point Q is located along \(\overline{R S}\) so that the ratio of RQ to QS is 2 to 3. What are the coordinates of point Q?
Big Ideas Math Answer Key Geometry Chapter 5 Congruent Triangles 263
(A) Q(1, 2, 3)
(B) Q(4, 2)
(C) Q(2, 3)
(D) Q(-6, 7)
Answer:

Question 5.
The coordinate plane shows that ∆ABC and ∆DEF
Big Ideas Math Answer Key Geometry Chapter 5 Congruent Triangles 264
a. Prove ∆ABC ≅ ∆DEF using the given information.
Answer:

b. Describe the composition of rigid motions that maps ∆ABC to ∆DEF
Answer:

Question 6.
The vertices of a quadrilateral are W(0, 0), X(- 1, 3), )(2, 7), and Z(4, 2). Your friend claims that point W will not change after dilatinig quadrilateral WXYZ by a scale factor of 2. Is your friend correct? Explain your reasoning.
Answer:

Question 7.
Which figure(s) have rotational symmetry? Select all that apply.
(A) Big Ideas Math Answer Key Geometry Chapter 5 Congruent Triangles 265
(B) Big Ideas Math Answer Key Geometry Chapter 5 Congruent Triangles 266
(C) Big Ideas Math Answer Key Geometry Chapter 5 Congruent Triangles 267
(D) Big Ideas Math Answer Key Geometry Chapter 5 Congruent Triangles 268
Answer:

Question 8.
Write a coordinate proof.
Given Coordinates of vertices of quadrilateral ABCD
Prove Quadrilateral ABCD is a rectangle.
Big Ideas Math Answer Key Geometry Chapter 5 Congruent Triangles 269
Answer:

Question 9.
Write a proof to verify that the construction of the equilateral triangle shown below is valid.
Big Ideas Math Answer Key Geometry Chapter 5 Congruent Triangles 270
Answer:

Big Ideas Math Algebra 2 Answers PDF Download | Free BIM Algebra 2 Textbook Solution Key

Big Ideas Math Algebra 2 Answers

If you get stuck during your exam preparation then here is the savior to master in Algebra 2 math concepts ie., Big Ideas Math Algebra 2 Answers Pdf for all chapters. This page contains chapterwise BigIdeasMath Answers for Algebra 2 in pdf format for easy access. All chapters Algebra 2 BIM Solution Key includes multiple preparation resources like practice exercises, chapter tests, performance tests, etc. for better & accurate preparation. Make use of the below available Common Core 2019 Curriculum Big Ideas Math Answers Algebra 2 Pdf and improve your subject & problem-solving skills. Go through this article and kickstart your preparation.

Big Ideas Math Answer Key for Algebra 2 Chapters | Free BIM Algebra 2 Solution Key Pdf

Are you excited to access the chapter-wise BigIdeasMath Algebra 2 Answers Pdf from here & practice frequently? Then, here are the quick links to move to the respective chapter of Big Ideas math solutions for Algebra 2 concepts and solve all the questions easily & accurately. After practicing the algebra 2 math topics from the Big ideas math book solution key pdf, you can get a good grip on the subject from the basic level to the advanced level. Below are the links to BIM Algebra 2 Answers Pdf that you are looking for. Simply tap on the required chapter link and start your learnings.

Benefits of Common Core 2019 High School BIM Algebra 2 Solutions

This section may realize how important to practice the questions involved in the big ideas math book algebra 2 answers. So, don’t skip these points while referring to the BIM Answers for Algebra 2 chapters. Below are some of the advantages of BigIideasMath Textbook Answer Key Algebra 2.

  • If you are seeking any homework help then this Algebra 2 BIM Answer Key can be the best solution for students.
  • Practice the problems available on our BIM textbook solution key and crack various exams faced during your education life.
  • Students can discover chapterwise big ideas math book answers for algebra 2 from this article for free of cost.
  • Ace up your preparation by using the experts prepared common core 2019 edition Big Ideas Math Algebra 2 Textbook Answers Pdf.
  • Step by step explanation is given for all questions covered in this algebra 2 BIM textbook solution key. Also, you can practice more questions by referring to the chapter tests, assessment tests, cumulative tests, etc.

FAQs on Big Ideas Math Algebra 2 Textbook Answers

1. Where can I find the free Algebra 2 Big Ideas Math Book Answer Key?

On the internet world, there are various official portals that help students to learn and prepare math concepts thoroughly & crack the exams. Among them, bigideasmathanswers.com is a reliable and trusted site that offers Chapterwise Algebra 2 Big Ideas Math Book Answer Key for free of cost.

2. How to access Big Ideas Math Textbook Answers Algebra 2?

Just tap on the direct links available on this page and easily access the Bigideas Math Algebra 2 Answer Key online & offline.

3. How to practice Algebra 2 concepts easily? 

With the help of the Big Ideas Math Algebra 2 Answer Key, students can practice all chapters of algebra 2 and enhance their solving skills to score good marks in the exams.

Big Ideas Math Answers Grade 5 Chapter 4 Multiply Whole Numbers

Big Ideas Math Answers Grade 5 Chapter 4 Multiply Whole Numbers

Get the free pdf of Big Ideas Math Answers Grade 5 Chapter 4 Multiply Whole Numbers. Every problem is explained with the solution and explanation on Big Ideas Multiply Whole Numbers Grade 5 Chapter 4 Math Answers. Our highly trained experts are provided the concepts in a simple way where every student can understand the solving methods. Also, various methods are included to solve the math problems. Therefore, practice all problems and score good marks in the exam. Every explanation is mainly explained with images, graphs, tables, and, etc. So, students will not face any problems to solve the basic questions too difficult questions.

Big Ideas 5th Grade Chapter 4 Multiply Whole NumbersMath Book Answer Key

Save your time by finding every concept and its related problems individually. Quickly go through the below links and check out all the topics with problems. Mainly, concentrate on concepts to prepare and solve the questions on your own. If you understand the concept, it becomes easier to solve difficult problems easily. Download Big Ideas Math Book 5th Grade Answers Chapter 4 Multiply Whole Numbers pdf and start your practice now. We have also included exercises for the better practice of students.

Lesson: 1 Multiplication Patterns

Lesson: 2 Estimate Products

Lesson: 3 Multiply by One-Digit Numbers

Lesson: 4 Multiply by Two-Digit Numbers

Lesson: 5 Multiply by Multi-Digit Whole Numbers

Performance Task

Lesson 4.1 Multiplication Patterns

Explore and Grow

Complete the table. Find the value of each expression.
Big Ideas Math Answer Key Grade 5 Chapter 4 Multiply Whole Numbers 4.1 1
What patterns do you notice?
Answer:

Power Expression Value
8 x 101 8 x 10 80
8 x 102 8 x 100 800
8 x 103 8 x 1,000 8,000
8 x 104 8 x 10,000 80,000

Explanation:

In each product, the number of zeros after 8 is the same as the exponent.

Patterns
Explain how to multiply any number by a power of 10.
Answer:
By multiplying the given number with the 10 as many times as to its the given power value.
Explanation :
Example  4 x 102  we take as  4 x 10 x 10 means 10 is multiplied twice
means as the power of 10 is 2 so we  multiply the number 4 with 10 twice,
if it 4 x 103 we take it as 4 x 10 x 10 x 10 means we take 10 by 3 times as power is 3

Think and Grow: Use Patterns to Find Products

Example
Find 46 × 103.
One Way:
Use powers and look for a pattern
Big Ideas Math Answer Key Grade 5 Chapter 4 Multiply Whole Numbers 4.1 2

46,000

Another Way:
Use place value.
46 × 103 = 46 × 1,000
= 46 thousands
= ___46,000___
Big Ideas Math Answer Key Grade 5 Chapter 4 Multiply Whole Numbers 4.1 3
So, 46 × 103.
Notice the pattern: In each product, the number of zeros after 46 is the same as the exponent.

Example
Find 40 × 600.
Use a multiplication fact and look for a pattern.
Big Ideas Math Answer Key Grade 5 Chapter 4 Multiply Whole Numbers 4.1 4
So, 40 × 600 = __24,000____.
Notice the pattern: In each product, the number of zeros after 24 is equal to the number of zeros in the factors.

Show and Grow

Find the product.
Question 1.
78 × 104
Answer:
The product of 78 × 10= 78 x 10,000 = 780,000
Explanation: As 78 is multiplied by 104  we get the result as 780,000

Question 2.
20 × 400 = ______
Answer:
The product of 20 x 400 = 2x 10 x 4 x 100 = 2 x 4 x 10 x 100 = 8,000
Explanation:
As 20 X 400 has 3 zeros we have 20 x 400 as 8,000

Apply and Grow: Practice

Find the product
Question 3.
11 × 103 = _______
Answer:
The product of 11 × 103 = 11 x 1,000=11,000
Explanation :
The result of  11 × 103  is 11,000

Question 4.
40 × 70 = _______
Answer:
The product of 40 x 70 = 4 x 10 x 7 x 10 =4 x 7 x 10 x 10 = 2,800
Explanation :
When 40 is multiplied by  70 the result is 2,800

Question 5.
300 × 90 = ________
Answer:
The product of 300 x 90 = 3 x 100 x 9 x 10 = 3 x 9 x 100 x 10 = 27,000
Explanation :
When 300 is multiplied by  90 the result is 27,000

Question 6.
104 × 60 = _______
Answer:
The product of 104 × 60 = 10 x 10 x 10 x 10 x 60 = 6,00,000
Explanation :
When 104  is multiplied by 60 the result is 6,00,000

Question 7.
206 × 102 = _______
Answer:
The product of 206 × 102 = 206 x 10 x 10 = 20,600
Explanation :
When 206 is multiplied by 102 the result is 20,600

Question 8.
15 × 200 = _________
Answer:
The product of 15 x 200 = 15 x 2 x 10 x 10 = 3,000
Explanation :
When 15 is multiplied by 200 the result is 3,000

Find the missing factor.
Question 9.
30 × ______ = 15,000
Answer:
30 x ____= 15,000,15,000/30 = 500
Explanation:
So when 15,000 is divided by 30 we get 500 the missing factor is 500

Question 10.
_____ × 103 = 54,000
Answer:
_____ × 103 = 54,000
the missing factor is 54
Explanation :
When 54,000 is divided by 10or 1000
we get 54,000 x 1/ 1000= 54

Question 11.
500 × ______ =40,000
Answer:
500 × ______ =40,000
the missing factor is 8,000
Explanation:
so when 40,000 is dived by 500 its is equal to
40,000 x 1/500=8,000

Question 12.
A heart pumps about 2,000 gallons of blood each day. How many gallons of blood does the heart pump in 7 days?
Answer:
Given that heart pumps about 2,000 gallons of blood each day and for 7 days it is
2,000 x 7 = 14,000 gallons
Explanation :
So we multiply 2000 by 7 for 7 days and the result is 14,000 gallons

Question 13.
Number Sense
Which expressions are equal to 24,000?
Big Ideas Math Answer Key Grade 5 Chapter 4 Multiply Whole Numbers 4.1 5
Answer:
240 x 100= 24 x 10 x 10 x 10  = 24,000 so it is equal
30 x 800 = 3 x 10 x 8 x 10 x 10 = 24,000 so this is also equal
400 x 600= 4 x 10 x 10 x 6 x 10 x 10 =24,000 so this also equal
and 24 x 1,000 = 24 x 10 x 10 x 10 = 24,000 which is also equal
so all the given expressions are equal to 24,000
Explanation :
When Big Ideas Math Answer Key Grade 5 Chapter 4 Multiply Whole Numbers 4.1 5

these numbers are multiplied all results are equal to 24,000

Question 14.
Logic
When you multiply any whole number by 100, what do you know about the digits in the product?
Answer:
When we multiply any whole number by 100, it becomes the digit with two zeros in the right , number x 100= number x 102 =number,00
or we get the whole number with additional 2 zeros in the right
Explanation:
For example if the number is 5 it becomes as 5 x 100 = 5 x 102= 500, 2 zeros after 5

Question 15.
DIG DEEPER!
Show how to find 20 × 103 using the Associative Property of Multiplication.
Answer:
The Associative Property of Multiplication of 20 × 103  is
(20 x 10) x 10 x 10 = 20 x ( 10 x 10 ) x 10
(200) x 10 x 10 = 20 x (100) x 10
20,000 = 20,000
Explanation:
Both sides are equal by using the Associative Property of Multiplication Law.

Think and Grow: Modeling Real Life

Example
A black bear consumes 12,000 calories each day for 30 days. A grizzly bear consumes 400,000 calories in 30 days. Which bear consumes more calories in 30 days?
Multiply 12,000 and 30 to find the number of calories the black bear consumes in 30 days. Use a multiplication fact and a pattern.
12 × 3 = ____36____ Multiplication fact
The multiplication fact is 12 × 3 = 36. The factors 12,000 and 30 have a total of _____4_ zeros. So, the product has __4___ zeros after 36.
12,000 × 30 = ___3,60,000______ calories
The black bear consumes 3,60,000 calories in 30 days. Compare the numbers of calories the bears consume in 30 days.
A grizzly bear consumes 4,00,000 x 30 = 120,00,000
So  grizzly bear consumes more calories than black bear
The ___grizzly ___ bear consumes more calories in 30 days.

Show and Grow

Question 16.
The attendance at a theme park is about 300,000 guests each month. The attendance at a water park is 2,280,000 guests in 1 year. Which park has a greater attendance in 1 year?
Answer:
The theme park has a greater attendance in 1 year.
Explanation:
The attendance at theme park is about 3,00,000 guests in one month so in a year it is 3,00,000 x 12 = 36,00,000 guests.
and given water park has 22,80,000 guests= 228 x 104  in 1 year
so as compared between the two parks 36,00,000 guests = 360 x104  is greater than 22,80,000 guests= 228 x 10 means the theme park has a greater attendance in 1 year.

Question 17.
Your heart beats 120,000 times each day. In 1 week, a professional athlete’s heart beats 607,000 times. How many more times does your heart beat in 1 week compared to the athlete?
Answer:
Our heart beats 233,000 times more compared to the athlete in 1 week
Explanation :
Each day our heart beats 120,000 times each day
so in 1 week it is 120,000 x 7 = 840,000 times our heart beats  in 1 week given  athlete’s heart beats in 1 week is 607,000 times so compared to both it is
840,000 – 607,000 = 233,000 times
So our heart beats 233,000 times more compared to the athlete in 1 week

Question 18.
A whale’s brain weighs 103 times as much as a platypus’s brain. The whale’s brain weighs 9,000 grams. How many grams does the platypus’s brain weigh?
Big Ideas Math Answer Key Grade 5 Chapter 4 Multiply Whole Numbers 4.1 6
Answer:
platypus’s brain weighs 9 grams
Explanation:
Given whale’ brain weighs 103 times  as much as a platypus’s brain and also given that whale’s brain weighs 9,000 grams
so platypus’s brain weighs 9,000 x 1/ 103 times = 9,000x 1/1000 = 9 grams.

Multiplication Patterns Homework & Practice 4.1

Find the product
Question 1.
37 × 104
Answer:
37 × 10= 370,000
Explanation:
The product of 37 × 10= 37 x 10 x 10 x 10 x 10= 370,000

Question 2.
400 × 90 = ______
Answer:
400 × 90 = 36,000
Explanation:
The product of 400 × 90 = 4 x 10 x 10 x 9 x 10 = 4 x 9 x 103 = 36,000

Question 3.
100 × 900 = _____

Answer:
100 x 900 = 90,000
Explanation:
The product of 100 x 900 = 10 x 10 x 9 x 10 x 10 = 9 x 10= 90,000

Question 4.
800 × 700 = _____
Answer:
800 x 700 = 560,000
Explanation:
The product of 800 x 700 = 8 x 10 x 10 x 7 x 10 x 10 = 8 x 7 x 10=560,000

Question 5.
867 × 103 = ______
Answer:
867 × 103 =867,000
Explanation:
The product of 867 × 103 = 867 x 10 x 10 x 10 = 867,000

Question 6.
12 × 6,000 = _______
Answer:
12 x 6,000 = 72,000
Explanation:
The product of 12 x 6,000 = 12 x 6 x 10 x 10 x 10= 12 x 6 x 103 = 72,000

Find the missing factor.
Question 7.
____ × 102 = 3,000
Answer:
30 x 102 = 3,000
Explanation:
The missing factor of  ____ × 102 = 3,000 is 3,000 x 1/100= 3 x 103 / 10=30

Question 8.
______ × 500 = 450,000
Answer:
900 x 500 = 450,000
Explanation:
The missing factor of ______ × 500 = 450,000 is 450,000 x 1/500= 450,000/500=
45 x 104  / 5 x 10= 9 x 10= 900

Question 9.
80 × _____ = 640,000
Answer:
80 x 8000 = 640,000
Explanation:
The missing factor of 80 × _____ = 640,000 is 640,000 x 1/80 = 64 x 104  / 8 x 10=
8 x 10=8,000

Question 10.
Reasoning
Do 50 × 8,000 and 50 × 9,000 have the same number of zeros in their products? Explain.
Answer:
Yes, 50 × 8,000 and 50 × 9,000 have the same number of zeros in their products
Explanation:
Why because 50 x 8,000 = 5 x 10 x 8 x 10 x 10 x 10 = 5 x 8 x 104
and 50 x 9,000 = 5 x 10 x 9 x 10 x 10 x 10 = 5 x 9 x 104
so when compared between 5 x 8 x 104  and 5 x 9 x 10we have 4 zeros in both the products.

Question 11.
DIG DEEPER!
The product of a number and twice that number is 180,000. Write the multiplication equation.
Answer:
2 n2 = 18 x 104
Explanation:
let the number be n and its twice is 2 n is 180,000
n x 2n = 18 x 10 x 10 x 10 x 10
2 n2 = 18 x (10 x 10) x (10 x 10)
so the multiplication equation is 2 n2 = 18 x 10

Question 12.
Modeling Real Life
A hummingbird flutters its wings 80 times in 1 second. A bumblebee flutters its wings 7,800 times in 1 minute. Which animal flutters its wings more times in 1 minute?
Big Ideas Math Answer Key Grade 5 Chapter 4 Multiply Whole Numbers 4.1 7
Answer:
bumblebee flutters more

Explanation:
Given humming bird flutters its wings 80 times in 1 second equals to 8 x 10,
bumblebee flutters its wings 7,800 times in 1 second equals to 78 x 102
as 78 x 10> 8 x 10 means bumblebee flutters its wings more than humming bird in 1 minute.

Question 13.
DIG DEEPER!
In 1 day, the blood in your body travels a total of 12,000 miles. This is about 4 times the distance across the United States from coast to coast. About how many miles is the distance across the United States?
Answer:
The distance across the United States is 3,000 miles.
Explanation:
Given in 1 day the blood in our body travels a total of 12,000 miles which is equal to 4 times the distance across the United States from coast to coast.
so the number of miles in the distance across the United States is
12,000 miles / 4=3,000 miles.

Review & Refresh

Classify the triangle by its sides.
Question 14.
Big Ideas Math Answer Key Grade 5 Chapter 4 Multiply Whole Numbers 4.1 8
Answer:
Equilateral Triangle
Explanation:
In the  given figure it is showing all the 3 sides are equal so we call  the triangle as Equilateral Triangle.

Question 15.
Big Ideas Math Answer Key Grade 5 Chapter 4 Multiply Whole Numbers 4.1 9
Answer:
Isosceles Triangle
Explanation:
In the above figure it is showing only 2 sides are equal so we call the given triangle as Isosceles Triangle.

Question 16.
Big Ideas Math Answer Key Grade 5 Chapter 4 Multiply Whole Numbers 4.1 10
Answer:
Scalene Triangle
Explanation:
In the above figure it is showing no side is equal so we call the given triangle as
Scalene Triangle.

Lesson 4.2 Estimate Products

Explore and Grow

Choose an expression to estimate each product. Write the expression. You may use an expression more than once.
Big Ideas Math Answers 5th Grade Chapter 4 Multiply Whole Numbers 4.2 1
Big Ideas Math Answers 5th Grade Chapter 4 Multiply Whole Numbers 4.2 2
Compare your answers with a partner’s. Did you choose the same expressions?
Answer:
29 x 26
30 x 25
Explanation:
so for 29 x 26 i am choosing the expression 30 x 25 as 29 is near to 30 and 26 is near to 26 i have selected the expression 30 x 25
21 x 24
20 x 25
Explanation :
for 21 x 24 i am choosing the expression 20 x 25 as 21 is near to 20 and 24 is near to 25 i have selected the expression 20 x 25
32 x 26
30 x 25
Explanation :
for 32 x 26 i am choosing the expression 30 x 25 as in the given expressions only 32 is near to 30 and 26 is near to 25 i have selected the expression 30 x 25
28 x 24
30 x 25
Explanation :
for 28 x 24 also i am choosing the expression 30 x 25 why because 28 is near to 30 and 24 is near to 25 so i am selecting the expression  30 x 25
Yes i have selected the same expression 30 x 25  for all the three products 29 x 26, 32 x 26,28 x 24 because for all the given three products the nearest expression we are getting is 30 x 25

Construct Arguments
You estimate the product of 23 and 26 using the expression 25 × 30. Without multiplying, determine whether the estimate is greater than or less than the actual product. Explain.
Answer:
Given the product 23 x 26 as 25 x 30
So the result of given estimate is greater than the actual value
Explanation:
Because as 25 is greater than 23 and 30 is greater than 26 so obviously the expression 25 x 30 has greater value than that of the product of 23 X 26

Think and Grow: Estimate Products

An overestimate is greater than the actual value, and an underestimate is less than the actual value. When you estimate a product, your answer will be an overestimate when both numbers are greater than the original factors. Your answer will be an underestimate when both numbers are less than the original factors.

Example
Use rounding to estimate 204 × 61. Explain whether your estimate is an overestimate or an underestimate.
Big Ideas Math Answers 5th Grade Chapter 4 Multiply Whole Numbers 4.2 3

12 x 1,000 = 12,000
So, 204 × 61 is about __12,444___. This is an __underestimate______ because about both rounded numbers are ___200___, __60____ the original factors.

Example
Use compatible numbers to estimate 23 × 194. Explain whether your estimate is an overestimate or an underestimate.
Choose compatible numbers. Then multiply.
Big Ideas Math Answers 5th Grade Chapter 4 Multiply Whole Numbers 4.2 4

50 x100 = 5,000
So, 23 × 194 is about __4,462____. This is an __overestimate_____ because both compatible numbers are __25____ __200____ the original factors.

Show and Grow

Estimate the product. Explain whether your estimate is an overestimate or an underestimate.
Question 1.
387 × 29
Answer:
387 x 29 =11,223
400  X 30 = 12,000 is overestimate
Explanation :
As our estimate after  rounding each factor becomes 387 as 400 nearest hundred  and 29 as 30 to nearest ten so it is 400 x 30 = 12 x 1,000=12,000 as 387 x 29 < 400 x 30 as the estimate result is greater than the given product results our estimate is an overestimate

Question 2.
52 × 913
Answer:
52 x 913 = 47,476
50 x 900 = 45,000 is underestimate

Explanation:
As our estimate after rounding each factor becomes 52 as 50 to nearest ten and 913 as 900 nearest hundred so it is 50 X 900 = 45,000
as 52 x 913 > 50 x 900 as the estimate result is less than the given product results
our estimate is an underestimate.

Apply and Grow: Practice

Estimate the product. Explain whether your estimate is an overestimate or an underestimate.
Question 3.
45 × 98
Answer:

45 x 98 = 4,410
50 x 100 = 5,000 is overestimate
Explanation:
As our estimate after rounding each factor becomes 45 as 50 to nearest ten and 98 as 100 to nearest hundred so it is 50 X 100 = 5,000
as 45 x 98 < 50 x 100 as the estimate result is greater than the given product results our estimate is an overestimate

Question 4.
21 × 404
Answer:
21 x 404 = 8,484
20 x 400 = 8,000 is underestimate

Explanation:
As our estimate after rounding each factor becomes 21 as 20 nearest ten and 404 as 400 to nearest hundred so it is 20 X 400 = 8,000
as 21 x 404 > 20 x 400 as the estimate result is less than the given product results our estimate is an underestimate.

Question 5.
394 × 285
Answer:
394 x 285 = 112,290
400 x 300 = 120,000 is overestimate
Explanation:
our estimate after rounding each factor becomes 394  as 400 to nearest hundred and 285 as 300 to nearest hundred so it is 400 X 300 = 120,000
as 394 x 285 < 400 x 300 as the estimate result is greater than the given product results our estimate is an overestimate

Question 6.
596 × 488
Answer:
596 x 488 = 290,848
600 x 500 = 300,000 is overestimate

Explanation:
As our estimate after rounding each factor becomes 596  as 600 to nearest hundred and 488 as 500 to nearest hundred so it is 600 X 500 = 300,000
as 596 x 488 < 600 x 500 as the estimate result is greater than the given product results our estimate is an overestimate

Question 7.
194 × 46
Answer:
194 x 46=8,924
200 x 50 = 10,000 overestimate

Explanation:
As our estimate after rounding each factor becomes 194  as 200 to nearest hundred and 46 as 50 to nearest ten so it is 200 X 50 = 10,000
as 194 x 46 < 600 x 500 as the estimate result is greater than the given product results our estimate is an overestimate

Question 8.
22 × 221
Answer:
22 x 221 =4,862
20 x 200 = 4,000 underestimate

Explanation :
As our estimate after rounding each factor becomes 22  as 20 to nearest ten and 221 as 200 to nearest hundred so it is 20 X 200 = 4,000
as 22 x 221 > 20 x 200 as the estimate result is less than the given product results our estimate is an underestimate.

Question 9.
Number Sense
You estimate 27 × 408 by rounding each number to the nearest ten. Your friend estimates the product using the compatible numbers 25 and 400. Without multiplying, determine which answer is an overestimate and which answer is an underestimate. Explain.
Answer:
27 x 408
30 x 410 Mine is overestimate and
27 x 408
25 x 400 Friend is underestimate
Explanation:
Given my estimate as 27 x 408 by rounding the number to the nearest ten it becomes 27 as 30 and 408 as 410 as 27 is less than 30 and 408 is also less than 410 so the product 27 x 408 is less than 30 x 410
27 x 408
30 x 410
as 27 x 408 < 30 x 410 mine is overestimate.
Now 27 x 408
as     25 x 400
my friend estimates the product using the compatible numbers 25 and 400 as 25 is less than 27 and 400 is less than 408 obviously the product results 25 X 400 <27 x 408 so my friend results are underestimate.
therefore mine is overestimate and friends is underestimate

Question 10.
Writing

Explain two different methods to estimate 49 × 305. Which do you prefer?
Answer:
I prefer 50 x 300
Explanation:
49 x 305 by rounding each number to the nearest ten it is 50 x 310 ,
49 x 305
50 x 310
49 x 305=14,945 and 50 x 310 = 15,500
49 as 50 and 305 as 310 product so 49 is less than 50 and 305 is also less than 310 so the product 49 x 305 is less than 50 x 310
49 x 305 <  50 x 310 which is overestimate
now 49 x 305
50 x 300
49 x 305 compatible numbers are 50 and 300 the results 49 x 305 = 14,945 and 50 x 300 = 15,000 means 49 x 305 < 50 x 300 which is also overestimate,
as both the results are overestimate but i prefer 50,300  compatible numbers only as these number results are very near to the given product 49 x 305 as compared to the rounding the numbers to nearest ten.
so i prefer 50 x 300
Question 11.
DIG DEEPER!
You estimate 9 × 26 using the compatible numbers 10 and 25. Is your estimate an overestimate or an underestimate? Explain.
Answer:
My estimate is underestimate.
Explanation:
My estimate is given as 9 x 26 = 234
and compatible numbers result is  10 x 25 = 250,
as 9 x26 < 10 x 25 as my estimate results is less than the compatible numbers result so my estimate is underestimate

Think and Grow: Modeling Real Life

Example
Earth travels about 1,118 miles in 1 minute. About how far does Earth travel in 1 hour?
Think: What do you know? What do you need to find? How will you solve?
Big Ideas Math Answers 5th Grade Chapter 4 Multiply Whole Numbers 4.2 5
There are 60 minutes in 1 hour, so multiply the distance by 60.
60 × 1,118 = ?
Use rounding to estimate the product.
Big Ideas Math Answers 5th Grade Chapter 4 Multiply Whole Numbers 4.2 6
So, Earth travels about _60,000_____ miles in 1 hour.

Show and Grow

Question 12.
Old Faithful erupts about 19 times in 1 day. Each eruption sprays about 6,050 gallons of water. About how many gallons of water does Old Faithful spray in 1 day?
Big Ideas Math Answers 5th Grade Chapter 4 Multiply Whole Numbers 4.2 7
Answer:
Old faithful spray in 1 day19 x 6,000= 114,000 gallons of water
Explanation :
Given Old Faithful erupts about 19 times in 1 day and each eruption sprays about 6,050 gallons of water, so Old faithful sprays about 19 x 6,050=114,950 gallons of water in 1 day.
Now using rounding to estimate the product it becomes 19 as 20 and 6,050 as 6,000
so Old faithful sprays about 19 x 6,000=114,000 gallons of water in 1 day.

Question 13.
Workers place a foam finger on every seat in a stadium for an upcoming football game. There are 29 sections in the stadium and about 245 seats in each section. About how many foam fingers are needed? Is it better to have an overestimate or an underestimate? Explain.
Big Ideas Math Answers 5th Grade Chapter 4 Multiply Whole Numbers 4.2 8
Answer:
Needed are 7,105 number of foam fingers and It is better to have overestimate
Explanation:
Given there are total 29 sections in the stadium
and 245 seats in each section.
The number of foam fingers needed would be 29 x 245 = 7,105 foam fingers
So we need 7,105 number of foam fingers
So now by using rounding 29 becomes 30 and 245 as 250 so it becomes
30 x 250=7,500 foam fingers.
Now on comparing 29 x 245 < 30 x 350 so the results is overestimate.
So it is better to have overestimate only

Question 14.
DIG DEEPER!
One acre of land is equal to the area of a rectangular piece of land that is 22 yards wide and 220 yards long. Central Park in New York City is 843 acres. What is the approximate area of Central Park in square yards?
Answer:
Central Park in New York City has 22 x 220 x 843 square yards
Explanation:
Given one acre of land is equal to 22 yards wide and 220 yards long
means one acre is 22 x 220 square yards
So Central Park in New York City has 22 x 220 x 843 square yards.

Estimate Products Homework & Practice 4.2

Use rounding to estimate the product. Explain whether your estimate is an overestimate or an underestimate.
Question 1.
85 × 96
Answer:
After rounding 90x 100= 9,000, overestimate
Explanation:
85 x 96=8,160  after rounding it becomes 90 x 100 =9,000
as 85 is less than 90 and 96 is less than 100 so it is overestimate

Question 2.
41 × 21
Answer:
After rounding 40 x 20=800, underestimate
Explanation:
41 x 21 = 861 after rounding it becomes 40 x 20 = 800
as 41 is greater than 40 and 21 is also greater than 20 so it becomes underestimate

Use compatible numbers to estimate the product. Explain whether your estimate is an overestimate or an underestimate.
Question 3.
56 × 106
Answer:
56 x 106 = 5,936 and 55 x 100 = 5,500
as 5,936 > 5,500 ,So my estimate is overestimate
Explanation:
For 56 x 106 compatible numbers are 55 and 100 and its product is 5,500,
56 is greater than 55 and 106 is greater than 100,so the product of given numbers is greater than the product of compatible numbers, so my estimate is overestimate

Question 4.
23 × 597
Answer:
23 x 597 = 13,731 and 25 x 600 = 15,000
as 13,731 < 15,000, So my estimate is underestimate
Explanation:
For 23 x 597 compatible numbers are 25 and 600
56 is greater than 55 and 106 is greater than 100,so the product of given numbers is lesser than the products of compatible numbers, so my estimate is underestimate

Estimate the product
Question 5.
203 × 85
Answer:
203 x 85 = 17,255 and 200 x 90 = 18,000
as 17,255 < 18,000 So my estimate is underestimate
Explanation:
For 203 x 85 compatible numbers are 200 and 90
203 is greater than 200 and 85 is lesser than 90,But the product of given numbers is lesser than the product of compatible numbers so my estimate is underestimate
Question 6.
67 × 405
Answer:
67 x 405 = 27,135 and 70 x 400 = 28,000
as 27,135 < 28,000 So my estimate is underestimate
Explanation:
For 67 x 405 compatible numbers are 70 and 400
As 67 is less than 70 and 405 is greater than 400,but the product of given numbers is lesser than the product of compatible numbers, so my estimate is underestimate

Question 7.
Number Sense
Estimate 426 × 2,045. Is your answer an overestimate or an underestimate? Explain.
Answer:
426 x 2,045=871,170 and 430 x 2,000= 860,000
as 871,170 > 860,000 so mine is overestimate
Explanation:
For 426 x 2,045 compatible numbers are 430 and 2000
As 426 is less than  and 430 and 2,045 is greater than 2,000 but the product of given numbers is greater than the product of compatible numbers, so my estimate is overestimate
Question 8.
Open-Ended
Write 2 three-digit numbers. Then estimate their product by rounding so that the answer is an underestimate.
Answer:
188 x 478 = 89,864
200 x 500 = 100,000
as 89,864 < 100,000 , so 188 x 478 is underestimate
Explanation:
To make the product as an underestimate let us take the 2 three-digit numbers as 188 and 478 and their compatible numbers 200 and 500,
As 188 is lesser than 200 and 478 is also less than 500 obviously the product becomes lesser than the product of compatible numbers and their product becomes underestimate.
Question 9.
Reasoning
Without multiplying, determine which estimate is closer to the actual product. Explain.
Big Ideas Math Answers 5th Grade Chapter 4 Multiply Whole Numbers 4.2 9
Answer:
The closer product to 21 x 372  is 20 x 400
Explanation :
If we compare 21 x 372 with 20 x 400 we see 20 is less than 21 which is very close and 372 is near to 400 and the result is much closer than when compared to the result of product of 25 x 400 as 25 is greater than to 21 and even 400 is greater than 372, so obviously the product is much far, So the closer product is 20 x 400

Question 10.
Modeling Real Life
Your cousin sells jars of jam at a farmers’ market. He sells 48 jars in 1 day for $12 each. About how much money does he earn in sales in 1 day?
Answer:
48 x $12 = $ 576 in 1 day
Explanation:
Given that cousin sells 48 jars in 1 day, each jar cost $12 , so the total cost per 1 day is product of number of jars X cost of 1 jar =48 x $12 = $ 576 in 1 day
So my cousin sells $ 576 in 1 day
Question 11.
DIG DEEPER!
SupersaurusA weighed 45 tons. About how many more pounds did an Argentinosaurus weigh than a Supersaurus?
Big Ideas Math Answers 5th Grade Chapter 4 Multiply Whole Numbers 4.2 10
Answer:
Argentinosaurus weighs 72,000 pounds more than a Supersaurus
Explanation:
As we know 1 ton = 2000 pounds, given Supersaurus weighed 45 tons = 45 x 2000=90,000 pounds and Argentinosaurus weighs 81 tons = 81 x 2000= 162,000 pounds, So 162,000 – 90,000= 72,000 means Argentinosaurus weigh 72,000 more than a Supersaurus

Review & Refresh

Use the figure.
Big Ideas Math Answers 5th Grade Chapter 4 Multiply Whole Numbers 4.2 11
Question 12.
Name a pair of lines that appear to be parallel.
Answer:
( A,G ) and ( C,J) appear as parallel
Explanation:
In geometry, parallel lines can be defined as two lines in the same plane that are at equal distance from each other and never meet so ( A,G ) and ( C,J) appear as parallel
Question 13.
Name two lines that are perpendicular.
Answer:
( D, F ) and ( B, H ) are perpendicular
Explanation:
In geometry,
Two distinct lines intersecting each other at 90° or a right angle are called perpendicular lines
Question 14.
Name two intersecting lines.
Answer:
(D,F) and (C,J) are two intersecting lines
Explanation:
As the line DF are CJ are intersecting at point F so DF,CJ are called as intersecting lines

Lesson 4.3 Multiply by One-Digit Numbers

Explore and Grow

Use the area model and partial products to find 1,985 × 4.
Big Ideas Math Answers Grade 5 Chapter 4 Multiply Whole Numbers 4.3 1
Answer:
1,985 x 4 = 7,940
Reasonable 7,940  is close to 8,000
Explanation :
Step 1: Multiply the ones. Regroup
4 x 5 ones = 20 ones, Regroup  20 ones as 2 tens and 0 ones
2
1985
x    4
      0
Step 2 : Multiply the tens. Regroup
4 x 8 tens  = 32 tens,  32 tens + 2 tens = 34 tens, Regroup 34 tens as
3 hundreds and 4 tens
3
1985
x    4 
40
Step 3 : Multiply the hundreds. Regroup hundreds
4 x 9 hundreds = 36 hundreds, 36 hundreds + 3 hundreds = 39 hundreds
Regroup 39 hundreds as 3 thousands and 9 hundred
3
1985
x    4 
940
Step 4 : Multiply the thousands. Add the regrouped thousands
4 x 1 thousands = 4 thousands,4 thousands +3 thousands=7 thousands
3
1985
x    4
7,940
1,985 x 4 = 7,940         Reasonable 7,940  is close to 8,000

Structure
Explain how you can use an area model and partial products to find 2,083 × 3.
Answer:
2,083 x 3 = 6,249
Reasonable 6,249  is close to 6,000 or 6,300
Explanation:

Step 1: Multiply the ones.
3 x 3 ones = 9 ones,
2083
x    3
     9
Step 2 : Multiply the tens. Regroup
3 x 8 tens  = 24 tens, Regroup 24 tens as 2 hundreds and 4 tens
2
2083
x    3
   49
Step 3 : Multiply the hundreds.
3 x 0 hundreds = 0 hundred, 0 hundred + 2 hundreds = 2 hundreds
2083
x    3
  249
Step 4 : Multiply the thousands.
3 x 2  thousands = 6 thousands
2083
x    3
6,249
2,083 x 3 = 6,249         Reasonable 6,249  is close to 6,000 or 6,300

Think and Grow: Multiply by One-Digit a Number

Example
Find 8,902 × 4. Estimate ___35,608___
Big Ideas Math Answers Grade 5 Chapter 4 Multiply Whole Numbers 4.3 2

So 8,902 x 4 = 35,608.                  Reasonable 35,608 is close to 35,600

Show and Grow

Find the product. Check whether your answer is reasonable.
Question 1.
Big Ideas Math Answers Grade 5 Chapter 4 Multiply Whole Numbers 4.3 3
Big Ideas Math Answers Grade 5 Chapter 4 Multiply Whole Numbers 4.3 4
Answer:
1. 7,346 x 2= 14,692               Reasonable 14,692 and  is close to 15,000
2. 12,053 x 3 = 36,159            Reasonable 36,159 and  is close to 36,000
Explanation:
1.7,346 x 2=14,692
Step 1: Multiply the ones. Regroup
2 x 6 ones = 12 ones, Regroup 12 ones as 1 tens and 2 ones
1
7,346
x    2
     2
Step 2 : Multiply the tens.
2 x 4 tens  = 8 tens,  8 tens + 1 tens = 9 tens
7,346
x    2
    92
Step 3 : Multiply the hundreds.
2 x 3 hundreds = 6 hundreds
7,346
x     2
   692
Step 4 : Multiply the thousands.
2 x 7 thousands = 14 thousands
7,346
x     2
14,692
So 7,346 x 2 = 14,692        Reasonable 14,692 and  is close to 15,000

2. 12,053 x 3 =36,159
Step 1: Multiply the ones.
3 x 3 ones = 9 ones
12,053
x      3
        9
Step 2 : Multiply the tens. Regroup
3 x 5 tens  = 15 tens, Regroup 15 tens as 1 hundred and 5 tens
1
12,053
x       3
       59
Step 3 : Multiply the hundreds. Regroup
3 x 0 hundreds = 0 hundreds, Regroup 0 hundred +1 hundred = 1 hundred
12,053
x      3
159
Step 4 : Multiply the thousands.
3 x 2 thousands = 6 thousands
12,053
x      3
  6,159
Step 5  : Multiply the ten thousands.
3 x 1 ten thousands = 3 ten thousands
12,053
x      3
36,159
12,053 x 3 = 36,159              Reasonable 36,159 is close to 36,000

Apply and Grow: Practice

Find the product. Check whether your answer is reasonable.
Question 3.
Big Ideas Math Answers Grade 5 Chapter 4 Multiply Whole Numbers 4.3 5
Answer:
503 x 5 = 2,515           Reasonable as 2,515 is close to 2,500
Explanation:
Step 1: Multiply the ones.
5 x 3 ones = 15 ones, Regroup 15 ones as 1 tens and 5 ones
1
503
x  5
    5
Step 2 : Multiply the tens. Regroup
5 x 0 tens  = 0 tens,  0 tens + 1 tens = 1 tens
503
x  5
  15
Step 3 : Multiply the hundreds.
5 x 5 hundreds = 25 hundreds
503
x  5
2,515
503 x 5 = 2,515        Reasonable 2,515 is close to 2,500

Question 4.
Big Ideas Math Answers Grade 5 Chapter 4 Multiply Whole Numbers 4.3 6
Answer:
5,295 x 9 = 47,655             Reasonable 47,655 is close to 48,000
Explanation:
Step 1: Multiply the ones. Regroup
9 x 5 ones = 45 ones, Regroup 45 ones as 4 tens and 5 ones
4
5,295
x     9
      5
Step 2 : Multiply the tens. Regroup
9 x 9 tens  = 81 tens, Regroup 81 tens+ 4 tens = 85 tens as 8 hundred and 5 tens
8
5,295
x     9
55
Step 3 : Multiply the hundreds. Regroup
9 x 2 hundreds = 18 hundreds, Regroup 18 hundred +8 hundred =
26 hundreds as 2 thousand and 6 hundreds
2
5,295
x     9
655
Step 4 : Multiply the thousands.
9 x 5 thousands = 45 thousands, Regroup 45 thousands+2 thousand=47 thousand
2
5,295
x     9 
47,655
5,295 x 9  =47,655          Reasonable 47,655 is close to 48,000

Question 5.
Big Ideas Big Ideas Math Answers Grade 5 Chapter 4 Multiply Whole Numbers 4.3 7
Answer:
87,294 x 3 = 2,61,882        Reasonable 2,61,888 is close to 2,60,000
Explanation:
Step 1: Multiply the ones. Regroup
3 x 4 ones = 12 ones, Regroup 12 ones as 1 ten and 2 ones
1
87,294
x      3
        2
Step 2 : Multiply the tens. Regroup
3 x 9 tens  = 27 tens, Regroup  27 tens + 1 ten = 28 tens as 2 hundred and 8 tens
2
87,294
x       3
82
Step 3 : Multiply the hundreds. Regroup
3 x 2 hundreds = 6 hundreds, Regroup 6 hundred +2 hundred = 8 hundreds
87,294
x      3
882
Step 4 : Multiply the thousands. Regroup
3 x 7 thousands = 21 thousands, Regroup as 2 ten thousands and 1 thousand
2
87,294
x      3
1882
Step 5  : Multiply the ten thousands.
3 x 8 ten thousands = 24 ten thousands,24 ten thousands+ 2 ten thousands
=26 ten thousands, regroup as 2 hundred thousand and 6 ten thousand
2
87,294
x      3
2,61,882
87,294 x 3 =2,61,882        Reasonable 2,61,888 is close to 2,60,000

Question 6.
399 × 2 = _____
Answer:
399 x 2 = 798        Reasonable 798 is close to 800
Explanation:
Step 1: Multiply the ones.
2 x 9 ones = 18 ones, Regroup 1 ones as 1 tens and 8 ones
1
399
x    2
      8
Step 2 : Multiply the tens. Regroup
2 x 9 tens  = 18 tens,  18 tens + 1 tens = 19 tens, Regroup 19 tens as 1 hundred  and 9 tens
1
399
x   2                            
   98
Step 3 : Multiply the hundreds.
2 x 3 hundreds = 6 hundreds, Regroup 6 hundreds+1 hundreds= 7 hundreds
399
x   2 
798
So 399 x 2 = 798       Reasonable 798 is close to 800

Question 7.
33,184 × 2 = _____
Answer:
33,184 x 2 =66,368  Reasonable 66,368 is close to 66,000
Explanation:
Step 1: Multiply the ones.
2 x 4 ones =8 ones
33,184
x      2
        8
Step 2 : Multiply the tens. Regroup
2 x 8 tens  = 16 tens, Regroup 16 tens as 1 hundred  and 6 tens
1
33,184
x      2
68
Step 3 : Multiply the hundreds. Regroup
2 x 1 hundreds = 2 hundreds, Regroup 2 hundreds +1 hundred =  3 hundreds
33,184
x      2
368
Step 4 : Multiply the thousands.
2 x 3 thousands = 6 thousands
33,184
x      2
  6368
Step 5  : Multiply the ten thousands.
2 x 3 ten thousands = 6 ten thousands
33184
x      2
66,368
66,368  x 2 = 66,368        Reasonable 66,368 is close to 66,000

Question 8.
60,759 × 4 = _______
Answer:
60,759 x 4 = 2,43,036        Reasonable 2,43,036 is close to 2,40,000
Explanation:
Step 1: Multiply the ones.
4 x 9 ones =36 ones, Regroup 36 ones as 3 tens and 6 ones
3
60759
x      4
        6
Step 2 : Multiply the tens. Regroup
4 x 5 tens  = 20 tens, Regroup 20 tens + 3 tens = 23 tens as 2 hundred and 3 tens
2
60759
x      4
36
Step 3 : Multiply the hundreds. Regroup
4 x 7 hundreds = 28 hundreds, Regroup 28 hundred +2 hundred =30 hundreds as 3 thousand and 0 hundreds
3
60759
x      4
036
Step 4 : Multiply the thousands.
4 x 0 thousands = 0 thousands , Regroup 0 thousands + 3 thousands=3 thousands
60759
x     4
3036
Step 5  : Multiply the ten thousands.
4 x 6 ten thousands =24 ten thousands
60759
x      4
243068
60,759 x 4 = 2,43,036        Reasonable 2,43,036 is close to 2,40,000

Question 9.
A multiplex has 9 different movie theaters. Each theater has 455 seats. How many seats are in the multiplex?
Answer:
9 x 455 = 4,095 seats
Explanation:
Given a multiplex has 9 different movie theaters, and each theater has 455 seats , so total number of seats in the multiplex is 9 x 455=4,095
there are total 4,095 seats available.

Question 10.
An interstellar object travels 85,700 miles in 1 hour. How far does the object travel in 5 hours?
Answer:
85,770 x 5 = 428,850 miles
Explanation:
Given interstellar object travels 85,700 miles in 1 hour, in 5 hours the interstellar object will travel 85,770 x 5 = 428,850 miles

Question 11.
Without multiplying, determine how much greater 4,395 × 7 is than 4,395 × 6. Explain.
Answer:
it is more by 4,395
Explanation:
Comparing the values both the sides 4,395 is same and as 7 is one time more than 6 we multiply the value by one more time,

so the value of 4,395 x 7 to 4,395 x 6 is one time greater by 4,395

Question 12.
DIG DEEPER!
Find the missing digits.
Big Ideas Math Answers Grade 5 Chapter 4 Multiply Whole Numbers 4.3 8
Answer:
40,716 ÷ 3 = 13572 , Missing digits are 1 , 5, 2
Explanation:
To get the missing digits we divide 40,716 by 3
3)40716(13572
3              3x 1=3
4-3 =1 , we take next digit 0 so it becomes10
——
10          3 x 3=9
09         10- 9=1 , we take next digit 7 becomes 17
——
17            3x 5 = 15
15           17-15=2, we take next digit 1 becomes 21
——
21           3 x 7 = 21
21           21-21 = 0,then we take next digit becomes 6
——–
06          3 x 2 =6
6            6-6=0 and we ends as next  no number
——
0

If 40,716 is divided  by 3 we will get results as 13572  and missing digits are 1 , 5, 2

Think and Grow: Modeling Real Life

Example
A Cuvier’s beaked whale can dive 1,324 feet deeper than 4 times the depth a beluga whale can dive. How deep can a Cuvier’s beaked whale dive?
Big Ideas Math Answers Grade 5 Chapter 4 Multiply Whole Numbers 4.3 9
Write and solve an equation to find the depth.
Big Ideas Math Answers Grade 5 Chapter 4 Multiply Whole Numbers 4.3 10
Let d represent the diving depth of a Cuvier’s beaked whale.
d = (4 × 2,123) + 1,324
= __8,492___ + 1,324
= __9,816___
A Cuvier’s beaked whale can dive   9,816 feet.

Show and Grow

Question 13.
The height of Chicago’s Willis Tower is 469 feet more than 2 times the height of the Gateway Arch. How tall is Willis Tower?
Big Ideas Math Answers Grade 5 Chapter 4 Multiply Whole Numbers 4.3 11
Answer:
The height of Chicago’s Willis Tower is 1,729 feet
Explanation:
Willis Tower = ( 2 X height of the Gateway Arch) +469 feet
Given the height of the Gateway Arch in  St.Louis as 630 feet
Willis Tower = ( 2 X 630 feet ) + 469 feet
Willis Tower = 1,260 + 469 = 1,729 feet

Question 14.
DIG DEEPER!
The Amazon River is 350 miles shorter than 3 times the length of the Colorado River. How much longer is the Amazon River than the Mississippi River?
Big Ideas Math Answers Grade 5 Chapter 4 Multiply Whole Numbers 4.3 12
Answer:
Amazon River is 1,660 miles longer than the Mississippi River
Explanation:
Given Amazon River is = ( 3 x Colorado River )- 350 miles
Amazon River = (3 x 1,450)-350 miles
Amazon River = 4,350 – 350 = 4,000 miles
Now comparing Amazon River with Mississippi River = Mississippi River – Amazon River = 4,000-2,340=1,660 miles
So Amazon River is 1,660 miles longer than the Mississippi River

Multiply by One-Digit Numbers Homework & Practice 4.3

Find the product. Check whether your answer is reasonable.
Question 1.
Big Ideas Math Answers Grade 5 Chapter 4 Multiply Whole Numbers 4.3 13
Answer:
814 x 2=1,628    Reasonable 1,624 is close to 1,600 or 2,000
Explanation:
Step 1: Multiply the ones.
2 x 4 ones = 8 ones.
814
x   2
     8
Step 2 : Multiply the tens.
2 x 1 tens  = 2 tens,
814
x   2                            
  28
Step 3 : Multiply the hundreds.
2 x 8 hundreds = 16 hundreds
814
x  2
1,628
814 x 2=1,628      Reasonable 1,624 is close to 2,000 or 1,600

Question 2.
Big Ideas Math Answers Grade 5 Chapter 4 Multiply Whole Numbers 4.3 14
Answer:
206 x 4 = 824             Reasonable 824 is close to 1,000
Step 1: Multiply the ones.
4 x 6 ones = 24 ones. Regroup 24 ones as 2 tens and 4 ones
2
206
x   4
     4
Step 2 : Multiply the tens. Regroup
4 x 0 tens  = 0 tens, Regroup 0 tens + 2 tens = 2 tens
206
x   4                            
   24
Step 3 : Multiply the hundreds.
4 x 2 hundreds = 8 hundreds
206
x  4
824
206 x 4=824      Reasonable 824 is close to 1,000

Question 3.
Big Ideas Math Answers Grade 5 Chapter 4 Multiply Whole Numbers 4.3 15
Answer:
2,425 x 6 = 14,550             Reasonable 14,550 is close to 14,500
Explanation:
Step 1: Multiply the ones. Regroup
6 x 5 ones = 30 ones, Regroup 30 ones as 3 tens and 0 ones
3
2,425
x     6
      0
Step 2 : Multiply the tens. Regroup
6 x 2 tens  = 12 tens, Regroup 12 tens+ 3 tens = 15 tens as 1 hundred  and 5 tens
1
2425
x    6
50
Step 3 : Multiply the hundreds. Regroup
6 x 4 hundreds = 24 hundreds, Regroup 24 hundred +1 hundred = 25 hundreds as 2 thousand and 5 hundreds
2
2425
x    6
550
Step 4 : Multiply the thousands.
6 x 2 thousands = 12 thousands, Regroup 12 thousands+2 thousands as
14 thousand
2425
x     6
14,550
2,425 x 6 = 14,550        Reasonable 14,550 is close to 14,500

Question 4.
Big Ideas Math Answers Grade 5 Chapter 4 Multiply Whole Numbers 4.3 16
Answer:
5,149 x 9 = 46,341      Reasonable 46,341 is close to 46,000
Explanation:
Step 1: Multiply the ones. Regroup
9 x 9 ones = 81 ones, Regroup 81 ones as 8 tens and 1 ones
8
5149
x     9
      1
Step 2 : Multiply the tens. Regroup
9 x 4 tens  = 36 tens, Regroup 36 tens+ 8 tens =44 tens as 4 hundred  and 4 tens
4
5149
x    9
41
Step 3 : Multiply the hundreds. Regroup
9 x 1 hundreds = 9 hundreds, Regroup 9 hundred +4 hundred =13 hundreds as
1 thousand and 3 hundreds
1
5149
x    9
341
Step 4 : Multiply the thousands.
9 x 5 thousands = 45 thousands, Regroup 45 thousands+1 thousand as 46 thousand
5149
x    9
46,341
5,149 x 9 = 46,341      Reasonable 46,341 is close to 46,000

Question 5.
Big Ideas Math Answers Grade 5 Chapter 4 Multiply Whole Numbers 4.3 17
Answer:
17,823 x 3 = 53,469        Reasonable 53,469 is close to 50,000 or 53,000
Explanation:
Step 1: Multiply the ones.
3 x 3 ones = 9 ones
17,823
x      3
        9
Step 2 : Multiply the tens.
3 x 2 tens  = 6 tens
17,823
x       3
69
Step 3 : Multiply the hundreds. Regroup
3 x 8 hundreds = 24 hundreds, Regroup as 2 thousands and 4 hundreds
2
17823
x      3
    469
Step 4 : Multiply the thousands. Regroup
3 x 7 thousands = 21 thousands, Regroup 21 thousands+ 2 thousands= 23 thousands as 2 ten thousands and 3 thousands
2
17823
x      3
3,469
Step 5  : Multiply the ten thousands.
3 x 1 ten thousands = 3 ten thousands ,Regroup 3 ten thousands + 2 ten thousands = 5 ten thousands
17823
x      3
53,469
17823 x 3 = 53,469      Reasonable 53,469 is close to 50,000 or 53,000

Question 6.
Big Ideas Math Answers Grade 5 Chapter 4 Multiply Whole Numbers 4.3 18
Answer:
61,837 x 8 = 494,696        Reasonable 494,696 is close to 500,000
Explanation:
Step 1: Multiply the ones.
8 x 7 ones =56 ones, Regroup 56 ones as 5 tens and 6 ones
5
61,837
x      8
        6
Step 2 : Multiply the tens. Regroup
8 x 3 tens  = 24 tens, Regroup 24 tens + 5 tens = 29 tens as 2 hundred and 9 tens
2
61,837
x      8
96
Step 3 : Multiply the hundreds. Regroup
8 x 8 hundreds = 64 hundreds, Regroup 64 hundred +2 hundred =66  hundreds as 6 thousand and 6 hundreds
6
61837
x      8
696
Step 4 : Multiply the thousands. Regroup
8 x 1 thousands = 8 thousands , Regroup 8 thousands + 6 thousands=
14 thousands as 1 ten thousand and 4 thousands
1
61837
x     8
4,696
Step 5  : Multiply the ten thousands. Regroup
8 x 6 ten thousands =48 ten thousands, Regroup 48 ten thousands +1 ten thousand = 49 ten thousands
61837
x      8
494,696
61,837 x 8 = 494,696        Reasonable 494,696 is close to 500,000

Find the product. Check whether your answer is reasonable.
Question 7.
4,257 × 5 = _______
Answer:
4,257 x 5 = 21,285                Reasonable 21,285 is close to 20,000
Explanation:
Step 1: Multiply the ones. Regroup
5 x 7 ones = 35 ones, Regroup  35 ones as 3 tens and 5 ones
3
4257
x    5
     5
Step 2 : Multiply the tens. Regroup
5 x 5 tens  = 25 tens, Regroup 25 tens+ 3 tens =28 tens as 2 hundreds and 8 tens
2
4257
x    5
85
Step 3 : Multiply the hundreds. Regroup
5 x 2 hundreds = 10 hundreds, Regroup 10 hundreds + 2 hundreds = 12 hundreds as 1 thousand and 2 hundreds
1
4257
x    5
285
Step 4 : Multiply the thousands.
5 x 4 thousands = 20 thousands, Regroup 20 thousands + 1 thousand =21 thousands
4257
x    5
21,285
4,257 x 5 = 21,285                Reasonable 21,285 is close to 20,000

Question 8.
6,702 × 6 = _____
Answer:
6,702 x 6 = 40,212                    Reasonable 40,212 is close to 40,000
Explanation:
Step 1: Multiply the ones. Regroup
6 x 2 ones = 12 ones, Regroup  12 ones as 1 ten and 2 ones
1
6702
x    6
     2
Step 2 : Multiply the tens. Regroup
6 x 0 tens  = 0 tens, Regroup  0 tens+ 1 tens =1 tens
6702
x    6
12
Step 3 : Multiply the hundreds. Regroup
6 x 7 hundreds = 42 hundreds, Regroup 42 hundreds as 4 thousands and 2 hundreds
4
6702
x    6
212
Step 4 : Multiply the thousands.
6 x 6 thousands = 36 thousands, Regroup 36 thousands+ 4 thousand = 40 thousands
6702
x    6
40,212
6,702 x 6 = 40,212                Reasonable 40,212 is close to 40,000
Question 9.
28,956 × 7 = ______
Answer:
28,956 x 7 = 2,02,692                 Reasonable 2,02,692 is close to 2,00,000
Explanation:
Step 1: Multiply the ones.
7 x 6 ones =42 ones, Regroup 42 ones as 4 tens and 2 ones
4
28,956
x       7
        2
Step 2 : Multiply the tens. Regroup
7 x 5 tens  = 35 tens, Regroup 35 tens + 4 tens = 39 tens as 3 hundreds and 9 tens
3
28956
x      7
92
Step 3 : Multiply the hundreds. Regroup
7 x 9 hundreds = 63 hundreds, Regroup 63 hundreds +3 hundred =
66  hundreds as 6 thousand and 6 hundreds
6
28956
x      7
692
Step 4 : Multiply the thousands. Regroup
7 x 8 thousands = 56 thousands , Regroup 56 thousands + 6 thousands=
62 thousands as 6 ten thousand and 2 thousands
6
28956
x      7
2692
Step 5  : Multiply the ten thousands. Regroup
7 x 2 ten thousands = 14  ten thousands, Regroup 14 ten thousands +
6 ten thousands = 20 ten thousands
28956
x      7
20,2692
28,956 x 7 = 2,02,692                 Reasonable 2,02,692 is close to 2,00,000

Question 10.
A parking garage has 8 levels. There are 194 parking spots on each level. How many parking spots does the garage have?
Answer:
194 x 8 =1,552 parking spots
Explanation:
Given a parking garage has 8 levels and each level has 194 parking spots so total number of parking spots are 194 is multiplied by 8 ,194 x 8 =1,552 , there are total
1,552 parking spots available in the garage.
Question 11.
YOU BE THE TEACHER
Newton finds 16,041 × 8. Is Newton correct? Explain.
Big Ideas Math Answers Grade 5 Chapter 4 Multiply Whole Numbers 4.3 19
Answer:
Yes, Newton is correct
Explanation:
Step 1: Multiply the ones.
8 x 1 ones = 8 ones
16,041
x       8
        8
Step 2 : Multiply the tens. Regroup
8 x 4 tens  = 32 tens, Regroup 3 hundreds and 2 tens
3
16,041
x      8
28
Step 3 : Multiply the hundreds. Regroup
8 x 0 hundreds = 0 hundreds, Regroup 0 hundreds +3 hundreds = 3 hundreds
16,041
x      8
328
Step 4 : Multiply the thousands. Regroup
8 x 6 thousands = 48 thousands , Regroup 48 thousands as 4 ten thousand and 8 thousands
4
16,041
x      8 
8328
Step 5  : Multiply the ten thousands. Regroup
8 x 1 ten thousands = 8 ten thousands, Regroup 8 ten thousands + 4 ten thousands = 12 ten thousands
16,041
x      8
128,328
As the value of 16,041 x 8 = 128,328, Yes Newton is correct
Question 12.
DIG DEEPER!
Write a different multiplication problem that has the same product as 726 × 5.
Answer:
726 x 5 =3,630
605 x 6 =3,630
Explanation:
we divide the product 3630 by 6


we get Quotient as 605 means  when we multiply 605 x 6 we get the  same product result 3,630 similarly when 726 x 5 is multiplied,
So 726 x 5 = 605 x 6

Question 13.
Modeling Real Life
The land area of Florida is 8,056 square miles more than 5 times the land area of Vermont. What is the land area of Florida?
Big Ideas Math Answers Grade 5 Chapter 4 Multiply Whole Numbers 4.3 20
Answer:
The land area of Florida is 49,496 square miles
Explanation:
Given land area of Florida is 8,056 square miles more than 5 times the land area of Vermont and land area of Vermont is 9,216 square miles
land area of Florida = ( 5 x land area of Vermont ) + 9,216 square miles
land area of Florida = ( 5 x 8056 ) + 9,216 square miles
land area of Florida =40280+9,216 = 49,496 square miles
So the land area of Florida is 49,496 square miles.

Question 14.
DIG DEEPER!
A solar farm has 6 rectangular arrays of solar panels. Each array has 105 rows with 8 panels in each row. How many solar panels are on the solar farm?
Answer:
6 x 105 x 8 = 5,040 panels
Explanation:
Given a solar farm has 6 rectangular arrays of solar panels and each array has 105 rows with 8 panels in each row so there are 105 x 8 = 840 panels and we have total 6 rectangular arrays
So total number of solar panels available are 6 x 840 =5,040 panels

Review & Refresh

Multiply.
Question 15.
2 × \(\frac{1}{4}\)
Answer:
2 x 1/4=1/2 or 1 by 2
Explanation:
When 2 is multiplied by fraction of 1,4 we get 2 x 1/4 as 1/2 or 1 by 2

Question 16.
3 × \(\frac{2}{6}\)
Answer:
3 x 2/6 = 6/6 =1
Explanation:
When 3 is multiplied by fraction of 2,6 we get 3 x 2/6 as 6/6 equals to 1

Question 17.
1 × \(\frac{7}{10}\)
Answer:
1 x 7/10 = 7/10 or 0.7
Explanation:
When 1 is multiplied by fraction of 7,10 we get 1 x 7/10 as 7/10 or 0.7

Lesson 4.4 Multiply by Two-Digit Numbers

Explore and Grow

Use the area model and partial products to find 28 × 13.
Big Ideas Math Solutions Grade 5 Chapter 4 Multiply Whole Numbers 4.4 1
Answer:
28 x 13 = 364            Reasonable 364 is close to 400
Explanation:
Step 1: Multiply 28 by 3 ones. Regroup
3 ones x 28 = 84 ones
2
28
x 13
  84
Step 2: Multiply 28 by 1 tens or 10, Regroup
1 tens x 28 =28 tens =280 ones
28
x 13
  84
280
Step 3 : Add the partial products.
28
x   13
    84
+280
364
28 x 13 =364            Reasonable 364 is close to 400

Repeated Reasoning
Explain how you can use an area model and partial products to find 128 × 13.
Answer:
128 x 13 =1,664           Reasonable 1,664 is close to 1,600
Explanation:

Step 1: Multiply 128 by 3 ones. Regroup
3 ones x 128 = 384 ones
2
128
x  13
 384
Step 2: Multiply 128 by 1 tens or 10, Regroup
1 tens x 128 =128 tens =1280 ones
128
    x 13
    384
1280
Step 3 : Add the partial products.
128
x     13
      384
+ 1280
1664
128 x 13 =1,664            Reasonable 1,664 is close to 1,600

Think and Grow: Multiply by a Two-Digit Number

Example
Find 312 × 82. Estimate _______
Big Ideas Math Solutions Grade 5 Chapter 4 Multiply Whole Numbers 4.4 2

So 312 x 82= 25,584                  Reasonable 25,584 is close to 25,500
Show and Grow

Find the product. Check whether your answer is reasonable.
Question 1.
Big Ideas Math Solutions Grade 5 Chapter 4 Multiply Whole Numbers 4.4 3
Answer:
184 x 23 = 4,232                      Reasonable 4,232 is close to 4,000

Explanation:
Step 1: Multiply 184 by 3 ones. Regroup
3 ones x 184 = 552 ones
21
184
x 23
552
Step 2: Multiply 184 by 2 tens or 20, Regroup
2 tens x 184 =368 tens =3680 ones
1
184
  x 23
  552
3680
Step 3 : Add the partial products.
184
x   23
     552
+3680
4232
184 x 23 =4,232            Reasonable 4,232 is close to 4,000

Question 2.
Big Ideas Math Solutions Grade 5 Chapter 4 Multiply Whole Numbers 4.4 4
Answer:
817 x 49 = 40,033           Reasonable 40,033 is close to 40,000
Explanation:
Step 1: Multiply 817 by 9 ones. Regroup
9 ones x 817 = 7353 ones
1 6
817
x 49
7353
Step 2: Multiply 817 by 4 tens or 40, Regroup
4 tens x 817 =3,268 tens =32680 ones
2
817
  x 49
  7353
32680
Step 3 : Add the partial products.
817
x   49
     7353
+32680
40033
817 x 49 =40,033            Reasonable 40,033 is close to 40,000

Question 3.
Big Ideas Math Solutions Grade 5 Chapter 4 Multiply Whole Numbers 4.4 5
Answer:
5046 x 91 = 4,59,186          Reasonable 4,59,186 is close to 4,60,000

Explanation:
Step 1: Multiply 5046 by 1 ones. Regroup
1 ones x  = 5046 ones
5046
x  91
5046
Step 2: Multiply 5046 by 9 tens or 90, Regroup
9 tens x 5046 =45,414 tens =45,4140 ones
45
5046
  x   91
    5046
454140
Step 3 : Add the partial products.
5046
x      91
        5046
+ 454140
4,59,186
5046 x 91 = 4,59,186        Reasonable 4,59,186 is close to 4,60,000

Apply and Grow: Practice

Find the product. Check whether your answer is reasonable.
Question 4.
Big Ideas Math Solutions Grade 5 Chapter 4 Multiply Whole Numbers 4.4 6
Answer:
96 x 24 = 2,304               Reasonable 2,304 is close to 2,300

Explanation:
Step 1: Multiply 96 by 4 ones. Regroup
4 ones x 96 =  ones
2
96
x 24
384
Step 2: Multiply 96 by 2 tens or 20, Regroup
2 tens x 96 =192 tens =1920 ones
1
96
 x 24
  384
1920
Step 3 : Add the partial products.
96
x   24
     384
+1920
2,304
96 x 24 = 2,304        Reasonable 2,304 is close to 2,300

Question 5.
Big Ideas Math Solutions Grade 5 Chapter 4 Multiply Whole Numbers 4.4 7
Answer:
108 x 19 = 2,052             Reasonable 2,052 is close to 2,000

Explanation:
Step 1: Multiply 108 by 9 ones. Regroup
9 ones x 108 = 972 ones
7
108
x 19
972
Step 2: Multiply 108 by 1 tens or 10, Regroup
1 tens x 108 =108 tens =1080 ones
108
  x 19
  972
1080
Step 3 : Add the partial products.
108
x  19
     972
+1080
2,052
108 x 19 = 2,052             Reasonable 2,052 is close to 2,000

Question 6.
Big Ideas Math Solutions Grade 5 Chapter 4 Multiply Whole Numbers 4.4 8
Answer:
6420 x 75 = 4,81,500           Reasonable 4,81,500 is close to 5,00,000
Explanation:
Step 1: Multiply 6420 by 5 ones. Regroup
5 ones x 6420 = 32100 ones
21
6420
x  75
32,100
Step 2: Multiply 6420 by 7 tens or 70, Regroup
7 tens x 6420 =44,940 tens =449400 ones
6420
  x 75
  32100
449400
Step 3 : Add the partial products.
6420
x   75
     32100
+449400
481500
6420 x 75 = 4,81,500           Reasonable 4,81,500 is close to 5,00,000

Question 7.
802 × 41 = ______
Answer:
802 x 41 = 32,882                  Reasonable 32,882 is close to 33,000

Explanation:
Step 1: Multiply 802 by 1 ones. Regroup
1 ones x 802 = 802 ones
802
x 41
802
Step 2: Multiply 802 by 4 tens or 40, Regroup
4 tens x 802 =3208 tens =32080 ones
802
  x 41
  802
32080
Step 3 : Add the partial products.
802
x       41
       802
+32080
32,882
802 x 41 = 32,882          Reasonable 32,882 is close to 33,000

Question 8.
577 × 86 = _____
Answer:
577 x 86 = 49,622                 Reasonable 49,622 is close to 50,000
Explanation:
Step 1: Multiply 577 by 6 ones. Regroup
6 ones x 577 =3,462 ones
4 4
577
x 86
3462
Step 2: Multiply 577 by 8 tens or 80, Regroup
8 tens x 577 =4616 tens =46160 ones
577
  x 86
  3462
46160
Step 3 : Add the partial products.
577
x      86
    3462
+46160
49622
577 x 86 = 49,622                 Reasonable 49,622 is close to 50,000

Question 9.
9,513 × 67 = _____
Answer:
9513 x 67 = 6,37,371             Reasonable  6,37,371   is close to 6,50,000
Explanation:
Step 1: Multiply 9513 by 7 ones. Regroup
7 ones x 9513 =66,591 ones
2
9513
x 67
66,591
Step 2: Multiply 9513 by 6 tens or 60, Regroup
6 tens x 9513 =57078 tens = 570780 ones
9513
  x 67
  66591
570780
Step 3 : Add the partial products.
9513
x      67
    66591
+570780
637371
9513 x 67 = 6,37,371             Reasonable  6,37,371   is close to 6,50,000

Question 10.
Is 6,364 a reasonable product for 706 × 45? Explain your reasoning.
Answer:
No 6,344 is not a reasonable product for 706 x 45 as 706 x 45 = 31,770
Explanation:
When we multiply 706 x 45 we get result as 31,770
as 6,364  is not at all close to 31,770, So 6,344 is not reasonable product for 706 x 45

Question 11.
DIG DEEPER!
Which two-digit number when multiplied by itself has a product of 625?
Answer:
25, because 25 x 25 = 625
Explanation:
Given that a two-digit number when multiplied by itself has a product of 625
lets take the two digit number as X and X x X = 625
X2=625 so X = square root of 625 = √625= √25 x 25, X = 25
So 25 when multiplied by itself we get 625

Think and Grow: Modeling Real Life

Example
A blood bank receives 58 donors in 1 day. Each person donates 1 pint, or about 473 milliliters of blood. About how many milliliters of blood are donated that day?
Big Ideas Math Solutions Grade 5 Chapter 4 Multiply Whole Numbers 4.4 9
Multiply the number of people by the amount of blood each person donates to find the total amount of blood donated.
Multiply 473 and 58.
Big Ideas Math Solutions Grade 5 Chapter 4 Multiply Whole Numbers 4.4 10
So, about _27,434_ milliliters of blood are donated that day.

Show and Grow

Question 12.
A store sells 15 drones. How much money does the store collect?
Big Ideas Math Solutions Grade 5 Chapter 4 Multiply Whole Numbers 4.4 11
Answer:
248 x 15 = $3,720
Explanation:
Given the cost of drone as $248 and the store sells 15 drones, total money collected in the store is $248 x 15=$3,720
Step 1: Multiply 248 by 5 ones. Regroup
5 ones x 248 = 1240 ones
4
248
x15
1240
Step 2: Multiply 248 by 1 tens or 10, Regroup
1 tens x 248 =248 tens = 2480 ones
248
  x 15
 1240
2480
Step 3 : Add the partial products.
248
x  15
  1240
+2480
3720
The store collected total amount is $3,720

Question 13.
What is the area of the soccer field?
Big Ideas Math Solutions Grade 5 Chapter 4 Multiply Whole Numbers 4.4 12
Answer:
The area of the soccer field is 120 x 80 =9,600 square yards
Explanation:
Step 1: Multiply 120 by 0 ones.
0 ones x 120 =120 ones
120
x 80
000
Step 2: Multiply 120 by 8 tens or 80, Regroup
8 tens x 120 =960 tens =  9600 ones
120
x 80
 000
9600
Step 3 : Add the partial products.
120
x 80
   000
+9600
9600
The total area of the soccer field is 120 x 80 =9,600 square yards

Question 14.
The Cassini spacecraft orbited Saturn for 13 years and 77 days. Three of those years were leap years. For how many days did the Cassini spacecraft orbit Saturn? Justify your answer.
Answer:
Cassini spacecraft orbited Saturn for total 4,825 days
Explanation:
Given the Cassini spacecraft orbited Saturn for 13 years and 77 days and three of those years were leap, First we need to convert years to days and add to 77 days, we know  each year has 365 days so first we multiply 365 with 13= 365 x 13 = 4,745 days,
365
x 13
4,745
And we know leap year has 366 days and in 13 years we have 3 leap years so we will add 3 more days to 4,745+3=4,748 days or
adding 3 days as it is saying in 13 years leap years 3 years are leap to 4,745 days=4,745+3=4,748 days
3 days as 3 are leap years
4745
+   3
4,748
Now adding 4,748 and 77 days
4,748
+   77
4,825
Cassini spacecraft orbited Saturn for total 4,825 days

Multiply by Two-Digit Numbers Homework & Practice 4.4

Find the product. Check whether your answer is reasonable.
Question 1.
Big Ideas Math Solutions Grade 5 Chapter 4 Multiply Whole Numbers 4.4 13
Answer:
107 x 11 =1,177     reasonable 1,177 is close to 1,000 or 12,00
Explanation:
Step 1: Multiply 107 by 1 ones.
1 ones x 107 =107 ones
107
x 11
107
Step 2: Multiply 107 by 1 tens or 10, Regroup
1 tens x 107 =107 tens =  1070 ones
107
x 11
 107
1070
Step 3 : Add the partial products.
107
x 11
    107
+1070
1,177
107 x 11 =1,177      reasonable 1,177 is close to 1,000 or 12,00

Question 2.
Big Ideas Math Solutions Grade 5 Chapter 4 Multiply Whole Numbers 4.4 14
Answer:
72 x 13 = 936       reasonable 936 is close to 1,000
Explanation:
Step 1: Multiply 72 by 3 ones.
3 ones x 72 =216 ones
72
x 13
216
Step 2: Multiply 72 by 1 tens or 10, Regroup
1 tens x 72 =72 tens =  720 ones
72
x 13
 216
 720
Step 3 : Add the partial products.
72
x 13
   216
+720
936
72 x 13 =936      reasonable 936 is close to 1,000

Question 3.
Big Ideas Math Solutions Grade 5 Chapter 4 Multiply Whole Numbers 4.4 15
Answer:
466 x 27 = 12,582       reasonable  12,582 is close to 13000 or 12,600
Explanation:
Step 1: Multiply 466 by 7 ones.
7 ones x 466 = 3262 ones
44
466
x 27
3262
Step 2: Multiply 466 by 2 tens or 20, Regroup
2 tens x 466 =932 tens = 9320 ones
466
x 27
 3262
 9320
Step 3 : Add the partial products.
466
x 27
  3262
+9320
12,582
466 x 27 = 12,582    reasonable   12,582 is close to 13000 or 12,600

Find the product. Check whether your answer is reasonable.
Question 4.
Big Ideas Math Solutions Grade 5 Chapter 4 Multiply Whole Numbers 4.4 16
Answer:
83 x 57 = 4,731             reasonable 4731 is close to 5,000
Explanation:
Step 1: Multiply 83 by 7 ones.
7 ones x 83 = 581 ones
2
83
x 57
581
Step 2: Multiply 83 by 5 tens or 50, Regroup
5 tens x 83 =415 tens =  4150 ones
83
x 57
  518
4150
Step 3 : Add the partial products.
83
x 57
     581
+4150
4731
83 x 57 = 4,731          reasonable 4731 is close to 5,000

Question 5.
Big Ideas Math Solutions Grade 5 Chapter 4 Multiply Whole Numbers 4.4 17
Answer:
893 x 44 = 39,292    reasonable 39,292 is close to 40,000
Explanation:
Step 1: Multiply 893 by 4 ones.
4 ones x 893 = 3,572 ones
3 1
893
x 44
3572
Step 2: Multiply 893 by 4 tens or 40, Regroup
4 tens x 893 =3572 tens = 35720 ones
893
x 44
  3572
35720
Step 3 : Add the partial products.
893
x 44
     3572
+35720
39292
893 x 44 = 39,292    reasonable 39,292 is close to 40,000

Question 6.
Big Ideas Math Solutions Grade 5 Chapter 4 Multiply Whole Numbers 4.4 18
Answer:
2079 x 82 = 170,478   reasonable 170,478 is close to 170,000
Explanation:
Step 1: Multiply 2079 by 2 ones.
2 ones x 2079 = 4,158 ones
11
2079
x  82
4158
Step 2: Multiply 2079 by 8 tens or 80, Regroup
8 tens x 2079 = 16,632 tens = 16,6320 ones
2079
x 82
    4158
16,6320
Step 3 : Add the partial products.
2079
x      82
       4158
+16,6320
170478
2079 x 82 = 170,478   reasonable 170,478 is close to 170,000

Question 7.
6,082 × 25 = ______
Answer:
6,082 x 25 = 152,050    reasonable 152,050 is close to 150,000
Explanation:
Step 1: Multiply  by 5 ones.
5 ones x 6082 = 30,410 ones
41
6082
x   25
30410
Step 2: Multiply 6082 by 2 tens or 20, Regroup
2 tens x 6082 = 12164 tens = 121640 ones
6082
x  25
  30410
12,1640
Step 3 : Add the partial products.
6082
x      25
     30410
+121640
15,2050
6,082 x 25 = 152,050    reasonable 152,050 is close to 150,000

Question 8.
158 × 96 = ______
Answer:
158 x 96 = 15,168         reasonable 15168 is close to 15,000
Explanation:
Step 1: Multiply 158 by 96 ones.
6 ones x 158 = 948 ones
34
158
x 96
948
Step 2: Multiply 158 by 9 tens or 90, Regroup
9 tens x 158 =1422 tens = 14220 ones
57
158
x 96
    948
14220
Step 3 : Add the partial products.
158
x 96
      948
+14220
15,168
158 x 96 = 15,168         reasonable 15168 is close to 15,000

Question 9.
98 × 74 = ______
Answer:
98 x 74 = 7252             reasonable 7252 is close to 7000
Explanation:
Step 1: Multiply 98 by 4 ones.
4 ones x 98 = 392 ones
3
98
x74
392
Step 2: Multiply 98 by 7 tens or 70, Regroup
7 tens x 98  = 686 tens = 6860 ones
98
x74
392
6860
Step 3 : Add the partial products.
98
x 74
     392
+6860
7252
98 x 74 = 7252             Reasonable 7252 is close to 7000

Question 10.
Writing
Why is one partial product always greater than the other partial product when multiplying by a two-digit number?
Answer:
The first partial product cannot have a multiplicand greater than 9.
The second partial product cannot have a multiplicand less than 10.
So one partial product is always greater than the other partial product when multiplying by a two-digit number
Explanation:
Lets multiply 315 with 26 a two digit number(26)
The multiplicand 26 can be decomposed to (20 + 6). The first partial product is the product of 315 x 6= 1890. The second partial product is the product of 315 x 20 = 6300.
Here the first partial product cannot have a multiplicand greater than 9  which is 6 and the second partial product cannot have a multiplicand less than 10 which is 20,therefore  one partial product always greater than the other partial product when multiplying by a two-digit number.

Question 11.
DIG DEEPER!
Use the numbers 2, 4, 6, and 8 once to form the greatest product.
Big Ideas Math Solutions Grade 5 Chapter 4 Multiply Whole Numbers 4.4 19
Answer:
The greatest product is 806 x 42 = 33,852
Explanation:
Given numbers are 2,4,6,8 , first greatest number among given numbers are  8 and next is 6 then 4 and then 2 we have the pattern  given as 3 digit number multiplied by  2 digit number, so first 3 great digits numbers are 8,6,0 in these the biggest is 8 then 6 but the middle digit is already given as 0 so the first three digits are 806,
Now the left two digits are 2,4 now to make the greatest 2 digit number we take from the combination of 2 and 4 ,in these the greatest number is 4 first then 2 making it as 42 as second greatest two digit number,
therefore the product is 806 X 42 =33,852 making it as the greatest product.
806
x    42
33,852

Question 12.
Modeling Real Life
A store sells 79 virtual reality headsets. How much money does the store collect?
Big Ideas Math Solutions Grade 5 Chapter 4 Multiply Whole Numbers 4.4 20
Answer:
$125 X 79 = $9,875
Explanation:
Given that the virtual reality headset cost as $ 79 and the store sells 79 virtual reality headsets, Total money collected by the store is $125 x 79=$9,875
125
x 79
9,875

Question 13.
Modeling Real Life
Your friend needs a trumpet for 12 months. She can rent a trumpet for $42 each month and pay a yearly fee of $25, or she can buy a trumpet for $550. Should she buy or rent the trumpet? Explain.
Answer:
Friend should take the trumpet for rent because it is costing less than buying it.
Rent for 12 months is $529
Buying costs to $550.
Explanation:
Friend needs a trumpet for 12 months and rent of trumpet is $42 for each month, so for 12 months it costs as $42 x 12=  $504 and also has to pay  a yearly fee of $25 means total cost for entire 12 months is $504+$25= $529.
Given that if she buys its cost to $550, now comparing between rent and buy as $529<$550 ,as rent is less than buying she has to take it for rent so that she can save $21
Review & Refresh

Use a place value chart to answer the question.
Question 14.
4,000 is 10 times as great as what number?
Answer:
4,000 is 10 times as great as 400
Explanation:
Lets us take the number as X,
Given 4,000 is 10 times as great as X means X  x 10= 4,000
the value of X equals to 4000 x 1/10= 4000/10=400
X=400 therefore 4,000 is 10 times as great as 400

Question 15.
50 is \(\frac{1}{10}\) of what number?
Answer:
50 is \(\frac{1}{10}\) of 500
Explanation:
Let the number be X and given 50 = 1/10 x X, so X = 50 x 10 = 500

Lesson 4.5 Multiply by Multi-Digit Whole Numbers

Explore and Grow

Use any strategy to find each product.
Big Ideas Math Answer Key Grade 5 Chapter 4 Multiply Whole Numbers 4.5 1.1
Answer:
425 x 2 = 850  , 425 x 12 =5,100  , 425 x 112 = 47,600
Explanation:
1) 425 x 2 =850, multiply 425 by 2 ones or 2
1
425
x  2
850
2) 425 x 12 = 51,00
Explanation:
Step 1 : Multiply 425 by 2 ones or 2
1
425
x 12
850
Step 2 : Multiply 425 by 1 tens or 10.
425
x 12
  850
4250
Step 3: Add the partial products.
425
x 12
    850
+4250
  5100
3) 425 x 112 = 47,600
Explanation:
Step 1 : Multiply 425 by 2 ones or 2
1
425
x112
850
Step 2 :Multiply 425 by 1 tens or 10.
425
x 112
  850
4250
Step 3 : Multiply 425 by 1 hundred or 100. Regroup
425
x 112
    850
4250
42500
Step 4: Add the partial products.
425
x 112
        850
4250
+ 42500
47,600 

Structure
How can you use the first product to find the second product? How can you use the second product to find the third product?
Answer:
Yes we can use the first product to find the second product and use the second product to find the third product.
Explanation:
For example let the numbers be 324 x 125 = 40,500
First product result will be product of 324 is first multiplied by last digit 5 at ones place as 324 x 5=1620
Second product result will be the first product results in addition to multiple of 324 with the second digit which is at tens place as 324 x 20 and add to first product results, so here we have used the first product result to find the second product results
324
x25
1620
6480
8100
Third product result will be the second product results and we add it to the multiple of product of third digit which is at hundreds place as  324 x 100 and add to second product results, so here we have used the second product result to find the third product results
324
x125
  8100
32400
40,500

Think and Grow: Multiply Multi-Digit Whole Numbers

Example
Find 2,043 × 132. Estimate __2,69,676_____
Big Ideas Math Answer Key Grade 5 Chapter 4 Multiply Whole Numbers 4.5 1

2,043 x 132 = 2,69,676    reasonable 2,69,676 is close to 2,70,000

Show and Grow

Find the product. Check whether your answer is reasonable.
Question 1.
Big Ideas Math Answer Key Grade 5 Chapter 4 Multiply Whole Numbers 4.5 2
Answer:
318 x 523 =166,314            reasonable 1,66,314 is close to 1,70,000
Explanation:
Step 1 : Multiply 318 by 3 ones or 3
2
318
x 523
  954
Step 2 :Multiply 318 by 2 tens or 20.
1
318
x 523
  954
6360
Step 3 : Multiply 318 by 5 hundred or 500. Regroup
318
x 523
      954
6360
159000
Step 4: Add the partial products.
318
x     523
        954
6360
+159000
166314
So 318 x 523 =166,314            reasonable 1,66,314 is close to 1,70,000

Question 2.
Big Ideas Math Answer Key Grade 5 Chapter 4 Multiply Whole Numbers 4.5 3
Answer:
7291 x 308 = 2,245,628  reasonable 2,245,628 is close to 2,300,000
Explanation:
Step 1 : Multiply 7291 by 8 ones or 8
27
7291
x 308
58328
Step 2 :Multiply 7219 by 0 tens .
7291
x 308
58328
00000
Step 3 : Multiply 7219 by 3 hundred or 300. Regroup
2
7291
x 308
   58328
00000
2187300
Step 4: Add the partial products.
7291
x 308__
     58328
00000
+2187300
2245628
7291 x 308 = 2,245,628     Reasonable 2,245,628 is close to 2,300,000

Apply and Grow: Practice

Find the product. Check whether your answer is reasonable.
Question 3.
Big Ideas Math Answer Key Grade 5 Chapter 4 Multiply Whole Numbers 4.5 4
Answer:
521 x 317 = 1,65,157    Reasonable 165,157  is close to 1,700,00
Explanation:
Step 1 : Multiply 521 by 7 ones or 7
1
521
x 317
3647
Step 2 :Multiply 521 by 1 tens or 10 .
521
x 317
3647
5210
Step 3 : Multiply  521 by 3 hundred or 300. Regroup
521
x 317
    3647
5210
156300
Step 4: Add the partial products.
 521
x 317
       3647
5210
+156300
165157
521 x 317 = 1,65,157    Reasonable 165,157  is close to 1,700,00

Question 4.
Big Ideas Math Answer Key Grade 5 Chapter 4 Multiply Whole Numbers 4.5 5
Answer:
631 x 574 = 3,62,194  Reasonable 3,62,194 is close to 3,60,000
Explanation:
Step 1 : Multiply 631 by 4 ones or 4
631
x 574
2524
Step 2 :Multiply 631 by 7 tens or 70 .
631
x 574
 2524
44170
Step 3 : Multiply 631 by 5 hundred or 500. Regroup
631
x 574
    2524
44170
315500
Step 4: Add the partial products.
631
x 574
     2524
44170
+315500
362194
        
631 x 574 = 3,62,194          Reasonable 3,62,194 is close to 3,60,000  

Question 5.
Big Ideas Math Answer Key Grade 5 Chapter 4 Multiply Whole Numbers 4.5 6
Answer:
2496 x 358 =8,93,568       Reasonable 8,93,568 is close to 9,00,000
Explanation:
Step 1 : Multiply 2496 by 8 ones or 8
374
2496
x 358
19,968
Step 2 :Multiply 2496 by 5 tens or 50 .
243
2496
x 358
 19968
124800
Step 3 : Multiply 2496 by 3 hundred or 300. Regroup
2496
x 358
 19968
124800
748800
Step 4: Add the partial products.
2496
x 358
  19968
124800
748800   
893568
2496 x 358 =8,93,568      Reasonable 8,93,568 is close to 9,00,000
Question 6.
155 × 956 = ______
Answer:
155 x 956 = 1,48,180      Reasonable 1,48,180 is close to 1,50,000
Explanation:
Step 1 : Multiply 155 by 6 ones or 6
33
155
x 956
  930
Step 2 :Multiply 155 by 5 tens or 50 .
22
155
x 956
  930
7750
Step 3 : Multiply 155 by 9 hundred or 900. Regroup
155
x 956
      930
7750
139500
Step 4: Add the partial products.
155
x 956
      930
7750
139500
148180
155 x 956 = 1,48,180      Reasonable 1,48,180 is close to 1,50,000

Question 7.
748 × 239 = ______
Answer:
748 x 239 = 1,78,772     Reasonable  1,78,772 is close to or 1,80,000 or  2,00,000
Explanation:
Step 1 : Multiply 748 by 9 ones or 9
47
748
x239
 6732
Step 2 :Multiply 748 by 3 tens or 30 .
12
748
x 239
  6732
22440
Step 3 : Multiply 748 by 2 hundred or 200. Regroup
1
748
x 239
   6732
22440
149600
Step 4: Add the partial products.
748
x 239___
       6732
22440
+149600
178772
748 x 239 = 1,78,772     Reasonable  1,78,772 is close to or 1,80,000 or  2,00,000

Question 8.
1,907 × 218 = ______
Answer:
1907 x 218 = 415,726  Reasonable 4,15,726 is close to 4,20,000
Explanation:
Step 1 : Multiply 1907 by 8 ones or 8
7 5
1907
x 218
15256
Step 2 :Multiply 1907 by 1 tens or 10 .
1907
x 218
 15256
19070
Step 3 : Multiply  1907 by 2 hundred or 200. Regroup
1  1
1907
x 218
  15256
19070
381400
Step 4 :Add the partial products.
1907
x 218
     15256
19070
+381400
415726
1907 x 218 = 415,726  Reasonable 4,15,726 is close to 4,20,000

Question 9.
The delta of the Nile River is about 150 miles wide. How many feet wide is the delta?
Big Ideas Math Answer Key Grade 5 Chapter 4 Multiply Whole Numbers 4.5 7
Answer:
5280 x 150 = 7,92,000 feet
Explanation:
Given the delta of the Nile River is about 150 miles wide and 1 mile is equal to 5,280 feet so total feet of Nile River is product of 5280 and 150 , 5280 x 150 = 7,92,000 feet

Question 10.
Reasoning
When you multiply a three-digit number by a four-digit number, what is the greatest number of digits the product can have? Explain.
Answer:
999 x 9999 = 9,989,001 the product will have 7 greatest number of digits.
Explanation:
We know 9 is the greatest digit, so the greatest three-digit number is 999 and the greatest four-digit number is 9999 when 999 x 9999 we get the results as 999 x 9999 = 9,989,001 which consists of total 7 digits in it, therefore when we multiply a three-digit number by a four-digit number, we get 7 greatest number of digits in the results.

Question 11.
DIG DEEPER!
Find the missing digit so that both products are the same.
Big Ideas Math Answer Key Grade 5 Chapter 4 Multiply Whole Numbers 4.5 8
Answer:
The missing digit is 0, as 850 x 150= 1,27,500 and 375 x 340 = 1,27,500 are same
Explanation:
To find the missing digit first we multiply 850 X 150 we get 1,27,500 and  it is mentioned that the products are same means
850 x 150= 1,27,500 = 375 x 34___. So to find the missing digit from 34___, we divide 1,27,500 by 375 and the result is 340. So obviously the missing  digit is 0.

Think and Grow: Modeling Real Life

Example
The employees at a baseball bat factory need to make 350,000 bats in 6 months. They make 2,750 bats each day and work 127 days in the 6 months. Do the employees make enough bats?
Big Ideas Math Answer Key Grade 5 Chapter 4 Multiply Whole Numbers 4.5 9
Multiply the number of bats produced each day by the number of days.
Big Ideas Math Answer Key Grade 5 Chapter 4 Multiply Whole Numbers 4.5 10

Compare the number of bats produced to the number of bats needed.
2750 x 127 = 3,49,250 bats as 3,49,250 < 3,50,000
The employees does not___ make enough bats.
Explanation:
Each day the employees make 2750 bats and has worked 127 days in 6 months , so in 6 months the employees have made
2750 x 127 =3,49,250 bats, given that the employees at a baseball bat factory need to make 350,000 bats in 6 months, now comparing  we are getting difference as the employees made to the required number of bats.
We are getting employees made is less when compared to the needed number of bats as 3,49,250 < 3,50,000. Both are not matching so employees have not made enough bats.

Show and Grow

Question 12.
The drivers at a warehouse need to deliver 40,000 packages each day. The warehouse has 128 trucks. Each truck has 350 packages. The drivers deliver all of the packages on the trucks. Do the warehouse drivers deliver enough packages?
Answer:
350 x 128= 44,800 and required is 40,000 as compared it is less,
Yes, the warehouse drivers can deliver enough packages each day.
Explanation:
Each truck has 350 packages and the ware house has 128 trucks, so the total number of packages in a day are 350 x 128 = 44,800 packages. Now comparing the drivers need only 40,000 packages each day as  44,800 is greater than 40,000, 44,800>40,000.Easily the drivers can deliver the packages each day.

Question 13.
A school buys 5 tablets and 5 laptops. Each tablet costs $379. Each laptop costs $449. How much more does the school pay for laptops than it pays for tablets?
Answer:
Tablets costs = $379 x 5=  $1,895
Laptop costs = $449 x 5 = $2,245, Laptop to Tablets = $ 2,245-$1,895=$350, School pays $350 more.
Explanation:
School buys 5 tablets and 5 laptops and each tablet costs $379 and laptop costs $449,the total amount for tablets costs = $379 x 5=  $1,895 and for laptop costs = $449 x 5 = $2,245,Now comparing the costs of laptops to tablets $2,245 to $1895 we are getting a difference of  $350 among them. So the school pays $379
Question 14.
DIG DEEPER!
A koala sleeps 18 hours each day. How many minutes does the koala sleep in 1 year?
Big Ideas Math Answer Key Grade 5 Chapter 4 Multiply Whole Numbers 4.5 11
Answer:
The koala sleeps 3,94,200 minutes in 1 year.
Explanation:
We know that in 1 hour equals to 60 minutes and in 1 year we have total 365 days a koala sleep 18 hours each day means
18 x 60 =1,080 minutes in each day, so in 1 year koala sleeps 1080 minutes x 365= 3,94,200 minutes in 1 year.

Multiply by Multi-Digit Whole Numbers Homework & Practice 4.5

Find the product. Check whether your answer is reasonable.
Question 1.
Big Ideas Math Answer Key Grade 5 Chapter 4 Multiply Whole Numbers 4.5 12
Answer:
102 x 104 =10,608     Reasonable 10,608  is close to 10,000
Explanation:
Step 1 : Multiply 102 by 4 ones or 4
102
x 104
  408
Step 2 : Multiply 102 by 0 tens .
102
x 104
408
000
Step 3 : Multiply 102 by 1 hundred or 100. Regroup
102
x 104
   408
000
10200
Step 4 : Add the partial products.
102
x 104
       408
000
+ 10200
10608
102 x 104 =10,608     Reasonable 10,608 is close to 10,000

Question 2.
Big Ideas Math Answer Key Grade 5 Chapter 4 Multiply Whole Numbers 4.5 13
Answer:
185 x 123 = 22,755  Reasonable 22,755 is close to 20,000
Explanation:
Step 1 : Multiply 18 by 3 ones or 3
21
185
x 123
  555
Step 2 : Multiply 185 by 2 tens or 20
11
185
x 123
  555
3700
Step 3 : Multiply 185 by 1 hundred or 100. Regroup
185
x 123
  555
3700
18500
Step 4 : Add the partial products.
185
x 123
       555
3700
+ 18500
22755

185 x 123 = 22,755  Reasonable 22,755 is close to 20,000
Question 3.
Big Ideas Math Answer Key Grade 5 Chapter 4 Multiply Whole Numbers 4.5 14
Answer:
3410 x 251 = 855,910    Reasonable 855,910 is close to 8,60,000
Explanation:
Step 1 : Multiply 3410 by 1 ones or 1
3410
x 251
3410
Step 2 : Multiply 3410 by 5 tens or 50
2
3410
x 251
    3410
170500
Step 3 : Multiply 3410 by 2 hundred or 200. Regroup
3410
x 251
    3410
170500
682000
Step 4 : Add the partial products.
3410
x 251
3410
170500
+682000
855910
3410 x 251 = 855,910    Reasonable 855,910 is close to 8,60,000

Find the product. Check whether your answer is reasonable.
Question 4.
Big Ideas Math Answer Key Grade 5 Chapter 4 Multiply Whole Numbers 4.5 15
Answer:
954 x 176 = 1,67,904      Reasonable 1,67,904 is close to 1,68,000 or 1,70,000
Explanation:
Step 1 : Multiply 954 by 6 ones or 6
32
954
x 176
5724
Step 2 : Multiply 954 by 7 tens or 70
32
954
x 176
   5724
66780
Step 3 : Multiply 954 by 1 hundred or 100. Regroup
954
x 176
   5724
66780
95400
Step 4 : Add the partial products.
954
x 176
    5724
66780
+95400
167904

954 x 176 = 1,67,904      Reasonable 1,67,904 is close to 1,68,000 or 1,70,000

Question 5.
Big Ideas Math Answer Key Grade 5 Chapter 4 Multiply Whole Numbers 4.5 16
Answer:
818 x 524 = 428,632    Reasonable 4,28,632 is close to 4,30,000 or 4,00,000
Explanation:
Step 1 : Multiply 818 by 4 ones or 4
3
818
x 524
3272
Step 2 : Multiply 818 by 2 tens or 20
1
818
x  524
   3272
16360
Step 3 : Multiply 818 by  5 hundred or 500. Regroup
4
818
x  524
   3272
16360
409000
Step 4 : Add the partial products.
818
x  524
       3272
16360
+ 409000
428632
818 x 524 = 428,632    Reasonable 4,28,632 is close to 4,30,000 or 4,00,000

Question 6.
Big Ideas Math Answer Key Grade 5 Chapter 4 Multiply Whole Numbers 4.5 17
Answer:
6301 x 472 =2,974,072   Reasonable 2,974,072 is close to 3,000,000
Explanation:
Step 1 : Multiply 6301 by 2 ones or 2
6301
x 472
12602
Step 2 :  Multiply 6301 by 7 tens or 70
2
6301
x 472
   12602
441070
Step 3 : Multiply 6301 by  4 hundred or 400. Regroup
6301
x 472
     12602
441070
2520400
Step 4 : Add the partial products.
6301
x 472
      12602
441070
+2520400
2974072
6301 x 472 =2,974,072   Reasonable 2,974,072 is close to 3,000,000

Question 7.
999 × 186 = ______
Answer:
999 x 186 = 185,814     Reasonable 185,814 is close to 2,00,000
Explanation:
Step 1 : Multiply 999 by 6 ones or 6
55
999
x 186
5994
Step 2 :  Multiply 999 by 8 tens or 80
77
999
x 186
  5994
79920
Step 3 : Multiply 999 by  1 hundred or 100. Regroup
999
x 186
    5994
79920
99900
Step 4 : Add the partial products.
999
x 186
    5994
79920
+99900
185814

999 x 186 = 185,814     Reasonable 185,814 is close to 2,00,000
Question 8.
2,445 × 356 = ______
Answer:
2445 x 356 = 870420     Reasonable 870420 is close to 8,70,000
Explanation:
Step 1 : Multiply 2445 by 6 ones or 6
223
2445
x 356
14670
Step 2 :  Multiply 2445 by 5 tens or 50
222
2445
x  356
  14670
122250
Step 3 : Multiply 2445 by  3 hundred or 300. Regroup
111
2445
x 356
  14670
122250
733500
Step 4 : Add the partial products.
2445
x 356
     14670
122250
+733500
  870420
2445 x 356 = 870420     Reasonable 870420 is close to 8,70,000

Question 9.
1,564 × 389 = ______
Answer:
1564 x 389 = 608396     Reasonable 6,08,396 is close to 6,00,000
Step 1 : Multiply 1564 by 9 ones or 9
553
1564
x 389
14076
Step 2 :  Multiply 1564 by 8 tens or 80
453
1564
x 389
  14076
125120
Step 3 : Multiply 1564 by  3 hundred or 300. Regroup
111
1564
x 389
  14076
125120
469200
Step 4 : Add the partial products.
1564
x 389
    14076
125120
+469200
608396 

1564 x 389 = 608396     Reasonable 6,08,396 is close to 6,00,000

Question 10.
Reasoning
Your friend says the product of 4,164 and 137 is 70,460. How will finding an estimate help your friend realize the answer is not reasonable?
Answer:
4164 X 137 = 570,468, 5,70,468 is not equal to 70,460.
Explanation:
Friend says product of 4,164 and 137 is 70,460 , Now finding the estimate the product of 4,164 x 137 = 5,70,468 but friend says it is 70,460. So after finding friend realize the answer is not reasonable and not even close.

Question 11.
YOU BE THE TEACHER
Your friend says that when multiplying 300 by 126, she can multiply 3 × 126 and write two zeros after the product. Is your friend correct? Explain.
Answer:
Yes, Friend is correct
Explanation:
When first we multiply 300 x 126 we get the product as 37,800 and when we multiply 3 x 126 we get result as 378 and if we write two zeros after the product it becomes 37800 which is similar to the result of 300 x 126, so friend is correct.
We can write 300 x 126 as 3 x 102 x 126=   (3 x 126 ) x 102 , We know that if any whole number is multiplied by 10 with powers  we get the product whole number with the  given number of power value additional number of  zeros in the right.
as (3 x 126 ) x 102   has 2 as power to 10 we write the result with 2 zeros after the product which is equal to 300 x 126,So friend is correct.

Question 12.
Modeling Real Life
Your friend starts a video channel and wants to have 50,000 subscribers by the end of 1 year. She gets 140 new subscribers each day for 365 days. Does she meet her goal?
Answer:
140 x 365 = 51,100 as 51,100 > 50,000, Yes friend meets her goal.
Explanation:
As friend starts a video channel and wants to have 50,000 subscribers by the end of 1 year, and each day friend gets 140 new subscribers for 365 days , So total number of subscribers for 365 days or 1 year is 140 x 365= 51,100.
As 51,100 is greater than 50,000 means friend can easily meets her goal.

Review & Refresh

Evaluate the expression.
Question 13.
(5 + 8) × 4
Answer:
(5+8) x 4 = 13 x 4 = 52
Explanation:
Expressions inside parentheses are evaluated first from left to right, i.e (5+8) and then we proceed with multiplication.
So first we add the values inside the parentheses or values of bracket (5+8) x 4 ,5+8=13 and multiply by 4= 13 x 4 = 52,
(5 + 8) × 4 = 52
Question 14.
16 + (9 ÷ 3)
Answer:
16 + (9 ÷ 3) = 16 + 3 = 19
Explanation:
First we take expressions inside parentheses (9 ÷ 3) and evaluate then we proceed with addition,
When  9 ÷ 3 we get  3 , Now we go with addition so 16 + 3 = 19.
16 + (9 ÷ 3) = 19

Question 15.
7 × (4 + 6)
Answer:
7 x (4 +6) = 7 x 10 = 70
Explanation:
First we take expressions inside parentheses ( 4 + 6)=10 and proceed with multiplication, 7 x 10 = 70,
7 x (4 +6) = 70

Multiply Whole Numbers Performance Task

The Grand Coulee Dam is located on the Columbia River in Washington. The dam is used to control flood waters, provide irrigation, and generate power.
Big Ideas Math Answers 5th Grade Chapter 4 Multiply Whole Numbers 1
Question 1.
The dam has 12 pumps that each transfer the same amount of water through pipes from Lake Roosevelt to Banks Lake. Together, the pumps can transfer about 12,000 gallons of water each second.
a. About how many gallons of water can each pump transfer in 1 second?
b. About how many gallons of water can each pump transfer in 1 hour? Explain.
Answer:
a. 12,000 ÷ 12 = 1,000 gallons of water can each pump transfer in 1 second.
b. 1000 x 3,600 seconds = 3,600,000 gallons of water can each pump transfer in 1 hour.
Explanation :
a. Given a dam has 12 pumps that each transfer the same amount of water through pipes from Lake Roosevelt to Banks Lake, and together, the pumps can transfer about 12,000 gallons of water each second.
12 pumps = 12,000 gallons of water each second, Each pump transfer in 1 second is equal to 12,000 divided by 12= 12,000 ÷ 12 =
1,000 gallons of water can each pump transfer in 1 second.
b. Now number of gallons of water can each pump transfer in 1 hour, we know in 1 hour we have 60 minutes and 1 minute has 60 seconds so making it as 3,600 seconds in an hour.1 hour equals to 3,600 seconds. If in 1 second each pump can transfer 1000 gallons of water then in hour it will transfer 1000 x 3,600= 3,600,000 gallons of water can each pump transfer in 1 hour.
Question 2.
The dam generates power Capacity of Power at the Grand Coulee Dam using 33 generators located in 4 different powerhouses.
Big Ideas Math Answers 5th Grade Chapter 4 Multiply Whole Numbers 2
a. What is the total Power for the Grand Coulee Dam?
b. Only about one-third of the capacity of power for each powerhouse is actually generated each year. About how many megawatts does the Third Powerhouse actually generate in 1 year?
c. There are 106 watts in 1 megawatt. Without calculating, how can you find about how many watts the Third Powerhouse actually generates in 1 year?
Answer:
a. The total Power for the Grand Coulee Dam is 6,701 megawatts.
b. The Third Powerhouse actually generates only 1/3 in 1 year is 1800 x 1/3 = 1800/3=1800 ÷ 1/3 = 600 megawatts
c. The number of watts the Third Powerhouse actually generates in 1 year =600 x 106 watts.
Explanation:
a. First we calculate the total capacity of the power house in each power house with the capacity of power from each generator with the number of generators as per the powerhouse, Total Capacity of the power house = capacity of powerhouse from each generator multiply by number of generators  separately. Then adding all capacities we get the total Power for the Grand Coulee Dam as shown in the below table, So total Power for the Grand Coulee Dam is 6,701 megawatts.

b. We have Third Powerhouse has total capacity as 1,800 megawatts as calculated in that only about one-third of the capacity of power for each powerhouse is actually generated each year means total capacity in third powerhouse multiply by 1/3=  1800 x 1/3 = 1800/3=1800 ÷ 1/3 = 600 megawatts.
Third Powerhouse actually generates 600 megawatts in 1 year.
c. Given 1 mega watt equals to 106 watts , We have Third Powerhouse generates 600 megawatts in 1 year, so to convert megawatts  into watts we multiply it by 106 watts =600 x 106 watts, Third Powerhouse actually generates 600 x 106 watts  in 1 year.

Multiply Whole Numbers Activity

Multiplication Adventure
Directions:
1. Players take turns rolling a die. Players solve problems on their boards to race the explorers to their destinations.
2. On your turn, solve the next multiplication problem in the row of your roll.
3. The first player to get an explorer to a destination wins!
Big Ideas Math Answers 5th Grade Chapter 4 Multiply Whole Numbers 3
Answer:

Multiply Whole Numbers Chapter Practice

4.1 Multiplication Patterns

Find the product.
Question 1.
72 × 103 = ______
Answer:
72 × 103 = 72,000
Explanation :
72 × 103 =  72 x 10 x 10 x 10 =72,000 , we multiply 72 with 10 three times as 10 power is 3.

Question 2.
30 × 900 = ______
Answer:
30 × 900 =27,000
Explanation:
When we multiply 30 by 900 we get 27,000 or 30 × 900 = 3 x 10 x 9 x 10 x 10 = 3 x 9 x 103= 27 x 103= 27,000
Question 3.
40 × 500 = ______
Answer:
40 × 500 = 20,000
Explanation:
When we multiply 40 by 500 we get 20,000 or 40 × 500 = 4 x 10 x 5 x 10 x 10 = 4 x 5 x 103= 20 x 103=20,000

Question 4.
60 × 800 = ______
Answer:
60 x 800 = 48,000
Explanation:
When we multiply 60 by 800 we get 48,000 or 60 × 800 = 6 x 10 x 8 x 10 x 10 = 6 x 8 x 103= 48 x 103=48,000

Question 5.
20 × 90 = ______
Answer:
20 x 90 = 1800
Explanation:
When we multiply 20 by 90 we get 1800 or 20 × 90 = 2 x 10 x 9 x 10 = 2 x 9 x 102= 18 x 102=1800

Question 6.
102 × 41 = _____
Answer:
102 × 41 =4100
Explanation:
When we multiply  102 × 41 =10 x 10 x 41 = 100 x 41 = 41,00

Find the missing factor.
Question 7.
_____ × 103 = 26,000
Answer:
The missing factor is 26
Explanation:
To get the missing factor, let us take it as X ,So  X x 103 = 26,000, therefore X = 26,000 / 103 =26,000/1000=26

Question 8.
600 × ____ = 24,000
Answer:
The missing factor is 40

Explanation:
To get the missing factor, let us take it as X , So 600 x X = 24,000, therefore X = 24,000/600 = 40

Question 9.
5,000 × _____ = 250,000
Answer:
The missing factor is 50

Explanation:
To get the missing factor, let us take it as X , So 5,000 x X = 250,000, therefore X = 250,000/5000 = 50

Question 10.
Logic
The product of a number and twice that number is 320,000. Write the multiplication equation.
Answer:
Let the number be X the multiplication equation is X x 2X =320,000
Explanation:
Let us take the number as X given the product of a number and twice that number equals to 320,000 so the equation becomes as x X x 2X = 320,000 means X multiplied by 2X result is 320,000.

4.2 Estimate Products

Estimate the product. Explain whether your estimate is an overestimate or an underestimate.
Question 11.
44 × 81
Answer:
44 x 81 = 3,564
50 x 80 = 4,000 is overestimate
Explanation :
As our estimate after rounding each factor becomes as 44 as 50 to nearest ten and 81 as 80 to nearest ten as 44 x 81 < 50 x 80 as the estimate result is greater than the given product results our estimate is an overestimate

Question 12.
29 × 67
Answer:
29 x 67 = 1,943
30 x 70 = 2,100 is overestimate
Explanation :
As our estimate after rounding each factor becomes 29 as 30  to nearest ten and 67 as 70 to nearest ten as 29 x 67 < 30 x 70 as the estimate result is greater than the given product results our estimate is an overestimate.

Question 13.
504 × 302
Answer:
504 x 302 = 152,208
500 x 300 = 150,000 is underestimate
Explanation :
As our estimate after rounding each factor becomes 504 as 500 to nearest hundred and 302 as 300 to nearest hundred as 504 x 302  > 500 x 300 as the estimate result is lesser than the given product results our estimate is an underestimate.

Question 14.
797 × 29
Answer:
792 x 29 = 22,968
800 x 30 = 24,000 is overestimate
Explanation :
As our estimate after rounding each factor becomes 792 as 800 to nearest hundred and 29 as 30 to nearest ten as 792 x 29 < 800 x 30 as the estimate result is greater than the given product results our estimate is an overestimate.

4.3 Multiply by One-Digit Numbers

Find the product. Check whether your answer is reasonable.
Question 15.
Big Ideas Math Answers 5th Grade Chapter 4 Multiply Whole Numbers chp 15
Answer:
663 x 5 = 3,315     Reasonable 3,315 is close to 3,300
Explanation:
Step 1: Multiply the ones. Regroup
5 x 3 ones = 15 ones, Regroup  15 ones as 1 tens and 5 ones
1
663
x   5
     5
Step 2 : Multiply the tens. Regroup
5 x 6 tens  = 30 tens,  30 tens + 1 tens = 31 tens, Regroup 31 tens as
3 hundreds and 1 tens
3
663
x    5 
15
Step 3 : Multiply the hundreds. Regroup hundreds
5 x 6 hundreds = 30 hundreds, 30 hundreds + 3 hundreds = 33 hundreds
Regroup 33 hundreds as 3 thousands and 3 hundred
663
x   5  
3315
663 x 5 = 3,315     Reasonable 3,315 is close to 3,300

Question 16.
Big Ideas Math Answers 5th Grade Chapter 4 Multiply Whole Numbers chp 16
Answer:
1973 x 7 = 13,811      Reasonable 13,811  is close to 14,000
Explanation:
Step 1: Multiply the ones. Regroup
7 x 3 ones = 21 ones, Regroup  21 ones as 2 tens and 1 ones
2
1973
x    7
     1
Step 2 : Multiply the tens. Regroup
7 x 7 tens  = 49 tens,  49 tens + 2 tens = 51 tens, Regroup 51 tens as
5 hundreds and 1 tens
5
1973
x    7 
11
Step 3 : Multiply the hundreds. Regroup hundreds
7 x 9 hundreds = 63 hundreds, 63 hundreds + 5 hundreds = 68 hundreds
Regroup 68 hundreds as 6 thousands and 8 hundreds
6
1973
x   7  
811
Step 4 : Multiply the thousands. Add the regrouped thousands
7 x 1 thousands = 7 thousands,7 thousands +6 thousands=13 thousands
1973
x    7
13811
1973 x 7 = 13,811      Reasonable 13,811  is close to 14,000

Question 17.
Big Ideas Math Answers 5th Grade Chapter 4 Multiply Whole Numbers chp 17
Answer:
75,028 x 6 = 450,168        Reasonable  4,50,168 is close to 4,50,000
Explanation:
Step 1: Multiply the ones. Regroup
6 x 8 ones = 48 ones, Regroup  48 ones as 4 tens and 8 ones
4
75028
x      6
       8
Step 2 : Multiply the tens. Regroup
6 x 2 tens  = 12 tens,  12 tens + 4 tens = 16 tens, Regroup 16 tens as
1 hundreds and 6 tens
1
75028
x      6 
68
Step 3 : Multiply the hundreds. Regroup hundreds
6 x 0 hundreds = 0 hundreds, 0 hundreds + 1 hundreds = 1 hundred
75028
x     6  
168
Step 4 : Multiply the thousands. Regroup
6 x 5 thousands = 30 thousands, Regroup 30 thousands as 3 ten thousands and 0 thousands
3
75028
x      6
   0168
Step 5  : Multiply the ten thousands.
6 x 7 ten thousands = 42 ten thousands,42 ten thousands+ 3 ten thousands
=45 ten thousands, regroup as 4 hundred thousand and 5 ten thousand
75028
x      6
450168
75,028 x 6 = 4,50,168     Reasonable 4,50,168 is close to 4,50,000

Question 18.
Modeling Real Life
The Great Western Trail is 355 miles longer than 4 times the length of the Iditarod Trail. The Iditarod Trail is 1,025 miles long. How long is the Great Western Trail?
Answer:
The Great Western Trail = 355 + ( 4 x 1,025)= 355 + 4,100 = 4,455 miles
Explanation :
Given that The Great Western Trail is 355 miles longer than 4 times the length of the Iditarod Trail,
The Great Western Trail= 355 + ( 4 x the Iditarod Trail),
and the Iditarod Trail is 1,025 miles ,
The Great Western Trail = 355 + ( 4 x 1025) = 355 + 4100 = 4,455 miles, The Great Western Trail is 4,455 miles long.
4.4 Multiply by Two-Digit Numbers

Find the product. Check whether your answer is reasonable.
Question 19.
Big Ideas Math Answers 5th Grade Chapter 4 Multiply Whole Numbers chp 19
Answer:
45 x 35 = 1575            Reasonable 1575 is close to 1500
Explanation:
Step 1: Multiply 45 by 5 ones. Regroup
5 ones x 45 = 225 ones
2
45
x 35
225
Step 2: Multiply 45 by 3 tens or 30, Regroup
3 tens x 45 =135 tens =1350 ones
45
x 35
  225
1350
Step 3 : Add the partial products.
45
x   35
     225
+1350
1575
45 x 35 = 1575            Reasonable 1575 is close to 1500

Question 20.
Big Ideas Math Answers 5th Grade Chapter 4 Multiply Whole Numbers chp 20
Answer:
206 x 74 = 15,244         Reasonable 15,244 is close to 15,000
Explanation:
Step 1: Multiply 206 by 4 ones. Regroup
4 ones x 206 = 824 ones
2
206
x 74
824
Step 2: Multiply 206 by 7 tens or by 70, Regroup
7 tens x 206 =1442 tens =14420 ones
206
x 74
    824
14420
Step 3 : Add the partial products.
206
x        74
        824
+ 14420
   15244
206 x 74 = 15,244         Reasonable 15,244 is close to 15,000

Question 21.
Big Ideas Math Answers 5th Grade Chapter 4 Multiply Whole Numbers chp 21
Answer:
5,082 x 69 =  350,658                Reasonable 3,50,658 is close to 3,50,000
Explanation:
Step 1: Multiply 5082 by 9 ones. Regroup
9 ones x 5082 = 45,738 ones
71
5082
x  69
45,738
Step 2: Multiply 5082 by 6 tens or 60, Regroup
6 tens x 5082 = 30492 tens =304920 ones
5082
x  69
  45738
304920
Step 3 : Add the partial products.
5082
x    69
    45738
+304920
350658
5,082 x 69 =  350,658                Reasonable 3,50,658 is close to 3,50,000

4.5 Multiply by Multi-Digit Whole Numbers

Find the product. Check whether your answer is reasonable.
Question 22.
Big Ideas Math Answers 5th Grade Chapter 4 Multiply Whole Numbers chp 22
Answer:
612 x 143 = 87,516      Reasonable 87,516 is close to 90,000
Explanation:
Step 1 : Multiply 612 by 3 ones or 3
612
x 143
1836
Step 2 :Multiply 612 by 4 tens or 40.
612
x 143
 1836
24480
Step 3 : Multiply 612 by 1 hundred or 100. Regroup
612
x 143
  1836
24480
61200
Step 4: Add the partial products.
 612
x 143
     1836
   24480
+61200
87516
612 x 143 = 87,516      Reasonable 87,516 is close to 90,000

Question 23.
Big Ideas Math Answers 5th Grade Chapter 4 Multiply Whole Numbers chp 23
Answer:
899 x 475 = 427,025    Reasonable 4,27,025 is close to 4,30,000
Explanation:
Step 1 : Multiply 899 by 5 ones or 5
44
899
x 475
 4495
Step 2 :Multiply 899 by 7 tens or 70.
66
899
x 475
 4495
62930
Step 3 : Multiply 899 by 4 hundred or 400. Regroup
33
899
x 475
 4495
62930
359600
Step 4: Add the partial products.
899
x 475
      4495
62930
+359600
427025
899 x 475 = 427,025    Reasonable 4,27,025 is close to 4,30,000

Question 24.
Big Ideas Math Answers 5th Grade Chapter 4 Multiply Whole Numbers chp 24
Answer:
6182 x 794 = 4,908,508   Reasonable 4,908,508  is close to 5,000,000
Explanation:
Step 1 : Multiply 6182 by 4 ones or 4
3
6182
x 794
24728
Step 2 :Multiply 6182 by 9 tens or 90.
71
6182
x 794
  24728
556380
Step 3 : Multiply 6182 by 7 hundred or 700. Regroup
51
6182
x 794
  24728
556380
4327400
Step 4: Add the partial products.
6182
x 794
    24728
556380
4327400
4908508
6182 x 794 = 4,908,508   Reasonable 4,908,508  is close to 5,000,000

Conclusion:

We wish the Big Ideas Math Grade 5 Answer Key Chapter 4 Multiply Whole Numbers is beneficial for you all. There are different methods to solve the problems in multiply whole numbers. Prepare the questions on your own and solve the problems. Students are notified to improve their math skills by referring to the Big Ideas Math Answers Grade 5 Chapter 4 Multiply Whole Numbers.

Big Ideas Math Answers Grade 6 Chapter 9 Statistical Measures

Big Ideas Math Answers Grade 6 Chapter 9 Statistical Measures

Big Ideas Math Answers Grade 6 Chapter 9 Statistical Measures Pdf is here. To help all the candidates, we took the help of various experts and professionals and prepared Big Ideas Math Answers Grade 6 in pdf format. The pdf format will help you to prepare at home or tuitions as it is available in offline mode. You can get the various chapters involved in Statistical Measures concepts.

Big Ideas Math Answers Grade 6 Chapter 9 Statistical Measures pdf is available for free, so you can start practicing all the problems to score maximum marks. Scroll down to the below sections to get the various concepts like Mean, Mass of Center, Measures of Variations, Mean Absolute Deviation, etc.

Big Ideas Math Book 6th Grade Answer Key Chapter 9 Statistical Measures

Follow all the concepts of statistical measures which are available in Big Ideas Math Grade 6 Solution Key Chapter 9. This material will help you as a guide for your preparation. If you are looking for all the concepts of statistical measures in one place, then BIM 6th Grade Chapter 9 Statistical Measures pdf is the best solution. Don’t waste your time in search of a topic-wise pdf, instead download this whole pdf once and start your preparation.

You can find various level problems here, i.e., easy, tough, and medium level which helps you to analyze your level of preparation and also to overcome your difficulties. Scoring good marks in the exam is the main aim for all the candidates, hence you have to use Big Ideas Math Book 6th Grade Answer Key Chapter 9 Statistical Measures book as a reference.

Performance Task

Lesson: 1 Introduction to Statistics

Lesson: 2 Mean

Lesson: 3 Measures of Center

Lesson: 4 Measures of Variation

Lesson: 5 Mean Absolute Deviation

Chapter 9: Statistical Measures

Statistical Measures STEAM Video/Performance Task

STEAM Video
Daylight in the Big City
Averages can be used to compare different sets of data. How can you use averages to compare the amounts of day light in different cities? Can you think of any other real-life situations where averages are useful?
Big Ideas Math Answer Key Grade 6 Chapter 9 Statistical Measures 1
Watch the STEAM Video “Daylight in the Big City.” Then answer the following questions.
1. Why do different cities have different amounts of daylight throughout the year?

Answer:
Our amount of daylight hours depends on our latitude and how Earth orbits the sun. This causes a seasonal variation in the intensity of sunlight reaching the surface and the number of hours of daylight. The variation in intensity results because the angle at which the sun’s rays hit the Earth changes with the time of year.

2. Robert’s table includes the difference of the greatest amount of daylight and the least amount of daylight in Lagos, Nigeria, and in Moscow, Russia.
Lagos: 44 minutes
Moscow:633 minutes
Use these values to make a prediction about the difference between the greatest amount of daylight and the least amount of daylight in a city in Alaska.

Answer:
The least daylight in Alaska is 1092 minutes in Juneau
The greatest daylight in Alaska is 1320 minutes in Fairbanks

Performance Task
Which Measure of Center Is Best: Mean, Median, or Mode?
After completing this chapter, you will be able to use the concepts you learned to answer the questions in the STEAM Video Performance Task. You will be given the greatest and least amounts of daylight in the 15 cities in the United States with the greatest populations.
s
Big Ideas Math Answer Key Grade 6 Chapter 9 Statistical Measures 3
You will determine which measure of center best represents the data. Why might someone be interested in the amounts of daylight throughout the year in a city?

Statistical Measures Getting Ready for Chapter 9

Chapter Exploration
Work with a partner. Write the number of letters in each of your first names on the board.
Big Ideas Math Answer Key Grade 6 Chapter 9 Statistical Measures 4
1. Write all of the numbers on a piece of paper. The collection of numbers is called data.
2. Talk with your partner about how you can organize the data. What conclusions can you make about the numbers of letters in the first names of the students in your class?
3. Draw a grid like the one shown below. Then use the grid to draw a graph of the data.
Big Ideas Math Answer Key Grade 6 Chapter 9 Statistical Measures 5

Answer:
3,6,9,5,6,7,6,5,5,8,6,8,5,6,4,4,7,6,3,5,6,5,5

4. THE CENTER OF THE DATA Use the graph of the data in Exercise 3 to answer the following.
a. Is there one number that occurs more than any of the other numbers? If so, write a sentence that interprets this number in the context of your class.
b. Complete the sentence, “In my class, the average number of letters in a student’s first name is __________.” Justify your reasoning.
c. Organize your data using a different type of graph. Describe the advantages or disadvantages of this graph.

Answer:
a. Yes, 6, 5, 8 are more than other numbers given in the data.
b. “In my class, the average number of letters in a student’s first name is 5 and 6.

Vocabulary
The following vocabulary terms are defined in this chapter. Think about what each term might mean and record your thoughts.
statistical question
measure of center
measure of variation
mean
median
range

Lesson 9.1 Introduction to Statistics

EXPLORATION 1

Using Data to Answer a Question
Work with a partner.
a. Use your pulse to find your heart rate in beats per minute.
Big Ideas Math Answer Key Grade 6 Chapter 9 Statistical Measures 9.1 1
b. Collect the recorded heart rates of the students in your class, including yourself. How spread out are the data? Use a diagram to justify your answer.
c. REASONING How would you answer the following question by using only one value? Explain your reasoning.
“What is the heart rate of a sixth-grade student?”
Answer: Your pulse is measured by counting the number of times your heart beats in one minute. For example, if your heart contracts 72 times in one minute, your pulse would be 72 beats per minute (BPM).

EXPLORATION 2

Identifying Types of Questions
Work with a partner.
a. Answer each question on your own. Then compare your answers with your partner’s answers. For which questions should your answers be the same? For which questions might your answers be different?
1. How many states are in the United States?
Answer: There are 50 states in the United States.

2. How much does a movie ticket cost? Math Practice
Answer: $9.16
3. What color fur do bears have? Build Arguments How can comparing your answers help you support your conjecture?
Answer: The color white becomes visible to our eyes when an object reflects back all.

4. How tall is your math teacher?
Big Ideas Math Answer Key Grade 6 Chapter 9 Statistical Measures 9.1 2
b. CONJECTURE
Some of the questions in part(a) are considered statistical questions. Which ones are they? Explain.
Answer: 5.10 inches

Big Ideas Math Answer Key Grade 6 Chapter 9 Statistical Measures 9.1 3

Statistics is the science of collecting, organizing, analyzing, and interpreting data. A statistical question is one for which you do not expect to get a single answer. Instead, you expect a variety of answers, and you are interested in the distribution and tendency of those answers.

Try It
Determine whether the question is a statistical question. Explain.
Question 1.
What types of cell phones do students have in your class?
Answer:
Smartphones, Cell phones give students access to tools and apps that can help them complete and stay on top of their class work. These tools can also teach students to develop better study habits, like time management and organization skills.

Question 2.
How many desks are in your classroom?
Answer: 25

Question 3.
How much do virtual-reality headsets cost?
Answer: $499

Question 4.
How many minutes are in your lunch period?
Answer: 45 minutes

A dot plot uses a number line to show the number of times each value in a data set occurs. Dot plots show the spread and the distribution of a data set.

Question 5.
Repeat parts (a)–(c)using the dot plot below that shows the times of students in a 100-meter race.
Big Ideas Math Answer Key Grade 6 Chapter 9 Statistical Measures 9.1 7
Answer:

Self-Assessment for Concepts & Skills

Solve each exercise. Then rate your understanding of the success criteria in your journal.
Question 6.
VOCABULARY
What is a statistical question? Give an example and a non-example.
Answer:
Eg for statistical question: a. How much do bags of pretzels cost at the grocery store?
Because you can anticipate that the prices will vary, it is a statistical question. table at the right may represent the prices of several bags of pretzels at a grocery store.
Eg for non-statistical question: b. How many days does your school have off for spring break this year?
Answer: Because there is only one answer, it is not a statistical question.

Question 7.
OPEN-ENDED
Write and answer a statistical question using the dot plot. Then find and interpret the number of data values.
Big Ideas Math Answer Key Grade 6 Chapter 9 Statistical Measures 9.1 8
Answer: There are 16 data values on the dot plot.

Question 8.
You record the amount of snowfall each day for several days. Then you create the dot plot.
Big Ideas Math Answer Key Grade 6 Chapter 9 Statistical Measures 9.1 11
a. Find and interpret the number of data values on the dot plot.
Answer: There are 13 data values on the dot plot.

b. How can you collect these data? What are the units?
Answer: We can collect the data by using the dots given in the above figure.
c. Write a statistical question that you can answer using the dot plot. Then answer the question.
Answer: dot plots are best used to show a distribution of data.

Question 9.
You conduct a survey to answer, “How many hours does a typical sixth-grade student spend exercising during a week?” Use the data in the table to answer the question.
Big Ideas Math Answer Key Grade 6 Chapter 9 Statistical Measures 9.1 12
Answer:
Given the data
5, 1, 5, 3, 5, 4, 5, 2, 5, 4, 3, 4, 6, 5, 6
The typical sixth-grade student spend exercising during a week is 6 hours.

Introduction to Statistics Homework & Practice 9.1

Review & Refresh

Solve the inequality. Graph the solution.
Question 1.
x – 16 > 8
Answer: x>3.

big ideas math answers grade 6 chapter 9 statistical measures img_1

Question 2.
p + 6 ≤ 8
Answer:   p ≤ 2

big ideas math answers grade 6 chapter 9 statistical measures img_2

Question 3.
54 > 6k
Answer: 9>k

big ideas math answers grade 6 chapter 9 statistical measures img_3

Question 4.
\(\frac{m}{12}\) ≥ 3
Answer: m ≤ 36

Tell whether the ordered pair is a solution of the equation.
Question 5.
y = 4x; (2, 8)
Answer: The given ordered pair is a solution of the equation.
Given : y = 4x;(2,8)
y=8;x=2
8=4 × 2
8=8 (satisfied)

Question 6.
y = 3x + 5; (3, 15)
Answer: Given order pair is not an absolute solution of ordered pair
Given: y = 3x + 5; (3, 15)
y=15;x=3
15=3(3)+5
15=9+5
15=14 (not satisfied)

Question 7.
y = 6x – 15; (4, 9)
Answer:
The given ordered pair is a solution of the equation.
Given: y = 6x – 15; (4, 9)
9=6(4)-15
9=24-15
9=9

Question 8.
A point is reflected in the x-axis. The reflected point is (4, −3). What is the original point?
A. (-3, 4)
B. (-4, 3)
C. (-4, -3)
D. (4, 3)
Answer: B,(-4,3)

Order the numbers from least to greatest.
Question 9.
24%, \(\frac{1}{4}\) , 0.2, \(\frac{7}{20}\) , 0.32
Answer:0.24,0.25,0.2.0.35,0.32
0.2,0.24,0.32,0.35

Question 10.
\(\frac{7}{8}\), 85%, 0.88, \(\frac{3}{4}\) , 78%
Answer:0.875,0.78,0.88,0.75,0.78
0.75,0.78,0.85,0.875,0.88

Concepts, Skills, &Problem Solving

IDENTIFYING TYPES OF QUESTIONS Answer the question. Tell whether your answer should be the same as your classmates’. (See Exploration 2, p. 413.)
Question 11.
How many inches are in 1 foot?
Answer: 12 inches

Question 12.
How many pets do you have?
Answer: none

Question 13.
On what day of the month were you born?
Answer: 27th April

Question 14.
How many senators are in Congress?
Answer: The Senate is composed of 100 Senators, 2 for each state. Until the ratification of the 17th Amendment in 1913, Senators were chosen by state legislatures, not by popular vote. Since then, they have been elected to six-year terms by the people of each state.

IDENTIFYING STATISTICAL QUESTIONS
Determine whether the question is a statistical question. Explain.
Big Ideas Math Answer Key Grade 6 Chapter 9 Statistical Measures 9.1 13
Question 15.
What are the eye colors of sixth-grade students?
Answer: brown

Question 16.
At what temperature (in degrees Fahrenheit) does water freeze?
Answer: 32 degrees Fahrenheit

Question 17.
How many pages are in the favorite books of students your age?
Answer: 200 pages

Question 18.
How many hours do sixth-grade students use the Internet each week?
Answer: 1.5 hour each

Question 19.
MODELING REAL LIFE
The vertical dot plot shows the heights of the players on a recent NBA championship team.
a. Find and interpret the number of data values on the dot plot.
b. How can you collect these data? What are the units?
c. Write a statistical question that you can answer using the dot plot. Then answer the question.
Answer:

Question 20.
MODELING REAL LIFE
The dot plot shows the lengths of earthworms.
Big Ideas Math Answer Key Grade 6 Chapter 9 Statistical Measures 9.1 14
a. Find and interpret the number of data values on the dot plot.
Answer: There are 21 data values on the plot.
b. How can you collect these data? What are the units?
Answer: Based on dot plots and units are measured in mm.
c. Write a statistical question that you can answer using the dot plot. Then answer the question.
Answer: Find the mode of the length of earthworms using the dot plot.
23 is repeated times.
So, the mode is 23.

DESCRIBING DATA
Display the data in a dot plot. Identify any clusters, peaks, or gaps in the data.
Question 21.
Big Ideas Math Answer Key Grade 6 Chapter 9 Statistical Measures 9.1 15
Answer:
bim grade 6 chapter 9 statictical measures answers key img_5

Data are clustered around 22 and around 25
Peak at 25
The gap between 16 and 21

Question 22.
Big Ideas Math Answer Key Grade 6 Chapter 9 Statistical Measures 9.1 16
Answer:
bim grade 6 chapter 9 statictical measures answers key img_6

No clusters
Peak at 83
No gaps

INTERPRETING DATA
The dot plot shows the speeds of cars in a traffic study. Estimate the speed limit. Explain your reasoning.
Question 23.
Big Ideas Math Answer Key Grade 6 Chapter 9 Statistical Measures 9.1 17
Answer: Most of the data clustered around 44 and 45 , hence the estimated speed is between 44-45 miles per hour

Question 24.
Big Ideas Math Answer Key Grade 6 Chapter 9 Statistical Measures 9.1 18
Answer: Most of the data clustered around 65 , there is a peak at 65 and gaps between”60-62″ and 63-65.

Question 25.
DIG DEEPER!
You conduct a survey to answer, “How many hours does a sixth-grade student spend on homework during a school night?” The table shows the results.
Big Ideas Math Answer Key Grade 6 Chapter 9 Statistical Measures 9.1 19
a. Is this a statistical question? Explain.
Answer: yes, it is a statistical question because students work in the different time zone based on individual student capacity.
b. Identify any clusters, peaks, or gaps in the data.
Answer: cluster is around 2. There is a peak at 2 and there is no gap.
c. Use the distribution of the data to answer the question.
Answer: A total of 29 data values are distributed.

RESEARCH
Use the Internet to research and identify the method of measurement and the units used when collecting data about the topic.
Question 26.
wind speed
Answer: The instruments used to measure wind are known as anemometers and can record wind speed, direction, and the strength of gusts. The normal unit of wind speed is the knot (nautical mile per hour = 0.51 m sec-1 = 1.15 mph).

Question 27.
amount of rainfall
Answer:
The standard instrument for the measurement of rainfall is the 203mm (8 inches) rain gauge. This is essentially a circular funnel with a diameter of 203mm which collects the rain into a graduated and calibrated cylinder. The measuring cylinder can record up to 25mm of precipitation

Question 28.
earthquake intensity
Answer: The Richter scale measures the largest wiggle (amplitude) on the recording, but other magnitude scales measure different parts of the earthquake. The USGS currently reports earthquake magnitudes using the Moment Magnitude scale, though many other magnitudes are calculated for research and comparison purposes.

Question 29.
REASONING
Write a question about letters in the English alphabet that is not a statistical question. Then write a question about letters that is a statistical question. Explain your reasoning.
Answer: Statistical Question: How many letters in the English alphabet are used to spell a student’s name in class?
Reasoning: The original question has one answer. This Question will have many answers.

Question 30.
REASONING
A bar graph shows the favorite colors of 30 people. Does it make sense to describe clusters in the data? peaks? gaps? Explain.
Answer: No, It doesn’t make sense to describe the distribution. Colors are not measures numerically.

Lesson 9.2 Mean

EXPLORATION 1

Finding a Balance Point
Work with a partner. The diagrams show the numbers of tokens brought to a batting cage. Where on the number line is the data set balanced ? Is this a good representation of the average? Explain.
Big Ideas Math Answers 6th Grade Chapter 9 Statistical Measures 9.2 1

EXPLORATION 2

Finding a Fair Share
Work with a partner. One token lets you hit 12 baseballs in a batting cage. The table shows the numbers of tokens six friends bring to the batting cage.
Big Ideas Math Answers 6th Grade Chapter 9 Statistical Measures 9.2 2
a. Regroup the tokens so that everyone has the same amount. How many times can each friend use the batting cage? Explain how this represents a “fair share. “Use Clear Definitions What does it mean for data to have an average? How does this help you answer the question?
b. how can you find the answer in part(a) algebraically?
c. Write a statistical question that can be answered using the value in part(a).
Answer:

Big Ideas Math Answers 6th Grade Chapter 9 Statistical Measures 9.2 3

Try It

Find the mean of the data.
Question 1.
Big Ideas Math Answers 6th Grade Chapter 9 Statistical Measures 9.2 6
Answer:
The sum of the data/no of values
The sum of the data=45+54+13+44+89+60+9+18;
no of values=8
The sum of the data=332:no of values=8; 332/8=41.5 is the mean of the data

Question 2.
Big Ideas Math Answers 6th Grade Chapter 9 Statistical Measures 9.2 7
Answer:
555 is mean for the above-given data.

Question 3.
WHA IT?
The monthly rainfall in May was 0.5 inch in City A and 2 inches in City B. Does this affect your answer in Example 2? Explain.
Answer:

Self-Assessment for Concepts & Skills

Solve each exercise. Then rate your understanding of the success criteria in your journal.
Question 4.
NUMBER SENSE
Is the mean always equal to a value in the data set? Explain.
Answer: It is the value that is most common. You will notice, however, that the mean is not often one of the actual values that you have observed in your data set. In addition, the mean is the only measure of central tendency where the sum of the deviations of each value from the mean is always zero.

Question 5.
WRITING
Explain why the mean describes a typical value in a data set.
Answer:
A measure of central tendency is a single value that attempts to describe a set of data by identifying the central position within that set of data. The mean (often called the average) is most likely the measure of central tendency that you are most familiar with, but there are others, such as the median and the mode.

Question 6.
NUMBER SENSE
What can you determine when the mean of one data set is greater than the mean of another data set? Explain your reasoning.
Answer:

Question 7.
COMPARING MEANS
Compare the means of the data sets.
Data set A: 43, 32, 16, 41, 24, 19, 30, 27
Data set B: 44, 18, 29, 24, 36, 22, 26, 21
Answer:
An outlier is a data value that is much greater or much less than the other values. When included in a data set, it can affect the mean.

Question 8.
DIG DEEPER!
The monthly numbers of customers at a store in the first half of a year are 282, 270, 320, 351, 319, and 252. The monthly numbers of customers in the second half of the year are 211, 185, 192, 216, 168, and 144. Compare the mean monthly customers in the first half of the year with the mean monthly customers in the second half of the year.
Answer:

Question 9.
The table shows tournament finishes for a golfer. What place does the golfer typically finish in tournaments? Explain how you found your answer.
Big Ideas Math Answers 6th Grade Chapter 9 Statistical Measures 9.2 12
Answer: Mean=sum of data/number of data values
Mean=118/16
Mean=7.375
a. The golfer’s mean finish was about 7th
b. The finishes 37 and 26 are much greater than other finishes. They are outliers

Mean Homework & Practice 9.2

Review & Refresh

Determine whether the question is a statistical question. Explain.
Question 1.
How tall are sixth-grade students?
Answer: The average height for a sixth grader (age 12) is about five feet. Girls tend to be about an inch taller on average. But there is a wide range. Any height from about 52 inches (4′4″) to 65 inches (5′5″) is in the normal range according to the CDC.

Question 2.
How many minutes are there in 1 Year?
Answer:
An average Gregorian year is 365.2425 days (52.1775 weeks, 8765.82 hours, 525949.2 minutes, or 31556952 seconds). For this calendar, a common year is 365 days (8760 hours, 525600 minutes, or 31536000 seconds), and a leap year is 366 days (8784 hours, 527040 minutes, or 31622400 seconds).

Question 3.
How many counties are in Tennessee?
Answer: Tennessee’s 95 counties are divided into four TDOT regions. Regional offices are located in Jackson (Region 4), Nashville (Region 3), Chattanooga (Region 2), and Knoxville (Region 1).

Question 4.
What is a student’s favorite sport?
Answer: cricket

Write the percent as a fraction or mixed number in simplest form.
Question 5.
84%
Answer:0.84

Question 6.
71%
Answer:0.71

Question 7.
353%
Answer:3.53

Question 8.
0.2%
Answer:0.002

Divide. Check your answer.
Question 9.
11.7 ÷ 9
Answer:1.3

Question 10.
\(\sqrt [ 5 ]{ 72.8 } \)
Answer: 2.35

Question 11.
\(\sqrt [ 6.8 ]{ 28.56 } \)
Answer: 1.63

Question 12.
93 ÷ 3.75
Answer:24.8

Concepts, Skills, & Problem Solving

FINDING A FAIR SHARE Regroup the amounts so that each person has the same amount. What is the amount? (See Exploration 2, p. 419.)
Question 13.
Dollars brought by friends to a fair: 11, 12, 12, 12, 12, 12, 13
Answer:
Given : 11,12,12,12,12,12,13.
Mean=Sum of data/number of data values
Mean=84/7
Mean=12
Answer = 12 dollars for each friend

Question 14.
Tickets earned by friends playing an arcade game: 0, 0, 0, 1, 1, 2, 3
Answer:
Given : 0,0,0,1,1,2,3.
Mean=Sum of data/number of data values
Mean= 7/7
Mean=1
Answer = 1 Tickets each friend

FINDING THE MEAN
Find the mean of the data.
Question 15.
Big Ideas Math Answers 6th Grade Chapter 9 Statistical Measures 9.2 13
Answer: 2 is the mean of the data.

Question 16.
Big Ideas Math Answers 6th Grade Chapter 9 Statistical Measures 9.2 14
Answer: 3 is the mean of the above-given data.

Question 17.
Big Ideas Math Answers 6th Grade Chapter 9 Statistical Measures 9.2 15
Answer: 103 is the mean of the above-given data

Question 18.
Big Ideas Math Answers 6th Grade Chapter 9 Statistical Measures 9.2 16
Answer: 14.8 is the mean of the above-given data.

Question 19.
MODELING REAL LIFE
You and your friends are watching a television show. One of your friends asks, “How long are the commercial breaks during this show?”Break Times (minutes)
Big Ideas Math Answers 6th Grade Chapter 9 Statistical Measures 9.2 17
a. Is this a statistical question? Explain.
Answer: Yes it is a statistical question.

b.Use the mean of the values in the table to answer the question.
Answer:
Given the data,
4.2, 3.5, 4.55, 2.75, 2.25
x̄ = (4.2 + 3.5 + 4.55 + 2.75 + 2.25)/5
x̄ = 17.25/5
= 3.45

Question 20.
MODELING REAL LIFE
The table shows the monthly rainfall amounts at a measuring station.
Big Ideas Math Answers 6th Grade Chapter 9 Statistical Measures 9.2 18
a. What is the mean monthly rainfall?
Answer:
x̄ = (22.5 + 1.51 + 1.86 + 2.06 + 3.48 + 4.47 + 3.37 + 5.40 + 5.45 + 4.34 + 2.64 + 2.14)/12
= 33.54/12
= 2.795

b. Compare the mean monthly rainfall for the first half of the year with the mean monthly rainfall for the second half of the year.
Answer:
Mean:
x̄ = (22.5 + 1.51 + 1.86 + 2.06 + 3.48 + 4.47)/6
= 15.6/6
= 2.6
For second 6 months:
x̄ = (3.37 + 5.40 + 5.45 + 4.34 + 2.64 + 2.14)/6
= 23.34/6
= 3.89
The mean value of the second 6 months is greater than the first 6 months.

Question 21.
OPEN-ENDED
Create two different data sets that have six values and a mean of 21.
Answer:
Mean of 21:
Set 1:
12, 31, 21, 24, 13, 25 for these numbers we can calculate the mean we get 21
Set 2:
12, 31, 20, 30, 10, 18 for these numbers we can calculate the mean we get 21

Question 22.
MODELING REAL LIFE
The bar graph shows your cell phone data usage for five months. Describe how the outlier affects the mean. Then use the data to answer the statistical question, “How much cell phone data do you use in a month?”
Big Ideas Math Answers 6th Grade Chapter 9 Statistical Measures 9.2 19
Answer: 288 is a lot less than the other data values so it is an outlier
Mean with outlier=10/5
Mean with outlier = 2
Mean without outlier = 6.18/5
Mean without outlier = 1.236
The outlier causes the mean to be about 0.76 data usage.

Question 23.
MODELING REAL LIFE
The table shows the heights of the volleyball players on two teams. Compare the mean heights of the two teams. Do outliers affect either mean? Explain.
Big Ideas Math Answers 6th Grade Chapter 9 Statistical Measures 9.2 20
Answer:
Dolphins=59+65+53+56+58+61+64+68+51+56+54+57=702
Total no of observations=12;Mean=702\12=58.5
Tigers=63+68+66+58+54+55+61+62+53+70+64+64=683
Total no of observations=12; Mean=683/12=56.9

Question 24.
REASONING
Use a dot plot to explain why the mean of the data set below is the point where the data set is balanced.
11, 13, 17, 15, 12, 18, 12
Answer:
mean = (11 + 13 + 17 + 15 + 18 + 12)/6
= 86/6
= 14.3

Question 25.
DIG DEEPER!
In your class, 7 students do not receive a weekly allowance, 5 students receive $3, 7 students receive $5, 3 students receive $6, and 2 students receive $8.
a. What is the mean weekly allowance? Explain how you found your answer.
b. A new student who joins your class receives a weekly allowance of $3.50. Without calculating, explain how this affects the mean.
Answer:
Given number of students receive no amount = 7
Number of students receive $3 = 5
Then, total amount 5 students receive = 5 × 3 = $15
Then, total amount 7 students receive = 5 × 7 = $35
Number of students receive $6 = 3
Then total amount 3 students receive = 6 × 3 = $18
Number of students receive $8 = 2
Then, total amount 2 students receive = 2 × 8 = $16
Now, the total amount all students receive =
15 + 35 + 18 + 6 = 84
The total students = 7 + 5 + 7 + 3 + 2 = 24
Mean = total amount/total amount = 84/24 = $3.5
Hence, the mean weekly allowance is $3.5

Question 26.
PRECISION
A collection of 8 geodes has a mean weight of 14 ounces. A different collection of 12 geodes has a mean weight of 14 ounces. What is the mean weight of the 20 geodes? Explain how you found your answer.
Big Ideas Math Answers 6th Grade Chapter 9 Statistical Measures 9.2 21
Answer:
Given,
A collection of 8 geodes has a mean weight of 14 ounces.
A different collection of 12 geodes has a mean weight of 14 ounces.
Total weight of the first 8 backpacks
8×14
112 pounds
Total weight of the second 12 backpacks
12×9
108
Total weight of the whole 20 backpacks
112+108
220
So the mean weight of the 20 backpacks
220 / 20
11

Lesson 9.3 Measures of Center

EXPLORATION 1

Finding the Median
Work with a partner.
a. Write the total numbers of letters in the first and last names of 15 celebrities, historical figures, or people you know. One person is already listed for you.
Big Ideas Math Answers Grade 6 Chapter 9 Statistical Measures 9.3 1

Dr. B. R. Ambedkar-8
Otto von Bismarck-15
A. P. J. Abdul Kalam-10
Vallabhbhai Patel-16
Alexander Hamilton-17
Jawaharlal Nehru -15
Mother Teresa -12
Thomas Jefferson-15
J. R. D. Tata -4
Indira Gandhi -12
Sachin Tendulkar-15
Napoleon Bonaparte-17
John Adams-9
Karl Marx-8
Andrew Jackson-13
b. Order the values in your data set from least to greatest. Then write the data on a strip of grid paper with 15 boxes.
Big Ideas Math Answers Grade 6 Chapter 9 Statistical Measures 9.3 2
c. The middle value of the data set is called the median. The value (or values) that occur most often is called the mode. Find the median and the mode of your data set. Explain how you found your answers.
Big Ideas Math Answers Grade 6 Chapter 9 Statistical Measures 9.3 3
d. Why are the median and the mode considered averages of a data set?
Answer:

Big Ideas Math Answers Grade 6 Chapter 9 Statistical Measures 9.3 4

A measure of center is a measure that describes the typical value of a data set. The mean is one type of measure of center. Here are two others.

Try It

Question 1.
Find the median and mode of the data.1, 2, 20, 4, 17, 8, 12, 9, 5, 20, 13
Answer: Given the data,
1, 2, 20, 4, 17, 8, 12, 9, 5, 20, 13
First, write the numbers in the ascending or descending order.
1, 2, 4, 5, 8, 9, 12, 13, 17, 20, 20
The Median is 9.
The mode is 20 because it is repeated more than once.

Question 2.
100, 75, 90, 80, 110, 102
Answer:
Given the data,
100, 75, 90, 80, 110, 102
First, write the numbers in the ascending or descending order.
75, 80, 90, 100, 102, 110
= (90+100)/2
= 85
Mode:
No mode in the data.

Question 3.
One member of the class was absent and ends up voting for horror. Does this change the mode? Explain.
Answer: No

Question 4.
The times (in minutes) it takes six students to travel to school are 8, 10, 10, 15, 20, and 45. Find the mean, median, and mode of the data with and without the outlier. Which measure does the outlier affect the most?
Answer:
Median:
Write the numbers in ascending or descending order
8, 10, 10, 15, 20, and 45
= (10 + 15)/2 = 25/2 = 12.5
Mode:
10 is the mode. Because it is the most repeated number.
Mean:
Adding up the values and then dividing by the number of values.
= (8 + 10 + 10 + 15 + 20 + 45)/6
= 108/6
= 18

Question 5.
WHAT IF?
The store decreases the price of each video game by$3. How does this decrease affect the mean, median, and mode?
Answer:

Self-Assessment for Concepts & Skills

Solve each exercise. Then rate your understanding of the success criteria in your journal.
Question 6.
FINDING MEASURES OF CENTER
Consider the data set below.
15, 18, 13, 11, 12, 21, 9, 11
a. Find the mean, median, and mode of the data.

Answer:
Given the data,
15, 18, 13, 11, 12, 21, 9, 11
x̄ = (15 + 18 + 13 + 11 + 12 + 21 + 9 + 11)/8
x̄ = 110/8
x̄ = 13.75
Median:
Write the numbers in ascending order and descending order.
9, 11, 11, 12, 13, 15, 18, 21
= (12 + 13)/2
= 12.5
Mode:
11 is the mode because this is repeated more than one time.

b. Each value in the data set is decreased by 7. How does this change affect the mean, median, and mode?
Answer:
Each value is decreased by 7 in the given data
8, 11, 6, 4, 5, 14, 2, 4
x̄ = (8 + 11 + 6 + 4 + 5 + 14 + 2 + 4)/8
x̄ = 54/8
x̄ = 6.75

Question 7.
WRITING
Explain why a typical value in a data set can be described by the median or the mode.
Answer:
For data from skewed distributions, the median is better than the mean because it isn’t influenced by extremely large values. The mode is the only measure you can use for nominal or categorical data that can’t be ordered

Question 8.
How does removing the outlier affect your answer in Example 5?
Answer:

Question 9.
It takes 10 contestants on a television show 43, 41, 62, 40, 44, 43, 44, 46, 45, and 41 seconds to cross a canyon on a zipline. Find the mean, median, and mode of the data with and without the outlier. Which measure does the outlier affect the most?
Answer:

Question 10.
The table shows the weights of several great white sharks. Use the data to answer the statistical question, “What is the weight of a great white shark?”
Big Ideas Math Answers Grade 6 Chapter 9 Statistical Measures 9.3 12
Answer:

Measures of Center Homework & Practice 9.3

Review & Refresh

Find the mean of the data.
Question 1.
1, 5, 8, 4, 5, 7, 6, 6, 2, 3
Answer: 4.7

Explanation:
Given the data,
1, 5, 8, 4, 5, 7, 6, 6, 2, 3
x̄ = ∑x/n
x̄ = (1 + 5 + 8 + 4 + 5 + 7 + 6 + 6 + 2 + 3)/16
x̄ = 49/16
x̄ = 3.06

Question 2.
9, 12, 11, 11, 10, 7, 4, 8
Answer: 9

Explanation:
Given the data,
9, 12, 11, 11, 10, 7, 4, 8
x̄ = ∑x/n
x̄ = (9 + 12 + 11 + 11 + 10 + 7 + 4 + 8)/8
x̄ = 72/8
x̄ = 9

Question 3.
26, 42, 31, 50, 29, 37, 44, 31
Answer: 36.25

Explanation:
Given the data,
26, 42, 31, 50, 29, 37, 44, 31
x̄ = ∑x/n
x̄ = (26+42+31+50+29+37+44+31)/8
x̄ = 290/8
x̄ = 36.25

Question 4.
53, 45, 43, 55, 28, 21, 61, 29, 24, 40, 27, 42
Answer: 39

Explanation:
Given the data,
53, 45, 43, 55, 28, 21, 61, 29, 24, 40, 27, 42
x̄ = ∑x/n
x̄ = (53+45+43+55+28+21+61+29+24+40+27+42)/12
x̄ = 468/12
x̄ = 39

Question 5.
A shelf in your room can hold at most 30 pounds.  ere are 12 pounds of books already on the shelf. Which inequality represents the number of pounds you can add to the shelf?
A. x < 18
B. x ≥ 18
C. x ≤ 42
D. x ≤ 18
Answer: x ≤ 18

Explanation:
12+x ≤ 30
12+x -12 ≤ 30-12
x ≤ 18

Find the missing values in the ratio table. Then write the equivalent ratios.
Question 6.
Big Ideas Math Answers Grade 6 Chapter 9 Statistical Measures 9.3 13
Answer:
Big-Ideas-Math-Answers-Grade-6-Chapter-9-Statistical-Measures-9.3-13

Question 7.
Big Ideas Math Answers Grade 6 Chapter 9 Statistical Measures 9.3 14
Answer:
Big-Ideas-Math-Answers-Grade-6-Chapter-9-Statistical-Measures-9.3-14

Find the surface area of the prism.

Question 8.
Big Ideas Math Answers Grade 6 Chapter 9 Statistical Measures 9.3 15
Answer:
Given,
l = 6m
w = 5m
h = 5m
We know that,
Surface Area of the Prism = 2lw + 2lh + 2hw
= 2(6 × 5) + 2(6 × 8) + 2(8 × 5)
= 60 + 96 + 80
= 236 sq. meters

Question 9.
Big Ideas Math Answers Grade 6 Chapter 9 Statistical Measures 9.3 16
Answer:
Given,
l = 4.5 ft
w = 2ft
h = 3.5ft
We know that,
Surface Area of the Prism = 2lw + 2lh + 2hw
= 2(4.5 × 2) + 2(4.5 × 3.5) + 2(2 × 3.5)
= 18 + 31.5 + 14
= 63.5 sq. ft

Question 10.
Big Ideas Math Answers Grade 6 Chapter 9 Statistical Measures 9.3 17
Answer:
Given,
l = 6 yd
w = 4 yd
h = 2 yd
We know that,
Surface Area of the Prism = bh + 2lh + lb
= 2 × 4 + 2(6 × 5) + 6 × 2
= 8 + 60 + 12
= 80 sq. yards

Concepts, Skills, & Problem Solving

FINDING THE MEDIAN Use grid paper to find the median of the data. (See Exploration 1, p. 425.)
Question 11.
9, 7, 2, 4, 3, 5, 9, 6, 8, 0, 3, 8
Answer:
First, arrange the numbers in ascending or descending order.
= 0, 2, 3, 3, 4, 5, 6, 7, 8, 8, 9, 9
= (5 + 6)/2
= 11/2
= 5.5

Question 12.
16, 24, 13, 36, 22, 26, 22, 28, 25
Answer:
First, arrange the numbers in ascending or descending order.
13, 16, 22, 22, 24, 25, 26, 28, 36
24 is the median.
The median is the middle score in a set of given data.

FINDING THE MEDIAN AND MODE
Find the median and mode of the data.
Question 13.
3, 5, 7, 9, 11, 3, 8
Answer: The Median is 7; The Mode is 3.
Given: 3, 5, 7, 9, 11, 3, 8
Sorted list: 3,3,5,7,8,9,11
Median is the middle number in a sorted list of numbers = 7
The mode is the value that appears most frequently in a data set = 3

Question 14.
14, 19, 16, 13, 16, 14
Answer: The Median is 15; The Modes are 14 and 16.
Given: 13,14,14,16,16,19
Sorted list: 14, 19, 16, 13, 16, 14
Median is the middle number in a sorted list of numbers = 15
The mode is the value that appears most frequently in a data set = 14,16

Question 15.
16. 93, 81, 94, 71, 89, 92, 94, 99
Answer: The Median is 90.5; The Mode is 94.
Given: 16, 93, 81, 94, 71, 89, 92, 94, 99
Sorted list: 16,71,81,89,92,93,94,94,99
Median is the middle number in a sorted list of numbers = 92
The mode is the value that appears most frequently in a data set = 94

Question 16.
44, 13, 36, 52, 19, 27, 33
Answer: The Median is 33; There are no modes.
Given: 44, 13, 36, 52, 19, 27, 33
Sorted list: 13,19,27,33,36,44,52
Median is the middle number in a sorted list of numbers = 33
The mode is the value that appears most frequently in a data set = no mode

Question 17.
12, 33, 18, 28, 29, 12, 17, 4, 2
Answer: The Median is 17; The Modes are 12.
Given: 12, 33, 18, 28, 29, 12, 17, 4, 2
Sorted list: 2,4,12,12,17,18,28,29,33
Median is the middle number in a sorted list of numbers = 17
The mode is the value that appears most frequently in a data set = 12

Question 18.
55, 44, 40, 55, 48, 44, 58, 67
Answer:
The Median is 51.5
The Modes are 44 and 55.
Given: 55, 44, 40, 55, 48, 44, 58, 67
Sorted list: 40,44,44,48,55,55,58,67
Median is the middle number in a sorted list of numbers = 51.5
The mode is the value that appears most frequently in a data set = 44,55

Question 19.
YOU BE THE TEACHER
Your friend finds the median of the data. Is your friend correct? Explain your reasoning.
Big Ideas Math Answers Grade 6 Chapter 9 Statistical Measures 9.3 18
Answer: No, first the given data is arranged in ascending order then after median is to be found. The median is 55

FINDING THE MODE
Find the mode of the data.
Question 20.
Big Ideas Math Answers Grade 6 Chapter 9 Statistical Measures 9.3 19
Answer: The modes are Black and Blue.

Question 21.
Big Ideas Math Answers Grade 6 Chapter 9 Statistical Measures 9.3 20
Answer: The modes are singing, dancing, comedy.

Question 22.
REASONING
In Exercises 20 and 21, can you find the mean and median of the data? Explain.
Answer: You can’t find the mean and median in exercises 20 and 21.
The data set is not made up of numbers

FINDING MEASURES OF CENTER
Find the mean, median, and mode of the data.
Question 23.
4.7, 8.51, 6.5, 7.42, 9.64, 7.2, 9.3
Answer: Given: 4.7, 8.51, 6.5, 7.42, 9.64, 7.2, 9.3
Sorted list: 4.7, 6.5, 7.2, 7.42, 8.51, 9.64
Mean: x̄ = ∑x/n
x̄ = (4.7+6.5+7.2+7.42+8.51+9.64)/6
x̄ = 43.97/6
x̄ =7.32
Median: 7.42.
Mode: no mode.

Question 24.
8\(\frac{1}{2}\), 6\(\frac{5}{8}\), 3\(\frac{1}{8}\), 5\(\frac{3}{4}\), 6\(\frac{5}{8}\), 5\(\frac{1}{4}\), 10\(\frac{5}{8}\), 4\(\frac{1}{2}\)
Answer: Given: 8.5, 6.62, 3.12, 5.75, 6.62, 5.25, 10.62, 4.5
Sorted list: 3.12, 4.5, 5.25, 5.75, 6.62, 6.62, 8.5, 10.62
Mean: x̄ = ∑x/n
x̄ = (3.12, 4.5, 5.25, 5.75, 6.62, 6.62, 8.5, 10.62)/8
x̄ =
x̄ =
Median: 6.18
Mode: 6.62

Question 25.
MODELING REAL LIFE
The weights (in ounces) of several moon rocks are shown in the table. Find the mean, median, and mode of the weights.
Big Ideas Math Answers Grade 6 Chapter 9 Statistical Measures 9.3 21
Answer:
Mean
x̄ = (2.2 + 2.2 + 3.2 + 2.4 + 2.8 + 3.4 + 2.6 + 3.0 + 2.5)/9
Median:
Write the moon rock weights in ascending or descending order.
2.6 is the median
Mode:
2.2 is repeated move times
So, 2.2 is the mode.

REMOVING AN OUTLIER Find the mean, median, and mode of the data with and without the outlier. Which measure does the outlier affect the most?
Question 26.
45, 52, 17, 63, 57, 42, 54, 58
Answer:
Outliners means removing of the small data value
17 is the outliner
x̄ = ∑x/n
= (45 + 52 + 17 + 63 + 57 + 42 + 54 + 58)/8
= 388/8 = 48.5
Mean without outliner:
= (45 + 52 + 63 + 57 + 42 + 54 + 58)/7
= 371/7 = 53
Median with outliner:
17, 42, 45, 52, 54, 57, 58, 63
= (52 + 54)/2
= 106/2
= 53
Median without outliner:
42, 45, 52, 54, 57, 58, 63
54 is the median
Mode:
There is no change of value in the without outliner and with the outliner.
So, there is no mode in the data values.

Question 27.
85, 77, 211, 88, 91, 84, 85
Answer:
77 is the outliner
Mean with outliner:
x̄ = (85 + 77 + 211 + 88 + 91 + 84 + 85)/7
=721/7
= 103
Mean without outliner:
x̄ = (85 + 211 + 88 + 91 + 84 + 85)/6
= 644/6
= 107
Median with outliner:
Write the data values in ascending or descending order.
77, 84, 85, 88, 91, 211
85 is the median.
Median without outliner:
84, 85, 85, 88, 91, 211
= (85 + 88)/2
= 173/2
= 86.5
Mode:
There is no change of value in the without outliner and with the outliner.
85 is the mode.

Question 28.
23, 73, 45, 27, 23, 25, 43, 45
Answer:
73 is the outliner
Mean with outliner:
Mean = (23 + 45 + 27 + 23 + 25 + 43 + 45)
= 231/7
= 33
Mean with outliner:
Mean = (23 + 45 + 27 + 23 + 25 + 43 + 45+ 73)
= 304/8
= 38

Question 29.
101, 110, 99, 100, 64, 112, 110, 111, 102
Answer:
64 is the outliner
Mean with outliner:
x̄ = (101 + 110 + 99 + 100 + 64 + 112 + 110 + 111 + 102)/9
= 901/9 = 101
Mean with outliner:
x̄ = (101 + 110 + 99 + 100 + 112 + 110 + 111 + 102)/8
= 755/8
= 94.37
Median:
Write the data values in ascending or descending order
64, 99, 100, 101, 102, 110, 111, 112
Median without outliner:
= (101 + 102)/2
= 203/2
= 101.5
Mode:
Mode with and without outliner = 110

Question 30.
REASONING
The table shows the monthly salaries for employees at a company.
Big Ideas Math Answers Grade 6 Chapter 9 Statistical Measures 9.3 22
a. Find the mean, median, and mode of the data.
b. Each employee receives a 5% raise. Find the mean, median, and mode of the data with the raise. How does this increase affect the mean, median, and mode of the data?
c. How are the mean, median, and mode of the monthly salaries related to the mean, median, and mode of the annual salaries?
Answer:

CHOOSING A MEASURE OF CENTER
Find the mean, median, and mode of the data. Choose the measure that best represents the data. Explain your reasoning.
Question 31.
48, 12, 11, 45, 48, 48, 43, 32
Answer:
Write the data in ascending order or descending order.
11, 12, 32, 43, 45, 48, 48, 48
= (32 + 43)/2
= 75/2
= 37.5
48 is the mode of the data

Question 32.
12, 13, 40, 95, 88, 7, 95
Answer:
Mean:
x̄ = ∑x/n
= (12 + 13 + 40 + 95 + 88 + 7 + 95)/7
= 350/7 = 50
Median:
7, 12, 13, 40, 88, 95, 95
40 is the median
mode:
95 is the mode.

Question 33.
2, 8, 10, 12, 56, 9, 5, 2, 4
Answer:
Mean:
x̄ = ∑x/n
= (2 + 8 + 10 + 12 + 56 + 9 + 5 + 2 + 4)/9
= 108/9
= 12
Median:
2, 2, 4, 5, 8, 9, 10, 12, 56
8 is the median
Mode:
2 is the mode.

Question 34.
126, 62, 144, 81, 144, 103
Answer:
Mean:
x̄ = ∑x/n
= (126 + 62 + 144 + 81 + 144 + 103)6
= 660/60
= 11
Median:
62, 81, 103, 126, 144, 144
= (103 + 126)/2
= 114.5

Question 35.
MODELING REAL LIFE
The weather forecast for a week is shown. Which measure of center best represents the high temperatures? the low temperatures? Explain your reasoning.
Big Ideas Math Answers Grade 6 Chapter 9 Statistical Measures 9.3 23
Answer:

Question 36.
RESEARCH
Find the costs of 10 different boxes of cereal. Choose one cereal whose cost will be an outlier.
a. Which measure of center does the outlier affect the most? Justify your answer.
b. Use the data to answer the statistical question, “How much does a box of cereal cost?”
Answer:

Question 37.
PROBLEM SOLVING
The bar graph shows the numbers of hours you volunteered at an animal shelter. What is the minimum number of hours you need to volunteer in the seventh week to justify that you volunteered an average of 10 hours per week for the 7 weeks? Explain your answer using measures of center.
Big Ideas Math Answers Grade 6 Chapter 9 Statistical Measures 9.3 24
Answer:

Question 38.
REASONING
Why is the mode the least frequently used measure of center to describe a data set? Explain.
Answer:
The mode can be helpful in some analyses, but generally it does not contain enough accurate information to be useful in determining the shape of a distribution. When it is not a “Normal Distribution” the Mode can be misleading, although it is helpful in conjunction with the Mean for defining the amount of skewness in a distribution.

Question 39.
DIG DEEPER!
The data are the prices of several fitness wristbands at a store.
$130 $170 $230 $130
$250 $275 $130 $185
a. Does the price shown in the advertisement represent the prices well? Explain.
Big Ideas Math Answers Grade 6 Chapter 9 Statistical Measures 9.3 25
b. Why might the store use this advertisement?
c. In this situation, why might a person want to know the mean? the median? the mode? Explain.
Answer:

Question 40.
CRITICAL THINKING
The expressions 3x, 9x, 4x, 23x, 6x, and 3x form a data set. Assume x> 0.
a. Find the mean, median, and mode of the data.
b. Is there an outlier? If so, what is it?
Answer:
Mean: This is an average of all the numbers. Add up the numbers and then divide by how many numbers there are.
(3 + 9 + 4 + 23 + 6 + 3)/6 = 48/6 = 8
Median: The number in the middle, when the numbers are in order. If there are 2 middle numbers, average them together.
3, 3, 4, 6, 9, 23 : 4 and 6 are the middle numbers. 4+6/2 = 10/2 = 5
Mode: What value occurs most frequently? 3 is the only duplicate
Outlier: What value is abnormal to our set of data? All of our numbers are small (single digits), except for 23. That makes it an outlier.

Lesson 9.4 Measures of Variation

EXPLORATION 1

Interpreting Statements
Work with a partner. There are 24 students in your class. Your teacher makes the following statements.
Big Ideas Math Solutions Grade 6 Chapter 9 Statistical Measures 9.4 1
• “The exam scores range from 75% to 96%.”
a. What does each statement mean? Explain.
b. Use your teacher’s statements to make a dot plot that can represent the distribution of the exam scores of the class.
c. Compare your dot plot with other groups’. How are they alike? different?

EXPLORATION 2

Grouping Data
Work with a partner. The numbers of U.S.states visited by students in a sixth-grade class are shown.
Big Ideas Math Solutions Grade 6 Chapter 9 Statistical Measures 9.4 2
a. Represent the data using a dot plot. Between what values do the data range?
b. Use the dot plot to make observations about the data.
c. How can you describe the middle half of the data?

A measure of variation is a measure that describes the distribution of a data set. A simple measure of variation to find is the range. The range of a data set is the difference of the greatest value and the least value.

Big Ideas Math Solutions Grade 6 Chapter 9 Statistical Measures 9.4 3

Try It
Question 1.
The ages of people in line for a roller coaster are 15, 17, 21, 32, 41, 30, 25, 52, 16, 39, 11, and 24. Find and interpret the range of the ages.
Answer:
Given,
The ages of people in line for a roller coaster are 15, 17, 21, 32, 41, 30, 25, 52, 16, 39, 11, and 24.
Range = (upper value – lower value)/2
= (52 – 11)/2
= 41/2
= 20.5

Question 2.
The data are the number of pages in each of an author’s novels. Find and interpret the interquartile range of the data.
356, 364, 390, 468, 400, 382, 376, 396, 350
Answer:
Given,
The data are the number of pages in each of an author’s novels.
356, 364, 390, 468, 400, 382, 376, 396, 350
Lower quartile = 360
Upper quartile = 398
Interquartile range = 38

Self-Assessment for Concepts & Skills

Solve each exercise. Then rate your understanding of the success criteria in your journal.
Question 3.
WRITING
Explain why the variability of a data set can be described by the range or the interquartile range.
Answer:
The interquartile range is the third quartile (Q3) minus the first quartile (Q1). But the IQR is less affected by outliers: the 2 values come from the middle half of the data set, so they are unlikely to be extreme scores. The IQR gives a consistent measure of variability for skewed as well as normal distributions.

Question 4.
DIFFERENT WORDS, SAME QUESTION
Which is different? Find “both” answers.
Big Ideas Math Solutions Grade 6 Chapter 9 Statistical Measures 9.4 8
Answer:

Question 5.
The table shows the distances traveled by a paper airplane. Find and interpret the range and interquartile range of the distances.
Big Ideas Math Solutions Grade 6 Chapter 9 Statistical Measures 9.4 11
Answer: Given: 13.5, 12.5, 21, 16.75, 10.25, 19, 32, 26.5, 29,16.25, 28.5, 18.5.

Question 6.
The table shows the years of teaching experience of math teachers at a school. How do the outlier or outliers affect the variability of the data?
Big Ideas Math Solutions Grade 6 Chapter 9 Statistical Measures 9.4 12
Answer:
Given the data
5, 10, 7, 8, 10, 11, 22, 8, 6, 35
22 is added to the data set
22 is the outliner
so there is no effect to measure of center and the measure of variability.

Measures of Variation Homework & Practice 9.4

Review & Refresh

Find the mean, median, and mode of the data.
Question 1.
4, 8, 11, 6, 4, 5, 9, 10, 10, 4
Answer:
Mean = x̄ = (4 + 8 + 11 + 6 + 4 + 5 + 9 + 10 + 10 + 4)/10
= 71/10
= 7.1
Median:
Write the data in ascending or descending order.
4, 4, 4, 5, 6, 8, 9, 10, 10, 11
= (5 + 8)/2
= 13/2
=6.5
Mode:
More number if data repeated is called mode.
4 is the mode.

Question 2.
74, 78, 86, 67, 80
Answer:
Mean = x̄ = (74 + 78 + 86 + 67 + 80)/5
= 385/5
= 77
Median:
Write the data in ascending or descending order.
67, 74, 78, 80, 86
78 is the median
Mode:
There is no mode in the data.

Question 3.
15, 18, 17, 17, 15, 16, 14
Answer:
Mean = x̄ = (15 + 18 + 17 + 17 + 15 + 16 + 14)/7
= 112/7 = 16
Median:
Write the data in ascending or descending order.
14, 15, 15, 16, 17, 17, 18
16 is the median
Mode:
17, 15 are the median.

Question 4.
31, 14, 18, 26, 17, 32
Answer:
Mean:
x̄ = (31 + 14 + 18 + 26 + 17 + 32)/6
Median:
Write the data in ascending or descending order.
14, 17, 18, 26, 31, 32
= (18 + 26)/2
= 44/2
= 22
Mode:
There is no mode in the data.

Copy and complete the statement using < or >.
Question 5.
Big Ideas Math Solutions Grade 6 Chapter 9 Statistical Measures 9.4 13
Answer:
A negative number is less than the positive number
6 > -7

Question 6.
Big Ideas Math Solutions Grade 6 Chapter 9 Statistical Measures 9.4 14
Answer:
A negative number is less than the positive number
-3 < 0

Question 7.
Big Ideas Math Solutions Grade 6 Chapter 9 Statistical Measures 9.4 15
Answer:
A negative number is less than the positive number
14 > -14

Question 8.
Big Ideas Math Solutions Grade 6 Chapter 9 Statistical Measures 9.4 16
Answer:
A negative number is less than the positive number
8 > -10

Find the surface area of the pyramid.
Question 9.
Big Ideas Math Solutions Grade 6 Chapter 9 Statistical Measures 9.4 17
Answer:
Given,
Length = 12 mm
Height = 14 mm
A = a² + 2a √a²/4 + h²
Area = 509.56 sq. mm

Question 10.
Big Ideas Math Solutions Grade 6 Chapter 9 Statistical Measures 9.4 18
Answer:
Given,
Length = 5 in
Height = 8.5 in
A = a² + 2a √a²/4 + h²
Area = 113.6 sq. inches

Question 11.
Big Ideas Math Solutions Grade 6 Chapter 9 Statistical Measures 9.4 19
Answer:
Given,
Length = 6 ft
Height = 9 ft
A = a² + 2a √a²/4 + h²
Area = 149.84 sq.ft

Concepts, Skills, &Problem Solving

INTERPRETING STATEMENTS There are 20 students in your class. Your teacher makes the two statements shown. Use your teacher’s statements to make a dot plot that can represent the distribution of the scores of the class. (See Exploration 1, p. 433.)
Question 12.
“The quiz scores range from 65% to 95%.”
“The scores were evenly spread out.”
Answer:

Question 13.
“The project scores range from 78% to 93%.”
“Most of the students received low scores.”
Answer:

FINDING THE RANGE Find the range of the data.
Question 14.
4, 8, 2, 9, 5, 3
Answer: 7

Explanation:
Range is the difference of higher value and lower value
lowest value = 2
highest value = 9
R = 9 – 2
R = 7

Question 15.
28, 42, 36, 23, 14, 47, 40
Answer: 33

Explanation:
The range is the difference between higher value and lower value
Lowest value: 14
Highest value: 47
Range = 47 – 14
R = 33

Question 16.
26, 21, 27, 33, 24, 29
Answer: 12

Explanation:
The range is the difference between higher value and lower value
Lowest value: 21
Highest value: 33
Range = 33 – 21
R = 12

Question 17.
52, 40, 49, 48, 62, 54, 44, 58, 39
Answer: 23

Explanation:
The range is the difference between higher value and lower value
Lowest value: 39
Highest value: 62
Range = 62 – 39
R = 23

Question 18.
133, 117, 152, 127, 168, 146, 174
Answer: 57

Explanation:
The range is the difference between higher value and lower value
Lowest value: 117
Highest value: 174
Range = 174 – 117
R = 57

Question 19.
4.8, 5.5, 4.2, 8.9, 3.4, 7.5, 1.6, 3.8
Answer: 7.3

Explanation:
The range is the difference of higher value and lower value
Lowest value: 1.6
Highest value: 8.9
Range = 8.9 – 1.6
R = 7.3

Question 20.
YOU BE THE TEACHER
Your friend finds the range of the data. Is your friend correct? Explain your reasoning.
Big Ideas Math Solutions Grade 6 Chapter 9 Statistical Measures 9.4 20
Answer:
The range is the difference between higher value and lower value
Lowest value: 28
Highest value: 59
Range =  59 – 28
Range = 31

FINDING THE INTERQUARTILE RANGE Find the interquartile range of the data.
Question 21.
4, 6, 4, 2, 9, 1, 12, 7
Answer: 6

Explanation:
This simple formula is used for calculating the interquartile range:
IQR = Xu – Xl
Lower quartile (xL): 2.5
Upper quartile (xU): 8.5
IQR = 8.5 – 2.5
IQR = 6

Question 22.
18, 22, 15, 16, 15, 13, 19, 18
Answer: 3.75

Explanation:
This simple formula is used for calculating the interquartile range:
IQR = Xu – Xl
Lower quartile (xL): 15
Upper quartile (xU): 18.75
IQR = 18.75 – 15
= 3.75

Question 23.
40, 33, 37, 54, 41, 34, 27, 39, 35
Answer: 7

Explanation:
This simple formula is used for calculating the interquartile range:
IQR = Xu – Xl
Lower quartile (xL): 33.5
Upper quartile (xU): 40.5
IQR = 40.5 – 33.5
= 7

Question 24.
84, 75, 90, 87, 99, 91, 85, 88, 76, 92, 94
Answer: 8

Explanation:
This simple formula is used for calculating the interquartile range:
IQR = Xu – Xl
Lower quartile (xL): 84
Upper quartile (xU): 92
IQR = 92 – 84
= 8

Question 25.
132, 127, 106, 140, 158, 135, 129, 138
Answer: 12

Explanation:
This simple formula is used for calculating the interquartile range:
IQR = Xu – Xl
Lower quartile (xL): 127.5
Upper quartile (xU): 139.5
IQR = 139.5 – 127.5
= 12

Question 26.
38, 55, 61, 56, 46, 67, 59, 75, 65, 58
Answer: 12.75

Explanation:
This simple formula is used for calculating the interquartile range:
IQR = Xu – Xl
Lower quartile (xL): 52.75
Upper quartile (xU): 65.5
IQR = 65.5  – 52.75
= 12.75

Question 27.
MODELING REAL LIFE
The table shows the number of tornadoes in Alabama each year for several years. Find and interpret the range and interquartile range of the data. Then determine whether there are any outliers.
Big Ideas Math Solutions Grade 6 Chapter 9 Statistical Measures 9.4 21
Answer:
The data is 65, 32, 54, 23, 55, 145,37, 80, 94, 42, 69, 77
Range:
Lowest value: 23
Highest value: 145
R = Highest value – Lowest value
R = 145 – 23
R = 122
IQR:
This simple formula is used for calculating the interquartile range:
IQR = Xu – Xl
Lower quartile (xL): 38.25
Upper quartile (xU): 79.25
IQR = 79.25 – 38.25
= 41

Question 28.
WRITING
Consider a data set that has no mode. Which measure of variation is greater, the range or the interquartile range? Explain your reasoning.
Answer:
It would be based on the set of numbers you have, but in most cases, it is the interquartile range, because the mode is usually closer to the median. This leaves the interquartile range as a larger number.

Question 29.
CRITICAL THINKING
Is it possible for the range of a data set to be equal to the interquartile range? Explain your reasoning.
Answer:
The interquartile range (IQR) is a measure of variability, based on dividing a data set into quartiles. Quartiles divide a rank-ordered data set into four equal parts.

Question 30.
REASONING
How does an outlier affect the range of a data set? Explain.
Answer:
Outlier An extreme value in a set of data that is much higher or lower than the other numbers. Outliers affect the mean value of the data but have little effect on the median or mode of a given set of data.

Question 31.
MODELING REAL LIFE
The table shows the numbers of points scored by players on a sixth-grade basketball team in a season.
Big Ideas Math Solutions Grade 6 Chapter 9 Statistical Measures 9.4 22
a. Find the range and interquartile range of the data.
b. Identify the outlier(s) in the data set. Find the range and interquartile range of the data set without the outlier(s). Which measure does the outlier or outliers affect more?
Answer:

Question 32.
DIG DEEPER!
Two data sets have the same range. Can you assume that the interquartile ranges of the two data sets are about the same? Give an example to justify your answer.
Answer:
Yes,
A data set with the least value of 2 and the greatest value of 20 will have the same range as a data set with the least value of 82 and the greatest value of 100 will have the same range of 18.

Question 33.
MODELING REAL LIFE
The tables show the ages of the finalists for two reality singing competitions.
Big Ideas Math Solutions Grade 6 Chapter 9 Statistical Measures 9.4 23
a. Find the mean, median, range, and interquartile range of the ages for each show. Compare the results.

Answer:
18, 15, 22, 18, 24, 17, 21, 16, 28, 21
Mean:
x̄ = ∑x/n = (18 + 15 + 22 + 18 + 24 + 17 + 21 + 16 + 28 + 21)/10
=200/10 = 20
Median:
15, 16,  17,  18,  18, 21, 22, 24, 28
= (18 + 21)/2
= 39/2
= 19.5
Range:
(28 – 15)/2
= 13/2
= 6.5
interquartile range:
Number of observations: 10
Xl = 16.75
Xu = 22.5
Xu – Xl = 5.75
Ages of show B:
Mean:
x̄ = ∑x/n = (21 + 20 + 23 + 13 + 15 + 18 + 17 + 22 + 36 + 25)/10
= 210/10 = 21
Median:
13, 15, 17, 18, 20, 21, 22, 23, 25, 36
= (20 + 21)/2 = 41/2 = 20.5
Range:
(36 – 13)/2
= 23/2
= 11.5
Interquartile Range:
Samples = 10
Xl = 16.5
Xu = 23.5

b. A 21-year-old is voted off Show A, and the 36-year-old is voted off Show B. How do these changes affect the measures in part(a)? Explain.
Answer:
Mean:
x̄ = ∑x/n = (18 + 17 + 15 + 22 + 16 + 18 + 28 + 24)/8
= 158/8
= 79
Median: 15, 16, 17, 18, 18, 22, 24, 28
(18 + 18)/2
= 36/2
= 18
Range:
(28 – 15)/2
= 13/2
= 6.5
Interquartile Range:
Samples = 8
Xl = 16.25
Xu = 23.5
Interquartile Range = 23.5 – 16.25
= 7.25
21, 20, 23, 13, 15, 18, 17, 22, 25
Mean = (21 + 20 + 23 + 13 + 15 + 18 + 17 + 22 + 25)/9
= 174/2
= 87
Median:
13, 15, 17, 18, 21, 20, 22, 23, 25
21 is the median
Range:
(25 – 13)/2
= 12/2
= 6
Interquartile Range:
data = 9
Xl = 16
Xu = 22.5
(Xu – Xl) = 22.5 – 16
= 6.5
In Part A there is no effect on the range and it affects the mean, median, interquartile.

Question 34.
OPEN-ENDED
Create a set of data with 7 values that has a mean of 30, a median of 26, a range of 50, and an interquartile range of 36.
Answer:
The first thing we need to do is to put the data in increasing order. This is needed to calculate the median:
30,31,32,33,34,35,35,36,37,39

Lesson 9.5 Mean Absolute Deviation

Big Ideas Math Answer Key Grade 6 Chapter 9 Statistical Measures 9.5 1

EXPLORATION 1

Finding Distances from the Mean
Work with a partner. The table shows the exam scores of 14 students in your class.
Big Ideas Math Answer Key Grade 6 Chapter 9 Statistical Measures 9.5 2
a. Which exam score deviates the most from the mean? Which exam score deviates the least from the mean? Explain how you found your answers.
b. How far is each data value from the mean?
c. Divide the sum of the values in part(b) by the number of values. In your own words, what does this represent?
d. REASONING Ina data set, what does it mean when the value you found in part(c) is close to 0? Explain.

Big Ideas Math Answer Key Grade 6 Chapter 9 Statistical Measures 9.5 3

Another measure of variation is the mean absolute deviation. The mean absolute deviation is an average of how much data values differ from the mean.

Try It
Question 1.
Find and interpret the mean absolute deviation of the data.
5, 8, 8, 10, 13, 14, 16, 22
Answer: Number of observations : 8
Mean: 12

Question 2.
WHAT IF?
The pitcher allows 4 runs in the next game. How would you expect the mean absolute deviation to change? Explain.
Answer:

Self-Assessment for Concepts & Skills

Solve each exercise. Then rate your understanding of the success criteria in your journal.
Question 3.
WRITING
Explain why the variability of a data set can be described by the mean absolute deviation.
Answer:

Question 4.
FINDING THE MEAN ABSOLUTE DEVIATION
Find and interpret the mean absolute deviation of the data. 8, 12, 4, 3, 14, 1, 9, 13
Answer: number of observations:8
Mean: 8
mean absolute deviation: 4

Question 5.
WHICH ONE do DOESN’T BELONG?
Which one does not belong with the other three? Explain your reasoning.
Big Ideas Math Answer Key Grade 6 Chapter 9 Statistical Measures 9.5 6
Answer: MEAN
A mean is different from all the above-given factors
A mean is the simple mathematical average of a set of two or more numbers.
The mean for a given set of numbers can be computed in more than one way, including the arithmetic mean method, which uses the sum of the numbers in the series, and the geometric mean method, which is the average of a set of products.

Question 6.
The tables show the numbers of questions answered correctly by members of two teams on a game show. Compare the mean, median, and mean absolute deviation of the numbers of correct answers for each team. What can you conclude?
Big Ideas Math Answer Key Grade 6 Chapter 9 Statistical Measures 9.5 9
Answer:
Tiger sharks
3, 6, 5, 4, 4, 2
Mean: (3 + 6 + 5 + 4 + 4 + 2)/6
= 24/6
= 4
Median:
2, 3, 4, 4, 5, 6
= (4 + 4)/2
= 4
MAD:
Number of observations: 6
Mean = 4
MAD = 1
Bear Cats:
Mean:
6, 1, 4, 1, 8, 4
(6 + 1 + 4 + 1 + 8 + 4)/6
= 24/6
= 4
Median:
1, 1, 4, 4, 6, 8
= (4 + 4)/2
= 4
MAD:
Number of observations: 6
Mean = 4
MAD = 2
The mean, Median, Mean Absolute Deviation of both tiger sharks and Bear Cats are the same.

Question 7.
DIG DEEPER!
The data set shows the numbers of books that students in your book club read last summer.
8, 6, 11, 12, 14, 12, 11, 6, 15, 9, 7, 10, 9, 13, 5, 8
A new student who read 18 books last summer joins the club. Is18 an outlier? How does including this value in the data set affect the measures of center and variation? Explain.
Answer: 8 is added to the dataset.
Yes, 18 is an outliner
No, it does not affect the measures of the center and variation by removing the outliner.
If the outliner is not removed then it affects the measures of center and variation.

Mean Absolute Deviation Homework & Practice 9.5

Review & Refresh

Find the range and interquartile range of the data.
Question 1.
23, 45, 39, 34, 28, 41, 26, 33
Answer:
Number of observations:8
Lower quartile (xL): 26.5
Upper quartile (xU): 40.5
interquartile range = 14
Range:
Number of observations:8
Lowest value: 23
Highest value: 45
Range = 45 – 23
= 22

Question 2.
63, 53, 48, 61, 69, 63, 57, 72, 46
Answer:
Number of observations:9
Lower quartile (xL): 50.5
Upper quartile (xU): 66
interquartile range = 15.5
Range:
Number of observations:9
Lowest value: 46
Highest value: 72
Range = 26

Graph the integer and its opposite.
Question 3.
15
Answer:
Big Ideas Math Grade 6 Chapter 9 Statistics Answer Key img_5

Question 4.
17
Answer:
Big Ideas Math Grade 6 Chapter 9 Statistics Answer Key img_6

Question 16.
– 22
Answer:
Big Ideas Math Grade 6 Chapter 9 Statistics Answer Key img_7

Question 7.
Find the numbers of faces, edges, and vertices of the solid.
Big Ideas Math Answer Key Grade 6 Chapter 9 Statistical Measures 9.5 10
Answer:
The name of the solid is a pentagon.
Number of vertices = 5
Number of faces = 5
Numver of edges = 5

Write the word sentence as an equation.
Question 8.
17 plus a number q is 40.
Answer:
We have to write the equation for the word sentence.
The phrase ‘plus’ indicates ‘+’
17 + q = 40

Question 9.
The product of a number s and 14 is 49.
Answer:
We have to write the equation for the word sentence.
The phrase product indicates ‘×’
s × 14 = 49

Question 10.
The difference of a number b and 9 is 32.
Answer:
We have to write the equation for the word sentence.
The phrase difference indicates ‘-‘
b – 9 = 32

Question 11.
The quotient of 36 and a number g is 9.
Answer:
We have to write the equation for the word sentence.
The phrase quotient indicates ‘÷’
36 ÷ g = 9

Concepts, Skills, &Problem Solving

FINDING DISTANCES FROM THE MEAN Find the average distance of each data value in the set from the mean. (See Exploration 1, p. 439.)
Question 12.
Model years of used cars on a lot: 2014, 2006, 2009, 2011, 2005
Answer:

Question 13.
Prices of kites at a shop: $7, $20, $9, $35, $12, $15, $7, $10, $20, $25
Answer:

FINDING THE MEAN ABSOLUTE DEVIATION Find and interpret the mean absolute deviation of the data.
Question 14.
69, 51, 71, 77, 71, 80, 75, 63, 73
Answer:
Given the data
69, 51, 71, 77, 71, 80, 75, 63, 73
Number of samples = 9
Mean Absolute Deviation = 70

Question 15.
94, 86, 95, 99, 88, 90
Answer:
Given the data
94, 86, 95, 99, 88, 90
Number of samples = 6
Mean Absolute Deviation = 92

Question 16.
46, 54, 43, 57, 50, 62, 78, 42
Answer:
Given the data
46, 54, 43, 57, 50, 62, 78, 42
Number of samples = 8
Mean Absolute Deviation = 54

Question 17.
25, 28, 20, 22, 32, 28, 35, 34, 30, 36
Answer:
Given the data
25, 28, 20, 22, 32, 28, 35, 34, 30, 36
Number of samples = 10
Mean Absolute Deviation = 29

Question 18.
101, 115, 124, 125, 173, 165, 170
Answer:
Given the data
101, 115, 124, 125, 173, 165, 170
Number of samples = 7
Mean Absolute Deviation = 139

Question 19.
1.1, 7.5, 4.9, 0.4, 2.2, 3.3, 5.1
Answer:
Given the data
1.1, 7.5, 4.9, 0.4, 2.2, 3.3, 5.1
Number of samples = 7
Mean Absolute Deviation = 3.5

Question 20.
\(\frac{1}{4}, \frac{5}{8}, \frac{3}{8}, \frac{3}{4}, \frac{1}{2}\)
Answer:
Number of observations:5
Mean (x̄): 0.5
Mean Absolute Deviation (MAD): 0.15

Question 21.
4.6, 8.5, 7.2, 6.6, 5.1, 6.2, 8.1, 10.3
Answer:
Number of observations:8
Mean (x̄): 7.075
Mean Absolute Deviation (MAD): 1.45

Question 22.
YOU BE THE TEACHER
Your friend finds and interprets the mean absolute deviation of the data set 35, 40, 38, 32, 42, and 41. Is your friend correct? Explain your reasoning.
Big Ideas Math Answer Key Grade 6 Chapter 9 Statistical Measures 9.5 11
Answer:
x̄ = ∑x/n = (35 + 40 + 38)/3
= 113/3
= 37.6
Yes, the data values are different from the mean by an average of 3.

Question 23.
MODELING REAL LIFE
The data set shows the admission prices at several glass-blowing workshops.
$20, $20, $16, $12, $15, $25, $11
Find and interpret the range, interquartile range, and mean absolute deviation of the data.
Big Ideas Math Answer Key Grade 6 Chapter 9 Statistical Measures 9.5 12
Answer:
Range = (25 – 11)
= 14/2
= 7
Interquartile range:
Samples = 7
Xl = 12
Xu = 20
Xu – Xl = 20 – 12
= 8
Absolute Deviation of the data:
Data = 7
Mean = 17
Mean Absolute Deviation = 4

Question 24.
MODELING REAL LIFE
The table shows the prices of the five most-expensive and least-expensive dishes on a menu. Find the MAD of each data set. Then compare their variations.
Big Ideas Math Answer Key Grade 6 Chapter 9 Statistical Measures 9.5 13
Answer:
Five expensive dishes
$28, $30, $28, $39, $25
MAD:
Dishes = 5
Mean $30
MAD = $3.6
First leasr expensive dishes:
$7, $7, $10, $8, $12
MAD:
Dishes = 5
Mean $8.8
MAD = $1.76
Mean Absolute Deviation of five most expensive dishes is greater than Mean Absolute Deviation of five least expensive dishes.

Question 25.
REASONING
The data sets show the years of the coins in two collections.
Your collection: 1950, 1952, 1908, 1902, 1955, 1954, 1901, 1910
Your friend’s collection: 1929, 1935, 1928, 1930, 1925, 1932, 1933, 1920
Compare the measures of center and the measures of variation for each data set. What can you conclude?
Big Ideas Math Answer Key Grade 6 Chapter 9 Statistical Measures 9.5 14
Answer:
The measure of center is a value of the center or middle of a data set.
There are 4 measures of center they are
Mean
Median
Mode
Midrange
four measures of variations
Range
Interquartile range
Variance
Standard deviation
your collection:
Mean: (1950 + 1952 + 1908 + 1902 + 1955 + 1954 + 1901 + 1910)/8
= 1,929
Median: 1901, 1902, 1908, 1910, 1950, 1952, 1954, 1955
= (1910 + 1952)/2
= 1930
Mode: There is no mode
Midrange:
(1955 + 1901)/2
= 3856/2
= 1928
Range:
(1955 – 1901)/2
= 54/2
= 27
Interquartile range:
Number of observations = 8
Xl = 1903.5
Xu = 1953.5
Interquartile range = 50
Variance = 655.14
Standard deviation = 25.59

Question 26.
MODELING REAL LIFE
You survey students in your class about the numbers of movies they watched last month. A new student joins the class who watched 22 movies last month. Is22 an outlier? How does including this value affect the measures of center and the measures of variation? Explain.
Big Ideas Math Answer Key Grade 6 Chapter 9 Statistical Measures 9.5 15
Answer:

REASONING
Which data set would have the greater mean absolute deviation? Explain your reasoning.
Question 27.
guesses for number of gumballs in a jar
guesses for number of baseballs in a jar
Answer:
Gumballs in the jar have a greater mean absolute deviation because baseballs are larger than baseballs.

Question 28.
monthly rainfall amounts in a city
monthly amounts of water used in a home
Answer:

Question 29.
REASONING
Range, interquartile range, and mean absolute deviation are all measures of variation. Which measure of variation is most reliable? Explain your reasoning.
Answer:

Question 30.
DIG DEEPER!
Add and subtract the MAD from the mean in the original data set in Exercise 26.
a. What percent of the values are within one MAD of the mean? two MADs of the mean? Which values are more than twice the MAD from the mean?
b. What do you notice as you get more and more MADs away from the mean? Explain.
Answer:

Statistical Measures Connecting Concepts

Using the Problem-Solving Plan

Question 1.
Six friends play a carnival game in which a person throws darts at balloons. Each person throws the same number of darts and then records the portion of the balloons that pop. Find and interpret the mean, median, and MAD of the data.
Big Ideas Math Answers 6th Grade Chapter 9 Statistical Measures cc 1
Understand the problem.
You know that each person throws the same number of darts. You are given the portion of balloons popped by each person as a fraction, a decimal, or a percent.

Make a plan.
First, write each fraction and each decimal as a percent. Next, order the percents from least to greatest. Then find and interpret the mean, median, and MAD of the data.

Solve and check.
Use the plan to solve the problem. Then check your solution.
Answer:

Question 2.
The cost c (in dollars) to rent skis at a resort for n days is represented by the equation c = 22n. The durations of several ski rentals are shown in the table. Find the range and interquartile range of the costs of the ski rentals. Then determine whether any of the costs are outliers.
Big Ideas Math Answers 6th Grade Chapter 9 Statistical Measures cc 2
Answer:
Given the equation c = 22n
c = 22(1) = 22
c = 22(5) = 1100
c = 22(1) = 22
c = 22(3) = 66
c = 22(5) = 110
c = 22(4) = 88
c = 22(3) = 66
c = 22(12) = 264
c = 22(1) = 22
c = 22(12) = 264
c = 22(5) = 110
c = 22(7) = 154
c = 22(4) = 88
c = 22(1) = 22
22, 110, 22, 66, 110, 88, 66, 264, 22, 264, 110, 154, 88, 22
Range = (264 – 22)/2 = 242/2
= 141
Interquartile range:
Number of observations: 14
lower quartile = 22
upper quartile = 121
Interquartile range = upper quartile – lower quartile
= 121 – 22
= 99

Performance Task
Which Measure of Center Is Best: Mean, Median, or Mode?
At the beginning of this chapter, you watched a STEAM Video called “Daylight in the Big City.“ You are now ready to complete the performance task related to this video, available at BigIdeasMath.com. Be sure to use the problem-solving plan as you work through the performance task.
Big Ideas Math Answers 6th Grade Chapter 9 Statistical Measures cc 3

Statistical Measures Chapter Review

Review Vocabulary

Write the definition and give an example of each vocabulary term.
Big Ideas Math Answers 6th Grade Chapter 9 Statistical Measures cr 1

Graphic Organizers

You can use a Definition and Example Chart to organize information about a concept. Here is an example of a Definition and Example Chart for the vocabulary term statistical question.
Big Ideas Math Answers 6th Grade Chapter 9 Statistical Measures cr 2

Choose and complete a graphic organizer to help you study the concept.
Big Ideas Math Answers 6th Grade Chapter 9 Statistical Measures cr 3
1. mean
2. outlier
3. median
4. mode
5. range
6. quartiles
7. interquartile range

Chapter Self-Assessment

As you complete the exercises, use the scale below to rate your understanding of the success criteria in your journal.
Big Ideas Math Answers 6th Grade Chapter 9 Statistical Measures crr 1

9.1 Introduction to Statistics (pp. 413–418)
Learning Target: Identify statistical questions and use data to answer statistical questions.

Determine whether the question is a statistical question. Explain.
Question 1.
How many positive integers are less than 20?
Answer: There are only 19 numbers in that group

Question 2.
In what month were the students in a sixth-grade class born?
Answer: February

Question 3.
The dot plot shows the number of televisions owned by each family on a city block.
Big Ideas Math Answers 6th Grade Chapter 9 Statistical Measures crr 3
a. Find and interpret the number of data values on the dot plot.
b. Write a statistical question that you can answer using the dot plot. Then answer the question.
Answer:

Display the data in a dot plot. Identify any clusters, peaks, or gaps in the data
Question 4.
Big Ideas Math Answers 6th Grade Chapter 9 Statistical Measures crr 4
Answer:

Question 5.
Big Ideas Math Answers 6th Grade Chapter 9 Statistical Measures crr 5
Answer:

Question 6.
You conduct a survey to answer, “What is the heart rate of a typical sixth-grade student?” e table shows the results. Use the distribution of the data to answer the question.
Big Ideas Math Answers 6th Grade Chapter 9 Statistical Measures crr 6
Answer:

9.2 Mean (pp. 419–424)
Learning Target: Find and interpret the mean of a data set.

Question 7.
Find the mean of the data.
Big Ideas Math Answers 6th Grade Chapter 9 Statistical Measures crr 7
Answer:
x̄ = ∑x/n =(1112+1409+675+536+1398+162)/6
x̄ = ∑x/n=6751/6
x̄ = ∑x/n=1125.16

Question 8.
The double bar graph shows the monthly profit for two toy companies over a four-month period. Compare the mean monthly profits.
Big Ideas Math Answers 6th Grade Chapter 9 Statistical Measures crr 8
Answer:
Company A:
3.6, 3, 3.4, 4
Mean: (3.6 + 3 + 3.4 + 4)/4 = 14/4 = 3.5
Company B:
3, 4.3, 2.2, 4.1
Mean: (3 + 4.3 + 2.2 + 4.1)/4
= 13.6/4
= 3.4

Question 9.
The table shows the test scores for a class of sixth-grade students. Describe how the outlier affects the mean. Then use the data to answer the statistical question, “What is the typical test score for a student in the class?”
Big Ideas Math Answers 6th Grade Chapter 9 Statistical Measures crr 9
Answer:

9.3 Measures of Center (pp. 425–432)
Learning Target: Find and interpret the median and mode of a data set.

Find the median and mode of the data.
Question 10.
8, 8, 6, 8, 4, 5, 6
Answer:
Median:
write the given data in ascending order or descending order.
4, 5, 6, 8, 8, 8
= (6 + 8)/2
= 14/2
= 7
Mode:
8 is the mode.

Question 11.
24, 74, 61, 29, 38, 27, 68, 54
Answer:
Median:
write the given data in ascending order or descending order.
24, 74, 61, 29, 38, 27, 68, 54
= 24, 27, 29, 38, 54, 61, 68, 74
= (38 + 54)/2
= 92/2
= 48
Mode:
There is no mode in the data.

Question 12.
Find the mean, median, and mode of the data set 67, 52, 50, 99, 66, 50, and 57 with and without the outlier. Which measure does the outlier affect the most?
Answer:
Given the data,
67, 52, 50, 99, 66, 50, and 57
Mean with outliner:
(67 + 52 + 50 + 99 + 66 + 50 + 57)/7
= 441/7
= 63
Mean without outliner:
66 is the median
Mode with outliner: 50
Mode without outliner:
No mode
Outliners affect the mean value of the data but have little effect on the median or mode of a given set of data.

Question 13.
The table shows the lengths of several movies. Which measure of center best represents the data? Explain your reasoning.
Big Ideas Math Answers 6th Grade Chapter 9 Statistical Measures crr 13
Answer:

Question 14.
Give an example of a data set that does not have a median. Explain why the data set does not have a median.
Answer:

9.4 Measures of Variation (pp. 433–438)
Learning Target: Find and interpret the range and interquartile range of a data set.

Find the range of the data.
Question 15.
45, 76, 98, 21, 52, 39
Answer:
Lowest value = 21
Highest value = 98
Range = (98 – 21)/2
= 77/2
= 38.5

Question 16.
95, 63, 52, 8, 93, 16, 42, 37, 62
Answer:
Lowest value = 8
Highest value = 95
Range = (95 – 8)/2
= 87/2
= 43.5

Find the interquartile range of the data.
Question 17.
28, 46, 25, 76, 18, 25, 47, 83, 44
Answer:
Given the data
28, 46, 25, 76, 18, 25, 47, 83, 44
Number of observations: 9
lower quartile: 25
upper quartile: 61.5
Interquartile range (Xu – Xl) = 36.5

Question 18.
14, 25, 97, 55, 66, 28, 92, 38, 94
Answer:
Given the data
14, 25, 97, 55, 66, 28, 92, 38, 94
Number of observations: 9
lower quartile: 26.5
upper quartile: 93
Interquartile range (Xu – Xl) = 66.5

Question 19.
The table shows the weights of several adult emperor penguins. Find and interpret the range and interquartile range of the data. Then determine whether there are any outliers.
Big Ideas Math Answers 6th Grade Chapter 9 Statistical Measures crr 19
Answer:
25, 27, 36, 23.5, 33.5, 31.25, 30.75, 32, 24, 29.25
Yes there are outliner
Range: (36  – 25)/2
= 11/2
= 5.5
Interquartile range:
Number of observations = 10
Mean = 29.225
MAD = 3.98

Question 20.
Two data sets have the same interquartile range. Can you assume that the ranges of the two data sets are about the same? Give an example to justify your answer.
Answer:
23
Yes, a data set with the least value of 2 and the greatest value of 20 will have the same range as a data set with the least value of 82 and the greatest value of 100 will have the same range of 18.

9.5 Mean Absolute Deviation (pp. 439–444)
Learning Target: Find and interpret the mean absolute deviation of a data set.

Find and interpret the mean absolute deviation of the data.
Question 21.
Big Ideas Math Answers 6th Grade Chapter 9 Statistical Measures crr 21
Answer:
Given data,
6, 8.5, 6, 9, 10, 7, 8, 9.5
No. of observations: 8
Mean = 8
Mean Absolute Deviation: 1.25

Question 22.
Big Ideas Math Answers 6th Grade Chapter 9 Statistical Measures crr 22
Answer:
Given data,
130, 150, 190, 100, 175, 120, 165, 140, 180, 190
No. of observations: 10
Mean = 154
Mean Absolute Deviation: 26

Question 23.
The table shows the prices of the five most-expensive and least-expensive manicures given by a salon technician on a particular day. Find the MAD of each data set. Then compare their variations.
Big Ideas Math Answers 6th Grade Chapter 9 Statistical Measures crr 23
Answer:
five most-expensive:
$58, $52, $70, $49, $56
No. of observations: 5
Mean = 57
Mean Absolute Deviation: 5.6
5 least-expensive manicures:
$10, $10, $15, $10, $15
No. of observations: 5
Mean = 12
Mean Absolute Deviation: 2.4
The Mean Absolute Deviation of the five most-expensive is greater than the Mean Absolute Deviation of the 5 least-expensive manicures.

Question 24.
You record the lengths of songs you stream. The next song is 276 seconds long. Is 276 an outlier? How does including this value affect the measures of center and the measures of variation? Explain.
Big Ideas Math Answers 6th Grade Chapter 9 Statistical Measures crr 24
Answer:
Given the data,
233, 219, 163, 213, 224, 208, 225, 220, 222, 240, 228, 219, 260, 249, 209, 236,  206
The next song is 276 seconds long.
276 is the outliner.
We can remove 276 from the given data set.
So, there is no effect on the center and the measure of variations.

Statistical Measures Practice Test

Find the mean, median, mode, range, and interquartile range of the data.
Question 1.
5, 6, 4, 24, 10, 6, 9, 8
Answer:
Mean = (5 + 6 + 4 + 24 + 10 + 6 + 9 + 8)/8
= 72/8
= 9
Median:
4, 5, 6, 6, 8, 9, 10, 24
= (6 + 8)/2 = 14/2
= 7
Mode:
6 is the mode
range = (24 – 4)/2
= 20/2
= 10
Range:
Lowest value: 4
Highest value: 24
Range: 20
Interquartile range:
Lower quartile (xL): 5.25
Upper quartile (xU): 9.75
Interquartile range (xU-xL): 4.5

Question 2.
46, 27, 94, 56, 53, 65, 43
Answer:
Given the data,
46, 27, 94, 56, 53, 65, 43
Mean = (46 + 27 + 94 + 56 + 53 + 65 + 43)/7
= 16.75
Median = 15.5
Mode: There is no mode
Range:
Number of observations = 7
Lowest value: 27
Highest value: 94
Range: 67
Interquartile range:
Lower quartile (xL): 43
Upper quartile (xU): 65
Interquartile range (xU-xL): 22

Question 3.
32, 58, 19, 36, 44, 57, 11, 26, 74
Answer:
Given the data,
32, 58, 19, 36, 44, 57, 11, 26, 74
Mean = (32 + 58 + 19 + 36 + 44 + 57 + 11 + 26 + 74)/9
= 357/9
= 39.66
Median:
Arrange the data in ascending or descending order.
11, 19, 26, 32, 36, 44, 57, 58, 74
Median = 36
Mode: There is no mode in the data
Range:
Lowest value: 11
Highest value: 74
Range: 63
Interquartile range:
Lower quartile (xL): 22.5
Upper quartile (xU): 57.5
Interquartile range (xU-xL): 35

Question 4.
36, 24, 49, 32, 37, 28, 38, 40, 39
Answer:
Given the data
36, 24, 49, 32, 37, 28, 38, 40, 39
Arrange the data in ascending or descending order.
24, 28, 32, 36, 37, 38, 39, 40, 49
Mean = (24 + 28 + 32 + 36 + 37 + 28 + 38 + 40 + 49)/9
= 34.66
Median: 37
Mode: There is no mode
Range:
Lowest value: 24
Highest value: 49
Range: 25
Interquartile range:
Lower quartile (xL): 30
Upper quartile (xU): 39.5
Interquartile range (xU-xL): 9.5

Find and interpret the mean absolute deviation of the data.
Question 5.
Big Ideas Math Answers Grade 6 Chapter 9 Statistical Measures pt 5
Answer:
Given the data,
312, 286, 196, 201, 158, 225, 206, 192
Mean (x̄): 0.5
Mean Absolute Deviation (MAD): 0.15

Question 6.
Big Ideas Math Answers Grade 6 Chapter 9 Statistical Measures pt 6
Answer:
Given the data,
15, 8, 19, 20, 18, 20, 22, 14, 10, 15
Mean (x̄): 16.1
Mean Absolute Deviation (MAD): 3.7

Question 7.
You conduct a survey to answer, “How many Times (minutes)minutes does it take a typical sixth-grade student to run a mile?” The table shows the results. Use the distribution of the data to answer the question.
Big Ideas Math Answers Grade 6 Chapter 9 Statistical Measures pt 7
Answer:

Question 8.
The table shows the weights of Alaskan malamute 8181808281dogs at a veterinarian’s office. Which measure of center best represents the weight of an Alaskan malamute? Explain your reasoning.
Big Ideas Math Answers Grade 6 Chapter 9 Statistical Measures pt 8
Answer:

Question 9.
The table shows the numbers of guests Numbers of Guests at a hotel on different days.
Big Ideas Math Answers Grade 6 Chapter 9 Statistical Measures pt 9
a. Find the range and interquartile range of the data.
b. Use the interquartile range to identify the outlier(s) in the data set. Find the range and interquartile range of the data set without the outlier(s). Which measure did the outlier or outliers affect more?
Answer:

Question 10.
The data sets show the numbers of hours worked each week by two people for several weeks.
Person A: 9, 18, 12, 6, 9, 21, 3, 12
Person B: 12, 18, 15, 16, 14, 12, 15, 18
Compare the measures of center and the measures of variation for each data set. What can you conclude?
Answer:

Question 11.
The table shows the lengths of several bearded dragons captured for a study. Find the mean, median, and mode of the data in centimeters and in inches. How does converting to inches affect the mean, median, and mode?
Big Ideas Math Answers Grade 6 Chapter 9 Statistical Measures pt 11
Answer:

Statistical Measures Cumulative Practice

Question 1.
Which statement can be represented by a negative integer?
A. The temperature rises 15 degrees.
B. A hot-air balloon ascends 450 yards.
C. You earn $50 completing chores.
D. A submarine submerges 260 feet.
Big Ideas Math Answers Grade 6 Chapter 9 Statistical Measures cp 1
Answer: D. A submarine submerges 260 feet.

Question 2.
What is the height h (in inches) of the prism?
Big Ideas Math Answers Grade 6 Chapter 9 Statistical Measures cp 2
Answer:
h = v/lw
h = 5850/30(12 1/4)
h = 5850/(30 × 12.25)
h = 5850/367.50
h = 15.91 inches

Question 3.
Which is the solution of the inequality \(\frac{2}{3}\)x < 6?
F. x < 4
G. x < 5\(\frac{1}{3}\)
H. x < 6\(\frac{2}{3}\)
I. x < 9
Answer: I. x < 9

Question 4.
The number of hours that each of six students spent reading last week is shown in the bar graph.
Big Ideas Math Answers Grade 6 Chapter 9 Statistical Measures cp 4
For the data in the bar graph, which measure is the?
A. mean
B. median
C. mode
D. range
Answer: C. mode

Explanation:
In the above bar graph, 10 is repeated two ways.
Thus the correct answer is option C.

Question 5.
Which list of numbers is in order from least to greatest?
F. – 5.41, – 3.6, – 3.2, – 3.06, – 1
G. – 1, – 3.06, – 3.2, – 3.6, – 5.41
H. – 5.41, – 3.06, – 3.2, – 3.6, – 1
I. – 1, – 3.6, – 3.2, – 3.06, – 5.41
Answer: F. – 5.41, – 3.6, – 3.2, – 3.06, – 1

Explanation:
We have to write the numbers from least to greatest
The negative sign with the highest number will be the least.
– 5.41, – 3.6, – 3.2, – 3.06, – 1
Thus the correct answer is option F.

Question 6.
What is the mean absolute deviation of the data shown in the dot plot, rounded to the nearest tenth?
Big Ideas Math Answers Grade 6 Chapter 9 Statistical Measures cp 6
A. 1.4
B. 3
C. 3.2
D. 57.
Answer:
Data from the dot plot
5, 5, 4, 4, 6, 1
Number of observations: 6
Mean = 4.166
Mean absolute deviation = 1.66
Thus the correct answer is option A.

Question 7.
A family wants to buy tickets to a theme park. There are separate ticket prices for adults and children.
Big Ideas Math Answers Grade 6 Chapter 9 Statistical Measures cp 7
Which expression represents the total cost (in dollars) for adult tickets c and child tickets?
F. 600 (a + c)
G. 50(a × c)
H. 30a + 20c
I. 30a × 20c
Answer: H. 30a + 20c

Question 8.
The dot plot shows the leap distances (in feet) of a tree frog. How many leaps were recorded?
Big Ideas Math Answers Grade 6 Chapter 9 Statistical Measures cp 8
Answer: 7 leaps were recorded

Question 9.
What is the value of the expression when a = 6 and b = 14?
0.8a + 0.02b
A. 0.4828
B. 0.8814
C. 5.08
D. 16.4
Answer:
Given the expression,
0.8a + 0.02b
a = 6
b = 14
0.8(6) + 0.02(14)
4.8 + 0.28
= 5.08
Thus the correct answer is option C.

Question 10.
Which property was not used to simplify the expression?
Big Ideas Math Answers Grade 6 Chapter 9 Statistical Measures cp 10
F. Distributive Property
G. Associative Property of Addition
H. Multiplication Property of One
I. Commutative Property of Multiplication
Answer: I. Commutative Property of Multiplication

Question 11.
What are the coordinates of Point P?
Big Ideas Math Answers Grade 6 Chapter 9 Statistical Measures cp 11
A. (- 3, – 2)
B. (3, – 2)
C. (- 2, – 3)
D. (-2, 3)
Answer: B. (3, – 2)

Explanation:
By seeing the above graph we can write the ordered pair P.
the x-axis is on 3 and the y-axis is on -2
Thus the correct answer is option B.

Question 12.
Create a data set with 5 numbers that has the following measures.
Think
Solve
Explain
• a mean of 7
• a median of 9
Explain how you created your data set.
Answer:
The data set is 3, 2, 9, 1, 20

Final Words:

We hope that the article on Big Ideas Math Answers Grade 6 Chapter 9 Statistical Measures helps you in all over preparation. If you are lagging in this concept, then you can check the above material. Feel free to post your doubts or comments in the comments section. If you need solutions to any of the questions, then you can ask us in the comment section itself. We clear all your doubts as early as possible. Stay tuned to our site to get more updates on BIM All Grades materials.

Big Ideas Math Answers Grade 7 Chapter 9 Geometric Shapes and Angles

Big Ideas Math Answers Grade 7 Chapter 9 Geometric Shapes and Angles

Get Big Ideas Math Answer Key for Grade 7 Chapter 9 Geometric Shapes and Angles Pdf here. We are providing free download links to all problems in the upcoming sections. You can understand the concept easily as we are explaining the concept with so many real-time examples. Candidates can improve their problem-solving skills and analytical thinking with the help of Big Ideas Math Answers Grade 7 Chapter 9 Geometric Shapes and Angles.

BIM Grade 7 Chapter 9 Geometric Shapes and Angles Answer Key pdf. Download all the pdf links from here for free of cost and kickstart your exam preparation. Follow the various concepts and solve as many problems as before going to the exam. We are providing the preparatory material for the candidates to score better marks in the exam. Grade 7 Big Ideas Math Answers is providing amazing tips and tricks in the next sections. Check it out!!

Big Ideas Math Book 7th Grade Answer Key 9 Geometric Shapes and Angles

If it is some theory, then you can easily learn it but if it is problematic, then you must have perfection and grip on the subject. Therefore, we are providing a lot of practice material with solutions. First, read the question and try to solve the problems without looking at the solutions. Then refer to the solution and know if it is correct or not. Also, check if it the easiest process or not. With the help of professionals and math experts, we gave the solution to the problems in the easiest methods.

You need not be the expert in all the topics but you must check the easy topics like Geometric Shapes and Angles and prepare perfectly. These topics act as a scoring factor because they are easy to understand and solve. If you have any further doubts, go through Big Ideas Math Answers Grade 7 Chapter 9 Geometric Shapes and Angles, you will get a clear idea about every detail.

Performance Task

Lesson: 1 Circle and Circumference

Lesson: 2 Areas of Circles

Lesson: 3 Perimeters and Areas of Composite Figures

Lesson: 4 Constructing Polygons

Lesson: 5 Finding Unknown Angle Measures

Chapter 9 – Geometric Shapes and Angles

Geometric Shapes and Angles STEAM Video/Performance Task

STEAM Video

Track and Field
Different lanes on a race track have different lengths. How can competitors run in different lanes and have the same finish line?

Watch the STEAM Video “Track and Field.” Then answer the following questions.
1. A track consists of a rectangle and two semicircles. The dimensions of the rectangle formed by the innermost lane are shown. What is the distance around each semicircle on the 400-meter, innermost lane?
Big Ideas Math Answer Key Grade 7 Chapter 9 Geometric Shapes and Angles 1
2. How does the width of the rectangle, 63.7 meters, compare to the distance around each semicircle? Explain.

Answer:
1. The distance around each semicircle on the 400-meter, innermost lane = 488 m
2. The distance around each semicircle = 90π + 320

Explanation:
1. The inside perimeter of the track = 400 m
the total length of the two straight portions = 90 + 90 = 180
therefore the length of the remaining portion = 400-180 = 220 m
circumference of the two remaining semi-circular portions = πr + πr = 2πr
2πr = 220
2 x 3.14 x r = 220
r = 35 m
Area of the track = 2 x 90 x 14 +3.14 x (49) x (49) – (35) x (35)
area of the track = 6216 square meter
length of the outer running track = 488 m
2. The perimeter of the track is the two circumferences of the circumferences.
The diameters of the circle and the width of the rectangle = 90 m
90 π + 320
Performance Task.
Finding the Area and Perimeter of a Track
After completing this chapter, you will be able to use the concepts you learned to answer the questions in the STEAM Video Performance Task. You will be given the dimensions of a race track.
Big Ideas Math Answer Key Grade 7 Chapter 9 Geometric Shapes and Angles 2
You will be asked to solve various perimeter and area problems about the track. Given a race track, what measures do you need to find the outer perimeter?

Answer:
The outer perimeter = 11,2610 sq m

Explanation:
perimeter of the semicircle = (π + 2 ) r
p = (3.14 + 2)36.5
p= (3.16) 36.5
p = 11,2610 sq m

Geometric Shapes and Angles Getting Ready for Chapter 9

Chapter Exploration
Work with a partner.
Question 1.
Perform the steps for each of the figures.

  • Measure the perimeter of the larger polygon to the nearest millimeter.
  • Measure the diameter of the circle to the nearest millimeter.
  • Measure the perimeter of the smaller polygon to the nearest millimeter.
  • Calculate the value of the ratio of the two perimeters to the diameter.
  • Take the average of the ratios. This average is the approximation of π(the Greek letter ).
    Big Ideas Math Answer Key Grade 7 Chapter 9 Geometric Shapes and Angles 3

Question 2.
Based on the table, what can you conclude about the value of π? Explain your reasoning.

Answer:
The value of π = 3.14

Explanation:
We can consider 3 values for the π.
they are (22/7) or 3.14
so i am considering the 3.14

Question 3.
The Greek mathematician Archimedes used the above procedure to approximate the value of π. He used polygons with 96 sides. Do you think his approximation was more or less accurate than yours? Explain your reasoning.
Answer:
The greek mathematician used polygons with the side of polygons as 12,14,48, and finally 96 sides.

Explanation:
The greek mathematician used polygons with the side of polygons as 12,14,48, and finally 96 sides.
yes the accuration is more than i think.

Vocabulary
The following vocabulary terms are defined in this chapter. Think about what each term might mean and record your thoughts.
diameter of a circle
semi circle
adjacent angles
circumference
composite figure
vertical angles

Answer:
The diameter of the circle = the diameter is the length of the line through the center that touches two points on the edge of the circle.
semi circle =  semicircle is a one-dimensional locus of points that forms half of the circle.
adjacent angles = adjacent angles are two angles that have a common vertex and a common side but do not overlap.
circumference = the circumference is the perimeter of the circle. the circumference would be the arc length of the circle.
composite figure = a figure that consists of two or more geometric shapes.
vertical angles = a pair of non-adjacent angles form when two lines intersect.

Explanation:
The diameter of the circle = the diameter is the length of the line through the center that touches two points on the edge of the circle.
semi circle =  semicircle is a one-dimensional locus of points that forms half of the circle.
adjacent angles = adjacent angles are two angles that have a common vertex and a common side but do not overlap.
circumference = the circumference is the perimeter of the circle. the circumference would be the arc length of the circle.
composite figure = a figure that consists of two or more geometric shapes.
vertical angles = a pair of non-adjacent angles form when two lines intersect.

Lesson 9.1 Circles and Circumference

EXPLORATION 1

Using a Compass to Draw a Circle
Work with a partner. Set a compass to 2 inches and draw a circle.
Big Ideas Math Answer Key Grade 7 Chapter 9 Geometric Shapes and Angles 9.1 1
a. Draw a line from one side of the circle to the other that passes through the center. What is the length of the line? This is called the diameter of the circle.
b. Estimate the distance around the circle. This is called the circumference of the circle. Explain how you found your answer.

Answer:
a. the length of the line = 4 inches
b.  The circumference of the circle = 12.56 inch

Explanation:
a. In the question they said that 2 inches
the length of the line = 4 in
b. the circumference of the circle = 2π r
circle = 2 x 3.14 x 2
circle = 12.56 in

EXPLORATION 2

Exploring Diameter and Circumference
Work with a partner.
a. Roll a cylindrical object on a flat surface to find the circumference of the circular base.
Big Ideas Math Answer Key Grade 7 Chapter 9 Geometric Shapes and Angles 9.1 2
b. Measure the diameter of the circular base. Which is greater, the diameter or the circumference? how many times greater?
c. Compare your answers in part(b) with the rest of the class. What do you notice?
d. Without measuring, how can you find the circumference of a circle with a given diameter? Use your method to estimate the circumference of the circle in Exploration 1.

Answer:
a. The circumference of the circle = 2πr
b. The circumference of the circle is 3.14 times greater than the diameter of the circle.
c. The circumference of the circle is greater than the diameter of the circle.
d. The diameter of the circle = 2r and the circumference of the circle = 2πr

Explanation:
a. The circumference of the circle = 2πr
b. The circumference of the circle is 3.14 times greater than the diameter of the circle.
c. The circumference of the circle is greater than the diameter of the circle.
d. The diameter of the circle = 2r and the circumference of the circle = 2πr

Big Ideas Math Answer Key Grade 7 Chapter 9 Geometric Shapes and Angles 9.1 3

Try It

Question 1.
The diameter of a circle is 16 centimeters. Find the radius.

Answer:
radius = 8 cm

Explanation:
The diameter of the circle = 2r
16 = 2r
r = 8 cm

Question 2.
The radius of a circle is 9 yards. Find the diameter.

Answer:
The diameter = 18 yds

Explanation:
The diameter of the circle = 2r
diameter = 2 x 9
r = 18 yds

Find the circumference of the object. Use 3.14 or \(\frac{22}{7}\) for π.
Question 3.
Big Ideas Math Answer Key Grade 7 Chapter 9 Geometric Shapes and Angles 9.1 4

Answer:
circumference = 12.56 cm

Explanation:
circumference of the circle =2πr
circle = 2 x 3.14 x 2 where r = 2cm given
circle = 6.28 x 2
circle = 12.56 cm

Question 4.
Big Ideas Math Answer Key Grade 7 Chapter 9 Geometric Shapes and Angles 9.1 5

Answer:
circumference =43.96 square feet

Explanation:
circumference of the circle =2πr
circle = 2 x 3.14 x 7 where r = 7 ft given
circle = 6.28 x 7
circle = 43.96 square feet

Question 5.
Big Ideas Math Answer Key Grade 7 Chapter 9 Geometric Shapes and Angles 9.1 6

Answer:
circumference =28.26 square in

Explanation:
circumference of the circle =2πr
circle = 2 x 3.14 x 4.5 where r = 4.5 is given
circle = 6.28 x 4.5
circle =28.26 in

Find the perimeter of the semicircular region.
Question 6.
Big Ideas Math Answer Key Grade 7 Chapter 9 Geometric Shapes and Angles 9.1 7

Answer:
perimeter of the semicircle =  5.14 ft

Explanation:
perimeter of the semicircle = (π + 2 ) r
perimeter = (3.14 + 2) 1 diameter = 2 given r= 1
perimeter = 5.14 feet

Question 7.
Big Ideas Math Answer Key Grade 7 Chapter 9 Geometric Shapes and Angles 9.1 8

Answer:
perimeter of the semicircle =  17.99 cm

Explanation:
perimeter of the semicircle = (π + 2 ) r
perimeter = (3.14 + 2) 3.5 diameter = 7 given r= 3.5
perimeter = 17.99 cm

Question 8.
Big Ideas Math Answer Key Grade 7 Chapter 9 Geometric Shapes and Angles 9.1 9

Answer:
the perimeter of the semicircle =  33.14 in

Explanation:
perimeter of the semicircle = (π + 2 ) r
perimeter = (3.14 + 2) 15 given r= 15
perimeter = 33.14 in

Self-Assessment for Concepts & Skills
Solve each exercise. Then rate your understanding of the success criteria in your journal.

Question 9.
WRITING
Are there circles for which the value of the ratio of circumference to diameter is not equal to π? Explain.

Answer:
circumference to diameter is equal to π

Explanation:
d. The diameter of the circle = 2r and the circumference of the circle = 2πr
circumference to diameter is equal to π

Question 10.
FINDING A PERIMETER
Find the perimeter of a semicircular region with a straight side that is 8 yards long.

Answer:
perimeter = 11.14 yd

Explanation:
perimeter of the semicircle = (π + 2 ) r
perimeter = (3.14 + 2) 4 given r= 4
perimeter = 11.14 yd
Question 11.
DIFFERENT WORDS, SAME QUESTION
Which is different? Find “both” answers.
Big Ideas Math Answer Key Grade 7 Chapter 9 Geometric Shapes and Angles 9.1 10

Answer:
What is π times the radius?
What is π times the diameter?

Explanation:
the radius of the circle = (c/2 π )
the diameter of the circle =  2r
Self-Assessment for Problem Solving
Solve each exercise. Then rate your understanding of the success criteria in your journal.

Question 12.
The wheels of a monster truck are 66 inches tall. Find the distance the monster truck travels when the tires make one 360-degree rotation.
Big Ideas Math Answer Key Grade 7 Chapter 9 Geometric Shapes and Angles 9.1 11

Answer:
The distance = 207.35 inches

Explanation:
The wheel is in the shape of a circle.
diameter = 66 given
radius = (66/2)
radius = 33
The circumference = 2πr
c = 2 x 3.14 x 33
c = 6.28 x 33
c = 207.24 in

Question 13.
DIG DEEPER!
The radius of a dog’s collar should be at least 0.5 inch larger than the radius of the dog’s neck. A dog collar adjusts to a circumference of 10 to 14 inches. Should the collar be worn by a dog with a neck circumference of 12.5 inches? Explain.
Big Ideas Math Answer Key Grade 7 Chapter 9 Geometric Shapes and Angles 9.1 12

Answer:
No, the collar should not be worn by this dog.

Explanation:
Given that the collar should be at least 0.5 inches.
dog collar adjusts to a circumference of 10 to 14 inches.

Question 14.
You resize a picture so that the radius of the midday Sun appears four times larger. How much larger does the circumference of the Sun appear? Explain.

Answer:
4 times larger

Explanation:
they said that if they resize it for 4 times.
therefore the sun appears 4 times larger.

Circles and Circumference Homework & Practice 9.1

Review & Refresh

Two jars each contain 1000 numbered tiles. The double box-and-whisker plot represents a random sample of 10 numbers from each jar.
Big Ideas Math Answer Key Grade 7 Chapter 9 Geometric Shapes and Angles 9.1 13
Question 1.
Compare the samples using measures of center and variation.

Answer:
a. Jar A = median 3, starting 2.
b. Jar B = median  6, starting 2

Explanation:
In the above-given figure, the jar A is starting from 2
jar A contains median = 3
the jar B is starting from 2
jar B contains median = 6

Question 2.
Can you determine which jar contains greater numbers? Explain.

Answer:
Jar B

Explanation:
jar B contains the numbers from 4 to 9
Question 3.
Find the percent of change from 24 to 18.
A. 25% decrease
B. 25% increase
C. 75% increase
D. 75% decrease

Answer:
option A is correct

Explanation:
if the percent of jar changes from 24 to 18
the decrease in the percent = 25

Concepts, Skills, & Problem Solving
EXPLORING DIAMETER AND CIRCUMFERENCE Estimate the circumference of the circular base of the object. (See Exploration 2, p. 361.)
Question 4.
tube of lip balm with radius 0.5 mm

Answer:
c = 3.14 mm

Explanation:
circumference of the circle =2πr
circle = 2 x 3.14 x 0.5 where r = 0.5 mm given
circle = 6.28 x 0.5
circle =3.14 mm

Question 5.
D battery with radius 0.65 in.

Answer:
c = 4.082 in

Explanation:
circumference of the circle =2πr
circle = 2 x 3.14 x 0.65 where r = 0.65 ingiven
circle = 6.28 x 0.65
circle =4.082 in

FINDING A RADIUS Find the radius of the button.
Question 6.
Big Ideas Math Answer Key Grade 7 Chapter 9 Geometric Shapes and Angles 9.1 14

Answer:
radius =2.5 cm

Explanation:
radius = (5/2)
radius = 2.5 cm

Question 7.
Big Ideas Math Answer Key Grade 7 Chapter 9 Geometric Shapes and Angles 9.1 15

Answer:
radius =14 mm

Explanation:
radius = (28/2)
radius = 14 mm

Question 8.
Big Ideas Math Answer Key Grade 7 Chapter 9 Geometric Shapes and Angles 9.1 16

Answer:
radius =1.75 in

Explanation:
radius = (3.5/2)
radius = 1.75 in

FINDING A DIAMETER Find the diameter of the object.
Question 9.
Big Ideas Math Answer Key Grade 7 Chapter 9 Geometric Shapes and Angles 9.1 17

Answer:
diameter = 4 in

Explanation:
diameter of the circle = 2r
where r = 2 given
d = 4 in

Question 10.
Big Ideas Math Answer Key Grade 7 Chapter 9 Geometric Shapes and Angles 9.1 18

Answer:
diameter = 0.64 ft

Explanation:
diameter of the circle = 2r
where r = 0.8 given
d = 0.64 ft

Question 11.
Big Ideas Math Answer Key Grade 7 Chapter 9 Geometric Shapes and Angles 9.1 19

Answer:
diameter = 1.2 cm

Explanation:
diameter of the circle = 2r
where r = 0.6 given
d = 1.2 cm

FINDING A CIRCUMFERENCE Find the circumference of the object. Use 3.14 or \(\frac{22}{7}\) for π.
Question 12.
Big Ideas Math Answer Key Grade 7 Chapter 9 Geometric Shapes and Angles 9.1 20

Answer:
c = 43.96 in

Explanation:
circumference of the circle =2πr
circle = 2 x 3.14 x 7 where r = 7 ingiven
circle = 6.28 x 7
circle =43.96 in

Question 13.
Big Ideas Math Answer Key Grade 7 Chapter 9 Geometric Shapes and Angles 9.1 21

Answer:
c = 18.84 cm

Explanation:
circumference of the circle =2πr
circle = 2 x 3.14 x 7 where r = 3 cmgiven
circle = 6.28 x 3
circle =18.84 cm

Question 14.
Big Ideas Math Answer Key Grade 7 Chapter 9 Geometric Shapes and Angles 9.1 22

Answer:
c = 6.28 mm

Explanation:
circumference of the circle =2πr
circle = 2 x 3.14 x 1where r = 1mgiven
circle = 6.28 x 1
circle =6.28 m

FINDING THE PERIMETER OF A SEMICIRCULAR REGION Find the perimeter of the window.
Question 15.
Big Ideas Math Answer Key Grade 7 Chapter 9 Geometric Shapes and Angles 9.1 23

Answer:
perimeter = 7.71 ft

Explanation:
perimeter of the semicircle = (π + 2 ) r
perimeter = (3.14 + 2) 1.5 given d =3 ,r = (d/2)
perimeter = 7.71 ft

Question 16.
Big Ideas Math Answer Key Grade 7 Chapter 9 Geometric Shapes and Angles 9.1 24

Answer:
perimeter = 64.8 cm

Explanation:
perimeter of the semicircle = (π + 2 ) r
perimeter = (3.14 + 2) 20 given  ,r = 20 cm
perimeter = 64.8 cm

ESTIMATING A RADIUS Estimate the radius of the object.
Question 17.
Big Ideas Math Answer Key Grade 7 Chapter 9 Geometric Shapes and Angles 9.1 25

Answer:
Radius = 1.417 mm

Explanation:
radius of the circle = (c/2π )
r = (8.9/6.28)
r = 1.417 mm

Question 18.
Big Ideas Math Answer Key Grade 7 Chapter 9 Geometric Shapes and Angles 9.1 26

Answer:
Radius = 19.426 in

Explanation:
radius of the circle = (c/2π )
r = (122/6.28)
r = 19.426 in

Question 19.
MODELING REAL LIFE
A circular sinkhole has a circumference of 75.36 meters. A week later, it has a circumference of 150.42 meters.
a. Estimate the diameter of the sinkhole each week.
b. How many times greater is the diameter of the sinkhole a week later?

Answer:
a. The diameter of the sinkhole each week = 4 in
b. 2 times greater is the diameter of the sinkhole a week later

Explanation:
a. The diameter of the sinkhole each week = 75.36 m
b. 2 times greater is the diameter of the sinkhole a week later
75.36 x 75.36 = 150.42 m
Question 20.
REASONING
Consider the circles A, B, C, and D.
Big Ideas Math Answer Key Grade 7 Chapter 9 Geometric Shapes and Angles 9.1 27
a. Without calculating, which circle has the greatest circumference? Explain.
b. Without calculating, which circle has the least circumference? Explain.

Answer:
a. option D has the greatest circumference.
b. option C has the least circumference.

Explanation:
D. circumference of the circle =2πr
circle = 2 x 3.14 x 50where r =50 ingiven
circle = 6.28 x 50
circle =314 in
Explanation:
C. circumference of the circle =2πr
circle = 2 x 3.14 x 1where r = 1given
circle = 6.28 x1
circle = 6.28
Explanation:
A. circumference of the circle =2πr
circle = 2 x 3.14 x 4 where r = 4given
circle = 6.28 x 4
circle = 25.12
Explanation:
A. circumference of the circle =2πr
circle = 2 x 3.14 x 10 where r = 10given
circle = 6.28 x 10
circle = 62.8

FINDING CIRCUMFERENCES Find the circumferences of both circles.
Question 21.
Big Ideas Math Answer Key Grade 7 Chapter 9 Geometric Shapes and Angles 9.1 28

Answer:
circumference of inside circle  =31.4 square cm
circumference of outside circle = 62.8  square cm

Explanation:
circumference of the inside circle =2πr
circle = 2 x 3.14 x 5 where r = 5 cm given
circle = 6.28 x 5
circle = 31.4 square cm
circumference of the outside circle =2πr
circle = 2 x 3.14 x 2 where r = 2 cm given
circle = 6.28 x 2
circle = 62.8 square cm

Question 22.
Big Ideas Math Answer Key Grade 7 Chapter 9 Geometric Shapes and Angles 9.1 29

Answer:
circumference of inside circle  =28.26 ft
circumference of outside circle = 31.4 square cm

Explanation:
circumference of the inside circle =2πr
circle = 2 x 3.14 x 4.5 where r = 4.5 feet given
circle = 6.28 x 4.5
circle = 28.26 ft
circumference of the outsideside circle =2πr
circle = 2 x 3.14 x 2.5 where r = 2.5 ft given
circle = 6.28 x 2.5
circle = 15.7 square ft

Question 23.
Big Ideas Math Answer Key Grade 7 Chapter 9 Geometric Shapes and Angles 9.1 30

Answer:
circumference of inside circle  =69.08  m
circumference of outside circle = 138.16 m

Explanation:
circumference of the inside circle =2πr
circle = 2 x 3.14 x 5.5 where r = 5.5 feet given
circle = 6.28 x 5.5
circle = 69.08 m
circumference of the outsideside circle =2πr
circle = 2 x 3.14 x 22 where r = 22given
circle = 6.28 x 22
circle = 138.16 m

Question 24.
MODELING REAL LIFE
A satellite is in an approximately circular orbit 36,000 kilometers from Earth’s surface. The radius of Earth is about 6400 kilometers. What is the circumference of the satellite’s orbit?
Big Ideas Math Answer Key Grade 7 Chapter 9 Geometric Shapes and Angles 9.1 31

Answer:
c = 40,192 km

Explanation:
circumference of the  satellite orbit =2πr
circle = 2 x 3.14 x 6400where r = 6400kmgiven
circle = 6.28 x 6400
circle =40,192km

Question 25.
STRUCTURE
The ratio of circumference to diameter is the same for every circle. Is the ratio of circumference to radius the same for every circle? Explain.

Answer:
The ratio of circumference to radius is  same for every circle.

Explanation:
c/r = 2πr/r
where r get canceled in both numerator and denominator.
c/r = 2π
radius = (c/2π)
the radius is same for every circle.

Question 26.
PROBLEM SOLVING
A wire is bent to form four semicircles. How long is the wire? Justify your answer.
Big Ideas Math Answer Key Grade 7 Chapter 9 Geometric Shapes and Angles 9.1 32

Answer:
The wire is 128 cm long

Explanation:
Given that the four semicircles are 32 cm
32 + 32 + 32 + 32 = 64

Question 27.
CRITICAL THINKING
Explain how to draw a circle with a circumference of π2 inches. Then draw the circle.

Answer:

Explanation:
circumference of circle = 2πr
c = π2

Question 28.
DIG DEEPER!
“Lines” of latitude on Earth are actually circles. The Tropic of Cancer is the northernmost line of latitude at which the Sun appears directly overhead at noon. The Tropic of Cancer has a radius of 5854 kilometers.
To qualify for an around-the-world speed record, a pilot must cover a distance no less than the circumference of the Tropic of Cancer, cross all meridians, and land on the same air field where the flight began.
Big Ideas Math Answer Key Grade 7 Chapter 9 Geometric Shapes and Angles 9.1 33
a. What is the minimum distance that a pilot must fly to qualify for an around-the-world speed record?
b. RESEARCH Estimate the time it will take for a pilot to qualify for the speed record. Explain your reasoning.

Answer:
a. The minimum distance that a pilot must fly to qualify for an around the world-speed record = 18.3376 km
b. The pilot will take for the speed record = 18.3376 km

Explanation:
a. The minimum distance that a pilot must fly to qualify for an around the world-speed record = 18.3376 km
b. The pilot will take for the speed record = 18.3376 km
Question 29.
PROBLEM SOLVING
Bicycles in the late 1800s looked very different than they do today.
Big Ideas Math Answer Key Grade 7 Chapter 9 Geometric Shapes and Angles 9.1 34
a. How many rotations does each tire make after traveling 600 feet? Round your answers to the nearest whole number.
b. Would you rather ride a bicycle made with two large wheels or two small wheels? Explain.

Answer:
a. The rotations each tire make after traveling 600 feet = 188.4 in in
b. two large wheels = 376.8 in
two small wheels = 113.04 in

Explanation:
the rotations each tire make after travelling = 2 x 3.14 x 30  = 188.4 in
b. two large wheels = 188.4 x 2 = 376.8 in
for two small wheels = 113.04 in

Question 30.
LOGIC
The length of the minute hand is 150% of the length of the hour hand.
Big Ideas Math Answer Key Grade 7 Chapter 9 Geometric Shapes and Angles 9.1 35
a. What distance will the tip of the minute hand move in 45 minutes? Justify your answer.
b. In 1 hour, how much farther does the tip of the minute hand move than the tip of the hour hand? Explain how you found your answer.

Answer:
The distance will the tip of minute hand move in 45 minutes = 140 %
b. the tip of the minute hand moves 60 times faster than hour hand.

Explanation:
The distance will the tip of minute hand move in 45 minutes = 140 %
b. the tip of the minute hand moves 60 times faster than hour hand.

Lesson 9.2 Areas of Circles

EXPLORATION 1

Estimating the Area of a Circle
Work with a partner. Each grid contains a circle with a diameter of 4 centimeters. Use each grid to estimate the area of the circle. Which estimate should be closest to the actual area? Explain.
Big Ideas Math Answers 7th Grade Chapter 9 Geometric Shapes and Angles 9.2 1

Answer:
Area of 1st circle = 200.96 cm
Area of 2nd circle =803.84 cm
Area of 3rd circle =3215.36 cm

Explanation:
area of 1st circle = πr x r
area = 3.14 x 8 x 8
a = 200.96 cm
area of 2nd circle = πr x r
area= 3.14 x 16 x 16
a = 803.84 cm
area of 3rd circle = πr x r
area= 3.14 x 32 x 32
a = 3215.36
EXPLORATION 2

Writing a Formula for the Area of a Circle
Work with a partner. A student draws a circle with radius and divides the circle into 24 equal sections. The student cuts out each section and arranges the sections to form a shape that resembles a parallelogram.
Big Ideas Math Answers 7th Grade Chapter 9 Geometric Shapes and Angles 9.2 2
a. Use the diagram to write a formula for the area of a circle in terms of the radius r. Explain your reasoning.Describe the relationship between the radius and the area of a circle.
b. Use the formula to check your estimates in Exploration 1.

Answer:
a. the area of the circle = 1808.64
b. the area of the circle in terms of radius r = 0.0084 cm

Explanation:
The area  of circle = πr x r
a = 3.14 x 24 x 24
a = 1808.64 cm
The radius of the circle = (c/2 π)
circumference = 2πr
c = 2 x 3.14 x 24
c = 150.72 cm
area = (150.72/6.28)
area = 0.0084 cm

Try It
Question 1.
Find the area of a circle with a radius of 6 feet. Use 3.14 for π.

Answer:
The area  of circle = 113.04 sq ft

Explanation:
The area  of circle = πr x r
a = 3.14 x 6 x 6
a = 113.04 square feet
Question 2.
Find the area of a circle with a diameter of 28 meters.Use \(\frac{22}{7}\) for π.

Answer:
The area  of circle = 175.84 sq meters

Explanation:
The area  of circle = πr x r
a = 3.14 x 14 x 14 where d = 28 so r = 14
a = 175.84 square meters

Find the area of the semicircle.
Question 3.
Big Ideas Math Answers 7th Grade Chapter 9 Geometric Shapes and Angles 9.2 3

Answer:
Area of semicircle =62.07 sq cm

Explamation:
Area of semicircle =( π+r x r/2)
area =( 3.14 +121/2)
area =(121.14/2)
area = 62.07 sq cm

Question 4.
Big Ideas Math Answers 7th Grade Chapter 9 Geometric Shapes and Angles 9.2 4

Answer:
Area of semicircle =9.57 sqm

Explamation:
Area of semicircle =( π+r x r/2)
area =( 3.14 +16/2)
area =(19.14/2)
area = 9.57 sq m

Question 5.
Big Ideas Math Answers 7th Grade Chapter 9 Geometric Shapes and Angles 9.2 5

Answer:
Area of semicircle =4.695 sq yd

Explanation:
Area of semicircle =( π+r x r/2)
area =( 3.14 +6.25/2)
area =(9.39/2)
area = 4.695 sq yd

Self-Assessment for Concepts & Skills
Solve each exercise. Then rate your understanding of the success criteria in your journal.

Question 6.
ESTIMATING AN AREA
The grid contains a circle with a diameter of 2 centimeters. Use the grid to estimate the area of the circle. How can you change the grid to improve your estimate? Explain.
Big Ideas Math Answers 7th Grade Chapter 9 Geometric Shapes and Angles 9.2 6

Answer:
The area  of circle = 50.24 sq centi meters

Explanation:
The area  of circle = πr x r
a = 3.14 x 4 x 4 where d = 8 so r = 4
a = 50.24 square centimeters

Question 7.
WRITING
Explain the relationship between the circumference and area of a circle.

Answer:
The area  of circle = πr x r
circumference of circle = 2πr

Explanation:
The circumference of the circle is 2 times greater than the area of the circle.
The area  of circle = πr x r
circumference of circle = 2πr

Question 8.
DIFFERENT WORDS, SAME QUESTION
Which is different? Find “both” answers.
Big Ideas Math Answers 7th Grade Chapter 9 Geometric Shapes and Angles 9.2 7

Answer:
What is area of a circle with a radius of 100 cm?
What is the area of a circle with a radius  of 500 mm?

Explanation:
The area  of circle = πr x r
area = 3.14 x 100 x 100
area = 31400 sq cm
The area  of circle = πr x r
area = 3.14 x 500 x 500
area = 785000 sq mm
Self-Assessment for Problem Solving
Solve each exercise. Then rate your understanding of the success criteria in your journal.

Question 9.
A local event planner wants to cover a circular region with mud for an obstacle course. The region has a circumference of about 157 feet. The cost to cover 1 square foot with mud is $1.50. Approximate the cost to cover the region with mud.
Big Ideas Math Answers 7th Grade Chapter 9 Geometric Shapes and Angles 9.2 8

Answer:
Cost to cover =

Question 10.
DIG DEEPER!
A manufacturer recommends that you use a frying pan with a radius that is within 1 inch of the radius of your stove top burner. The area of the bottom of your frying pan is 25π square inches. The circumference of your cook top burner is 9π inches. Does your frying pan meet the manufacturer’s recommendation?

Answer:
no the frying pan does not meet the manufacture

Explanation:
Given that frying pan has radius = 1 inch
area of frying pan = 25π square inches
circumference = 9π  inches

Areas of Circles Homework & Practice 9.2

Review & Refresh

Find the circumference of the object. Use 3.14 or \(\frac{22}{7}\) for π.
Question 1.
Big Ideas Math Answers 7th Grade Chapter 9 Geometric Shapes and Angles 9.2 9

Answer:
c = 28.26cm

Explanation:
circumference of the circle =2πr
circle = 2 x 3.14 x 4.5where r = 4.5cmgiven
circle = 6.28 x 4.5
circle =28.26 cm

Question 2.
Big Ideas Math Answers 7th Grade Chapter 9 Geometric Shapes and Angles 9.2 10

Answer:
c = 21.98 sq in

Explanation:
circumference of the circle =2πr
circle = 2 x 3.14 x 3.5where r = 3.5ingiven
circle = 6.28 x 3.5
circle =21.98 sq in

You spin the spinner shown.
Big Ideas Math Answers 7th Grade Chapter 9 Geometric Shapes and Angles 9.2 11
Question 3.
How many possible outcomes are there?

Answer:
3 possible outcomes

Explanation:
There are 3 possible outcomes.
3 numbers are there in the spin.

Question 4.
In how many ways can spinning an odd number occur?

Answer:
2 ways the spinning an odd number occur.

Explanation:
There are 2 possible ways that the odd numbers can occur.

Concepts, Skills, & Problem Solving
ESTIMATING AN AREA Use the grid to estimate the area of the circle. (See Exploration 1, p. 369.)

Question 5.
diameter of 3 centimeters
Big Ideas Math Answers 7th Grade Chapter 9 Geometric Shapes and Angles 9.2 12

Answer:
area of the circle = 6.75 sq cm

Explanation:
The area  of circle = πr x r
area = 3.14 x 1.5 x 1.5
area = 6.75 sq cm

Question 6.
diameter of 1.6 inches
Big Ideas Math Answers 7th Grade Chapter 9 Geometric Shapes and Angles 9.2 13

Answer:
area of the circle = 141.41 sq in

Explanation:
The area  of circle = πr x r
area = 3.14 x 6.4 x 6.4
area = 141.41 sq in

FINDING AN AREA Find the area of the circle. Use 3.14 or \(\frac{22}{7}\) for π.
Question 7.
Big Ideas Math Answers 7th Grade Chapter 9 Geometric Shapes and Angles 9.2 14

Answer:
The area  of circle = 254.34 sq milli meters

Explanation:
The area  of circle = πr x r
a = 3.14 x 4 x 4 where d = 8 so r = 4
a = 254.34 square millimeters

Question 8.
Big Ideas Math Answers 7th Grade Chapter 9 Geometric Shapes and Angles 9.2 15

Answer:
The area  of circle = 615.44 sq centi meters

Explanation:
The area  of circle = πr x r
a = 3.14 x 14 x 14 where r = 14
a = 615.44 square centimeters

Question 9.
Big Ideas Math Answers 7th Grade Chapter 9 Geometric Shapes and Angles 9.2 16

Answer:
The area  of circle = 314 sq inches

Explanation:
The area  of circle = πr x r
a = 3.14 x 10 x 10 where r = 10
a = 314 square inches

Question 10.
Big Ideas Math Answers 7th Grade Chapter 9 Geometric Shapes and Angles 9.2 17

Answer:
The area  of circle = 7.065 sq inches

Explanation:
The area  of circle = πr x r
a = 3.14 x 1.5 x 1.5 where r = 1.5
a = 7.065 square inches

Question 11.
Big Ideas Math Answers 7th Grade Chapter 9 Geometric Shapes and Angles 9.2 18

Answer:
The area  of circle = 3.14 sq cm

Explanation:
The area  of circle = πr x r
a = 3.14 x 1 x 1 where r = 1
a = 3.14 square cm

Question 12.
Big Ideas Math Answers 7th Grade Chapter 9 Geometric Shapes and Angles 9.2 19

Answer:
area  of circle = 1.76625sq ft

Explanation:
The area  of circle = πr x r
a = 3.14 x 0.75 x 0.75 where r = 0.75
a = 1.76625 square ft

Question 13.
YOU BE THE TEACHER
Your friend finds the area of a circle with a diameter of 7 meters. Is your friend correct? Explain.
Big Ideas Math Answers 7th Grade Chapter 9 Geometric Shapes and Angles 9.2 20

Answer:
No, my friend is not correct.

Explanation:
The area  of circle = πr x r
a = 3.14 x 3.5 x 3.5 where r = 0.75
a = 38.465 square meters
Question 14.
MODELING REAL LIFE
The diameter of a flour tortilla is 12 inches. What is the total area of two tortillas?

Answer:
The area  of tortilla = 226.08 sq inches

Explanation:
The area  of tortilla = πr x r
a = 3.14 x 6 x 6 where r = 6
a = 113.04 square inches
for 2 tortilla = 226.08 sq inches

Question 15.
MODELING REAL LIFE
The diameter of a coaster is 7 centimeters. What is the total area of five coasters?

Answer:
The  total area  of coaster = 192.325 cm

Explanation:
The area  of tortilla = πr x r
a = 3.14 x 3.5 x 3.5 where r = 3.5
a = 38.465 square cm
for 5 tortilla = 192.325 centimeters

Question 16.
PROBLEM SOLVING
The HillsboroInlet Lighthouse lights up how much more area than the Jupiter Inlet Lighthouse?
Big Ideas Math Answers 7th Grade Chapter 9 Geometric Shapes and Angles 9.2 21

Answer:
The HillsboroInlet Lighthouse lights are 2 times greater than the Jupiter Inlet Lighthouse.

Explanation:
Hillsboro inlet Lighthouse = 3.14 x 28 x 28
area = 2,461.76 sq mi
jupiter inlet Lighthouse = 3.14 x 18 x 18
area = 1,017.36 sq mi

FINDING THE AREA OF A SEMICIRCLE Find the area of the semicircle.
Question 17.
Big Ideas Math Answers 7th Grade Chapter 9 Geometric Shapes and Angles 9.2 22

Answer:
Area of semicircle = 628 sq cm

Explanation:
Area of semicircle =( π+r x r/2)
area =( 3.14 +400/2)
area =(403.14/2)
area = 628 sq cm

Question 18.
Big Ideas Math Answers 7th Grade Chapter 9 Geometric Shapes and Angles 9.2 23
Answer:
Area of semicircle =201.57 sq cm

Explanation:
Area of semicircle =( π+r x r/2)
area =( 3.14 +400/2)
area =(403.14/2)
area = 201.57 sq cm

Question 19.
Big Ideas Math Answers 7th Grade Chapter 9 Geometric Shapes and Angles 9.2 24

Answer:
Area of semicircle =1.57 sq ft

Explanation:
Area of semicircle =( π+r x r/2)
area =( 3.14 +1/2)
area =(3.14/2)
area = 1.57 sq ft

Question 20.
MODELING REAL LIFE
The plate for a microscope has a circumference of 100π millimeters. What is the area of the plate?

Answer:
Area of the plate = 200π mm

Explanation:
Area of the plate = π x r x r
area = 3.14 x 200 x 200

Question 21.
MODELING REAL LIFE
A dog is leashed to the corner of a house. How much running area does the dog have? Explain how you found your answer.
Big Ideas Math Answers 7th Grade Chapter 9 Geometric Shapes and Angles 9.2 25

Answer:
Area of the circle = 942 sq ft

Explanation:
Area of the circle = π x r x r
area = 3.14 x 20 x 20
area = 942 sq ft
The running area is 3/4 the area of a circle with a radius of 20 feet.

Question 22.
REASONING
Target A has a circumference of 20 feet. Target B has a diameter of 3 feet. Both targets are the same distance away. Which target is easier to hit? Explain your reasoning.

Answer:
Target B is easier to hit

Explanation:
Target A =2  π x r
A  = 2 x 3.14 x 3.18
A = 19.9704

Target B = 1.5

Question 23.
DIG DEEPER!
A circular oil spill has a radius of 2 miles. After a day, the radius of the oil spill increases by 3 miles. By how many square miles does the area of the oil spill increase?

Answer:
The area of oil spill increases by 65.94 miles.

Explanation:
Given that the circular oil spill has a radius of 2 miles.
The radius of the oil spill increases by 65.94 sq miles.

Question 24.
FINDING AN AREA
Find the area of the circle in square yards.
Big Ideas Math Answers 7th Grade Chapter 9 Geometric Shapes and Angles 9.2 26

Answer:
Area of the circle = 7.057935 sq yd

Explanation:
Area of the circle = π x r x r
area = 3.14 x 4.5 x 4.5
area = 63.585 sq ft
area = 7.057935 sq yd

Question 25.
REPEATED REASONING
What happens to the circumference and the area of a circle when you double the radius? triple the radius? Justify your answer.

Answer:
If we double the radius ,area  = π x r x r x r x r
If we double the radius, circumference  = 2πr x r x r
If we triple the radius ,area  = π x r x r x r x r x r
If we triple the radius, circumference  = 2πr x r x r x r x r

Explanation:
circumference doubles and area quadruples;
circumference triples and area is 9 times greater;
double the radius: circumference = 2π2r = 4πr
4πr /2πr  = 2 times larger, area =π (2r) x r =4πrx r
4πrx r/ πrx r = 4 times larger.

Question 26.
CRITICAL THINKING
Is the area of a semicircle with a diameter of x greater than, less than, or equal to the area of a circle with a diameter of \(\frac{1}{2}\)x? Explain.

Answer:
The area of a semicircle with a diameter of x is greater than the area of a circle with a diameter of (0.5)

Explanation:
Area of semicircle = (3.14 + (0.5 x 0.5)/2)
area = 1.695
Area of circle = (3.14 x 0.5 x 0.5)
area = 0.785

Lesson 9.3 Perimeters and Areas of Composite Figures

EXPLORATION 1

Submitting a Bid
Work with a partner. You want to bid on a project for the pool shown. The project involves ordering and installing the brown tile that borders the pool, and ordering a custom-made tarp to cover the surface of the pool. In the figure, each grid square represents 1 square foot. You pay $5 per linear foot for the tile.

  • You pay $4 per square foot for the tarp.
  • It takes you about 15 minutes to install each foot of tile.

a. Estimate the total cost for the tile and the tarp.
b. Write a bid for how much you will charge for the project.Include the hourly wage you will receive.Estimate your total profit.
Big Ideas Math Answers Grade 7 Chapter 9 Geometric Shapes and Angles 9.3 1

Answer:

Big Ideas Math Answers Grade 7 Chapter 9 Geometric Shapes and Angles 9.3 2

Try It

Question 1.
Estimate the perimeter and the area of the figure.
Big Ideas Math Answers Grade 7 Chapter 9 Geometric Shapes and Angles 9.3 3

Answer:
50.24 sq mm

Explanation:
The above-given figure is about  50.24 sq mm
Question 2.
Find the perimeter and the area of the figure.
Big Ideas Math Answers Grade 7 Chapter 9 Geometric Shapes and Angles 9.3 4

Answer:
perimeter of the figure = 3.16 sq in
area of the figure = 3.14 sq in

Explanation:
perimeter of the semicircle = ( π + 2) r
p = (3.14 + 2 ) 1
p = 3.16 in
area of the figure = π x r x r
area = 3.14 x 1 x1
area = 3.14 in

Self-Assessment for Concepts & Skills
Solve each exercise. Then rate your understanding of the success criteria in your journal.

Question 3.
ESTIMATING PERIMETER AND AREA
Estimate the perimeter and area of the figure at the right.
Big Ideas Math Answers Grade 7 Chapter 9 Geometric Shapes and Angles 9.3 5

Answer:
The perimeter  and area = 30 ft
area =  π  x r x r

Explanation:

The perimeter = ( π + 2) r
area =  π  x r x r

Question 4.
FINDING PERIMETER AND AREA
Identify the shapes that make up the figure at the left. Then find the perimeter and area of the figure.
Big Ideas Math Answers Grade 7 Chapter 9 Geometric Shapes and Angles 9.3 6

Answer:
The perimeter = 9.48 sq ft
area =  27.36 sq ft

Explanation:
The perimeter = ( π + 2) r
perimeter = 3.14 +2 x 3
perimeter = 9.48 sq ft
area = 3.14 x 3 x 3
area = 27.36 sq feet

Self-Assessment for Problem Solving
Solve each exercise. Then rate your understanding of the success criteria in your journal.

Question 5.
A farmer wants to seed and fence a section of land. Fencing costs $27 per yard. Grass seed costs $2 per square foot. How much does it cost to fence and seed the pasture?
Big Ideas Math Answers Grade 7 Chapter 9 Geometric Shapes and Angles 9.3 7

Answer:
1 m

Explanation:
Given that farmer has the fencing cost = $ 27
seed cost = $ 2
5.10 $ is used to cost for  grass seed
$ 27 is used to fence = 1 m

Question 6.
DIG DEEPER!
In each room shown, you plan to put down carpet and add a wallpaper border around the ceiling. Which room needs more carpeting? more wallpaper?
Big Ideas Math Answers Grade 7 Chapter 9 Geometric Shapes and Angles 9.3 8

Answer:
Room A needs more carpeting.

Explanation:
Room A =  10 x 11
where length = 11 , breadth = 10 given
Room A = 110
Room B = 12 x 8
B = 96

Perimeters and Areas of Composite Figures Homework & Practice 9.3

Review & Refresh

Find the area of the circle. Use 3.14 or \(\frac{22}{7}\) for π.
Question 1.
Big Ideas Math Answers Grade 7 Chapter 9 Geometric Shapes and Angles 9.3 9

Answer:
Area of the circle = 50.24 sq mm

Explanation:
Area of the circle = π x r x r
area = 3.14 x 4 x 4
area = 3.14 x 16
area = 50.24 sq mm

Question 2.
Big Ideas Math Answers Grade 7 Chapter 9 Geometric Shapes and Angles 9.3 10

Answer:
Area of the circle = 63.585 sq ft

Explanation:
Area of the plate = π x r x r
area = 3.14 x 4.5 x 4.5
area = 3.14 x
area = 63.585 sq ft

Find the missing dimension. Use the scale 1 : 5.
Big Ideas Math Answers Grade 7 Chapter 9 Geometric Shapes and Angles 9.3 11

Answer:
3. Height = 30 ft
4. Length = 6 ft
5. Depth = 100 cm
6. Diameter = 2 in

Explanation:
3. house : height = 6 ft , height = 30 ft given that scale = 1 : 5
4. garden hose : length = 6 ft , length = 20 yd
4. fountain : depth = 20 cm, depth = 100 cm
5. bicycle wheel :  = diameter = 2 in  diameter = 2 ft

Concepts, Skills, & Problem Solving

ESTIMATING PERIMETER AND AREA You build a patio with a brick border. (See Exploration 1, p. 375.)
Big Ideas Math Answers Grade 7 Chapter 9 Geometric Shapes and Angles 9.3 12
Question 7.
Estimate the perimeter of the patio.

Answer:
The perimeter of a patio = 24 units

Explanation:
In the above-given figure,
the perimeter of the patio = 24

Question 8.
Estimate the area of the patio.

Answer:
area of the patio = π r

ESTIMATING PERIMETER AND AREA Estimate the perimeter and the area of the shaded figure.
Question 9.
Big Ideas Math Answers Grade 7 Chapter 9 Geometric Shapes and Angles 9.3 13

Answer:
Perimeter = 19.5 units
area =13.5 units

Explanation:
given figure is trapezoid
Perimeter = a + b + c + d
area =( (a + b) x h /2)

Question 10.
Big Ideas Math Answers Grade 7 Chapter 9 Geometric Shapes and Angles 9.3 14

Answer:
area =(  3 √ 3/2) a square
perimeter = 6 a

Explanation:
given figure is hexagon
area =(  3 √ 3/2) a square
perimeter = 6 a

Question 11.
Big Ideas Math Answers Grade 7 Chapter 9 Geometric Shapes and Angles 9.3 15

Answer:
The perimeter = 24.6 units
Area of the plate = 41.1 sq units

Explanation:
given figure is semicircle
The perimeter = ( π + 2) r
Area of the plate = π x  r x r

Question 12.
Big Ideas Math Answers Grade 7 Chapter 9 Geometric Shapes and Angles 9.3 16

Answer:
Perimeter = a + b + c + d
area =( (a + b) x h /2)

Explanation:
given figure is trapezoid
Perimeter = a + b + c + d
area =( (a + b) x h /2)

Question 13.
Big Ideas Math Answers Grade 7 Chapter 9 Geometric Shapes and Angles 9.3 17

Answer:
Perimeter = 19 units
area = 24  squnits

Explanation:
given figure is pentagon
Perimeter = 5 a
area = (  perimeter x apotherm /2 )

Question 14.
Big Ideas Math Answers Grade 7 Chapter 9 Geometric Shapes and Angles 9.3 18

Answer:
Perimeter = a + b + c
area = (  height x breadth /2 )

Explanation:
given figure is triangle
Perimeter = a + b + c
area = (  height x breadth /2 )

FINDING PERIMETER AND AREA Find the perimeter and the area of the figure.
Question 15.
Big Ideas Math Answers Grade 7 Chapter 9 Geometric Shapes and Angles 9.3 19

Answer:
area = 137 sq m
perimeter = 56 m

Explanation:
area of the rectangle = l x w
l = length, w = width
area = 12 x 11
area = 137 sq m
perimeter of the rectangle = 2 ( l + w)
perimeter = 2 (28)
perimeter = 56 m

Question 16.
Big Ideas Math Answers Grade 7 Chapter 9 Geometric Shapes and Angles 9.3 20

Answer:
area = 114.07 sq ft
perimeter = 47.4 sq ft

Explanation:
Area of semicircle =( π+r x r/2)
area =( 3.14 +225/2)
area =(228.14/2)
area = 114.07 sq ft
perimeter of the semicircle = (π + 2 ) r
perimeter = (3.14 + 2) 15 given r= 15
perimeter = 3.16 x 15
perimeter = 47.4 sq ft

Question 17.
Big Ideas Math Answers Grade 7 Chapter 9 Geometric Shapes and Angles 9.3 21

Answer:
area = 49.5 cm
perimeter = 29 cm

Explanation:
area of the rectangle = l x w
l = length, w = width
area = 7 x 7
area = 49 cm
perimeter of the rectangle = 2 ( l + w)
perimeter = 2 (14)
perimeter = 29 cm

Question 18.
YOU BE THE TEACHER
Your friend finds the perimeter of the figure. Is your friend correct? Explain your reasoning.
Big Ideas Math Answers Grade 7 Chapter 9 Geometric Shapes and Angles 9.3 22

Answer:
Yes my friend is correct.

Explnation:
perimeter = length + side +  height + breadth + width + base
p = 4 + 3 + 4 + 5 + 4 + 5
p = 25 in

Question 19.
LOGIC
A running track has six lanes. Explain why the starting points for the six runners are staggered. Draw a diagram as part of your explanation.
Big Ideas Math Answers Grade 7 Chapter 9 Geometric Shapes and Angles 9.3 23

Answer:

Explanation:
The starting points for the six runners are staggered because each runner can run the same distance.

Explanation:
The starting points are staggered so that each runner can run the same distance and use the same finish line.
this is necessary because the circumference is different for each lane.
the above-diagram shows this because the diameter is greater n the outer lanes.

Question 20.
PROBLEM SOLVING
You run around the perimeter of the baseball field at a rate of 9 feet per second. How long does it take you to run around the baseball field?
Big Ideas Math Answers Grade 7 Chapter 9 Geometric Shapes and Angles 9.3 24

Answer:
It take to run around the baseball field = 1,58,962.5 sq feet

Explanation:
The area of the circle = π x r x r
area = 3.14 x 225 x 225
area = 1,58,962.5 sq feet

Question 21.
STRUCTURE
The figure at the right is made up of a square and a rectangle. Find the area of the shaded region.
Big Ideas Math Answers Grade 7 Chapter 9 Geometric Shapes and Angles 9.3 25

Answer:
The area of the shaded region =24 sq m

Ex planation:
Area of triangle = ( b x h )/2
area =( 8 x 7)/ 2
area = 48/2
area = 24 sq m
Question 22.
DIG DEEPER!
Your friend makes a two-dimensional model of a dividing cell as shown. The total area of the dividing cell is 350 square inches. What is the area of the shaded region?
Big Ideas Math Answers Grade 7 Chapter 9 Geometric Shapes and Angles 9.3 26

Answer:
The area of the shaded region = 1.89 sq in

Explanation :
area of semicircle = (π + r x r/2)
area = (3.14 + 64/2)
area = ( 3.78 / 2)
area = 1.89 sq in
Question 23.
CRITICAL THINKING
How can you add a figure to a composite figure without increasing its perimeter? Can this be done for all figures? Draw a diagram to support your answer.

Answer:

Explanation:
The perimeter does not increases.

Lesson 9.4 Constructing Polygons

EXPLORATION 1

Using Technology to Draw Polygons
Work with a partner.
a. Use geometry software to draw each polygon with the given side lengths or angle measures, if possible. Complete the table.
Big Ideas Math Solutions Grade 7 Chapter 9 Geometric Shapes and Angles 9.4 1
b. Without constructing, how can you tell whether it is possible to draw a triangle given three angle measures? three side lengths? Explain your reasoning.
c. Without constructing, how can you tell whether it is possible to draw a quadrilateral given four angle measures? four side lengths? Explain your reasoning.

Answer:
b. Yes it is possible to draw a triangle with the given three angles measures, three side lengths.
c. yes it is possible to draw a quadrilateral with the given 4 angle measures, four side lengths.

Explanation:
1. given that sides = 4 cm , 6 cm , 7cm

2. given that sides = 2 cm , 3 cm , 3 cm, 5 cm

Try It

Draw a triangle with the given angle measures, if possible.
Question 1.
45°, 45°, 90°

Answer:

Explanation:
The above triangle is an equilateral triangle.
it forms with the given angles 45°, 45°, 90°.

Question 2.
100°, 55°, 25°

Answer:

Explanation:
The above triangle is scalene  triangle.
it forms with the given angles 100°, 55°, 25°.

Question 3.
60°, 60°, 80°

Answer:

Explanation:
The above triangle is an equilateral triangle.
it forms with the given angles60°, 60°, 80°

Question 4.
Draw a triangle with side lengths of 1 inch and 2 inches that meet at a 60° angle.

Answer:

Explanation:
The above triangle is a scalene triangle.
it forms with the given angles 60° , 1 inch and 2 inch.

Draw a triangle with the given side lengths, if possible.
Question 5.
2 cm, 2 cm, 5 cm

Answer:

Explanation:
given the sides of a triangle 2cm , 2 cm , 5 cm

Question 6.
4 cm, 3 cm, 3 cm

Answer:

Explanation:
given that 2 sides are same and one side is different.

Question 7.
1 cm, 4 cm, 5 cm

Answer:

Draw a quadrilateral with the given angle measures, if possible.
Question 8.
100°, 90°, 65°, 105°

Answer:

Explanation:
The quadrilateral formed with the given angles 100°, 90°, 65°, 105°.

Question 9.
100°, 40°, 20°, 20°

Answer:

Explanation:
The quadrilateral formed with the given angles 100°, 40°, 20°, 20°.

Question 9.

Self-Assessment for Concepts & Skills
Solve each exercise. Then rate your understanding of the success criteria in your journal.

DRAWING POLYGONS Draw a polygon with the given side lengths or angle measures, if possible.
Question 10.
25 mm, 36 mm, 38 mm

Answer:

Explanation:
The polygon formed with the given sides is a triangle.

Question 11.
10°, 15°, 155°

Answer:

Explanation:
The polygon formed with the given sides is a triangle.

Question 12.
20°, 45°, 50°, 65°

Answer:

Explanation:
The polygon formed with the given sides is a  hexagon.

Question 9.
100°, 40°, 20°, 20°

Answer:

Explanation:
The polygon formed with the given sides is a  hexagon.
Question 9.
100°, 40°, 20°, 20°
Answer:

Question 9.
100°, 40°, 20°, 20°
Answer:

Explanation:
The polygon formed with the given sides is a  quadrilateral.

Question 13.
50°, 90°, 110°, 110°

Answer:

Question 14.
USING SIDE LENGTH
Can you construct one, many, or triangle(s) with side lengths of 3 inches, 4 inches, and 8 inches? Explain.
Answer:
We can construct only one triangle

Explanation:
Given the side lengths of 3 inches, 4 inches, and 8 inches.

Self-Assessment for Problem Solving
Solve each exercise. Then rate your understanding of the success criteria in your journal.

Question 15.
A triangular pen has fence lengths of 6 feet, 8 feet, and 10 feet. Create a scale drawing of the pen.
Answer:

Question 16.
The front of a cabin is the shape of a triangle. The angles of the triangle are 40°, 70°, and 70°. Can you determine the height of the cabin? If not, what information do you need?
Answer:

Question 17.
DIG DEEPER!
Two rooftops have triangular patios. One patio has side lengths of 9 meters,10 meters, and 11 meters.e other has side lengths of 6 meters,10 meters, and 15 meters. Which patio has a greater area? Explain.
Big Ideas Math Solutions Grade 7 Chapter 9 Geometric Shapes and Angles 9.4 2
Answer:
The patio which has a side length of 6 meters, 10 meters, and 15 meters.

Explanation:
The patio has a greater side length.

Constructing Polygons Homework & Practice 9.4

Review & Refresh

Find the perimeter and area of the figure.
Question 1.
Big Ideas Math Solutions Grade 7 Chapter 9 Geometric Shapes and Angles 9.4 3
Answer:
area = 12 in
perimeter = 14 in

Explanation:
area of the rectangle = l x w
l = length, w = width
area = 4 x 3
area = 12 in
perimeter of the rectangle = 2 ( l + w)
perimeter = 2 (7)
perimeter = 14 in

Question 2.
Big Ideas Math Solutions Grade 7 Chapter 9 Geometric Shapes and Angles 9.4 4
Answer:

perimeter of the figure = 9.48 sq cm
area of the figure = 28.26 sq cm

Explanation:
perimeter of the semicircle = ( π + 2) r
p = (3.14 + 2 ) 3
p = 9.48 cm
area of the figure = π x r x r
area = 3.14 x 3 x3
area = 28.26 sq cm

Use a tree diagram to find the sample space and the total number of possible outcomes of the indicated event.
Big Ideas Math Solutions Grade 7 Chapter 9 Geometric Shapes and Angles 9.4 5
Question 3.
choosing a toothbrush
Big Ideas Math Solutions Grade 7 Chapter 9 Geometric Shapes and Angles 9.4 6
Answer:
Extra soft, soft, Medium

Explanation:
In the above given figure the strength of the toothbrush = extra soft , soft , meedium

Question 4.
Big Ideas Math Solutions Grade 7 Chapter 9 Geometric Shapes and Angles 9.4 7
Answer:
The size of the toy hop is small, medium , large.

Explanation:
given that the colour of the toy hoop is blue , green , orange, pink, purple , yellow.

Concepts, Skills, & Problem Solving

USING TECHNOLOGY TO DRAW POLYGONS Use geometry software to draw the polygon with the given side lengths or angle measures, if possible. (See Exploration 1, p. 381.)
Question 5.
30°, 65°, 85°
Answer:

Question 6.
2 in., 3 in., 5 in.
Answer:

Question 7.
80°, 90°, 100°, 110°
Answer:
Not possible.

Question 8.
2 cm, 2 cm, 5 cm, 5 cm
Answer:

CONSTRUCTING TRIANGLES USING ANGLE MEASURES Draw a triangle with the given angle measures, if possible.
Question 9.
40°, 50°, 90°
Answer:

Question 10.
20°, 40°, 120°
Answer:

Question 11.
38°, 42°, 110°
Answer:

Question 12.
54°, 60°, 66°
Answer:

Question 13.
YOU BE THE TEACHER
Your friend determines whether he can draw a triangle with angle measures of 10°, 40°, and 130°. Is your friend correct? Explain your reasoning.
Big Ideas Math Solutions Grade 7 Chapter 9 Geometric Shapes and Angles 9.4 8
Answer:
Yes .

Explanation:
yes we cannot draw the triangle with the angle measures of 10, 40, 130

CONSTRUCTING TRIANGLES USING ANGLES AND SIDES Draw a triangle with the given description.
Question 14.
side lengths of 1 inch and 2 inches meet at a 50° angle
Answer:
yes.

Explanation:
we can draw a triangle with 1 inch 2 inch that meets at  50 degrees.

Question 15.
side lengths of 7 centimeters and 9 centimeters meet at a 120° angle
Answer:
yes.

Explanation:
we can draw a triangle with 7 cm 9 cm that meets at  120 degrees.

Question 16.
a 95° angle connects to a 15° angle by a side of length 2 inches
Answer:
no.

Explanation:
we cannot draw a triangle with 2 inches 15 degrees that meets at  120 degrees.

Question 17.
a 70° angle connects to a 70° angle by a side of length 4 centimeters
Answer:
yes.

Explanation:
we can draw an angle with 4 cm  70 degrees that meets at  120 degrees.

CONSTRUCTING TRIANGLES USING SIDE LENGTHS Draw a triangle with the given side lengths, if possible.
Question 18.
4 in., 5 in., 10 in.
Answer:

Question 19.
10 mm, 30 mm, 50 mm
Answer:

Question 20.
5 cm, 5 cm, 8 cm
Answer:

Question 21.
8 mm, 12 mm, 13 mm
Answer:

Question 22.
MODELING REAL LIFE
Can you construct a triangular case using two pieces of wood that are 12 inches long and one piece of wood that is 25 inches long? Explain.
Answer:
Yes we can construct a triangle .

Explanation:
We can costruct the triangle by using two pieces of wood that are 12 inches long and the one piece of wood is 25 inches.

Question 23.
MODELING REAL LIFE
Can you construct a warning triangle using three pieces of plastic that are each 6 inches long? Explain.
Big Ideas Math Solutions Grade 7 Chapter 9 Geometric Shapes and Angles 9.4 9
Answer:
Yes.

Explanation:
we can construct the three pieces of plastic by using 3 6 inches long.

Question 24.
LOGIC
You are constructing a triangle. You draw the first angle, as shown. Your friend says that you must be constructing an acute triangle. Is your friend correct? Explain your reasoning.
Big Ideas Math Solutions Grade 7 Chapter 9 Geometric Shapes and Angles 9.4 10
Answer:
Yes my friend is correct.

Explanation:
it is a acute angle triangle.

USING ANGLES AND SIDES Determine whether you can construct one, many, or no triangle(s) with the given description. Explain your reasoning.
Question 25.
a triangle with one angle measure of 60and one side length of 4 centimeters
Answer:

Explanation:
we cannot construct one trinangle with the help of given sidelengths.

Question 26.
a scalene triangle with side lengths of 3 centimeters and 7 centimeters
Answer:

Question 27.
an isosceles triangle with two side lengths of 4 inches that meet at an 80° angle
Answer:

Question 28.
a triangle with one angle measure of 60°, one angle measure of 70°, and a side length of 10 centimeters between the two angles
Answer:

Question 29.
a triangle with one angle measure of 20°, one angle measure of 35°, and a side of length 3 inches that is between the two angles
Answer:

Question 29.
REASONING
A triangle is shown.
Big Ideas Math Solutions Grade 7 Chapter 9 Geometric Shapes and Angles 9.4 11
a. Construct a triangle with side lengths twice those of the triangle shown. Does the new triangle have the same angle measures?
b. How can you change the side lengths of the triangle so that the measure of ∠A increases?
Answer:
a. Yes the new triangle have the same angle.
b. angle A increases .

Explanation:
Given that the triangle with side lengths twice those of the triangle shown.
If we can change the side lengths of triangle .

CONSTRUCTING QUADRILATERALS Draw a quadrilateral with the given angle measures, if possible.
Question 31.
60°, 60°, 120°, 120°
Answer:

Question 32.
50°, 60°, 110°, 150°
Answer:

Question 33.
20°, 30°, 150°, 160°
Answer:

Question 34.
10°, 10°, 10°, 150°
Answer:

Explanation:
Given angles are 10 degrees, 10 degrees, 10 degrees, 10 degrees.

CONSTRUCTING SPECIAL QUADRILATERALS Construct a quadrilateral with the given description.
Question 35.
a rectangle with side lengths of 1 inch and 2 inches
Answer:

Question 36.
a kite with side lengths of 4 centimeters and 7 centimeters
Answer:

Question 37.
a trapezoid with base angles of 40°
Answer:
Answer

Question 38.
a rhombus with side lengths of 10 millimeters
Answer:

Question 39.
REASONING
A quadrilateral has side lengths of 6 units, 2 units, and 3 units as shown. How many quadrilaterals can be formed given a fourth side with a fixed length? Explain.
Big Ideas Math Solutions Grade 7 Chapter 9 Geometric Shapes and Angles 9.4 12
Answer:
2 quadrilaterals can be formed.

Explanation:
Given that the quadrilateral has side lengths of 6 units, 2 units, and 3 units.
so 2 quadrilaterals can be formed.

Question 40.
REASONING
What types of quadrilaterals can you form using four side lengths of 7 units? Use drawings to support your conclusion.
Answer:

Question 41.
MODELING REAL LIFE
A triangular section of a farm is enclosed by fences that are 2 meters, 6 meters, and 7 meters long. Estimate the area of the section.
Answer:
Area of the section = 12 sq meters.

Question 42.
MODELING REAL LIFE
A chemical spill expert sets up a triangular caution zone using cones. Cones A and B are 14 meters apart. Cones B and C are 22 meters apart. Cones A and C are 34 meters apart. Estimate the area of the caution zone.
Big Ideas Math Solutions Grade 7 Chapter 9 Geometric Shapes and Angles 9.4 13
Answer:
Area of the area of the caution Zone = 308 sq meters.

Explanation:
Area of the triangle = l x b
area = 22 x 14
area = 308 sq meters.

Question 43.
MODELING REAL LIFE
A search region is in the shape of an equilateral triangle. The measure of one side of the region is 20 miles. Make a scale drawing of the search region. Estimate the area of the search region.
Answer:

Explanation:
Given that the equilateral triangle .

Question 44.
REASONING
A triangle has fixed side lengths of 2 and 14.
a. How many triangles can you construct? Use the figure below to explain your reasoning.
Big Ideas Math Solutions Grade 7 Chapter 9 Geometric Shapes and Angles 9.4 14
b. Is the unknown side length of the triangle also fixed? Explain.
Answer:
We can construct 14 triangles.
b. No the side length of the triangle cannot fixed.

Explanation:
a. We can construct 14 triangles.
b. No the side length of the triangl cannot fixed.

Lesson 9.5 Finding Unknown Angle Measures

EXPLORATION 1

Using Rules About Angles
Work with a partner. The diagram shows pairs of angles and vertical angles. Vertical angles cannot be adjacent.
Big Ideas Math Answer Key Grade 7 Chapter 9 Geometric Shapes and Angles 9.5 1
a. Which pair(s) of angles are adjacent angles? Explain.
b. Which pair(s) of angles are vertical angles? Explain.
c. Without using a protractor, find the values of x, y, and z. Explain your reasoning.
d. Make a conjecture about the measures of any two vertical angles.
e. Test your conjecture in part(d) using the diagram below. Explain why your conjecture is or is not true.
Big Ideas Math Answer Key Grade 7 Chapter 9 Geometric Shapes and Angles 9.5 2
Answer:
A. ∠ACD, ∠AEB
b. ∠ACD, ∠AEB
c. 125
d. ∠ACD, ∠AEB
Big Ideas Math Answer Key Grade 7 Chapter 9 Geometric Shapes and Angles 9.5 3

Try It

Question 1.
Name a pair of (a) adjacent angles, (b) complementary angles, (c) supplementary angles, and (d) vertical angles in the figure.
Big Ideas Math Answer Key Grade 7 Chapter 9 Geometric Shapes and Angles 9.5 4
Answer:
a. ∠JKL, ∠JKQ, . ∠MNJ,. ∠PJN
b. ∠JKQ
c. ∠JNK, ∠ JPL. ∠JMQ
D. ∠JMQ, ∠JPL.

Explanation:
The above angles are adjacent, vertical, supplementary,complementary.

Classify the pair of angles. Then find the value of x.
Question 2.
Big Ideas Math Answer Key Grade 7 Chapter 9 Geometric Shapes and Angles 9.5 5
Answer:
x = 95 ˚

Explanation:
x = (180 – 85)
x = 95 ˚

Question 3.
Big Ideas Math Answer Key Grade 7 Chapter 9 Geometric Shapes and Angles 9.5 6
Answer:
x =  180 ˚

Explanation:
x = 180 ˚

Question 4.
Big Ideas Math Answer Key Grade 7 Chapter 9 Geometric Shapes and Angles 9.5 7
Answer:
x = 30 ˚

Explanation:
(2x – 3) = 60
2x = (60/3)
2x = 20
x = 10

Find the measure of the indicated angle in the diagram.
Big Ideas Math Answer Key Grade 7 Chapter 9 Geometric Shapes and Angles 9.5 8.1
Question 5.
∠NJM
Answer:
12 x ˚

Question 6.
∠KJP
Answer:
16 x ˚

Question 7.
∠KJM
Answer:
6x ˚

Self-Assessment for Concepts & Skills
Solve each exercise. Then rate your understanding of the success criteria in your journal.

Question 8.
NAMING ANGLES
Name a pair of (a) adjacent angles, (b) complementary angles, (c) supplementary angles, and (d) vertical angles in the figure at the left.
Big Ideas Math Answer Key Grade 7 Chapter 9 Geometric Shapes and Angles 9.5 8
Answer:
a. ∠ABC
b. ∠ABD
c. ∠ABE
d. ∠ABE

Explanation:
The above angles are adjacent, vertical, supplementary,complementary.

FINDING ANGLE MEASURES Find the value of x.
Question 9.
Big Ideas Math Answer Key Grade 7 Chapter 9 Geometric Shapes and Angles 9.5 9
Answer:
x = 60˚

Explanation:
4x = x
4x – x = 180
3x = 180
x = 60˚

Question 10.
Big Ideas Math Answer Key Grade 7 Chapter 9 Geometric Shapes and Angles 9.5 10
Answer:
x = 12.5˚

Explanation:
2x  – 10= 2x + 40
4x  = 50
x = 12.5˚

Question 11.
WHICH ONE DOESN’T BELONG?
Which pair of angles does not belong with the other three? Explain your reasoning.
Big Ideas Math Answer Key Grade 7 Chapter 9 Geometric Shapes and Angles 9.5 11
Answer:
∠FBA, ∠EBD does not belong with the other three.

Explanation:
the 3 angles are different measures,

Self-Assessment for Problem Solving
Solve each exercise. Then rate your understanding of the success criteria in your journal.

Question 12.
What is the angle between any two windmill blades in the windmill at the left? Justify your answer.
Big Ideas Math Answer Key Grade 7 Chapter 9 Geometric Shapes and Angles 9.5 12

Answer:
The angle between any two wind mills blades in the windmill at the left = 60 °

Explanation:
60 + 60 + 60 = 180

Question 13.
A hockey puck strikes a wall at an angle of 30°. The puck then travels away from the wall at the same angle. Find the value of y. Explain your reasoning.
Big Ideas Math Answer Key Grade 7 Chapter 9 Geometric Shapes and Angles 9.5 13
Answer:
y = 150 °

Explanation:
In the above figure said that hockey puck strikes a wall at an angle of 30 °.
so 180 – 30 = 150

Question 14.
DIG DEEPER!
The laptop screen turns off when the angle between the keyboard and the screen is less than 20°. How many more degrees can the laptop screen close before the screen turns off?
Big Ideas Math Answer Key Grade 7 Chapter 9 Geometric Shapes and Angles 9.5 14
Answer:
The laptop screen close before the screen turns off = 60 degrees.

Explanation:
(z + 40) = (z – 20)
z – z = (-  20 -40)
z = -60

Finding Unknown Angle Measures Homework & Practice 9.5

Review & Refresh

Draw a triangle with the given side lengths, if possible.
Question 1.
1 in., 3 in., 4 in.
Answer:

Explanation:
In the above question , they said to draw 1 in, 3 in, 4 in.

Question 2.
4 cm, 4 cm, 7 cm
Answer:

Solve the inequality. Graph the solution.
Question 3.
– 8y ≤ 40
Answer:

Explanation:
– 8y ≤ 40
y = (40/8)
y = 5

Question 4.
1.1z > – 3.3
Answer:

Explanation:
z = 3.3

Question 5.
\(\frac{1}{3}\)x ≥ 2.5
Answer:

Concepts, Skills, & Problem Solving

USING RULES ABOUT ANGLES The diagram shows pairs of adjacent vertical angles and angles. B(See Exploration 1, p. 389.)
Big Ideas Math Answer Key Grade 7 Chapter 9 Geometric Shapes and Angles 9.5 15
Question 6.
Which pair(s) of angles are adjacent angles? Explain.
Answer:
angle AEC, angle ABD.

Explanation:
In the above given figure angle AEC, angle ABD are adjacent.

Question 7.
Which pair(s) of angles are vertical angles? Explain.
Answer:
angle ABC, angle ADE

Explanation:
In the above given figure angle AEC, angle ABD are adjacent.

NAMING ANGLES Use the figure shown.
Big Ideas Math Answer Key Grade 7 Chapter 9 Geometric Shapes and Angles 9.5 16
Question 8.
Name a pair of adjacent angles.
Answer:
∠ADC, ∠AEF, ∠ABC

Question 9.
Name a pair of complementary angles.
Answer:
∠ADE, ∠ABD

Question 10.
Name a pair of supplementary angles.
Answer:
∠ABE, ∠ACF

Question 11.
Name a pair of vertical angles.
Answer:
∠AEF, ∠ABC

Question 12.
YOU BE THE TEACHER
Your friend names a pair of angles with the same measure. Is your friend correct? Explain your reasoning.
Big Ideas Math Answer Key Grade 7 Chapter 9 Geometric Shapes and Angles 9.5 17
Answer:
yes my friend is correct

Explanation:
The angles both have the same measure.

ADJACENT AND VERTICAL ANGLES Tell whether the angles are adjacent, vertical, or neither. Explain.
Question 13.
Big Ideas Math Answer Key Grade 7 Chapter 9 Geometric Shapes and Angles 9.5 18
Answer:
vertical.

Explanation:
The given angles are vertical.

Question 14.
Big Ideas Math Answer Key Grade 7 Chapter 9 Geometric Shapes and Angles 9.5 19
Answer:
Adjacent.

Explanation:
The given angles are adjacent.

Question 15.
Big Ideas Math Answer Key Grade 7 Chapter 9 Geometric Shapes and Angles 9.5 20

Answer:
Adjacent, vertical

Explanation:
The given angles are adjacent, vertical.

COMPLEMENTARY AND SUPPLEMENTARY ANGLES Tell whether the angles are complementary supplementary, or neither. Explain.
Question 16.
Big Ideas Math Answer Key Grade 7 Chapter 9 Geometric Shapes and Angles 9.5 21
Answer:
The angles are neither complementary nor supplementary.

Explanation:
complementary = 90 degree
supplementary = 180 degree

Question 17.
Big Ideas Math Answer Key Grade 7 Chapter 9 Geometric Shapes and Angles 9.5 22
Answer:
The angles are complementary.

Explanation:
complementary = 90 degree
supplementary = 180 degree

Question 18.
Big Ideas Math Answer Key Grade 7 Chapter 9 Geometric Shapes and Angles 9.5 23
Answer:
The angles are complementary.

Explanation:
complementary = 90 degree
supplementary = 180 degree

Question 19.
YOU BE THE TEACHER
Your friend names a pair of supplementary angles. Is your friend correct? Explain.
Big Ideas Math Answer Key Grade 7 Chapter 9 Geometric Shapes and Angles 9.5 24
Answer:
yes my friend is correct.

Explanation:
angle LMN and angle PMQ are supplementary angles.

USING PAIRS OF ANGLES Classify the pair of angles. Then find the value of x.
Question 20.
Big Ideas Math Answer Key Grade 7 Chapter 9 Geometric Shapes and Angles 9.5 25
Answer:
Acute angle.
x = 145

Explanation:
x = (180 – 35)
x = 35

Question 21.
Big Ideas Math Answer Key Grade 7 Chapter 9 Geometric Shapes and Angles 9.5 26
Answer:
verticle angle.
x = 52

Explanation:
x = (180 – 128)
x = 52

Question 22.
Big Ideas Math Answer Key Grade 7 Chapter 9 Geometric Shapes and Angles 9.5 27
Answer:
obtuse angle.
x = 63

Explanation:
x = (180 – 117)
x = 63

Question 23.
Big Ideas Math Answer Key Grade 7 Chapter 9 Geometric Shapes and Angles 9.5 28
Answer:
intersection angles
x = 25

Explanation:
(4x – 25) = 75
4x = 75 + 25
4x = 100
x = (100/4)
x = 25

Question 24.
Big Ideas Math Answer Key Grade 7 Chapter 9 Geometric Shapes and Angles 9.5 29
Answer:
x = 15

Explanation:
2x = 30
x = (30/2)
x = 15
4x = 60
x = (60/4)
x = 15

Question 25.
Big Ideas Math Answer Key Grade 7 Chapter 9 Geometric Shapes and Angles 9.5 30
Answer:
x = 3.33

Explanation:
(x + 20 ) = 7 x
20 = 7x – x
20 = 6x
x = (20/6)
x = 3.33

Question 26.
Big Ideas Math Answer Key Grade 7 Chapter 9 Geometric Shapes and Angles 9.5 31
Answer:
x = 15

Explanation:
3x = 45
x = (45/3)
x = 15

Question 27.
Big Ideas Math Answer Key Grade 7 Chapter 9 Geometric Shapes and Angles 9.5 32
Answer:
x = 20

Explanation:
(x – 20 ) = x
20 = x – x
x = 20

Question 28.
Big Ideas Math Answer Key Grade 7 Chapter 9 Geometric Shapes and Angles 9.5 33
Answer:
x = 25

Explanation:
(3x + 25) = 2x
3x – 2x = 25
x = 25

Question 29.
MODELING REAL LIFE
What is the measure of each angle formed by the intersection? Explain.
Big Ideas Math Answer Key Grade 7 Chapter 9 Geometric Shapes and Angles 9.5 34
Answer:
angle 2 = 50°
angle 3 = 40°
angle 1 = 40°

Explanation:
In the above figure the angle 4 is given.

Question 30.
MODELING REAL LIFE
A tributary joins a river at an angle x. Find the value of x. Explain.
Big Ideas Math Answer Key Grade 7 Chapter 9 Geometric Shapes and Angles 9.5 35
Answer:
x = 21

Explanation:
(2x + 21 ) = x
2x – x = 21
x = 21

Question 31.
MODELING REAL LIFE
The iron cross is a skiing trick in which the tips of the skis are crossed while the skier is airborne. Find the value of x in the iron cross shown.
Big Ideas Math Answer Key Grade 7 Chapter 9 Geometric Shapes and Angles 9.5 36
Answer:
The value of x in the iron cross = 43

Explanation:
(2x + 41) = 127
2x = 127 – 41
2x = 86
x = 43

FINDING ANGLE MEASURES Find all angle measures in the diagram.
Question 32.
Big Ideas Math Answer Key Grade 7 Chapter 9 Geometric Shapes and Angles 9.5 37
Answer:
x = 90˚

Question 33.
Big Ideas Math Answer Key Grade 7 Chapter 9 Geometric Shapes and Angles 9.5 38
Answer:
23.33

Explanation:
(3x + 5) = 75
3x = 75 – 5
3x = 70
x = (70/3)
x = 23.33

Question 34.
Big Ideas Math Answer Key Grade 7 Chapter 9 Geometric Shapes and Angles 9.5 39
Answer:
x = 68
x = 67

Explanation:
(2x + 4) = 140
2x = (140 – 4)
2x = 136
x = (136/2)
x = 68
(2x + 6) = 140
2x = (140 – 6)
2x = 134
x = (134/2)
x = 67

OPEN-ENDED Draw a pair of adjacent angles with the given description.
Question 35.
Both angles are acute.
Answer:

Question 36.
One angle is acute, and one is obtuse.
Answer:

Question 37.
The sum of the angle measures is 135°.
Answer:

REASONING Copy and complete each sentence with always, sometimes, or never.
Question 38.
If x and y are complementary angles, then both x and y are________ acute.
Answer:
Right acute.

Explanation:
if x and y are complimentary then the x and y are right acute.

Question 39.
If x and y are supplementary angles, then is x ________ acute.
Answer:
left acute.

Explanation:
if x and y are complimentary then the x and y are right acute.

Question 40.
If x is a right angle, then is x ________ acute.
Answer:
Right acute.

Explanation:
if x and y are complimentary then the x and y are right acute.

Question 41.
If x and y are complementary angles, then x and y are ________ adjacent.
Answer:
Right adjacent.

Explanation:
if x and y are complimentary then the x and y are right adjacent.

Question 42.
If x and y are supplementary angles, then x and y are _______ vertical.
Answer:
left vertical.

Explanation:
if x and y are supplementary then the x and y are left vertical.

Question 43.
REASONING
Draw a figure in which ∠1 and ∠2 are acute vertical angles, ∠3 is a right angle adjacent to ∠2, and the sum of the measure of ∠1 and the measure of ∠4 is 180°.
Answer:

Question 44.
STRUCTURE
Describe the relationship between the two angles represented by the graph shown at the right.
Big Ideas Math Answer Key Grade 7 Chapter 9 Geometric Shapes and Angles 9.5 40
Answer:
90°

Explanation:
The relationship between the two angles represented by the graph =90°

Question 45.
STRUCTURE
Consider the figure shown at the left. Use a ruler to extend both rays into lines. What do you notice about the three new angles that are formed?
Big Ideas Math Answer Key Grade 7 Chapter 9 Geometric Shapes and Angles 9.5 41
Answer:
The 3 angles that are formed = 30°, 60°, 90°

Explanation:
The given angles are right angles.

Question 46.
OPEN-ENDED
Give an example of an angle that can be a supplementary angle but cannot be a complementary angle to another angle. Explain.
Answer:
Acute angle

Question 47.
MODELING REAL LIFE
The vanishing point of the picture is represented by point B.
Big Ideas Math Answer Key Grade 7 Chapter 9 Geometric Shapes and Angles 9.5 42
a. The measure of ∠ABD is 6.2 times greater than the measure of ∠CBD. Find the measure of ∠CBD.
b. ∠FBE and ∠EBD are congruent. Find the measure of ∠FBE.
Answer:
a. The measure of  ∠CBD = 30°
b. The measure of ∠FBE = 60°

Explanation:
Given that the measure of ∠ABD is 6.2 times greater than the measure of ∠CBD  = 30°
∠FBE and ∠EBD are congruent so ∠FBE = 60°

Question 48.
CRITICAL THINKING
The measures of two complementary angles have a ratio of 3 : 2. What is the measure of the larger angle?
Answer:
The measure of the larger angle = 3

Explanation:
given that, the measures of two complementary angles have a ratio = 3 : 2

Question 49.
REASONING
Two angles are vertical angles. What are their measures if they are also complementary angles? supplementary angles?
Answer:
when two angles are vertical.
complementary angles = Two angles are called complementary when their measures add to 90°
supplementary angles = two angles are called supplementary when their measures add to 180°

Question 50.
PROBLEM SOLVING
Find the values of x and y.
Big Ideas Math Answer Key Grade 7 Chapter 9 Geometric Shapes and Angles 9.5 43
Answer:
x = 2.857
y = 2
x = 4

Explanation:
7  x = 20
x = (20/7)
x = 2.857
2y = 20
y = (20/10)
y  = 2
5x = 20
x = (20/5)
x = 4

Geometric Shapes and Angles Connecting Concepts

Using the Problem-Solving Plan
Question 1.
A dart is equally likely to hit any point on the board shown. Find the theoretical probability that a dart hitting the board scores 100 points.
Understand the problem.
You are given the dimensions of a circular dartboard. You are asked to find the theoretical probability of hitting the center circle.
Big Ideas Math Answers 7th Grade Chapter 9 Geometric Shapes and Angles cc 1
Make a plan.
Find the area of the center circle and the area of the entire dart board. To find the theoretical probability of scoring 100 points, divide the area of the center circle by the area of the entire dart board.
Solve and check.
Use the plan to solve the problem. Then check your solution.
Answer:
Area of center =31,400 sq in
area of entire dart board = 1,962.5 sq in

Explanation:
Area of center circle = π r ²
a = 3.14 x 100 x 100
a = 31,400  sq in
area of entire dart board =  π r ²
a = 3.14 x 25 x 25
a = 3.14 x 625
a = 1,962.5 sq in

Question 2.
A scale drawing of a window is shown. Find the perimeter and the area of the actual window. Justify your answer.
Big Ideas Math Answers 7th Grade Chapter 9 Geometric Shapes and Angles cc 2
Answer:
Area of semicircle = 1.695sq ft
perimeter of semicircle = 1.58 sq ft

Explanation:
area of the semicircle = ( π + r x r /2)
s . c = (3.14 + 0.5 x 0.5 /2)
s. c = (3.14 + 0.25 /2)
s . c = (3.39 /2
s. c = 1.695 sq ft
perimeter of the semicircle = (π + 2 ) x r
p= 3.14 + 2 x 0.5
p = 3.16 x 0.5
p = 1.58 sq ft

Question 3.
∠CAD makes up 20% of a pair of supplementary angles. Find the measure of ∠DAE. Justify your answer.
Big Ideas Math Answers 7th Grade Chapter 9 Geometric Shapes and Angles cc 3
Answer:
∠DAE = 30 %

Explanation:
Given that ∠CAD = 20%
so
∠DAE = 30 %

Performance Task

Finding the Area and Perimeter of a Track
At the beginning of the this chapter, you watched a STEAM video called “Track and Field”. You are now ready to complete the performance task related to this video, available at BigIdeasMath.com. Be sure to use the problem-solving plan as you work through the performance task.
Big Ideas Math Answers 7th Grade Chapter 9 Geometric Shapes and Angles cc 4

Geometric Shapes and Angles Chapter Review

Review Vocabulary

Write the definition and give an example of each vocabulary term.
Big Ideas Math Answers 7th Grade Chapter 9 Geometric Shapes and Angles cr 1

Graphic Organizers

You can use a Four Square to organize information about a concept. Each of the four squares can be a category, such as definition, vocabulary, example, non-example, words, algebra, table, numbers, visual, graph, or equation. Here is an example of a Four Square for circumference.
Big Ideas Math Answers 7th Grade Chapter 9 Geometric Shapes and Angles cr 2

Choose and complete a graphic organizer to help you study each topic.
Big Ideas Math Answers 7th Grade Chapter 9 Geometric Shapes and Angles cr 3
1. area of a circle
2. semicircle
3. composite figure
4. constructing triangles
5. constructing quadrilaterals
6. complementary angles
7. supplementary angles
8. vertical angles

Answer:
1. area of a circle = π r ²
2. semicircle = ( π  +  r ²/2)
3. composite figure = The figure that consists of two or more geometric shapes.
4. constructing triangles = A triangle is a 3 – sided polygon made up of three sides having 3 angles.
5. constructing quadrilateral = quadrilateral can be categorized by the lengths of its sides and the size of its angles.
6. complementary angles = Two angles are called complimentary when their measures add to 90°
7. supplementary angles = two angles are called supplementary when their measures add to 180°
8. vertical angles = The angles opposite each other when two lines cross.

Chapter Self-Assessment

As you complete the exercises, use the scale below to rate your understanding of the success criteria in your journal.
Big Ideas Math Answers 7th Grade Chapter 9 Geometric Shapes and Angles cr 4

9.1 Circles and Circumference (pp. 361–368)
Learning Target: Find the circumference of a circle.

Question 1.
What is the radius of a circular lid with a diameter of 5 centimeters?
Answer:
radius = 50 mm

Explanation:
radius  = ( d / 2)
radius  = (5/ 2) cm
r = 2.5  cm

Question 2.
The radius of a circle is 25 millimeters. Find the diameter.
Answer:
Diameter = 50 mm

Explanation:
diameter = 2 x radius
diameter = 2 x 25  mm
d = 50 mm

Find the circumference of the object. Use 3.14 or \(\frac{22}{7}\) for π.
Question 3.
Big Ideas Math Answers 7th Grade Chapter 9 Geometric Shapes and Angles cr 5
Answer:
circumference of the object = 37.68 sq mm

Explanation:
D. circumference of the circle =2πr
circle = 2 x 3.14 x 6 where r =6 given
circle = 6.28 x  6
circle =37.68 sq mm

Question 4.
Big Ideas Math Answers 7th Grade Chapter 9 Geometric Shapes and Angles cr 6
Answer:
circumference of the object = 4.71 sq ft

Explanation:
D. circumference of the circle =2πr
circle = 2 x 3.14 x 0.75 where r =0.75 given
circle = 6.28 x  0.75
circle =4.71 sq ft

Question 5.
Big Ideas Math Answers 7th Grade Chapter 9 Geometric Shapes and Angles cr 7
Answer:
circumference of the object = 4.71 sq cm

Explanation:
D. circumference of the circle =2πr
circle = 2 x 3.14 x 3 .5 where r =3.5 given
circle = 6.28 x  3.5
circle =21.98  sq cm

Question 6.
You are placing non-slip tape along the perimeter of the bottom of a semicircle-shaped doormat. How much tape will you save applying the tape to the perimeter of the inside semicircle of the doormat? Justify your answer.
Big Ideas Math Answers 7th Grade Chapter 9 Geometric Shapes and Angles cr 8
Answer:
the tape saved = 47 .4 sq in
Explanation:
perimeter of the semicircle = ( π + 2 ) r
p = ( 3.14 + 2) 15
p = (3 .16 ) 15
p = 47.4 sq in

Question 7.
You need to carry a circular cake through a 32-inch wide doorway without tilting it. The circumference of the cake is 100 inches. Will the cake fit through the doorway? Explain.
Answer:
yes the cake fit through the doorway.
Explanation:
radius of the circle = (c/2π)
r= (100/6.28)
r = 15.923 sq in

Question 8.
Estimate the radius of the Big Ben clock face in London.
Big Ideas Math Answers 7th Grade Chapter 9 Geometric Shapes and Angles cr 9
Answer:
Radius of the Big Ben clock = 7.0063 m

Explanation:
radius of the circle = (c/2π)
r= (44/6.28)
r = 7.0063 m

Question 9.
Describe and solve a real-life problem that involves finding the circumference of a circle.
Answer:
The circumference of a circle = 2 π r

Explanation:
circle = 2 π r
where r = radius , π = 3.14

9.2 Areas of Circles (pp. 369-374)
Learning Target: Find the area of a circle.

Find the area of the circle. Use 3.14 or \(\frac{22}{7}\) for π.
Question 10.
Big Ideas Math Answers 7th Grade Chapter 9 Geometric Shapes and Angles cr 10
Answer:
The area of the circle = 50.24 sq in

Explanation:
Area of the circle = π x r x r
area = 3.14 x 4 x 4
area = 3.14 x 16
area = 50.24 sq in

Question 11.
Big Ideas Math Answers 7th Grade Chapter 9 Geometric Shapes and Angles cr 11
Answer:
The area of the circle = 379.94 sq cm

Explanation:
Area of the circle = π x r x r
area = 3.14 x 11 x 11
area = 3.14 x 121
area = 379.94 sq cm

Question 12.
Big Ideas Math Answers 7th Grade Chapter 9 Geometric Shapes and Angles cr 12
Answer:
The area of the circle = 1384.74 sq mm

Explanation:
Area of the circle = π x r x r
area = 3.14 x 21 x 21
area = 3.14 x 441
area = 1384.74 sq mm

Question 13.
A desktop is shaped like a semicircle with a diameter of 28 inches. What is the area of the desktop?
Answer:
The area of the desktop = 615.44 sq in

Explanation:
Area of the desktop = π x r x r
area = 3.14 x 14 x 14
area = 3.14 x 196
area = 615.44 sq in

Question 14.
An ecologist is studying an algal bloom that has formed on the entire surface of a circular pond. What is the area of the surface of the pond covered by the algal bloom?
Big Ideas Math Answers 7th Grade Chapter 9 Geometric Shapes and Angles cr 14
Answer:
The area of the surface of the pond covered by the algol bloom = 615.44 ft

Explanation:
Area of the pond = π x r x r
area = 3.14 x 14 x 14
area = 3.14 x 196
area = 615.44 sq ft

Question 15.
A knitted pot holder is shaped like a circle. Its radius is 3.5 inches. What is its area?
Answer:
The area of the pot holder = 38.465 sq in

Explanation:
Area of the pot holder = π x r x r
area = 3.14 x 3.5 x 3.5
area = 3.14 x 12.25
area = 38.465 sq in

9.3 Perimeters and Areas of Composite Figures (pp. 375–380)
Learning Target: Find perimeters and areas of composite figures.

Find the perimeter and the area of the figure.
Question 16.
Big Ideas Math Answers 7th Grade Chapter 9 Geometric Shapes and Angles cr 16
Answer:
Area of semicircle = 1.695 sq in
perimeter of semicircle = 15.8 sq in

Explanation:
area of the semicircle = ( π + r x r /2)
s . c = (3.14 + 5 x 5 /2)
s. c = (3.14 + 25/2)
s . c = (3.39 /2 )
s. c = 1.695 sq in
perimeter of the semicircle = (π + 2 ) x r
p= 3.14 + 2 x 5
p = 3.16 x 5
p = 15.8 sq in

Question 17.
Big Ideas Math Answers 7th Grade Chapter 9 Geometric Shapes and Angles cr 17
Answer:
Area of semicircle = 6.07 sq ft
perimeter of semicircle = 9.48 sqft

Explanation:
area of the semicircle = ( π + r x r /2)
s . c = (3.14 + 3 x 3 /2)
s. c = (3.14 + 9/2)
s . c = (12.14 /2 )
s. c = 6.07 sq ft
perimeter of the semicircle = (π + 2 ) x r
p= 3.14 + 2 x 3
p = 3.16 x 3
p = 9.48 sq ft

Question 18.
GARDEN
You want to fence part of a yard to make a vegetable garden. How many feet of fencing do you need to surround the garden?
Big Ideas Math Answers 7th Grade Chapter 9 Geometric Shapes and Angles cr 18
Answer:
The fencing need to surround the garden = 32 sq feet

Explanation:
area  of the rectangle = l + b
area = 18 + 14
area = 32 sq feet

9.4 Constructing Polygons (pp. 381-388)

Learning Target: Construct a polygon with given measures.

Draw a triangle with the given description, if possible.
Question 19.
a triangle with angle measures of 15°, 75°, and 90°
Answer:

Explanation:
Given triangle with angle measures.

Question 20.
a triangle with a 3-inch side and a 4-inch side that meet at a 30° angle
Answer:

Question 21.
a triangle with side lengths of 5 centimeters, 8 centimeters, and 2 centimeters
Answer:

Draw a quadrilateral with the given angle measures, if possible.
Question 22.
110°, 80°, 70°, 100°
Answer:

Question 23.
105°, 15°, 20°, 40°
Answer:

9.5 Finding Unknown Angle Measures (pp. 389–396)
Learning Target: Use facts about angle relationships to find unknown angle measures.

Use the figure shown.
Big Ideas Math Answers 7th Grade Chapter 9 Geometric Shapes and Angles cr 24
Question 24.
Name a pair of adjacent angles.
Answer:
x , y , v , w .

Explanation:
In the above-given figure, the adjacent angles are x, y, v, w.

Question 25.
Name a pair of complementary angles.
Answer:
u and z

Explanation:
complementary angles = u , z

Question 26.
Name a pair of supplementary angles.
Answer:
x , y , v , z

Explanation:
supplementary are x , y , v , z
Question 27.
Name a pair of vertical angles.
Answer:
x , y , v, w

Explanation:
pair of vertical angles are x , y , v , w

Classify the pair of angles. Then find the value of x.
Question 28.
Big Ideas Math Answers 7th Grade Chapter 9 Geometric Shapes and Angles cr 28
Answer:
x = 111 degrees.

Explanation :
x = 56
x =180  – 69
x = 111 degree

Question 29.
Big Ideas Math Answers 7th Grade Chapter 9 Geometric Shapes and Angles cr 29
Answer:
x = 81 degrees.

Explanation :
x + 3  = 84
x =84   – 3
x = 81  degree

Question 30.
Big Ideas Math Answers 7th Grade Chapter 9 Geometric Shapes and Angles cr 30
Answer:
x = 3.33degrees.

Explanation :
(4x + 10) = x
10 = x – 4 x
3 x = 10
x = 3.33  degree

Question 31.
Describe two ways to find the measure of ∠2.
Big Ideas Math Answers 7th Grade Chapter 9 Geometric Shapes and Angles cr 31
Answer:
angle 2 = 65

Explanation:
x = 180 – 115
x = 65
2 = 65

Question 32.
Using the diagram from Exercises 24–27, find all the angle measures when ∠XUY = 40°.
Answer:

Geometric Shapes and Angles Practice Test

Question 1.
Find the radius of a circle with a diameter of 17 inches.
Answer:
radius of a circle = 8.5 in

Explanation:
radius of a circle = (d / 2)
radius =( 17 / 2)
radius = 8.5 in
Find (a) the circumference and (b) the area of the circle. Use 3.14 or \(\frac{22}{7}\) for π.
Question 2.
Big Ideas Math Answers Grade 7 Chapter 9 Geometric Shapes and Angles pt 2
Answer:
Area of the circle = 3.14 m
circumference of the circle = 6.28 m

Explanation:
Area of the circle = π x r x r
area = 3.14 x 1 x 1
area = 3.14 m
circumference of the circle = 2 x π x r
c = 2 x 3.14 x 1
c = 6.28 m

Question 3.
Big Ideas Math Answers Grade 7 Chapter 9 Geometric Shapes and Angles pt 3
Answer:
Area of the circle = 3846.5 sq in
circumference of the circle = 219. 8 sq in

Explanation:
Area of the circle = π x r x r
area = 3.14 x 35 x 35
area = 3.14 x 1,225 sq in
area = 3846.5 sq in
circumference of the circle = 2 x π x r
c = 2 x 3.14 x 35
c = 6.28 x 35
c = 219.8 sq in

Find (a) the perimeter and (b) the area of the figure. Use 3.14 or \(\frac{22}{7}\) for π.
Question 4.
Big Ideas Math Answers Grade 7 Chapter 9 Geometric Shapes and Angles pt 4
Answer:
Area of semicircle = 2.695 sq ft
perimeter of semicircle = 4. 74 sq ft

Explanation:
area of the semicircle = ( π + r x r /2)
s . c = (3.14 + 1.5 x 1.5 /2)
s. c = (3.14 + 2.25 /2)
s . c = (5.39 /2 )
s. c = 2.695 sq ft
perimeter of the semicircle = (π + 2 ) x r
p= 3.14 + 2 x 1.5
p = 3.16 x 1.5
p = 4. 74 sq ft

Question 5.
Big Ideas Math Answers Grade 7 Chapter 9 Geometric Shapes and Angles pt 5
Answer:
Area of semicircle = 9.57 sq ft
perimeter of semicircle = 12.64 sq ft

Explanation:
area of the semicircle = ( π + r x r /2)
s . c = (3.14 + 4 x 4 /2)
s. c = (3.14 + 16 /2)
s . c = (19.14 /2 )
s. c = 9.57  sq ft
perimeter of the semicircle = (π + 2 ) x r
p= 3.14 + 2 x 4
p = 3.16 x 4
p = 12.64 sq ft

Draw a figure with the given description, if possible.
Question 6.
a triangle with sides of length 5 inches and 6 inches that meet at a 50° angle.
Answer:

Question 7.
a triangle with side lengths of 3 inches, 4 inches, and 5 inches
Answer:

Question 8.
a quadrilateral with angle measures of 90°, 110°, 40°, and 120°
Answer:

Classify each pair of angles. Then find the value of x.
Question 9.
Big Ideas Math Answers Grade 7 Chapter 9 Geometric Shapes and Angles pt 9
Answer:
x = 9 degrees.

Explanation:
(8x + 2) = 74
8x = 74 – 2
8x = 72
x = (72/8)
x = 9

Question 10.
Big Ideas Math Answers Grade 7 Chapter 9 Geometric Shapes and Angles pt 10
Answer:
x = 50 degrees.

Explanation:
(x + 6) = 56
x = 56 – 6
x = 50
Question 11.
Big Ideas Math Answers Grade 7 Chapter 9 Geometric Shapes and Angles pt 11
Answer:
x = 67 degrees.

Explanation:
x = 180 – 113
x = 67 degrees.

Question 12.
A museum plans to rope off the perimeter of the 60 ftL-shaped exhibit. How much rope does it need?
Big Ideas Math Answers Grade 7 Chapter 9 Geometric Shapes and Angles pt 12
Answer:
Area of the museum = 2,826 sq ft

Explanation:
Area of the museum  = π x r x r
area = 3.14 x 30 x 30
area = 3.14 x 900
area =  2,826 sq ft

Geometric Shapes and Angles Cumulative Practice

Question 13.
Draw a pair of adjacent angles that are neither complementary nor supplementary.
Answer:

Question 14.
The circumference of a circle is 36.2 centimeters. What is the length of the diameter of the circle?
Answer:
Diameter of the circle = 11.52866 cm

Explanation:
Diameter of the circle = 2 x r
radius of the circle = (c / 2 π )
circumference = 36.2 cm
radius = (36.2 / 6.28)
radius = 5.7643
daimeter = 2 x r
diameter = 5.7643 x 2
diameter = 11.52866 cm

Question 15.
The circular rug is placed on a square floor. The rug touches all four walls. How much of the floor space is not covered by the rug?
Big Ideas Math Answers Grade 7 Chapter 9 Geometric Shapes and Angles pt 15
Answer:
Area of the circle = 176.625sq ft

Explanation:
Area of the circle  = π x r x r
area = 3.14 x 7.5 x 7.5
area = 3.14 x 56.25
area =  176.625 sq ft

Geometric Shapes and Angles Cumulative Practice

Big Ideas Math Answers Grade 7 Chapter 9 Geometric Shapes and Angles cp 1
Question 1.
To make 6 servings of soup, you need 5 cup of chicken broth. You want to know how many servings you can make with 2 quarts of chicken broth. Which proportion should you use?
A. \(\frac{6}{5}=\frac{2}{x}\)
B. \(\frac{6}{5}=\frac{x}{2}\)
C. \(\frac{6}{5}=\frac{x}{8}\)
D. \(\frac{5}{6}=\frac{x}{8}\)
Answer:
option B is correct.

Explanation:
Given that in the question to make 6 servings of soup you need 5 cup of chicken broth.

Question 2.
What is the value of x?
Big Ideas Math Answers Grade 7 Chapter 9 Geometric Shapes and Angles cp 2
Answer:
x = 42 degrees.

Explanation:
(2x + 1) = 85
2x = 85 – 1
2x = 84
x = (84/2)
x = 42

Question 3.
Your mathematics teacher described an inequality in words.
Big Ideas Math Answers Grade 7 Chapter 9 Geometric Shapes and Angles cp 3
Which inequality matches your mathematics teacher’s description?
F. 7n – 5 < 42 G. (7 – 5)n > 42
H. 5 – 7n > 42
I. 7n – 5 > 42
Answer:
option G is correct.

Explanation:
5 is less than the product of 7 and an unknown number is greater than 42.
(7 – 5)n > 42

Question 4.
What is the approximate area of the circle below? (Use \(\frac{22}{7}\) for π).
Big Ideas Math Answers Grade 7 Chapter 9 Geometric Shapes and Angles cp 4
A. 132 cm2
B. 264 cm2
C. 5544 cm2
D. 22,176 cm2
Answer:
Area of the circle = 63.585 sq ft

Explanation:
Area of the circle  = π x r x r
area = 3.14 x 42 x 42
area = 3.14 x 1,764
area =  5,538.96 cm

Question 5.
You have a 50% chance of selecting a blue marble from Bag A and a 20% chance of selecting a blue marble from Bag B. Use the provided simulation to answer the question, “What is the estimated probability of selecting a blue marble from both bags?”
Big Ideas Math Answers Grade 7 Chapter 9 Geometric Shapes and Angles cp 5
F. 12%
G. 16%
H. 24%
I. 88%
Answer:
option F is correct.

Explanation:
The digits 1 and 2 in the ones place represent selecting a blue marble from bag B.
The digits 1 through 5  in the tens place represent selecting a blue marble from bag A.

Question 6.
Which proportion represents the problem?
“What number is 12% of 125?”
A. \(\frac{n}{125}=\frac{12}{100}\)
B. \(\frac{12}{125}=\frac{n}{100}\)
C. \(\frac{125}{n}=\frac{12}{100}\)
D. \(\frac{12}{n}=\frac{125}{100}\)
Answer:
option B is correct.

Explanation:
(12/125 ) x 100
(12/5) x 4
Question 7.
What is the approximate perimeter of the figure below? (Use 3.14 for π)
Big Ideas Math Answers Grade 7 Chapter 9 Geometric Shapes and Angles cp 7
Answer:
The perimeter of the semicircle = 18. 84

Explnation:
perimeter = ( π + 2 x r)
perimeter = (6.28 x 3 )
perimeter = 18 . 84

Question 8.
A savings account earns 2.5% simple interest per year. The principal is $850. What is the balance after 3 years?
F. $63.75
G. $871.25
H. $913.75
J. $7225
Answer:

Question 9.
Two ponds each contain about 400 fish. The double box-and-whisker plot represents the weights of a random sample of 12 fish from each pond. Which statement about the measures of center and variation is true?
Big Ideas Math Answers Grade 7 Chapter 9 Geometric Shapes and Angles cp 9
A. The variation in the samples is about the same, but the sample from Pond A has a greater median.
B. The variation in the samples is about the same, but the sample from Pond B has a greater median.
C. The measures of center and variation are about the same for both samples.
D. Neither the measures of center nor variation are the same for the samples.
Answer:
option D is correct.

Explanation:
Neither the measures of center nor variation are same for the samples.

Question 10.
A lawn sprinkler sprays water onto part of a circular region, as shown below.
Big Ideas Math Answers Grade 7 Chapter 9 Geometric Shapes and Angles cp 11
Part A What is the area, in square feet, of the region that the sprinkler sprays with water? Explain your reasoning. (Use 3.14 for π.)
Part B What is the perimeter, in feet, of the region that the sprinkler sprays with water? Explain your reasoning. (Use 3.14 for π.)
Answer:
part A The region that sprinkler sprays with water = 1,256 ft
part B The region that sprinkler sprays with water = 125 .6 ft

Explanation:
area of the circle = π x r x r
area = 3.14 x 20 x 20
area = 1256 ft
perimeter of the circle =  2 x π x r
perimeter = 2 x 3.14 x 20
perimeter = 125. 6 ft

Question 11.
What is the least value of x for which x – 12 ≥ – 8 is true?
F. – 20
G. – 4
H. 4
I. 5
Answer:
option F is correct.

Explanation:
x – 12 ≥ – 8
x = -20

Final Words:

Access Big Ideas Math Book 7th Grade Answer Key 9 Geometric Shapes and Angles from the direct links presented above. Hit the direct links and prepare yourself for the exam. With the help of the problems, you can test yourself and your capability of solving the problems. Cumulative practice, Chapter review, the Practice test will help you throughout your preparation.

Big Ideas Math Answers Grade 4 Chapter 6 Factors, Multiples, and Patterns

Big Ideas Math Answers Grade 4 Chapter 6

Get free step-by-step solutions and explanations for all the available questions here. The students who are looking for Big Ideas Math Answers Grade 4 Chapter 6 Factors, Multiples, and Patterns can download the pdf for free of cost. The BIM 4th Grade Chapter 6 contains all the topics that help the students to score good marks in the exams. The main aim of providing the Big Ideas Math Book 4th Grade Answer Key Chapter 6 Factors, Multiples, and Patterns is to make the students understand the concepts in an easy manner. Learn Big Ideas Grade 4 Math Answer Key with the help of the simple methods.

Big Ideas 4th Grade Math Book Answer Key Chapter 6 Factors, Multiples, and Patterns

The advantage of referring to our Big Ideas Math Answers Grade 4 Chapter 6 is you can refer to all the problems and also take practice tests for free of cost. Improve your math skills and score better marks in the exams with the help of the BIM 4th Grade Math Answer Key. Check out the below links for a better understanding of every topic. Just click on the links and grab the depth of knowledge on each topic. Practice real-time problems to gain more knowledge on concepts.

Lesson: 1 Understand Factors

Lesson: 2 Factors and Divisibility

Lesson: 3 Relate Factors and Multiples

Lesson: 4 Identify Prime and Composite Numbers

Lesson: 5 Number Patterns

Lesson: 6 Shape Patterns

Performance Task

Lesson 6.1 Understand Factors

Explore and Grow

Draw two different rectangles that each have an area of 24 square units. Label their side lengths.
Big Ideas Math Answer Key Grade 4 Chapter 6 Factors, Multiples, and Patterns 6.1 1
Compare your rectangles to your partner’s rectangles. How are they the same? How are they different?

Answer:

Explanation:
My rectangle is 6 × 4 and my partner’s rectangle is 3 × 8
We know that,
Area of rectangle = l × b
Hence,
According to the above formula, the areas of rectangles are the same irrespective of different lengths and different breadths.

Structure
How is each side length related to 24?

Answer: 
We know that,
The factors of 24 = 4 × 6, 3 × 8, 1 × 24, 2 × 12
Remember that,
a × b = b × a
So,
4 × 6 = 6 × 4
This pattern will be applicable to all factor pairs
Hence, from the above,
We can conclude that each side length is related to 24 because of the factor pair.

Think and Grow: Find Factor Pairs

You can write whole numbers as products of two factors. The two factors are called a factor pair for the number
Big Ideas Math Answer Key Grade 4 Chapter 6 Factors, Multiples, and Patterns 6.1 2

Example
Find the factor pairs for 20.
Find the side lengths of as many different rectangles with an area of 20 square units as possible.

The side lengths of each rectangle are a factor pair.
So, the factor pairs for 20 are 12 and 1,  10 and 2,  and 5 and 4.

Show and Grow

Question 1.
Use the rectangles to find the factor pairs for 12.
Big Ideas Math Answer Key Grade 4 Chapter 6 Factors, Multiples, and Patterns 6.1 4
Answer: Factor pairs of 12 are: 12 and 1, 6 and 2, and 4 and 3

Explanation:

The factor pairs are nothing but the side lengths of a rectangle and the area of a rectangle gives the factor
Hence,
The factor pairs of 12 are: 12 and 1, 6 and 2, and 4 and 3

Question 2.
Draw rectangles to find the factor pairs for 16.
Big Ideas Math Answer Key Grade 4 Chapter 6 Factors, Multiples, and Patterns 6.1 5
Answer: The factor pairs of 16 are: 1 and 16, 2 and 8, 4 and  4

Explanation:

The factor pairs are nothing but the side lengths of a rectangle and the area of a rectangle gives the factor
Hence,
The factor pairs of 16 are: 1 and 16, 2 and 8, 4 and  4

Apply and Grow: Practice

Draw rectangles to find the factor pairs for the number.
Question 3.
14
Big Ideas Math Answer Key Grade 4 Chapter 6 Factors, Multiples, and Patterns 6.1 6

Answer: The factor pairs of 14 are: 1 and 14, 2 and 7

Explanation:

Factors are the numbers that divide the original number completely. Hence,
The factor pairs of 14 are: 1 and 14, 2 and 7

Question 4.
15
Big Ideas Math Answer Key Grade 4 Chapter 6 Factors, Multiples, and Patterns 6.1 7

Answer:  The factor pairs of 15 are: 1 and 15, 3 and 5

Explanation:

Factors are the numbers that divide the original number completely. Hence,
The factor pairs of 15 are: 1 and 15, 3 and 5

Question 5.
20
Big Ideas Math Answer Key Grade 4 Chapter 6 Factors, Multiples, and Patterns 6.1 8

Answer: The factor pairs of 20 are: 1 and 20, 2 and 10, 4 and 5

Explanation:

Factors are the numbers that divide the original number completely. Hence,
The factor pairs of 15 are: 1 and 15, 3 and 5

Question 6.
36
Big Ideas Math Answer Key Grade 4 Chapter 6 Factors, Multiples, and Patterns 6.1 9
Answer: The factor pairs of 36 are: 1 and 36, 2 and 18, 3 and 12, 4 and 9, 6 and 6

Explanation:

Factors are the numbers that divide the number completely.
Hence,
The factor pairs of 36 are: 1 and 36, 2 and 18, 3 and 12, 4 and 9, 6 and 6

Find the factor pairs for the number.
Question 7.
11
Answer: The factor pairs of 11 are: 1 and 11

Explanation:

Factors are the numbers that divide the original number completely.Hence,
The factor pairs of 15 are: 1 and 15, 3 and 5

Question 8.
9
Answer: The factor pairs of 9 are: 1 and 9, and 3 and 3

Explanation:

Factors are the numbers that divide the original number completely. Hence,
The factor pairs of 15 are: 1 and 15, 3 and 5

Question 9.
4

Answer: The factor pairs of 4 are: 1 and 4, 2 and 2

Explanation:
Factors are the numbers that divide the original number completely. Hence,
The factor pairs of 4 are:
1 × 4, 2 × 2 ( Since 4 ×1 and 1 × 4 are equal, we will take any 1 factor )

Question 10.
25
Answer:  The factor pairs of 25 are: 1 and 25, 5 and 5

Explanation:
Factors are the numbers that divide the original number completely. Hence,
The factor pairs of 25 are:
1 × 25, 5 × 5

Question 11.
10
Answer: The factor pairs of 10 are: 1 and 10, 2 and 5

Explanation:
Factors are the numbers that divide the original number completely. Hence,
The factor pairs of 10 are:
1 × 10, 5 × 5

Question 12.
40
Answer: The factor pairs of 40 are: 1 and 40, 2 and 20, 4 and 10, 5 and 8

Explanation:
Factors are the numbers that divide the original number completely. Hence,
The factor pairs of 40 are:
1 × 40, 2 × 20, 4 × 10 and 5 × 8

Question 13.
Writing
Use the word to explain one way that 2 and 6 are related.
Answer: The factor pair of 6 are: 1 and 6, 2 and 3

Explanation:
The one way 2 and  are related is the “Factor-pair method”
According to the factor-pair method,
The factors of 6 are:
1 × 6 and 2 × 3
Hence, from the above,
We can conclude that 2 and 6 are related due to the factor-pair method.

Think and Grow: Modeling Real Life

Example
You want to organize 30 pictures into a rectangular array on a wall. How many different arrays can you make?
Big Ideas Math Answer Key Grade 4 Chapter 6 Factors, Multiples, and Patterns 6.1 10
To find the number of arrays you can make, find the number of factor pairs for 30.
There are 3-factor pairs for 30.
You can use each factor pair to make 30 arrays.
So, there are 30 ×3 = 90 ways to organize the pictures in different arrays.

Show and Grow

Question 14.
A city mayor buys 27 solar panels. She wants to organize the panels into a rectangular array. How many different arrays can she make?

Answer: The different arrays she can make are: 1 × 27, 3 × 9, 9 × 3, and 27 × 1

Explanation:
Given that a city mayor buys 27 solar panels and she wants to organize the panels into a rectangular array.
The “Array” is nothing but the number of patterns ( Factor pairs)  that we can arrange the given things.
Hence,
The factors of 27 are: 1 and 27, 3 and 9, 9 and 3, and 27 and 1
Hence,
The different arrays she can make in arranging the solar panels are:
1 × 27, 3 × 9, 9 × 3, and 27 × 1

Question 15.
DIG DEEPER!
A store owner has 42 masks to hang in a rectangular array on a wall. The owner does not have room for more than 10 masks in each row or column. What are the possible numbers of masks the owner should hang in each row?
Big Ideas Math Answer Key Grade 4 Chapter 6 Factors, Multiples, and Patterns 6.1 11

Answer: The number of possible masks the owner should hang in each row is: 7 masks

Explanation:
Given that a store owner has 42 masks to hang in a rectangular array on a wall.
So now,
The number of arrays ( Factor pairs ) of 42 are:
1 × 42, 2 × 21, 3 × 14, 6 × 7, 7 × 6, 14 × 3, 21 × 2, and 41 × 1
It is also given that the owner does not have enough room for more than 10 masks in each row or column.
So, from the above factor pairs, we can say that the possible array of masks is: 6 × 7 and 7 × 6
Hence, from the above,
we can conclude that the maximum number of masks the owner can put either in rows or columns is: 7 masks

Question 16.
A teacher wants to set up a chair for each of the 48 students in chorus. He wants to set up the chairs in a rectangular array. He can fit no more than 20 rows and no more than 30 chairs in each row in the room. What are the possible numbers of rows that he could set up?

Answer:  The possible number of rows that he could set up are: 16 rows

Explanation:
Given that a teacher wants to set up a chai for each of the 48 students in the chorus and he wants to set up the chairs in a rectangular array.
Hence, the number of arrays ( factor pairs) of 48 are:
1 × 48, 2 × 24, 3 × 16, 4 × 12, 6 × 8, 8 × 6, 12 × 4, 16 × 3, 24 × 2, and 48 × 1
It is also given that he can fit no more than 20 rows and no more than 30 chairs in each row in the room.
Hence, from the above,
We can conclude that the possible number of rows that he could set up are: 16 rows

Understand Factors Homework & Practice 6.1

Question 1.
Use the rectangles to find the factor pairs for 8.
Big Ideas Math Answer Key Grade 4 Chapter 6 Factors, Multiples, and Patterns 6.1 12

Answer: The factors of 8 are: 1 and 8, 2 and 4

Explanation:

Factors are the numbers that divide the original number completely. Hence,
The factor pairs of 8 are: 1 × 8 and 2 × 4

Question 2.
Draw rectangles to find the factor pairs for 21.
Big Ideas Math Answer Key Grade 4 Chapter 6 Factors, Multiples, and Patterns 6.1 13

Answer: The factor pairs of 21 are: 1 × 21, 3 × 7

Explanation:

Factors are the numbers that divide the original number completely. Hence,
The factor pairs of 21 are: 1 × 21, 3 × 7

Question 3.
Draw rectangles to find the factor pairs for 28.
Big Ideas Math Answer Key Grade 4 Chapter 6 Factors, Multiples, and Patterns 6.1 14

Answer: The factor pairs of 28 are: 1 × 28, 2 × 14, 4 × 7

Explanation:


Factors are the numbers that divide the original number completely. Hence,
The factor pairs of 28 are: 1 × 28, 2 × 14, 4 × 7

Find the factor pairs for the number.
Question 4.
13
Answer: The factor pairs of 13 are: 1 × 13 and 13 × 1

Explanation:
Factors are the numbers that divide the original number completely. Hence,
The factor pairs of 13 are:
1 × 13 and 13 × 1

Question 5.
5
Answer: The factor pairs of 5 are: 1 × 5 and 5 × 1

Explanation:
Factors are the numbers that divide the original number completely.
Hence,
The factor pairs of 5 are:
1 × 5 and 5 × 1

Question 6.
35
Answer: The factor pairs of 35 are: 1 × 35, 5 × 7

Explanation:
Factors are the numbers that divide the original number completely.
Hence,
The factor pairs of 35 are:
1 × 35, 5 × 7

Question 7.
45
Answer: The factor pairs of 45 are: 1 × 45, 3 × 15, 5× 9

Explanation:
Factors are the numbers that divide the original number completely.
Hence,
The factor pairs of 45 are:
1 × 45, 3 × 15, 5× 9

Question 8.
18
Answer: The factor pairs of 18 are: 1 × 18, 2 × 9, 3 × 6

Explanation:
Factors are the numbers that divide the original number completely.
Hence,
The factor pairs of 18 are:
1 × 18, 2 × 9, 3 × 6

Question 9.
36
Answer: The factor pairs of 36 are: 1 × 36, 2 × 18, 3 × 12, 4 × 9, 6 × 6

Explanation:
Factors are the numbers that divide the original number completely.
Hence,
The factor pairs of 36 are:
1 × 36, 2 × 18, 3 × 12, 4 × 9, 6 × 6

Question 10.
YOU BE THE TEACHER
Descartes says there are 5-factor pairs for 16. Newton says there are 3-factor pairs for 16. Who is correct? Explain.
Big Ideas Math Answer Key Grade 4 Chapter 6 Factors, Multiples, and Patterns 6.1 15

Answer:  Newton is correct

Explanation:
We know that,
a × b = b × a
Hence,
The factor pairs of 16 are:
1 × 16, 2 × 8, 4 × 4, 8 × 2, 16 × 1
But according to the above,
We don’t have to consider the factor pairs 8 × 2 and 16 × 1
Hence,
There are only 3-factor pairs.
But, Descartes says that there are 5-factor pairs for 16 while Newton says it three.
Hence, from the above,
We can conclude that Newton is correct.

Question 11.
Modeling Real Life
A race volunteer has 50 cases of bottled water. He wants to arrange the cases into a rectangular array. How many different arrays can he make?
Big Ideas Math Answer Key Grade 4 Chapter 6 Factors, Multiples, and Patterns 6.1 16

Answer: The number of different arrays he can make is: 6 arrays

Explanation:
Given that a race volunteer has 50 cases of bottled water and he wants to arrange the cases into a rectangular array.
The array is nothing but the factor pairs.
Hence, the number of arrays of 50 he can make = 1 × 50, 2 × 25, 5 × 10, 10 × 5, 25 × 2, 50 × 1
Hence, from the above,
we can conclude that the number of different arrays to arrange the 50 cases of bottled water are:
a) 1 × 50 b)  2 × 25 c)  5 × 10 d)  10 × 5 e)  25 × 2 f)  50 × 1

Review & Refresh

Find the product.
Question 12.
2 × 14 = _____
Answer: 2 × 14 = 28

Explanation:
According to the Distributive Property of Multiplication,
2 × 14 = ( 10 + 4 ) × 2
= ( 2 × 10 ) + ( 2 × 4 )
= 20 + 8
= 28
Hence, 2 × 14 = 28

Question 13.
22 × 7 = ______
Answer: 22 × 7 = 154

Explanation:
According to the Distributive Property of Multiplication,
22 × 7 = ( 20 + 2 ) × 7
= ( 20 × 7 ) + ( 2 × 7 )
= 140 + 14
= 154
Hence, 22 × 7 = 154

Question 14.
9 × 27 = ______
Answer: 27 × 9 = 243

Explanation:
According to the Distributive Property of Multiplication,
27 × 9 = ( 20 + 7 ) × 9
= ( 20 × 9 ) + ( 7 × 9 )
= 180 + 63
= 243
Hence, 27 × 9 = 243

Lesson 6.2 Factors and Divisibility

Explore and Grow

List any 10 multiples of 3. What do you notice about the sum of the digits in each multiple?
Answer:
The 10 multiples of 3 are:
3, 6, 9, 12, 15, 18 ,21, 24, 27, and 30
From the sum of the digits in the above multiples,
0 + 3 =3
0 + 6 = 6
0 + 9 = 9
1 + 2 = 3
1 + 5 = 6
2 + 1 = 3
2 + 4= 6
2 + 7 = 9
3 + 0 = 3
Hence, from the above sums,
We can conclude that the sum of the digits in the 10 multiples are also the multiples of 3.

List any 10 multiples of 9. What do you notice about the sum of the digits in each multiple?
Answer:
The 10 multiples of 9 are:
9, 18, 27, 36, 45, 54, 63, 72, 81, and 90
From the sum of the digits in the above multiples,
0 + 9 = 9
1 + 8 = 9
2 + 7 = 9
3 + 6 = 9
4 + 5 = 9
5 + 4 = 9
6 + 3 = 9
7 + 2 = 9
8 + 1 = 9
9 + 0 = 9
Hence, from the above,
We can conclude that the sum of the digits in all the multiples of 9 is equal to 9 only.

Structure
How can you use your observations above to determine whether 3 and 9 are factors of a given number? Explain.
Answer: The factors of 9 are: 1,3, and 9

Explanation:
The factor pairs of 9 are:
1 × 9, 3 × 3 and 9 × 1
Hence, from the above,
We can conclude that 3 and 9 are the factors of 9.

Think and Grow: Find Factors and Factor Pairs

A number is divisible by another number when a quotient is a whole number and the remainder is 0.

Some numbers have divisibility rules that you can use to determine whether they are factors of other numbers.
Big Ideas Math Answers 4th Grade Chapter 6 Factors, Multiples, and Patterns 6.2 1
Example
Find the factor pairs for 48.
Use divisibility rules and division to find the factors of 48.

The factors of 48 are 1, 2, 3, 4, 6, 8, 12, 16, 24, and 48.
The factor pairs for 48 are 6.

Show and Grow

Find the factor pairs for the number.
Question 1.
30
Answer:
The factor pairs of 30 are:
1 × 30, 2 × 15, 3 × 10, 5 × 6, 6 × 5, 10 × 3, 15 × 2, 30 × 1

Explanation:
Factors are the numbers that divide the original number completely.
Hence,
The factor pairs of 30 are:
1 × 30, 2 × 15, 3 × 10, 5 × 6, 6 × 5, 10 × 3, 15 × 2, 30 × 1

Question 2.
54
Answer:
The factor pairs of 54 are:
1 × 54, 2 × 27, 3 × 18, 6 × 9, 9 × 6, 18 × 3, 27 × 2, 54 × 1

Explanation:
Factors are the numbers that divide the original number completely.
Hence,
The factor pairs of 54 are:
1 × 54, 2 × 27, 3 × 18, 6 × 9, 9 × 6, 18 × 3, 27 × 2, 54 × 1

Apply and Grow: Practice

Find the factor pairs for the number.
Question 3.
29
Answer:
The factor pairs of 29 are:
1 × 29 and 29 × 1

Explanation:
Factors are the numbers that divide the original numbers completely.
Hence,
The factor pairs of 29 are:
1 × 29 and 29 × 1

Question 4.
50
Answer:
The factor pairs of 50 are:
1 × 50, 2 × 25, 5 × 10, 10 × 5, 25 × 2, and 50 × 1

Explanation:
Factors are the numbers that divide the original numbers completely.
Hence,
The factor pairs of 50 are:
1 × 50, 2 × 25, 5 × 10, 10 × 5, 25 × 2, and 50 × 1

Question 5.
63
Answer:
The factor pairs of 63 are:
1 × 63, 3 × 21, 7 × 9, 9 × 7, 21 × 3 and 63 × 1

Explanation:
Factors are the numbers that divide the number originally.
Hence,
The factor pairs of 63 are:
1 × 63, 3 × 21, 7 × 9, 9 × 7, 21 × 3 and 63 × 1

Question 6.
33
Answer:
The factor pairs of 33 are:
1 × 33, 3 × 11, 11 × 3 and 33 × 1

Explanation:
Factors are the numbers that divide the number originally.
Hence,
The factor pairs of 33 are:
1 × 33, 3 × 11, 11 × 3 and 33 × 1

Question 7.
60
Answer:
The factor pairs of 60 are:
1 × 60, 2 × 30, 3 × 20, 4 × 15, 5 × 12, and 6 × 10

Explanation:
Factors are the numbers that divide the number originally.
Hence,
The factor pairs of 60 are:
1 × 60, 2 × 30, 3 × 20, 4 × 15, 5 × 12, and 6 × 10

Question 8.
64
Answer:
The factor pairs of 64 are:
1 × 64, 2 × 32, 4 × 16, and 8 ×8

Explanation:
Factors are the numbers that divide the number originally.
Hence,
The factor pairs of 64 are:
1 × 64, 2 × 32, 4 × 16, and 8 ×8

List the factors of the number.
Question 9.
39
Answer:
The factors of 39 are: 1, 3, 13, and 39

Explanation:
Factors are the numbers that divide the number originally.
Hence,
The factors of 39 are: 1, 3, 13, and 39

Question 10.
44
Answer:
The factors of 44 are: 1, 2, 4, 11, 22, and 44

Explanation:
Factors are the numbers that divide the number originally.
Hence,
The factors of 44 are: 1, 2, 4, 11, 22, and 44

Question 11.
72
Answer:
The factors of 72 are: 1, 2, 3,4, 6, 8, 9, 12, 18, 24, 36, and 72

Explanation:
Factors are the numbers that divide the number originally.
Hence,
The factors of 72 are: 1, 2, 3,4, 6, 8, 9, 12, 18, 24, 36, and 72

Question 12.
67
Answer:
The factors of 7 are: 1 and 67

Explanation:
Factors are the numbers that divide the number originally.
Hence,
The factors of 7 are: 1 and 67

Question 13.
42
Answer:
The factors of 42 are: 1, 2, 3, 6, 7, 14, 21, and 42

Explanation:
Factors are the numbers that divide the number originally.
Hence,
The factors of 42 are: 1, 2, 3, 6, 7, 14, 21, and 42

Question 14.
28
Answer:
The factors of 28 are:
1, 2, 4, 7, 14, and 28

Explanation:
Factors are the numbers that divide the number originally
Hence,
The factors of 28 are:
1, 2, 4, 7, 14, and 28

Question 15.
Reasoning
Can an odd number have an even factor? Explain.
Answer: No, an odd number doesn’t have an even factor

Explanation:
Let the number which we want to find the factors is: 15
The factors of 15 are:
1, 3, 5, and 15
Hence, from the above factors,
We can conclude that an odd number doesn’t have an even factor.
Note: The “Odd number” is the number that can’t be divisible by 2.

Question 16.
Writing
Use the diagram to explain why you do not have to check whether any numbers greater than 4 are factors of 12.
Big Ideas Math Answers 4th Grade Chapter 6 Factors, Multiples, and Patterns 6.2 3
Answer: We don’t have to check the numbers greater than 4 are the factors of 12 because the factors of 12 greater than 4 are repeating.

Explanation:
The given number is 12
The factors of 12 are:
1, 2, 3, 4, 6, and 12
Hence, from the above
We can conclude that we don’t have to check the numbers greater than 4 are the factors of 12.

Think and Grow: Modeling Real Life

Example
There are 4 classes going on a field trip. The classes will use 3 buses. Can the teachers have an equal number of students on each bus?
Big Ideas Math Answers 4th Grade Chapter 6 Factors, Multiples, and Patterns 6.2 4
Think: What do you know? What do you need to find? How will you solve it?
Step 1: Add to find how many students are going on the field trip.
24 + 23 + 25 + 20 = 92
92 students are going on the field trip.
Step 2: Is the total number of students divisible by the number of buses?
Find the sum of the digits of 92. 9 + 2 =11
The sum of the digits not divisible by 3.
The teachers don’t have an equal number of students on each bus.

Show and Grow

Question 17.
A teacher is making a 5-page test with 28 vocabulary problems and 7 reading problems. Can the teacher put an equal number of problems on each page?
Answer: Yes, the teacher can put an equal number of problems on each page

Explanation:
Given that a teacher is making a 5- page test.
It is also given that there are 28 vocabulary problems and 7 reading problems.
So,
Total number of problems = 28 + 7 = 35 problems
So,
The number of problems that each page contains = 35 ÷ 5
Now,
By using the Distributive Property of Multiplication,
35 ÷ 5 = ( 30 + 5 ) ÷ 5
= ( 30 ÷ 5 ) + ( 5 ÷ 5 )
= 6 + 1
= 7
Hence, from the above,
We can conclude that the teacher can put an equal number of problems on each page.

Question 18.
A relay race is 39 laps long. Each team member must bike the same number of laps. Could a team have 8, 6, or 3 members? Explain.
Answer: A team has 3 members.

Explanation:
Given that a relay race is 39 laps long and each member must bike the same number of laps.
Now,
The factors of 39 are: 1, 3, 13, and 39
The factors are the numbers that divide the number originally.
So, 39 can be divided by 3 only.
Hence, from the above,
We can conclude that each team has only 3 members.

Question 19.
DIG DEEPER!
You have 63 clay figures to display on 7 shelves. Not all of the shelves need to be used and each shelf can hold no more than 25 figures. Each shelf must have the same number of figures. What are all the ways you could arrange the figures?
Big Ideas Math Answers 4th Grade Chapter 6 Factors, Multiples, and Patterns 6.2 5
Answer: The number of ways you could arrange the figures is to find the number of factors of 63.
So,
The number of factors of 63 is: 1, 3, 7,9 21, and 63

Explanation:
Given that you have 63 clay figures to display on 7 shelves.
It is also given that each shelf holds no more than 25 figures.
Now,
The factors of  63 are: 1, 3, 7,9 21, and 63
The “Factors” are the ways to arrange the given clay figures.
So,
The number of ways to arrange the clay figures that do not hold more than 25 is: 3, 7, 9, and 21
Hence, from the above,
We can conclude that the number of ways to arrange the clay figures in each shelf that do not hold more than 25 figures is: 3, 7, 9, and 21 ways

Factors and Divisibility Homework & Practice 6.2

Find the factor pairs for the number.
Question 1.
24
Answer:
The factor pairs of 24 are:
1 × 24, 2 × 12, 3 × 8, 4 × 6, 6 × 4, 8 × 3, 12 × 2, and 24 × 1

Explanation:
Factors are the numbers that divide the number originally.
Hence,
The factor pairs of 24 are:
1 × 24, 2 × 12, 3 × 8, 4 × 6, 6 × 4, 8 × 3, 12 × 2, and 24 × 1

Question 2.
48
Answer:
The factor pairs of 48 are:
1 × 48, 2 × 24, 3 × 16, 4 × 12, 6 × 8

Explanation:
Factors are the numbers that divide the number originally.
Hence,
The factor pairs of 48 are:
1 × 48, 2 × 24, 3 × 16, 4 × 12, 6 × 8

Question 3.
31
Answer:
The factor pairs of 31 are:
1 × 31 and 31 × 1

Explanation:
Factors are the numbers that divide the number originally.
Hence,
The factor pairs of 31 are:
1 × 31 and 31 × 1

Question 4.
99
Answer:
The factor pairs of 99 are:
1 × 99, 3 × 33, 9 × 11

Explanation:
Factors are the numbers that divide the number originally.
Hence,
The factor pairs of 99 are:
1 × 99, 3 × 33, 9 × 11

Question 5.
45
Answer:
The factor pairs of 45 are:
1 × 45, 3 × 15, and 5 × 9

Explanation:
factors are the numbers that divide the number originally.
Hence,
The factor pairs of 45 are:
1 × 45, 3 × 15, and 5 × 9

Question 6.
26
Answer:
The factor pairs of 26 are:
1 × 26, 2 × 13

Explanation:
Factors are the numbers that divide the number originally.
Hence,
The factor pairs of 26 are:
1 × 26, 2 × 13

List the factors of the number.
Question 7.
25
Answer:
The factors of 25 are: 1, 5, and 25

Explanation:
Factors are the numbers that divide the numbers originally.
Hence,
The factors of 25 are: 1, 5, and 25

Question 8.
56
Answer:
The factors of 56 are: 1, 2, 4, 7, 8, 14, 28 and 56

Explanation:
Factors are the numbers that divide the number originally.
Hence,
The factors of 56 are: 1, 2, 4, 7, 8, 14, 28 and 56

Question 9.
75
Answer:
The factors of 75 are: 1, 3, 5, 15, 25, and 75

Explanation:
Factors are the numbers that divide the number originally.
Hence,
The factors of 75 are: 1, 3, 5, 15, 25, and 75

Question 10.
80
Answer:
The factors of 80 are: 1, 2, 4, 5, 8, 10, 16, 20,40 and 80

Explanation:
Factors are the numbers that divide the numbers originally.
Hence,
The factors of 80 are: 1, 2, 4, 5, 8, 10, 16, 20,40 and 80

Question 11.
93
Answer:
The factors of 93 are: 1, 3, 31 and 93

Explanation;
Factors are the numbers that divide the number originally.
Hence,
The factors of 93 are: 1, 3, 31 and 93

Question 12.
61
Answer:
The factors of 61 are: 1 and 61

Explanation:
Factors are the numbers that divide the number originally.
Hence,
The factors of 61 are: 1 and 61

Question 13.
Reasoning
Why does a number that has 9 as a factor also have 3 as a factor?
Answer: The number that has 9 as a factor also have 3 as a factor because 9 is a multiple of 3

Explanation:
Let the number that has 9 as a factor and that has also 3 as a factor be: 18
Now,
The factors of 18 are: 1, 2, 3, 6, 9, and 18
Hene, from the above,
We can conclude that the number that has 9 as a factor also have 3 as a factor.

Question 14.
DIG DEEPER!
The number below has 3 as a factor. What could the unknown digit be?
3 _____ 5.
Answer: The unknown digit could be: 1 or 4 or 7

Explanation:
Given that the number has 3 as a factor.
To have 3 as a factor, the sum of the digits in the given number should be a multiple of 3
Hence,
The unknown digit in 3____5 could be: 1 or 4 or 7

Question 15.
Number Sense
Which numbers have 5 as a factor?
Big Ideas Math Answers 4th Grade Chapter 6 Factors, Multiples, and Patterns 6.2 6
Answer: The numbers which have 5 as a factor are: 50, 25, 1,485 and 100

Explanation:
Given numbers are: 50, 34, 25, 1,485, 100 and 48
The numbers that have the factor of 5 must have the last digits 0 or 5
Hence,
The numbers that have 5 as a factor are: 50, 25, 1,485, and 100

Question 16.
Modeling Real Life
You and a partner are conducting a bottle flipping experiment. You have 3 bottles with different amounts of water in each. You need to flip each bottle 15 times. If you take turns, will you and your partner each get the same number of flips?
Answer: Yes, you and your partner will each get the same number of flips.

Explanation:
Given that you and your partner are conducting a bottle flipping experiment and you have 3 bottles with different amounts of water in each and you need to flip each bottle 15 times.
So,
The number of flips each will get = 15 ÷ 3
Now,
By using the Distributive Property of Multiplication,
15 ÷ 3 = ( 12 + 3 ) ÷ 3
= ( 12 ÷ 3 ) + ( 3 ÷ 3 )
= 4 + 1
= 5
Hence, from the above,
We can conclude that each bottle will flip 5 times.

Question 17.
Modeling Real Life
A florist has 55 flowers. She wants to put the same number of flowers in each vase without any leftover Should she put 2, 3, or 5 flowers in each vase? Explain.
Big Ideas Math Answers 4th Grade Chapter 6 Factors, Multiples, and Patterns 6.2 7
Answer: The florist put 5 flowers in each vase.

Explanation:
Given that a florist has 55 flowers and she wants to put the same number of flowers in each vase without any leftover.
The number 55 will be divided by to not have any leftover because the last digit is 5.
Hence,
The florist puts 5 flowers in each vase.

Review & Refresh

Compare.
Question 18.
Big Ideas Math Answers 4th Grade Chapter 6 Factors, Multiples, and Patterns 6.2 8
Answer: 7,914 is greater than 7,912

Explanation:
Given numbers are 7,914 and 7,912
Hence, from the above,
We can conclude that 7,914 is greater than 7,912

Question 19.
Big Ideas Math Answers 4th Grade Chapter 6 Factors, Multiples, and Patterns 6.2 9
Answer: 65,901 is less than 67,904

Explanation:
Given numbers are 65,901 and 67,904
Hence, from the above,
We can conclude that 65,901 is less than 67,904

Question 20.
Big Ideas Math Answers 4th Grade Chapter 6 Factors, Multiples, and Patterns 6.2 10
Answer: 839,275 is equal to 839,275

Explanation:
Given numbers are 839,275 and 839,275
Hence, from the above,
We can conclude that 839,275 is equal to 839,275

Lesson 6.3 Relate Factors and Multiples

Explore and Grow

List all factors of 24.
Answer:
The factors of 24 are: 1, 2, 3, 4, 6, 8, 12 and 24

Explanation:
Factors are the numbers that divide the numbers originally.
Hence,
The factors of 24 are: 1, 2, 3, 4, 6, 8, 12 and 24

List several multiples of each factor. What number appears in each list?
Answer:
The factors of 24 are: 1, 2, 3, 4, 6, 8, 12 and 24

Explanation:
The factors of 24 are: 1, 2, 3, 4, 6, 8, 12 and 24
Now,
There are no other multiples of 1
The multiples of 2 are: 1, 2
The multiples of 3 are: 1,3
The multiples of 4 are: 1, 2, 4
The multiples of 6 are: 1, 2, 3 and 6
The multiples of 8 are: 1, 2, 4 and 8
The multiples of 12 are: 1, 2, 3, 4, 6, and 12
The multiples of 24 are: 1, 2, 3, 4, 6, 8, 12 and 24
Hence, from the above,
We can conclude that 1 appears in each list

Number Sense
How are factors and multiples related?
Answer:
Factors are the numbers that can divide the numbers originally.
Multiples are the numbers that can be divided by another number a certain number of times without any remainder.

Think and Grow: Identify Multiples

A whole number is a multiple of each of its factors.
12 is a multiple of 1, 2, 3, 4, 6, and 12.
1 × 12 = 12
2 × 6 = 12
3 × 4 = 12
4 × 3 = 12
6 × 2 = 12
12 × 1 = 12

Example
Is 56 a multiple of 7?
Big Ideas Math Answers Grade 4 Chapter 6 Factors, Multiples, and Patterns 6.3 1
One Way:
List multiples of 7.
7, 14, 21, 28, 35, 42, 49, 56
So, 56  is a multiple of 7.
Another Way:
Use division to determine whether 7 is a factor of 56.
56 ÷ 7 = 8
7 is a factor of 56.
So, 56 is a multiple of 7.

Example
Is 9 a factor of 64?
One Way:
Use divisibility rules to determine whether 9 is a factor of 64.
9 is not a factor of 64 because 6 + 4 = 10 is not divisible by 9.
Another Way:
List the multiples of 9.
9, 18, 27, 36, 45, 54, 63, 72
64 is not a multiple of 9.
So, 9 is not a factor of 64.

Show and Grow

Question 1.
Is 23 a multiple of 3? Explain.
Answer: 23 is not a multiple of 3

Explanation:

Multiples are the numbers that can be divided by another number a certain number of times without any remainder.
The multiples of 3 are: 3, 6, 9, 12, 15, 18, 21, 24, 27 and 30
Hence, from the above,
We can conclude that 23 is not a multiple of 3.

Question 2.
Is 8 a factor of 56? Explain.
Answer: 8 is a factor of 56.

Explanation:
Factors are the numbers that divide the numbers originally.
Now,
The factors of 56 are: 1, 2, 4, 7, 8, 14, 28, and 56
Hence, from the above,
We can conclude that 8 is a factor of 56.

Apply and Grow: Practice

Question 3.
Is 65 a multiple of 5? Explain.
Answer: 65 is a multiple of 5

Explanation:
Multiples are the numbers that can be divided by another number a certain number of times without any remainder.
The multiples of 5 are: 5, 10, 15, 20, 25, 30, 35,40, 45, 50, 55, 60 and 65
Hence, fro the above,
We can conclude that 65 is a multiple of 5.

Question 4.
Is 14 a multiple of 4? Explain.
Answer: 14 is not a multiple of 4.

Explanation:
Multiples are the numbers that can be divided by another number a certain number of times without any remainder.
The multiples of 4 are: 4, 8, 12, 16, 20, 24, 28, 32, 36, and 40
Hence, from the above,
We can conclude that 14 is not a multiple of 4.

Question 5.
Is 23 a multiple of 2? Explain.
Answer: 23 is not a multiple of 2

Explanation:
Multiples are the numbers that can be divided by another number a certain number of times without any remainder.
The multiples of 2 are: 2, 4, 6, 8, 10, 1, 14, 16, 18, 20, 22, 24 and 26
Hence, from the above,
We can conclude that 23 is not a multiple of 2.

Question 6.
Is 6 a factor of 96? Explain.
Answer: 6 is a factor of 96

Explanation:
Factors are the numbers that divide the number originally.
The factors of 96 are: 1, 2, 3, 4, 6, 8, 12, 16, 28, 32, 48 and 96
Hence, from the above,
We can conclude that 6 is a factor of 96.

Question 7.
Is 3 a factor of 82? Explain.
Answer: 82 is not a factor of 3

Explanation:
Factors are the numbers that divide the number originally.
The given number is 82
The sum of digits of 82 = 8 + 2 = 11
The number is a multiple of 3 only then the sum of the digits of that number is a multiple of 3.
But the sum of digits of 82 is not a multiple of 3
Hence, from the above,
we can conclude that 3 is not a factor of 82

Question 8.
Is 9 a factor of 72? Explain.
Answer: 9 is a factor of 72

Explanation:
Factors are the numbers that divide the number originally.
The factors of 72 are: 1, 2, 3, 4, 6, 8, 9, 12, 18, 24, 36, and 72
Hence, from the above,
We can conclude that 9 is a factor of 72.

Tell whether 8 is a multiple or a factor of the number. Write multiple, factor or both.
Question 9.
4
Answer: 8 is a multiple of 4.

Explanation:
Multiples are the numbers that can be divided by another number a certain number of times without any remainder.
So,
The multiples of 4 are: 4, 8, 12, 16, and 20
Hence, from the above,
We can conclude that 8 is a multiple of 4

Question 10.
8
Answer: 8 is a multiple of 8

Explanation:
Multiples are the numbers that can be divided by another number a certain number of times without any remainder.
So,
The multiples of 8 are: 8, 16, 24, 32, 0, etc.
Hence, from the above,
We can conclude that 8 is a multiple of 8

Question 11.
32
Answer: 32 is a multiple of 8

Explanation:
Multiples are the numbers that can be divided by another number a certain number of times without any remainder.
So,
The multiples of 8 are: 8, 16, 24, 32, 40, 48 etc
Hence, from the above,
We can conclude that 32 is a multiple of 8

Question 12.
Writing
Use numbers 6 and 12 to explain how factors and multiples are related.
Answer:
The multiplication Expression using numbers 6 and 12 is:
6 × 2 = 12
From the multiplication Expression,
6 is a factor of 12
12 is a multiple of 6

Explanation:
The given numbers re 6 and 12
From the given numbers, the multiplication Expression is:
6 × 2 = 12
Hence, from the above multiplication Expression,
6 is a factor of 12
12 is a multiple of 6.

Question 13.
Complete the Venn diagram.
Big Ideas Math Answers Grade 4 Chapter 6 Factors, Multiples, and Patterns 6.3 2
Answer:

Explanation:
From the given Venn diagram,
The factors of 40 are: 1, 2, 4, 5, 8, 10, 20, and 40
The first 10 multiples of 5 are: 5, 10, 15, 20, 25, 30, 35, 40, 45, and 50
Hence, from the above,
The common numbers from both the factors of 4 and the multiples of 5 are:
5, 10, 20 and 40

Think and Grow: Modeling Real Life

Example
You need 96 balloons for a school dance. Balloons come in packs of 4, packs of 6, and packs of 9. Which packs could you buy so you have no leftover balloons?
Big Ideas Math Answers Grade 4 Chapter 6 Factors, Multiples, and Patterns 6.3 3
Use division to determine whether 96 is a multiple of 4.
4 √96 So, 4  is a factor of 96, and 96  is a multiple of 4.
Use the divisibility rules to check whether 96 is a multiple of 6.
96 is even and 9 + 6 = 15 divisible by 3. So, 6  is a factor of 96 and 96  is a multiple of 6.
Use the divisibility rules to check whether 96 is a multiple of 9.
9 + 6 = 15  is not  divisible by 9. So, 9  is not a factor of 96 and
96  is not a multiple of 9.
You could buy packs of 4 balloons or packs of 6 balloons.

Show and Grow

Question 14.
A teacher needs 88 batteries for science experiments. Batteries are sold in packs of 2, packs of 6, and packs of 8. Which packs could the teacher buy so she has no leftover batteries?
Big Ideas Math Answers Grade 4 Chapter 6 Factors, Multiples, and Patterns 6.3 4
Answer: The teacher should buy the packs of 2 and packs of 8 so that there will be no leftover batteries.

Explanation:
Given that a teacher needs 88 batteries for science experiments and that batteries are sold in packs of 2, 6, and 8.
So,
The factors of 88 are: 1, 2, 4, 8, 11, 22, 44, and 88
So, from the factors of 88, we see that there is no 6 as a factor of 88
So, from this, we can say that we can’t pack 88 batteries in packs of 6 but in the packs of 2 and 8 ( Since 2 and 8 both are the factors of 88 )
We know that,
Factors are the numbers that divide the number originally.
Hence, from the above,
We can conclude that the 88 batteries can be packed in packs of 2 or packs of 8.

Question 15.
DIG DEEPER!
Descartes buys 2 books for a total of $15. Each book costs a multiple of $3. How much could each book cost?
Big Ideas Math Answers Grade 4 Chapter 6 Factors, Multiples, and Patterns 6.3 5

Answer: The cost of each book that Descartes bought is: $15

Explanation:
Given that Descartes bought 3 books for a total of $15 and each book costs a multiple of $3
So,
The cost of each book = The total cost of 2 books ÷ Total number of books
= 15 ÷ 2
Now,
By using the Distributive Property of Multiplication,
15 ÷ 2 = ( 12 + 3 ) ÷ 2
( 12 ÷ 2 ) + ( 3 ÷ 2 )
= 6 + 1.5
= 7.5
Hence, from the above,
We can conclude that the cost of each book is: $7.5 and the cost of each book is also a multiple of $3

Question 16.
Newton buys some boxes of dog treats for $9 each. Descartes buys some bags of cat treats for $6 each. Newton and Descartes spend the same amount of money on treats. What is the least amount of money they could have spent?

Answer: The least amount of money that Newton and Descartes spent on dog treats = $18

Explanation:
Given that Newton buys some boxes of dog treats for $9 each and Descartes buys some bags of cat treats for $6 each.
It is also given that Newton and Descartes spend the same amount of money on treats
So, we have to find the least number that can be a multiple of both 6 and 9.
Now,
the multiples of 6 are: 6, 12, 18, 24, 30, 36, 42, 48, 54, and 60
The multiples of 9 are: 9, 18, 27, 36, 45, 54, 63, 72, 81, and 90
From the multiples of 6 and 9,
We can see that the least number that can be the multiple of both 6 and 9 is: 18
Hence, from the above,
We can conclude that the least amount of money that Newton and Descartes spent on dog treats is: $18

Relate Factors and Multiples Homework & Practice 6.3

Question 1.
Is 16 a multiple of 3? Explain.
Answer: 16 is not a multiple of 3

Explanation:
Multiples are the numbers that can be divided by another number a certain number of times without any remainder.
So,
The multiples of 16 are 3, 6,9, 12, 15, 18, etc.
Hence, from the above,
We can conclude that 16 is not a multiple of 3.

Question 2.
Is 21 a multiple of 7? Explain.
Answer: 21 is a multiple of 7

Explanation:
Multiples are the numbers that can be divided by another number a certain number of times without any remainder.
So,
The multiples of 7 are: 7, 14, 21, 28, 35, 42, 49, 56, 63, 70
Hence, from the above,
We can conclude that 21 is a multiple of 7

Question 3.
Is 46 a multiple of 2? Explain.
Answer: 46 is a multiple of 2

Explanation:
Multiples are the numbers that can be divided by another number a certain number of times without any remainder.
So,
For the multiple of 2, the last digit should be 2, 4, 6, 8, 0
So,
46 can be said as a multiple of 2
Hence, from the above,
We can conclude that 46 is a multiple of 2.

Question 4.
Is 5 a factor of 71? Explain.
Answer: 5 is not a factor of 71

Explanation:
Factors are the numbers that divide the number completely.
So,
The factors of 71 are: 1, 71
Hence, from the above,
We can conclude that 5 is not a factor of 71.

Question 5.
Is 8 a factor of 88? Explain.
Answer: 8 is a factor of 88

Explanation:
Factors are the numbers that divide the numbers completely
So,
The factors of 88 are: 1, 2, 4, 8, 11, 22, 44, 88
Hence, from the above factors,
We can conclude that 8 is a factor of 88

Question 6.
Is 4 a factor of 80? Explain.
Answer: 4 is a factor of 80

Explanation:
Factors are the numbers that divide the number completely.
So,
The factors of 80 are: 1, 2, 4, 5, 8, 10, 16, 20, 40, 80
Hence, from the above factors,
We can conclude that 4 is a factor of 80.

Tell whether 30 is a multiple or a factor of the number. Write multiple, factor, or both.
Question 7.
30
Answer: 30 is a multiple and factor of 30

Explanation:
Multiples are the numbers that can be divided by another number a certain number of times without any remainder.
Factors are the numbers that divide the number completely
So,
The multiples of 30 are: 30, 60, 90, 120, 150, 180, 210, 240, 270 and 300
The factors of 30 are: 1, 2, 3, 5, 6, 10, 15, 30
Hence,
From the above,
We can conclude that 30 is a multiple and factor of 30

Question 8.
90
Answer: 90 is a multiple of 30

Explanation:
Multiples are the numbers that can be divided by another number a certain number of times without any remainder.
So,
The multiples of 30 are: 30, 60, 90, 120, 150, 180, 210, 240, 270, 300
Hence, from the above multiples,
We can conclude that 90 is a multiple of 30

Question 9.
10
Answer: 10 is a factor of 30

Explanation:
Factors are the numbers that divide the number originally.
So,
The factors of 30 are: 1, 2, 3, 5, 6, 10, 15, 30
Hence, from the above factors,
We can conclude that 10 is a factor of 30

Tell whether 10 is a multiple or a factor of the number. Write multiple, factor, or both.
Question 10.
5
Answer: 5 is a factor of 10

Explanation:
Factors are the numbers that divide the number completely.
So,
The factors of 10 are: 1, 2, 5, 10
Hence, from the above factors,
We can conclude that 5 is a factor of 10

Question 11.
60
Answer: 60 is a multiple of 10

Explanation:
Multiples are the numbers that can be divided by another number a certain number of times without any remainder.
So,
The multiples of 10 are: 10, 20, 30, 40, 50, 60, 70, 80 , 90, 100
Hence, from the above multiples,
We can conclude that 60 is a multiple of 10

Question 12.
10
Answer: 10 is a multiple and factor of 10

Explanation:
Multiples are the numbers that can be divided by another number a certain number of times without any remainder.
Factors are the numbers that divide the number completely
So,
The number of factors of 10 is: 1, 2, 5, 10
The multiples of 10 are: 10, 20, 30, 40, 50, 60, 70, 80 , 90, 100
Hence, from the above,
We can conclude that 10 is a factor and a multiple of 10

Question 13.
DIG DEEPER!
Name two numbers that are each a multiple of both 3 and 4. What do you notice about the two multiples?
Answer: 12 is a multiple of both 3 and 4

Explanation:
We know that,
If the number is divisible by 3, then the sum of the digits of that given number must be divisible by 3
If the number is divisible by 4, then the last 2 digits of that given number must also be divisible by 4
Hence,
The number which is a multiple of both 3 and 4 is: 12
12 will satisfy both the conditions of the number divisible by 3 and 4
Hence, from the above,
We can conclude that 12 is a multiple of both 3 and 4

Question 14.
YOU BE THE TEACHER
Is Newton correct? Explain.
Big Ideas Math Answers Grade 4 Chapter 6 Factors, Multiples, and Patterns 6.3 6

Answer: Yes, Newton is correct

Explanation:
Given that,
According to Newton, all numbers that are multiples of 10 have 2 as a factor.
Now,
The multiples of 10 are: 10, 20, 30, 40, 50, 60, 70, 80, 90, 100
So,
Let a multiple of 10 be 20
So,
The factors of 20 are: 1, 2, 4, 5, 10, 20
Hence, from the above,
We can conclude that Newton’s statement is correct

Question 15.
Logic
A quotient is a multiple of 4. The dividend is a multiple of 8. The divisor is a factor of 6. Write one possible equation for the problem.

Answer:
The possible equation for the problem is:
24 ÷ 6 = 4

Explanation:
Given that a quotient is a multiple of 4.
It is also given that the dividend is a multiple of 8 and the divisor is a factor of 6.
So,
The possible Equation for the given problem is:
24 ÷ 6 = 4
Where,
24 is a dividend
6 is a divisor
4 is a quotient
Hence, from the above,
We can conclude that the possible  equation according to the given conditions is:
24 ÷ 6 = 4

Question 16.
Modeling Real Life
Your friend needs our friend needs t0 50 US state capitals. She wants to memorize the same number of capitals each day. Which numbers of capitals can she memorize each day: 2, 3, 4, or 5?

Answer: The number of capitals she can memorize each day in packs of 2 and 5

Explanation:
Given that your friend needs 50 US state capitals and she wants to memorize the same number of capitals each day.
It is also given that the number of capitals she had to memorize in the packs of 2, 3, 4, or 5
So,
If the number has to be divided by 5, the last digit of that given number should be 5 (or) 0
So,
The factors of 50 are: 1, 2, 5,  10, 25, 50
Hence, from the above,
We can conclude that 50 US capitals she had to memorize is in the packs of 2 or packs of 5

Question 17.
Modeling Real Life
Zookeepers plan an enrichment day for the animals every 7 days and bathe the elephants every 2 days. You want to go to the zoo when both events are happening. What other dates in May will this happen?
Big Ideas Math Answers Grade 4 Chapter 6 Factors, Multiples, and Patterns 6.3 7

Answer: The events of both Enrichment day and Elephant Day come in the month of May other than 14 May is:
21 May and 28 May

Explanation:
Given that Zookeepers plan an enrichment day for the animals every 7 days
It is also given that the Elephant bathing takes place every 2 days.
So, other than 14 May,
The other days that these two events will take place on May are: 21 May and 28 May with 7 days gap from 14 May
Hence, from the above,
We can conclude that  both the events are happening on 21 and 28 May

Review & Refresh

Estimate the sum or difference.
Question 18.
Big Ideas Math Answers Grade 4 Chapter 6 Factors, Multiples, and Patterns 6.3 8
Answer: 71,606 – 49,641 = 21,965

Question 19.
Big Ideas Math Answers Grade 4 Chapter 6 Factors, Multiples, and Patterns 6.3 9
Answer: 75,294 + 36,043 = 111,337

Question 20.
Big Ideas Math Answers Grade 4 Chapter 6 Factors, Multiples, and Patterns 6.3 10
Answer: 93,294 – 40,293 = 53,001

Lesson 6.4 Identify Prime and Composite Numbers

Explore and Grow

Draw as many different rectangles as possible that each has the given area. Label their side lengths.
Big Ideas Math Solutions Grade 4 Chapter 6 Factors, Multiples, and Patterns 6.4 1
Compare the numbers of factors of 32 and 13.

Answer:
The factors of 32 are: 1, 2, 4, 8, 16, 32
The factors of 13 are: 1, 13
On comparison of the factors of 32 and 13, we can say that 13 has less number of factors than 32

Reasoning
Can a whole number have fewer than two factors? exactly two factors? more than two factors?

Answer: Yes, a whole number has exactly 2 factors and more than 2 factors but not less than 2 factors.

Explanation:
A whole number will be divided into 2 types based on the number of factors. They are:
A) Composite numbers:
The numbers that have more than 2 factors are called “Composite numbers”
B) Prime numbers:
The numbers that have exactly 2 factors are called “Prime numbers”
There will no less than 2 factors for any number.

Think and Grow: Identify Prime and Composite Numbers
A prime number is a whole number greater than 1 with exactly two factors, 1 and itself. A composite number is a whole number greater than 1 with more than two factors.

Example
Tell whether 27 is a prime composite
Use divisibility rules.
Big Ideas Math Solutions Grade 4 Chapter 6 Factors, Multiples, and Patterns 6.4 2
• 27 is odd, so it is not divisible by 2 or any other even number.
• 2 + 7 = 9 is divisible by 3,
so 27 is divisible by 3.
27 has factors in addition to 1 and itself.
So, 27 is a Composite number.

Example
Tell whether 11 is a prime composite
Use divisibility rules.
• 11 is odd, so it is not divisible by 2 or any other even number.
• 1 + 1 = 2 is not divisible by 3 or 9,
so 11 is not divisible by 3 or 9.
• The ones digit is not 0 or 5,
so 11 is not divisible by 5.
11 has exactly two factors, 1 and itself.
So, 11 is a prime number.

Show and Grow

Tell whether the number is prime or composite. Explain.
Question 1.
7
Answer: 7 is a prime number

Explanation:
Prime number:
The numbers which have exactly 2 factors 1 and itself is called “Prime numbers”
Now,
The factors of 7 are: 1, 7
Hence, from the above,
We can conclude that 7 is a prime number

Question 2.
12
Answer: 12 is a Composite number

Explanation:
Composite numbers:
The numbers which have more than 2 factors are called “Composite numbers”
Now,
The factors of 12 are: 1, 2, 3, 4, 6, 12
Hence, from the above,
We can conclude that 12 is a Composite number

Question 3.
2
Answer: 2 is a prime number.

Explanation:
Prime number:
The numbers which have exactly 2 factors 1 and itself are called “Prime numbers”
Now,
The factors of 2 are: 1, 2
Hence, from the above,
We can conclude that 2 is a prime number

Question 4.
19
Answer: 19 is a prime number

Explanation:
Prime number:
The numbers which have exactly 2 factors 1 and itself are called “Prime numbers”
Now,
The factors of 19 are: 1, 19
Hence, from the above,
we can conclude that 19 is a prime number

Question5.
45
Answer: 45 is a Composite number

Explanation:
Composite numbers:
The numbers which have more than 2 factors are called “Composite numbers”
Now,
The factors of 45 are: 1, 3, 5, 9, 15, 45
Hence, from the above,
we can conclude that 45 is a Composite number

Question 6.
54
Answer 54 is a  Composite number

Explanation:
Composite numbers:
The numbers which have more than 2 factors are called “Composite numbers”
Now,
The factors of 54 are: 1, 2. 3. 6, 9, 18, 27, 54
Hence, from the above,
We can conclude that 54 is a Composite number

Apply and Grow: Practice

Tell whether the number is prime or composite. Explain.
Question 7.
35
Answer:35 is a Composite number

Explanation:
Composite numbers:
The numbers which have more than 2 factors are called “Composite numbers”
Now,
The factors of 35 are: 1, 5, 7, 35
Hence, from the above,
We can conclude that 35 is a Composite number

Question 8.
5
Answer: 5 is a prime number

Explanation:
Prime numbers:
The numbers which have exactly 2 factors 1 and itself are called “prime numbers”
Now,
The factors of 5 are: 1, 5
Hence, from the above,
we can conclude that 5 is a prime number

Question 9.
23
Answer: 23 is a prime number

Explanation:
Prime numbers:
The numbers which have exactly 2 factors 1 and itself are called ” Prime numbers”
Now,
The factors of 23 are: 1, 23
Hence, from the above,
We can conclude that 23 is a prime number

Question 10.
40
Answer: 40 is a Composite number

Explanation:
Composite numbers:
The numbers which have more than 2 factors are called “Composite numbers”
Now,
The factors of 40 are: 1, 2, 4, 5, 8, 10, 20, 40
Hence, from the above,
We can conclude that 40 is a Composite number

Question 11.
41
Answer: 41 is a prime number

Explanation:
Prime numbers:
The numbers which have exactly 2 factors 1 and itself are called “Prime numbers”
Now,
The factors of 41 are: 1, 41
Hence, from the above,
we can conclude that 41 is a prime number

Question 12.
81
Answer: 81 is a Composite number

Explanation:
Composite numbers:
The numbers which have more than 2 factors are called “Composite numbers”
Now,
The factors of 81 are: 1, 3, 9, 27, 81
hence, from the above,
we can conclude that 81 is a Composite number

Question 13.
Structure
To create a list of the prime numbers that are less than 100, do the following.
Big Ideas Math Solutions Grade 4 Chapter 6 Factors, Multiples, and Patterns 6.4 3

  • Place a square around 1. It is neither prime nor composite.
  • Circle 2 and cross out all other multiples of 2.
  • Circle 3 and cross out all other multiples of 3.
  • Circle 5 and cross out all other multiples of 5.
  • Circle the next number that is not crossed out. This number is prime. Cross out all other multiples of this number.
  • Continue until every number is either circled or crossed out.

What are the prime numbers that are less than 100? Explain why these numbers not were crossed out on the chart.
Answer:
The prime numbers below 100 are:
2, 3, 5, 7, 11, 13, 17, 19, 23, 29, 31, 37, 41, 43, 47, 53, 59, 61, 67, 73, 79, 83, 87, 89, 93, and 97

Think and Grow: Modeling Real Life

Example
A museum volunteer has 76 shark teeth to display. Can the volunteer arrange the teeth into a rectangular array with more than 1 row and more than 1 tooth in each row? Explain.
Big Ideas Math Solutions Grade 4 Chapter 6 Factors, Multiples, and Patterns 6.4 4
Use divisibility rules to determine whether 76 is prime or composite.
76 is even, so it is divisible by 2.
76 has factors in addition to 1 and itself.
So, 76 is the Composite number.
So,
The volunteer will arrange the teeth into a rectangular array with more than 1 row and more than 1 tooth in each row.
Show and Grow

Question 14.
A teacher has 29 students in class. Can the teacher separate the students into equal groups? Explain.
Answer: No, the teacher can’t separate the students into equal groups.

Explanation:
Given that a teacher has 29 students in the class
Now,
Let find whether 29 is Prime or Composite
Now,
Factors of 29 are: 1, 29
Hence, from the above factors,
We can conclude that 29 is a prime number
Hence,
The teacher can’t divide 29 students into equal groups.

Question 15.
A band instructor wants to have several ways to organize band members into rectangular arrays on the field for a performance.Should the instructor have 89 members or 99 members on the field? Explain.
Answer: The Instructor should have 99 members on the field so that he can arrange the band members into rectangular arrays.

Explanation:
Given that a band instructor wants to organize band members into rectangular arrays on the field for a performance.
It is also given that the instructor wants to arrange into an array of whether 89 members or 99 members
Now,
Factors of 89 are: 1, 89
Factors of 99 are: 1, 3, 9, 11, 33, 99
Hence, from the above,
Since the 99 members can be arranged into different arrays,
We can conclude that the instructor can arrange the participants into an array of 99 members.

Question 16.
DIG DEEPER!
A paramedic is arranging bandages into 4 bins. An equal number of bandages are in each bin. Did the paramedic arrange a prime number or a composite number of bandages? Explain.
Big Ideas Math Solutions Grade 4 Chapter 6 Factors, Multiples, and Patterns 6.4 5
Answer: The paramedic has to arrange a composite number of bandages.

Explanation:
Gien that a paramedic is arranging bandages into 4 bins.
It is also given that there is an equal number of bandages in each bin.
Now,
Given there are 4 bins and 4 is a Composite number
So, 4 can divide only a Composite number but not a prime number.
Prime numbers:
The numbers which have exactly only 2 factors 1 and itself are “Prime numbers”
Composite numbers:
The numbers which have more than 2 factors are “Composite numbers”
Hence, from the above,
We can conclude that the paramedic has to arrange a Composite number of bandages.

Identify Prime and Composite Numbers Homework & Practice 6.4

Tell whether the number is prime or composite. Explain.
Question 1.
3
Answer: 3 is a prime number

Explanation:
Prime numbers:
The numbers which have exactly 2 factors 1 and itself are “Prime numbers”
Now,
The factors of 3 are: 1, 3
Hence, from the above,
We can conclude that 3 is a prime number

Question 2.
27
Answer: 27 is a composite number

Explanation:
Composite numbers:
The numbers which have more than 2 factors are “Composite numbers”
Now,
The factors of 27 are: 1, 3, 9, 27
Hence, from the above,
We can conclude that 27 is a composite number

Question 3.
46
Answer: 46 is a composite number

Explanation:
Composite numbers:
The numbers which have more than 2 factors are “Composite numbers”
Now,
The factors of 46 are: 1, 2, 23, 46
Hence, from the above,
We can conclude that 46 is a composite number

Question 4.
17
Answer: 17 is a prime number

Explanation:
Prime numbers:
The numbers which have exactly 2 factors 1 and itself are “Prime numbers”
Now,
The factors of 17 are: 1, 17
Hence, from the above,
We can conclude that 17 is a prime number

Question 5.
53
Answer: 53 is a prime number

Explanation:
Prime numbers:
The numbers which have exactly 2 factors 1 and itself are “Composite numbers”
Now,
The factors of 53 are: 1, 53
Hence, from the above,
we can conclude that 53 is a prime number

Question 6.
63
Answer: 63 is a composite number

Explanation:
Composite numbers:
The numbers which have more than 2 factors are “Composite numbers”
Now,
The factors of 63 are: 1, 3, 7, 9, 21, 63
Hence, from the above,
We can conclude that 63 is a composite number

Question 7.
29
Answer: 29 is a prime number

Explanation:
Prime numbers:
The numbers which have exactly 2 factors 1 and itself are “Prime numbers”
Now,
The factors of 29 are: 1, 29
Hence, from the above,
We can conclude that 29 is a prime number

Question 8.
31
Answer: 31 is a prime number

Explanation:
Prime numbers:
The numbers which have exactly 2 factors 1 and itself are “Prime numbers”
Now,
The factors of 31 are: 1, 31
Hence, from the above,
We can conclude that 31 is a prime number

Question 9.
75
Answer: 75 is a composite number

Explanation:
Composite numbers:
The numbers which have more than 2 factors are “Composite numbers”
Now,
The factors of 75 are: 1, 3, 5, 15, 25, 75
Hence, from the above,
We can conclude that 75 is a composite number

Question 10.
DIG DEEPER!
Can a number be both prime and composite? Explain.
Answer: No, a number can’t be both prime and composite

Explanation:
Prime numbers:
The numbers which have exactly 2 factors 1 and itself are “Prime numbers”
Composite numbers:
The numbers which have more than 2 factors are “Composite numbers”
Hence, from the number of factors,
We can say a number can’t be both prime and composite at the same time

Question 11.
Logic
Your friend is thinking of a prime number between 60 and 80. The tens digit is one less than the ones digit. What is the number?
Answer: The number is 67

Explanation:
Prime numbers:
The numbers which have exactly 2 factors 1 and itself are “Prime numbers”
So,
The prime numbers between 60 and 80 are: 61, 67, 73, 79
It is also given that the tens digit is one less than the ones digit
So,
Tens position value – 1 = ones position value
Hence, from the above,
We can conclude that 67 is the number.

Number Sense
Write true or false for the statement. If false, provide an example to support your answer.
Question 12.
All odd numbers are prime. _______
Answer: False

Explanation:
Given that all odd numbers are prime.
Prime numbers:
The number of factors which have exactly 2 factors 1 and itself is “Prime numbers”
So, let take the off numbers from 1 to 10
The odd numbers from 1 to 10 are: 1, 3, 5, 7, 9
So, from 1 to 10,
The prime numbers are: 3, 5, 7
So, from the above
9 is an odd number but it is not prime.
Hence, from the above,
We can conclude that all odd numbers are not prime

Question 13.
All even numbers, except 2, are composite. _______
Answer: True

Explanation:
Given that all even numbers except 2 are composite numbers
Now,
The even numbers from 1 to 0 are: 2, 4, 6, 8
Composite numbers:
The numbers which have more than 2 factors are “Composite numbers”
So,
The composite numbers from 1 to 10 are: 4, 6, 8
Note: 2 is an even prime number
Hence, from the above,
WE can conclude that all even numbers except  are composite numbers
Question 14.
A composite number cannot have exactly three factors. _______
Answer: True

Explanation:
Given that a composite number has exactly 3 factors
Composite numbers:
The numbers which have more than 2 factors are “Composite numbers”
Let the number be 63
Now,
The factors of 63 are: 1, 3, 7, 9, 21, 63
Hence, from the above,
We can conclude that a composite number don’t have exactly 3 factors

Question 15.
Modeling Real Life
There are 43 students trying out for a basketball team. Can the coach separate the students into equal groups? Explain.
Big Ideas Math Solutions Grade 4 Chapter 6 Factors, Multiples, and Patterns 6.4 6
Answer: No, the coach can’t separate the students into equal groups

Explanation:
Prime numbers:
The numbers which have exactly 2 factors 1 and itself are “Prime numbers”
Now,
The factors of 43 are: 1, 43
Hence, from the above,
We can conclude that the coach can’t separate the students into equal groups since 43 is a prime number

Question 16.
Modeling Real Life
Which planet not does have a prime number of rings?
Big Ideas Math Solutions Grade 4 Chapter 6 Factors, Multiples, and Patterns 6.4 7
Answer: Saturn does not have a prime number of rings.

Explanation:
Given that,
1 full circle = 2 rings
So,
1 half circle = 1 ring
Big Ideas Math Solutions Grade 4 Chapter 6 Factors, Multiples, and Patterns 6.4 7
So, from the above table,
The number of rings of Jupiter is: 3
The number of rings of Saturn is: 9
The number of rings of Uranus is: 13
The number of rings of Neptune is: 5
Prime numbers:
The numbers which have exactly 2 factors 1 and itself are “Prime numbers”
Hence, from the above,
We can conclude that Saturn does not have a prime number of rings ( Because 9 is a composite number )

Review & Refresh

Use properties to find the product. Explain your reasoning
Question 17.
4 × 9 × 25
Answer: 4 × 9 × 25 = 900

Explanation:
Using the Distributive Property of multiplication,
4 × 9 × 25 = 4 × 9 × ( 20 + 5 )
= 36 × ( 20 + 5 )
= ( 36 × 20 ) + ( 36 × 5 )
= 720 + 180
= 900
Hence, 4 × 9 × 25 = 900

Question 18.
405 × 3
Answer: 403 × 3 = 1,215

Explanation:
Using the Distributive Property of Multiplication,
405 × 3 = ( 400 + 5 ) × 3
= ( 400 × 3 ) + ( 5 × 3 )
= 1,200 + 15
= 1,215
Hence, 405 × 3 = 1,215

Question 19.
698 × 7
Answer: 698 × 7 = 4,886

Explanation:
Using the Distributive Property of Multiplication,
698 × 7 = ( 600 + 90 + 8 ) × 7
= ( 600 × 7 ) + ( 90 × 7 ) + ( 8 × 7 )
= 4,200 + 630 + 56
= 4,886
Hence, 698 × 7 = 4,886

Lesson 6.5 Number Patterns

Explore and Grow

Shade every third square in the table.
Big Ideas Math Answer Key Grade 4 Chapter 6 Factors, Multiples, and Patterns 6.5 1
Write the shaded numbers. What patterns do you see?

What other patterns do you see in the table?

Answer:
The shaded numbers are:
3, 6, 9, 12, 15, 18, 21, 24, 27, 30, 33, 36, 39, 42, 45, 48, 51, 54, 57, 60
From the shaded numbers, we can see that each third square is a multiple of 3
The other patterns we can observe in the given table is:
a) If we first shaded the second square and then shaded each second square, then we get the multiples of 2
b) The same pattern will have to be applied for the fourth square
There will also be other patterns by shading 5th square, 6th square, etc.

Structure
Circle every fourth square in the table. Write the circled numbers. What patterns do you see?
Answer:
The circled numbers are:
4, 8, 12, 16, 20, 24, 28, 32, 36, 40, 44, 48, 52, 56, 60
From the circled numbers of every fourth square, we can see that every fourth square is a multiple of 4.

Think and Grow: Create Number Patterns

A rule tells how numbers or shapes in a pattern are related.
Example
Use the rule “Add 3.” to create a number pattern. The first number in the pattern is 3. Then describe another feature of the pattern.
Create a pattern.

So,
The numbers in the pattern are multiples of 3.
Big Ideas Math Answer Key Grade 4 Chapter 6 Factors, Multiples, and Patterns 6.5 3

Example
Use the rule “Multiply by 2.” to create a number pattern. The first number in the pattern is 10. Then describe another feature of the pattern.
Create a pattern.

So,
The one’s digit of each number in the pattern is 0.

Show and Grow

Write the first six numbers in the pattern. Then describe another feature of the pattern.
Question 1.
Rule: Add 5.
First number: 1
1, ____, _____, _____, _____, _____
Answer: The first 6 numbers are: 1, 6, 11, 16, 21 and 26

Explanation:
For the formation of the pattern,
The given rules are:
Rule 1 : Add 5
Rule 2: Add 5
Hence,
The given pattern is

Question 2.
Rule: Multiply by 3.
First number: 3
3, _____, _____, ____, _____, _____
Answer: The first 6 numbers are: 3, 9, 27, 81, 243, 729

Explanation:
For the formation of the pattern,
the given rules are:
Rule 1: Multiply by 3
Rule 2: First number: 3
Hence,
The given pattern is

Question 3.
Rule: Subtract 2.
First number: 20
Answer: The first 6 numbers are: 20, 18, 16, 14, 12, 10

Explanation:
For the formation of the pattern,
the given rules are:
Rule 1: Subtract 20
Rule 2: First number: 20
Hence,
The given pattern is:

Question 4.
Rule: Divide by 2.
First number: 256
Answer: The first 6 numbers are: 256, 128, 64, 32, 16, 8

Explanation:
For the formation of the pattern,
the given rules are:
Rule 1: Divide by 2
Rule 2: First number: 256
Hence,
The given pattern is:

Apply and Grow: Practice

Write the first six numbers in the pattern. Then describe another feature of the pattern.
Question 5.
Rule: Add 11.
First number: 11
Answer: The first 6 numbers are: 11, 22, 33, 44, 55, 66

Explanation:
For the formation of the pattern,
the given rules are:
Rule: Add 11
First number: 11
Hence,
The given pattern is:

Question 6.
Rule: Multiply by 4.
First number: 4
Answer: The first 6 numbers are: 4, 16, 64, 256, 1024, 4096

Explanation:
For the formation of the pattern,
the given rules are:
Rule 1: Multiply by 4
First number: 4
Hence,
The given pattern is

Question 7.
Rule: Subtract 3.
First number: 21
Answer: The first 6 numbers are: 21, 18, 15, 12, 9, 6

Explanation:
For the formation of the pattern,
the given rules are:
Rule: Subtract 3
First number: 21
Hence,
The given pattern is:

Question 8.
Rule: Divide by 3.
First number: 729
Answer: The first 6 numbers are: 729, 243, 81, 27, 9, 3

Explanation:
For the formation of the pattern,
the given rules are:
Rule 1: Divide by 3
First number: 729
Hence,
The given pattern is

Question 9.
Rule: Add 9.
First number: 8
Answer: The first  numbers are: 8, 17, 26, 35, 44, 53

Explanation:
For the formation of the pattern,
the given rules are:
Rule 1: Add 9
First number: 8
Hence,
The given pattern is:

Question 10.
Rule: Multiply by 5.
First number: 5
Answer: The first 6 numbers are: 5, 25, 125, 625, 3125, 15,625

Explanation:
For the formation of the pattern,
the given rules are:
Rule 1: Multiply by 5
First number: 5
Hence,
The given pattern is:

Open-Ended
Use the rule to generate a pattern of four numbers.
Question 11.
Rule: Multiply by 2.
Answer:
Let the first number be 2.
Hence,
The pattern of four numbers is: 2, 4, 8, 16

Explanation:
For the formation of the pattern,
the given rules are:
Rule: Multiply by 2
Let the first number be: 2
Hence,
The obtained pattern will be:

Question 12.
Rule: Subtract 9.
Answer:
Let the first number be 36
Hence,
The pattern of four numbers is: 36, 27, 18, 9

Explanation:
For the formation of the pattern,
the given rules are:
Rule: Subtract 9
Let the first number be 36
Hence,
The obtained pattern will be:

Question 13.
Rule: Divide by 4.
Answer:
Let the first number be 16
Hence,
The pattern of 4 numbers are: 16, 12, 8, 4

Explanation:
For the formation of the pattern,
the given rules are:
Rule: Divide by 4
Let the first number be 16
Hence,
The obtained pattern will be:

Question 14.
Rule: Add 7.
Answer:
Let the first number be 7
Hence,
The pattern of the four numbers are: 7, 14, 21, 28

Explanation:
For the formation of the pattern,
the given rules are:
Rule: Add 7
Let the first number be 7
Hence,
The obtained pattern will be:

Question 15.
Patterns
Write a rule for the pattern below. Then write a different pattern that follows the same rule.
3, 6, 12, 24, 48
Answer:  The rule for the given pattern is: Multiply by 2

Explanation:
Given numbers are: 3, 6, 12, 24, 48
So, from the given numbers,
We can say that the given pattern follows the “Multiply by 2” rule

Question 16.
Reasoning
What is the missing number in the pattern? Explain.
39, 37, 35, _____, 31, 29
Answer: The missing number in the pattern is: 33

Explanation:
The given numbers are: 39, 37, 35, 31, 29
From the given pattern,
the numbers that are following the rule is “Subtract by 2”
Hence, from the above,
The missing number will be: 35 – 2 = 33

Think and Grow: Modeling Real Life

Example
A presidential election is held every 4 years. There was a presidential election in 2016. How many presidential elections will occur between 2017 and 2030?
Big Ideas Math Answer Key Grade 4 Chapter 6 Factors, Multiples, and Patterns 6.5 5
The rule is to add 4 years to each presidential election year. Start with 2016. Then count the years in the pattern that is between 2017 and 2030.

So,
3 presidential elections will occur between 2017 and 2030.

Show and Grow

Question 17.
The pattern of animals on a Chinese calendar repeats every 12 years. The year 2000 was the year of the dragon. How many times will the year of the dragon occur between 2001 and 2100?
Big Ideas Math Answer Key Grade 4 Chapter 6 Factors, Multiples, and Patterns 6.5 7
Answer: The number of times the dragon will appear between 2001 and 2100 is: 8

Explanation:
Given that the pattern of animals on a Chinese calendar repeats every 12 years
It is also given that the year 2000 was the year of the dragon
Now, between the year 2001 and 2100, there are 99 years
So,
The number of times the dragon will appear = The difference between the years 2001 and 2100 ÷ The number of years the pattern will change
= 99 ÷ 12
Now,
99 ÷ 12 = 8 R 3
Hence, from the above,
We can conclude that the dragon will appear 8 times between the years 2001 and 2021

Question 18.
A robotics team raised $25 the first month of school. Each month of school, the team wants to raise 2 times as much money as the month before. How much money should they raise in the fifth month of school?
Answer: The money should they raise in the fifth month of school is: $400

Explanation:
Given that a robotics team raised $25 for the first month of the school and each month of the school, the team wants to raise 2 times as much money as the month before.
Hence,
The rule followed here is: Multiply by 2
The given first number is: $25
Hence,
The pattern we will obtain is:

Hence, from the above,
We can conclude that the money they should raise in the fifth month is: $400

Question 19.
DIG DEEPER!
You start with 128 pictures on your tablet. You take 6 pictures and delete 3 pictures each day. How many pictures do you have on your tablet after 6 days?
Answer: The pictures you have on your tablet after 6 days is: 110 pictures

Explanation:
Given that you have 128 pictures on your tablet and you take 6 pictures and delete 3 pictures each day.
So,
The number of pictures you have each day = 6 – 3 = 3 pictures
So,
The total number of pictures in 6 days = 6 × 3 = 18 pictures
So,
The total number of pictures you have in 6 days = Total number of pictures – Total number f pictures in 6 days
= 128 – 18
= 110 pictures
Hence, from the above,
We can conclude that there are 110 pictures you have in 6 days.

Number Patterns Numbers Homework & Practice 6.5

Write the first six numbers in the pattern. Then describe another feature of the pattern.
Question 1.
Rule: Subtract 8.
First number: 88
88, ____, ____, ____, _____, _____
Answer:
The first 6 numbers in the given pattern are: 88, 80, 72, 64, 56, 48

Explanation:
For the formation of the pattern,
the given rules are:
Rule: Subtract 8
First number: 88
Hence,
The pattern we will obtain is:

Question 2.
Rule: Multiply by 10.
First number: 2
2, _____, _____, _____, ____, ____
Answer:
The first 6 numbers of the given pattern are: 2, 20, 200, 2,000, 20,000, 200,000

Explanation:
For the formation of the pattern,
the given rules are:
Rule: Multiply by 10
First number: 2
Hence,
The pattern we will obtain is:

Question 3.
Rule: Add 9.
First number: 17
Answer:
The first 6 numbers for the given pattern is: 17, 26, 35, 44, 53, 62

Explanation:
For the formation,
the given rules are:
Rule: Add 9
First number: 17
Hence,
The pattern we will obtain is:

Question 4.
Rule: Divide by 2.
First number: 1,600
Answer:
The first 6 numbers of the given pattern are: 1,600, 800, 400, 200, 100, 50

Explanation:
For the formation of the pattern,
the given rules are:
Rule: Divide by 2
First number: 1,600
Hence,
The pattern we will obtain is:

Open-Ended Use the rule to generate a pattern of four numbers.
Question 5.
Rule: Divide by 5.
Answer:
Let the first number be: 625
Hence,
The  four numbers of the given pattern will be: 625, 125, 25, 5

Explanation:
For the formation of the pattern,
the given rules are:
Rule: Divide by 5
Let the first number be: 625
Hence,
The pattern we will obtain is:

Question 6.
Rule: Add 8.
Answer:
Let the first number be 8
Hence,
The four numbers of the given pattern are: 8, 16, 24, 32

Explanation:
For the formation of the pattern,
the given rules are:
Rule: Add 8
Let the first number be 8
Hence,
The pattern we will obtain will be:

Question 7.
Rule: Multiply by 9.
Answer:
Let the first number be 9
Hence,
The four numbers of the pattern will be: 9, 81, 729, 6551

Explanation:
For the formation of the pattern,
the given rules are:
Rule: Multiply 9
Let the first number be 9
Hence,
The pattern we obtain will be:

Question 8.
Rule: Subtract 3.
Answer:
Let the first number be 27
Hence,
The four numbers of the given pattern will be: 27, 24, 21, 18

Explanation:
For the formation of the pattern,
the given rules are:
Rule: Subtract 3
Let the first number be 27
Hence,
The pattern we obtain will be:

Question 9.
Structure
List the first ten multiples of 9. What patterns do you notice with the digits in the one’s place? in the tens place?
Does this pattern continue beyond the tenth number in the pattern?
Answer: The first 10 multiples of 9 are: 9, 18, 27, 36, 45, 54, 63, 72, 81, 90

Explanation:
The first 10 multiples of 9 are: 9, 18, 27, 36, 45, 54, 63, 72, 81, 90
So,
from the above 10 multiples of 9,
The pattern we can observe in the one’s place is: Decreasing of numbers from 9 to 0 i.e., 9, 8, 7, ………..0
The pattern we can observe in the ten’s place is: Increasing of numbers from 0 to 9 i.e., 0, 1, 2 …………..9
The above 2 patterns will continue beyond the first 10 multiples of 9
Hence, from the above,
We can conclude that the patterns we observed are:
The pattern we can observe in the one’s place is: Decreasing of numbers from 9 to 0 i.e., 9, 8, 7, ………..0
The pattern we can observe in the ten’s place is: Increasing of numbers from 0 to 9 i.e., 0, 1, 2 …………..9

Question 10.
Modeling Real Life
It takes the moon about 28 days to orbit Earth. How many times will the moon orbit Earth in 1 year?
Big Ideas Math Answer Key Grade 4 Chapter 6 Factors, Multiples, and Patterns 6.5 8
Answer: The moon will orbit the earth 10,220 times in 1 year

Explanation:
Given that it takes the moon about 28 days to orbit the earth.
We know that,
1 year = 365 days
So,
The number of times the moon will orbit around the earth in 1 year = 365 × 28
Now,
By using the partial products method,
365 × 28  = ( 300 + 65 ) ×  ( 20 + 8 )
= ( 300 ×  20 ) + ( 300 ×  8 ) + ( 65 ×  20 ) + ( 65 ×  8 )
= 6,000 + 2,400 + 1,300 + 520
= 10,220 times
Hence, from the above,
We can conclude that the moon will orbit around the Earth 10,220 times

Question 11.
DIG DEEPER!
In each level of a video game, you can earn up to 10 points and lose up to 3 points. Your friend earns 9 points in the first level. If he earns and loses the maximum number of points at each level, how many total points will he have after level 6?
Answer: The total points your friend will have is: 44 points

Explanation:
Given that in each level of a video game, you can earn up to 10 points and lose up to 3 points
So,
The maximum number of points you can get = 10 – 3 = 7 points
It is also given that your friend earns 9 points in the first level
So,
The number of points he will have at level 6 = The number of points he has at the first level + 5 × ( The maximum number of points he can get )
= 9 + ( 5 ×  7 )
= 9 + 35
= 44 points
Hence, from the above,
We can conclude that he will have 44 points at the sixth level.

Review & Refresh

Find the product.
Question 12.
14 × 23 = _____
Answer: 14 ×  23 = 322

Explanation:
By using the partial products method,
14 ×  23 = ( 10 + 4 ) × ( 20 + 3 )
= ( 10 ×  20 ) + ( 10 ×  3 ) + ( 4 ×  20 ) + ( 4 ×  3 )
= 200 + 30 + 80 + 12
= 322
Hnece, 14 × 23 = 322

Question 13.
48 × 60 = _____
Answer: 48 × 60 = 2,880

Explanation:
By using the Distributive Property of Multiplication,
48 ×  60 = ( 40 + 8 ) × 60
= ( 40 ×  60 ) + ( 8 ×  60 )
= 2,400 + 480
= 2,880
Hence, 48 × 60 = 2,880

Question 14.
55 × 31 = _____
Answer: 55 ×  31 = 1,705

Explanation:
By using the partial products method,
55 × 31 = ( 50 + 5 ) ×  ( 30 + 1 )
= ( 50 ×  30 ) + ( 50 ×  1 ) + ( 5 ×  30 ) + ( 5 ×  1 )
= 1,500 + 50 + 150 + 5
= 1,705
Hence, 55 ×  31 = 1,705

Lesson 6.6 Shape Patterns

Explore and Grow

Create a rule using 3 different shapes. Draw the first six shapes in the pattern.

What is the next shape in the pattern?

What is the 9th shape in the pattern? Explain.

What is the 99th shape? 1,000th shape? Explain?

Answer: 
From the above 3 shapes,
The rule we can create is: Add 1
So,
We can say the above pattern will repeat after every 3 shapes.
So,
The 9th shape will be: Pentagon
The 99th shape will be: 99 / 3  =  33 R 0
So, The 99th shape will be: Pentagon
The 100th shape will be: Triangle (Because the remainder will be 1 i.e., the shapes will completely repeat 99 times and for the 100th time, it will start from 1st shape)

Structure
You want to show the first 40 shapes in the pattern above. Without modeling, how many of each shape do you think you will need?
Answer:
The number of times each shape will appear in the first 40 shapes is:
Triangle: 14 times
Square: 13 times
Pentagon: 13 times

Explanation:
In the above pattern, the shapes are: Triangle, Square, Pentagon
The rule we obtained by using these 3 shapes is: Add 1
So,
The pattern for the first 40 shapes will be: 40 ÷ 3
Now,
By using the Distributive Property of Division,
40 ÷ 3 = ( 36 + 3 ) ÷ 3
= ( 36 ÷ 3 ) + ( 3 ÷ 3 )
= 12 + 1
= 13 R 1
Hence, the 3 shapes will appear 39 times
So,
The number of times each shape will appear = 13 times
Hence, from the above,
We can conclude that the number of times the shape will appear is:
Triangle: 14 times ( 13 + 1. This 1 is because 40th time, the triangle will repeat again )
Square: 13 times
Pentagon: 13 times

Think and Grow: Create Shape Patterns

Example
Create a shape pattern by repeating the rule “triangle, hexagon, square, rhombus.”What is the 42nd shape in the pattern?
Create a pattern.
Big Ideas Math Answers 4th Grade Chapter 6 Factors, Multiples, and Patterns 6.6 1
42 ÷ 4 is 10 R2, so when the pattern repeats 10 times, the 40th shape is a rhombus So, the 41st shape is a Triangle and the 42nd shape is a hexagon.
Big Ideas Math Answers 4th Grade Chapter 6 Factors, Multiples, and Patterns 6.6 2
Big Ideas Math Answers 4th Grade Chapter 6 Factors, Multiples, and Patterns 6.6 3
Figure 1 has 1 column of 4 dots, so it has 1 × 4 = 4 dots.
Figure 2 has 2 columns of 4 dots, so it has 2 × 4 = 8 dots.
Figure 3 has 3 columns of 4 dots, so it has 3 × 4 = 12 dots.
So,
The 25th figure has 1 column of 4 dots, ( Because the 3 figures will be repeated 24 times and the first figure will appear again 25th time)
So, it has 1 × 4 =4 dots.

Show and Grow

Question 1.
Extend the pattern of shapes by repeating the rule “square, trapezoid, triangle, hexagon, triangle.”What is the 108th shape in the pattern?
Big Ideas Math Answers 4th Grade Chapter 6 Factors, Multiples, and Patterns 6.6 4
Answer: The 108th shape in the pattern will be: Triangle

Explanation:
The given pattern is: Square, trapezoid, triangle, hexagon, triangle
So, there is a total of 5 shapes.
So,
for this, when we divide any number by the total number of patterns, the value of the quotient will be a total repetition of the patterns and the remainder will be the counting of the pattern from the 1st figure.
So,
108 ÷ 5 = ( 100 + 5 ) ÷ 5
= ( 100 ÷ 5 ) + ( 5 ÷ 5 )
= 20 + 1
= 21 R 3
Hence, from the above,
We can conclude that the 108th shape will be the triangle.

Question 2.
Describe the dot pattern. How many dots are in the 76th figure?
Big Ideas Math Answers 4th Grade Chapter 6 Factors, Multiples, and Patterns 6.6 5
Answer: The number of dots in the 76th figure is: 3 dots

Explanation:
Big Ideas Math Answers 4th Grade Chapter 6 Factors, Multiples, and Patterns 6.6 5
From the above pattern,
In figure 1, the number of dots = 1 × 3 = 3 dots
In figure 2, the number of dots = 2 × 3 =  dots
In figure 3, the number of dots = 3 × 3 = 9 dots
So,
The total number of patterns is: 3
So,
for this, when we divide any number by the total number of patterns, the value of the quotient will be a total repetition of the patterns and the remainder will be the counting of the pattern from the 1st figure.
So,
By using the partial quotients method,
76 ÷ 3 = ( 66 +9 ) ÷ 3
= ( 66 ÷ 3 ) + ( 9 ÷ 3 )
= 22 + 3
= 25 R 1
Hence, from the above,
We can conclude that there are 3 dots in the 76th figure.

Apply and Grow: Practice
Question 3.
Extend the pattern of shapes by repeating the rule “oval, triangle.”What is the 55th shape?
Big Ideas Math Answers 4th Grade Chapter 6 Factors, Multiples, and Patterns 6.6 6
Answer: The 55th shape is: Oval

Explanation:
The given pattern of shapes is: Oval, Triangle
So,
The total number of patterns is: 2
So,
for this, when we divide any number by the total number of patterns, the value of the quotient will be a total repetition of the patterns and the remainder will be the counting of the pattern from the 1st figure.
So,
By using the partial quotients method,
55 ÷ 2 = ( 50 + 4 ) ÷ 2
= ( 50 ÷ 2 ) + ( 4 ÷ 2 )
= 25 + 2
= 27 R 1
Hence, from the above,
We can conclude that the 55th shape is: Oval

Question 4.
Extend the pattern of symbols by repeating the rule “add, subtract, multiply, divide.”What is the 103rd symbol?
Big Ideas Math Answers 4th Grade Chapter 6 Factors, Multiples, and Patterns 6.6 7
Answer: The 103rd symbol is: Multiply

Explanation:
The given patterns are: Add, Subtract, Multiply, Divide
So,
The total number of patterns is: 4
So,
for this, when we divide any number by the total number of patterns, the value of the quotient will be a total repetition of the patterns and the remainder will be the counting of the pattern from the 1st figure.
So,
By using the partial quotients method,
103 ÷ 4 = ( 80 + 20 ) ÷ 4
= ( 80 ÷ 4 ) + ( 20 ÷ 4 )
= 20 + 5
= 25 R 3
Hence, from the above,
We can conclude that the 103rd shape is: Multiply

Question 5.
Describe the pattern. How many squares are in the 24th figure?
Big Ideas Math Answers 4th Grade Chapter 6 Factors, Multiples, and Patterns 6.6 8
Answer: The number of squares in the 24th figure is: 4 squares

Explanation:
Big Ideas Math Answers 4th Grade Chapter 6 Factors, Multiples, and Patterns 6.6 8
In figure 1, the number of squares is: 2
In figure 2, the number of squares is: 3
In figure 3, the number of squares is: 4
So,
Total number of figures = 3
So,
for this, when we divide any number by the total number of patterns, the value of the quotient will be a total repetition of the patterns and the remainder will be the counting of the pattern from the 1st figure.
So,
By using the partial quotients method,
24 ÷ 3 = ( 21 + 3 ) ÷ 3
= ( 21 ÷ 3 ) + ( 3 ÷ 3 )
= 7 + 1
= 8 R 0
Hence, from the above,
We can conclude that  in the 24th figure, the number of squares is: 4

Question 6.
Describe the pattern of the small triangles. How many small triangles are in the 10th figure?
Big Ideas Math Answers 4th Grade Chapter 6 Factors, Multiples, and Patterns 6.6 9
Answer: The small triangles in the 10th figure is: 1

Explanation:
Big Ideas Math Answers 4th Grade Chapter 6 Factors, Multiples, and Patterns 6.6 9
From the given figure,
In fig 1, the number of triangles is: 1
In fig 2, the number of triangles is: 2
In fig 3, the number of triangles is: 4
So,
Total number of figures = 3
So,
for this, when we divide any number by the total number of patterns, the value of the quotient will be a total repetition of the patterns and the remainder will be the counting of the pattern from the 1st figure.
So,
By using the partial quotients method,
10 ÷ 3 = ( 3 + 6 ) ÷ 9
= ( 3 ÷ 3 ) + ( 6 ÷ 3 )
= 1 + 2
= 3 R 1
Hence, from the above,
We can conclude that the 10th figure has 1 triangle

Question 7.
Structure
Make a shape pattern that uses twice as many squares as triangles.
Answer:

Question 8.
Number Sense
Which shape patterns have a heart as the 12th shape?
Big Ideas Math Answers 4th Grade Chapter 6 Factors, Multiples, and Patterns 6.6 10
Answer:
Let the patterns be named as A), B), C) and D)
So,
The shape patterns that have a heart as the 12th shape is: A), B) and D)

Explanation:
Big Ideas Math Answers 4th Grade Chapter 6 Factors, Multiples, and Patterns 6.6 10
Let the patterns be named as A), B), C) and D)
A) In pattern A,
The total number of figures = 2
So,
for this, when we divide any number by the total number of patterns, the value of the quotient will be a total repetition of the patterns and the remainder will be the counting of the pattern from the 1st figure.
So,
12 ÷ 2 = 6
Hence, A) will have a heart as the 12th shape
B) In pattern B,
The total number of figures = 4
So,
for this, when we divide any number by the total number of patterns, the value of the quotient will be a total repetition of the patterns and the remainder will be the counting of the pattern from the 1st figure.
So,
12 ÷ 4 = 3
Hence, B) will have a heart as the 12th shape
C) In pattern C,
The total number of figures = 2
So,
for this, when we divide any number by the total number of patterns, the value of the quotient will be a total repetition of the patterns and the remainder will be the counting of the pattern from the 1st figure.
So,
12 ÷ 2 = 6
But, the 12th shape in C) will be a circle
D) In pattern D,
The total number of figures = 3
So,
for this, when we divide any number by the total number of patterns, the value of the quotient will be a total repetition of the patterns and the remainder will be the counting of the pattern from the 1st figure.
So,
12 ÷ 3 = 4
Hence, C) will have a heart as the 12th shape

Think and Grow: Modeling Real Life

Example
You make a necklace with a cube, hexagon, and star beads. You string the beads in a pattern. You use the rule “cube, star, cube, hexagon.”It takes 64 beads to complete the necklace. How many times do you repeat the pattern?
Big Ideas Math Answers 4th Grade Chapter 6 Factors, Multiples, and Patterns 6.6 11
Divide the number of beads it takes to complete the necklace by the number of beads in the rule. There are 4 beads in the rule.
So,
4√64 = 64 ÷ 4
By using the Distributive Property of division,
64 ÷ 4 = ( 60 + 4 ) ÷ 4
= ( 60 ÷ 4 ) + ( 4 ÷ 4 )
= 15 + 1
= 16
Hence,
You repeat the pattern 16 times

Show and Grow

Question 9.
The path on a board game uses the rule “red, green, pink, yellow, blue.”There are 55 spaces on the game board. How many times does the pattern repeat?
Big Ideas Math Answers 4th Grade Chapter 6 Factors, Multiples, and Patterns 6.6 12
Answer: The number of times the pattern will repeat is: 11 times

Explanation:
Given that the path on a board game uses the rule ” red, green, pink, yellow, blue ”
It is also given that there are 55 spaces on the game board
So,
for this, when we divide any number by the total number of patterns, the value of the quotient will be a total repetition of the patterns and the remainder will be the counting of the pattern from the 1st figure.
So,
By using the Distributive Property of division,
55 ÷ 5 = ( 50 + 5 ) ÷ 5
= ( 50 ÷ 5 ) + ( 5 ÷ 5 )
= 10 + 1
= 11
Hence, from the above,
We can conclude that the number of times the patterns repeat is: 11 times

Question 10.
You make a walkway in a garden using different-shaped stepping stones. You use the rule “square, circle, square, hexagon.”You use 24 square stepping stones. How many circle and hexagon stepping stones do you use altogether? How many stones do you use in all?
Answer:
The Total number of stones used are: 24
The number of  circle and hexagon stepping stones use are: 12

Explanation:
Given that there is a walkway in a garden using different-shaped stepping stones.
The rule used here is: Square, circle, square, hexagon
It is also given that there are 24 stepping stones you used
So,
for this, when we divide any number by the total number of patterns, the value of the quotient will be a total repetition of the patterns and the remainder will be the counting of the pattern from the 1st figure.
So,
By using the Distributive Property of division,
24 ÷ 4 = ( 20 + 4 ) ÷ 4
= ( 20 ÷ 4 ) + ( 4 ÷ 4 )
= 5 + 1
= 6
So,
The number of stones used each time is: 6 times
So,
The number of  circle and hexagon stepping stones used altogether is: 6 + 6 = 12
Hence, from the above,
We can conclude that circle and hexagon stepping stones you used altogether is: 12

Question 11.
DIG DEEPER!
You make a rectangular picture frame using square tiles. The picture frame is 12 tiles long and 8 tiles wide. You arrange the tiles in a pattern. You use the rule “red, orange, yellow.” How many of each color tile do you use?
Answer: The number of each color tiles do you use is: 32

Explanation:
Given that you make a rectangular picture frame using square tiles and it is also given that the picture frame is 12 tiles long and 8 tiles wide.
So,
The area of a rectangular picture frame = 12 × 8 = 96 square- meters
The rule used is: red, orange, yellow
So,
The total number of colors is: 3
So,
The number of each color tile you used = Area of rectangular picture frame ÷ Total number of colors
= 96 ÷ 3
Now,
By using the Distributive Property of division,
96 ÷ 3 = ( 90 + 6 ) ÷ 3
= ( 90 ÷ 3 ) + ( 6 ÷ 3 )
= 30 + 2
= 32
Hence, from the above,
we can conclude that each number of color tile used is: 32

Shape Patterns Numbers Homework & Practice 6.6

Question 1.
Extend the pattern of shapes by repeating the rule “up, right, down, left.”What is the 48th shape in the pattern?
Big Ideas Math Answers 4th Grade Chapter 6 Factors, Multiples, and Patterns 6.6 13
Answer: The 48th shape in the pattern is: Left

Explanation:
Given pattern is:
Big Ideas Math Answers 4th Grade Chapter 6 Factors, Multiples, and Patterns 6.6 13
The rule for the given pattern is: up, right, down, left
So,
The total number of patterns is: 4
So,
for this, when we divide any number by the total number of patterns, the value of the quotient will be a total repetition of the patterns and the remainder will be the counting of the pattern from the 1st figure.
So,
By using the partial quotients method,
48 ÷ 4 = ( 40 + 8 ) ÷ 4
= ( 40 ÷ 4 ) + ( 8 ÷ 4 )
= 10 + 2
= 12 R 0
Hence, from the above
We can conclude that the 48th figure in the given pattern is: Left

Question 2.
Extend the pattern of shapes by repeating the rule “small circle, medium circle, large circle.”What is the 86th shape in the pattern?
Big Ideas Math Answers 4th Grade Chapter 6 Factors, Multiples, and Patterns 6.6 14
Answer: The 86th shape in the pattern is: Medium circle

Explanation:
Given pattern is:
Big Ideas Math Answers 4th Grade Chapter 6 Factors, Multiples, and Patterns 6.6 14
The rule for the pattern is: Small circle, Medium circle, large circle
So,
The total number of patterns = 3
So,
for this, when we divide any number by the total number of patterns, the value of the quotient will be a total repetition of the patterns and the remainder will be the counting of the pattern from the 1st figure.
So,
By using the partial quotients method,
86 ÷ 3 = ( 75 + 9 ) ÷ 3
= ( 75 ÷ 3 ) + ( 9 ÷ 3 )
= 25 + 3
= 28 R 2
Hence, from the above,
We can conclude that the 86th shape in the pattern is: Medium circle

Question 3.
Describe the dot pattern. How many dots are in the 113th figure?
Big Ideas Math Answers 4th Grade Chapter 6 Factors, Multiples, and Patterns 6.6 15
Answer: The number of dots in the 113th figure is: 4 dots

Explanation:
Given pattern is:
Big Ideas Math Answers 4th Grade Chapter 6 Factors, Multiples, and Patterns 6.6 15
So,
In fig 1, the number of dots = 2
In fig 2, the number of dots = 4
In fig 3, the number of dots = 6
So,
The total number of figures = 3
So,
for this, when we divide any number by the total number of patterns, the value of the quotient will be a total repetition of the patterns and the remainder will be the counting of the pattern from the 1st figure.
So,
By using the partial quotients method,
113 ÷ 3 = ( 99 + 12 ) ÷ 3
= ( 99 ÷ 3 ) + ( 12 ÷ 3 )
= 33 + 4
= 37 R 2
Hence, from the above,
we can conclude that the number of dots in the 113th figure is: 4 dots

Question 4.
YOU BE THE TEACHER
You and your friend each create a shape pattern with 100 shapes. Your friend says both patterns will have the same number of circles. Is your friend correct? Explain.
Big Ideas Math Answers 4th Grade Chapter 6 Factors, Multiples, and Patterns 6.6 16
Answer: Yes, both patterns will have the same number of circles.

Explanation:
Given patterns are:
Big Ideas Math Answers 4th Grade Chapter 6 Factors, Multiples, and Patterns 6.6 16
The pattern of the friend is: Star, circle
The pattern of you si: Heart, circle, pentagon, circle
It is also given that,
The total number of figures = 100
So,
The total number of figures for your friend = 2
The total number of figures for you = 4
So,
for this, when we divide any number by the total number of patterns, the value of the quotient will be a total repetition of the patterns and the remainder will be the counting of the pattern from the 1st figure.
So
For your friend,
By using the partial quotients method,
100 ÷ 2 = ( 50 + 50 ) ÷ 2
= ( 50 ÷ 2 ) + ( 50 ÷ 2 )
= 25 + 25
= 50
Hence,
The number of circles = 50
The number of stars = 50
For you,
By using the partial quotients method,
100 ÷ 4 = ( 80 + 20 ) ÷ 4
= ( 80 ÷ 4 ) + ( 20 ÷ 4 )
= 20 + 5
= 25
Hence,
The number of circles altogether = 25 + 25 = 50
Hence, from the above,
We can conclude that there are equal number of circles in both the patterns

Question 5.
Structure
Draw the missing figure in the pattern. Explain the pattern.
Big Ideas Math Answers 4th Grade Chapter 6 Factors, Multiples, and Patterns 6.6 17
Answer:

Question 6.
Reasoning
Newton uses the rule “bone, bone, paw print” to make a shape pattern. He wants the pattern to repeat 8 times. How many bones will be in Newton’s pattern?
Big Ideas Math Answers 4th Grade Chapter 6 Factors, Multiples, and Patterns 6.6 18
Answer: The number of bones in Newton’s pattern is: 16

Explanation:
Given pattern is:
Big Ideas Math Answers 4th Grade Chapter 6 Factors, Multiples, and Patterns 6.6 18
The rule followed in the pattern is: Bone, Bone, pawprint
So,
The total number of figures in the given pattern = 3
So,
for this, when we divide any number by the total number of patterns, the value of the quotient will be a total repetition of the patterns and the remainder will be the counting of the pattern from the 1st figure.
So,
It is also given that the pattern repeated 8 times.
So,
The total number of figures when the pattern repeated 8 times = 8 × 3 = 24 figures
So,
By using the partial quotients method,
24 ÷ 3 = ( 21 + 3 ) ÷ 3
= ( 21 ÷ 3 ) + ( 3 ÷ 3 )
= 7 + 1
= 8
So,
The number of each figure in a pattern = 8
From the pattern, we can say there are 2 bones
So,
The total number of bones when the pattern repeated 8 times = 2 × 8 = 16
Hence, from the above,
We can conclude that there are 16 bones in the pattern when the pattern repeated 8 times

Question 7.
Modeling Real Life
The black keys on a piano follow the pattern “two black keys, three black keys.” There are 36 black keys on a standard piano. How many times does this entire pattern repeat?
Answer: The entire pattern repeat 12 times

Explanation:
Given that the black keys on a pattern follow the pattern
The given pattern is: two black keys, three black keys
So,
The total number of patterns = 2
It is also given that there are 36 black keys on a standard piano.
So,
The total number of black keys on a piano = 36
So,
for this, when we divide any number by the total number of patterns, the value of the quotient will be a total repetition of the patterns and the remainder will be the counting of the pattern from the 1st figure.
So,
By using the partial quotients method,
36 ÷ 2 = ( 30 + 6 ) ÷ 2
= ( 30 ÷ 2 ) + ( 6 ÷ 2 )
= 15 + 3
= 18
Hence, from the above,
We can conclude that the entire pattern will repeat 18 times.

Review & Refresh

Find the quotient.
Question 8.
30 ÷ 5 = _____
Answer: 30 ÷ 5 = 6

Explanation:
By using the partial products method,
30 ÷ 5 = ( 25 + 5 ) ÷ 5
= ( 25 ÷ 5 ) + ( 5 ÷ 5 )
= 5 + 1
= 6
Hence, 30 ÷ 5 = 6

Question 9.
360 ÷ 9 = ______
Answer: 360 ÷ 9 = 40

Explanation:
By using the partial quotients method,
360 ÷ 9 = ( 270 + 90 ) ÷ 9
= ( 270 ÷ 9 ) + ( 90 ÷ 9 )
= 30 + 10
= 40
Hence, 360 ÷ 9 = 40

Question 10.
6,400 ÷ 8 = _____
Answer: 6,400 ÷ 8 = 800

Explanation:
By using the place -value method,
6,400 ÷ 8 = 64 hundreds ÷ 8
= 8 hundred
= 8 × 100
= 800
Hence, 6,400 ÷ 8 = 800

Question 11.
140 ÷ 2 = _____
Answer: 140 ÷ 2 = 70

Explanation:
By using the place-value method,
140 ÷ 2 = 14 tens ÷ 2
= 7 tens
= 7 × 10
= 70
Hence, 140 ÷ 2 = 70

Question 12.
4,200 ÷ 7 = _____
Answer: 4,200 ÷ 7 = 600

Explanation:
By using the place-value method,
4,200 ÷ 7 = 42 hundreds ÷ 7
= 6 hundred
= 6 × 100
= 600
Hence, 4,200 ÷ 7 = 600

Question 13.
40 ÷ 2 = _____
Answer: 40 ÷ 2 = 20

Explanation:
By using the place-value method,
40 ÷ 2 = 4 tens ÷ 2
= 2 tens
= 2 × 10
= 20
Hence, 40 ÷ 2 = 20

Factors, Multiples, and Patterns Performance Task

You play basketball in a youth basketball program.
Big Ideas Math Answers Grade 4 Chapter 6 Factors, Multiples, and Patterns 1
Question 1.
There are 72 players in the program. Each team needs an equal number of players and must have at least 5 players. What are two different ways the teams can be made?
Answer: The different ways the teams can be made and to have at least 5 members are: 8 × 9 and 9 × 8

Explanation:
Given that there are 72 players in the program and each team needs an equal number of players.
It is also given that each team must have at least 5 players.
Now,
To find the different ways the teams can be made, we have to find the factors of 72
Now,
The factors of 72 are: 1, 2, 3, 4, 6, 8, 9, 12, 18, 24, 36, 72
Now, to make an equal number of players and to have at least 5 players,
The different ways the teams can be made is: 8 × 9, 9 × 8
Hence, from the above,
We can conclude that the 2 different ways the teams can be made are: 8 × 9 and 9 × 8

Question 2.
The width of a basketball court is 42 feet and the length is 74 feet. You run around the perimeter of the court 4 times to warm up. How many feet do you run?
Big Ideas Math Answers Grade 4 Chapter 6 Factors, Multiples, and Patterns 2
Answer: The number of feet you run is: 464 feet

Explanation:
Given that the width of a basketball court is 42 feet high and the length is 74 feet
So,
The perimeter of the basketball court = 42 + 74 = 116 feet
It is also given that you run around the perimeter of the basketball court 4 times to warm up.
So,
The number of feet you run to warm up = The perimeter of the basketball court × 4
= 116 × 4
Now,
By using the partial products method,
116 × 4 = ( 100 + 16 ) × 4
= ( 100 × 4 ) + ( 16 × 4 )
= 400 + 64
= 464 feet
Hence, from the above,
We can conclude that you will run up 464 feet to warm up your body.

Question 3.
You and your friend are on the same team. You played your first game last week.
a. Your team scored 4 more points than the other team. The total number of points scored by both teams was 58. How many points did your team score?
b. You and your friend scored the same number of points. You made 2-point shots and your friend made 3-point shots. What could be the greatest number of points you and your friend each scored?
Answer:

Question 4.
Your team uses the pattern below to decide which jersey color to wear to each game. Which color jersey will your team wear on the 20th game?
Big Ideas Math Answers Grade 4 Chapter 6 Factors, Multiples, and Patterns 3
Answer: The color of jersey your team will wear on the 20th game is: Game 4

Explanation:
Given that your team uses the following pattern:
Big Ideas Math Answers Grade 4 Chapter 6 Factors, Multiples, and Patterns 3
From the above pattern,
The total number of shirts is: 4
So,
for this, when we divide any number by the total number of patterns, the value of the quotient will be a total repetition of the patterns and the remainder will be the counting of the pattern from the 1st figure.
So,
By using the partial quotients method,
20 ÷ 4 = 5 R 0
Hence, from the above,
We can conclude that the color of the shirt your team will wear on the 20th game is: Game 4

Factors, Multiples, and Patterns Activity

Multiple Lineup
Directions:
1. Players take turns rolling a die.
2. On your turn, place a counter on a multiple of the number of your roll. If there is not a multiple of the number of your roll, you lose your turn.
3. The first player to create a line of 5 in a row, horizontally, vertically, or diagonally, wins!
Big Ideas Math Answers Grade 4 Chapter 6 Factors, Multiples, and Patterns 4
Answer:

Factors, Multiples, and Patterns Chapter Practice

6.1 Understand Factors

Question 1.
Use the rectangles to find the factor pairs for 6.
Big Ideas Math Answers Grade 4 Chapter 6 Factors, Multiples, and Patterns chp 1
Answer: The factor pairs of 6 are: 1 and 6, 2 and 3

Explanation:

The factor pairs are nothing but the side lengths of a rectangle and the area of a rectangle gives the factor
Hence,
The factor pairs of 6 are: 1 and 6, 2 and 3

Question 2.
Draw rectangles to find the factor pairs for 12.
Big Ideas Math Answers Grade 4 Chapter 6 Factors, Multiples, and Patterns chp 2
Answer: The factor pairs of 12 are: 1 and 12, 2 and 6, 3 and 4

Explanation:

The factor pairs are nothing but the side lengths of a rectangle and the area of a rectangle gives the factor
Hence,
The factor pairs of 12 are: 1 and 12, 2 and 6, 3 and 4

Find the factor pairs for the number.
Question 3.
17
Answer: The factor pairs of 17 are: 1 and 17

Explanation:
Factors are the numbers that divide the original completely.
Hence,
The factor pairs of 17 are: 1 × 17

Question 4.
10
Answer: The factor pairs of 10 are: 1 and 10, 2 and 5

Explanation:
Factors are the numbers that divide the original number completely.
Hence,
The factor pairs of 10 are: 1 × 10, 2 × 5

Question 5.
21
Answer: The factor pairs of 21 are: 1 and 21, 3 and 7

Explanation:
Factors are the numbers that divide the original number completely.
Hence,
The factor pairs of 21 are: 1 × 21, 3 × 7

Question 6.
20
Answer: The factor pairs of 20 are: 1 and 20, 2 and 10, 4 and 5

Explanation:
Factors are the numbers that divide the original number completely.
Hence,
The factor pairs of 20 are: 1 × 20, 2 × 10, 5 × 4

Question 7.
36
Answer: The factor pairs of 36 are: 1 and 36, 2 and 18, 3 and 12, 4 and 9, 6 and 6

Explanation:
Factors are the numbers that divide the original number completely.
Hence,
The factor pairs of 36 are: 1 × 36, 2 × 18, 3 × 12, 4 × 9, 6 × 6

Question 8.
50
Answer: The factor pairs of 50 are: 1 and 50, 2 and 25, 5 and 10

Explanation:
factors are the numbers that divide the original number completely.
Hence,
The factor pairs of 50 are: 1 × 50, 2 × 25, 5 × 10

6.2 Factors and Divisibility

Find the factor pairs for the number.
Question 9.
16
Answer: The factor pairs of 16 are: 1 and 16, 2 and 8, 4 and 4

Explanation:
Factors are the numbers that divide the original number completely.
Hence,
The factor pairs of 16 are: 1 × 16, 2 × 8, 4 × 4

Question 10.
24
Answer: The factor pairs of 24 are: 1 and 24, 2 and 12, 3 and 8, 4 and 6

Explanation:
factors are the numbers that divide the original number completely.
Hence,
The factor pairs of 24 are: 1 × 24, 2 × 12, 3 × 8, 4 × 6

Question 11.
56
Answer: The factor pairs of 56 are: 1 and 56, 2 and 28, 4 and 14, 7 and 8

Explanation:
Factors are the numbers that divide the original number completely.
Hence,
The factor pairs of 56 are: 1 × 56, 2 × 28, 4 × 14, 7 × 8

List the factors of the number.
Question 12.
25
Answer: The factors of 25 are: 1, 5, 25

Explanation:
Factors are the numbers that divide the original number completely.
Hence,
The factors of 25 are: 1, 5, 25

Question 13.
60
Answer: The factors of 60 are: 1, 2, 3, 4, 5, 6, 10, 12, 15, 20, 30, 60

Explanation:
Factors are the numbers that divide the original number completely.
Hence,
The factors of 60 are: 1, 2, 3, 4, 5, 6, 10, 12, 15, 20, 30, 60

Question 14.
72
Answer: The factors of 72 are: 1, 2, 3, 4, 6, 8, 9, 12, 18, 24, 36, 72

Explanation:
Factors are the numbers that divide the original number completely.
Hence,
The factors of 72 are: 1, 2, 3, 4, 6, 8, 9, 12, 18, 24, 36, 72

Question 15.
Number Sense
Which numbers have 3 as a factor?
Big Ideas Math Answers Grade 4 Chapter 6 Factors, Multiples, and Patterns chp 15
Answer: The numbers which have 3 as a factor are: 21, 36, 48, 93

Explanation:
Given numbers are: 56, 21, 3, 48, 93, 71
For the given number to have 3 as a factor, the sum of the digits of the given number must be a multiple of 3
So,
The sum of digits of:
56: 5 + 6 = 11: 1 + 1 = 2
21: 2 + 1 = 3
36: 3 + 6 = 9
48: 4 + 8 = 12: 1 + 2 = 3
93: 9 + 3 = 12: 1 + 2 = 3
71: 7 + 1 = 8
Hence, from the above,
we can conclude that the numbers which have 3 as a factor are: 21, 36, 48, 93

6.3 Relate Factors and Multiples

Question 16.
Is 54 a multiple of 3? Explain.
Answer: 54 is a multiple of 3

Explanation:
Multiples are the numbers that can be divided by another number a certain number of times without any remainder.
The multiples of 3 are: 3, 6, 9, 12, 15, 18, 21, 24, 27, 30, 33
To be a multiple of 3, the sum of the digits of the given number should also be a multiple of 3.
So,
54: 5 + 4 = 9 ( Divisible by 3 )
Hence, from the above,
we can conclude that 54 is a multiple of 3

Question 17.
Is 45 a multiple of 7? Explain.
Answer: 45 is not a multiple of 7

Explanation:
Multiples are the numbers that can be divided by another number a certain number of times without any remainder.
The multiples of 7 are: 7, 14, 21, 28, 35, 42, 49, 56, 63, 70
Hence, from the above,
we can conclude that 45 is not a multiple of 7

Question 18.
Is 2 a factor of 97? Explain.
Answer: 2 is not a factor of 97

Explanation:
Factors are the numbers that divide the original number completely.
The factors of 97 are: 1, 97
Hence, from the above,
We can conclude that 2 is not a factor of 97

Question 19.
Is 5 a factor of 60? Explain.
Answer: 5 is a factor of 60

Explanation:
Factors are the numbers that divide the original number completely.
The factors of 60 are: 1, 2, 3, 4, 5, 6, 10, 12, 15, 20, 30, 60
Hence, from the above,
We can conclude that 5 is a factor of 60

Tell whether 20 is a multiple or a factor of the number. Write multiple, factor, or both.
Question 20.
60
Answer: 20 is a factor of 60

Explanation:
Factors are the numbers that divide the original number completely.
The factors of 60 are: 1, 2, 3, 4, 5, 6, 10, 12, 15, 20, 30, 60
Hence, from the above,
We can conclude that 20 is a factor of 60

Question 21.
4
Answer: 20 is a multiple of 4

Explanation:
Multiples are the numbers that can be divided by another number a certain number of times without any remainder.
The multiples of 4 are: 4, 8, 12, 16, 20, 24, 28, 32, 36, 40
Hence, from the above,
We can conclude that 20 is a multiple of 4

Question 22.
20
Answer: 20 is a multiple and factor of 20

Explanation:
Multiples are the numbers that can be divided by another number a certain number of times without any remainder.
Factors are the numbers that divide the original number completely
Now,
Factors of 20 are: 1, 2, 4, 5, 10, 20
Multiples of 20 are: 20, 40, 60, 80, 100
Hence, from the above,
we can conclude that 20 is a multiple and factor of 20

Question 23.
Number Sense
Name two numbers that are each a multiple of both 5 and 2. What do you notice about the two multiples?
Answer: The 2 numbers that are each a multiple of both 5 and 2 are: 10 and 20

Explanation;
Multiples of 2 are: 2, 4, 6, 8, 10, 12, 14, 16, 18, 20
Multiples of 5 are: 5, 10, 15, 20, 25, 30, 35, 40, 45, 50
So,
From the multiples of both 2 and 5,
The numbers which have common multiples of both 5 and 2 are: 10, 20
The multiples of 2 have the one’s digit as 2 or 4 or 6 or 8 or 0
The multiples of 5 have the one’s digit as 5 or 0

Question 24.
Logic
A quotient is a multiple of 5. The dividend is a multiple of 4. The divisor is a factor of 8. Write one possible equation for the problem.
Answer: The possible equation is:
80 ÷ 8 = 10

Explanation:
Given that,
Quotient: Multiple of 5
Dividend: Multiple of 4
Divisor: Factor of 8
Hence, from the above,
we can conclude that the possible equation for the problem is:
80 ÷ 8 = 10

6.4 Identify Prime and Composite Numbers

Tell whether the number is prime or composite. Explain.
Question 25.
5
Answer: 5 is a prime number

Explanation;
Prime numbers:
The numbers which have exactly 2 factors 1 and itself are “Prime numbers”
Now,
The factors of 5 are: 1, 5
Hence, from the above,
We can conclude that 5 is a prime number

Question 26.
25
Answer: 25 is a composite number

Explanation:
Composite numbers:
The numbers which have more than 2 factors are “Composite numbers”
Now,
The factors of 25 are: 1, 5, 25
Hence, from the above,
We can conclude that 25 is a composite number

Question 27.
51
Answer: 51 is a composite number

Explanation:
Composite numbers:
The numbers which have more than 2 factors are “Composite numbers”
Now,
The factors of 51 are: 1, 3, 17, 51
Hence, from the above,
We can conclude that 51 is a composite number

Question 28.
21
Answer: 21 is a composite number

Explanation:
Composite numbers:
The numbers which have more than 2 factors are “Composite numbers”
Now,
The factors of 21 are: 1, 3, 7, 21
Hence, from the above,
We can conclude that 21 is a composite number

Question 29.
50
Answer: 50 is a composite number

Explanation:
Composite numbers:
The numbers which have more than 2 factors are “Composite numbers”
Now,
The factors of 50 are: 1, 2, 5, 10, 25, 50
Hence, from the above,
We can conclude that 50 is a composite number

Question 30.
83
Answer: 83 is a prime number

Explanation:
Prime numbers:
The numbers which have exactly 2 factors 1 and itself are “Prime numbers”
Now,
The factors of 83 are: 1, 83
Hence, from the above,
We can conclude that 83 is a prime number

Question 31.
Modeling Real Life
A prime number of students have which type of fingerprint?
Big Ideas Math Answers Grade 4 Chapter 6 Factors, Multiples, and Patterns chp 31

Answer: A prime number of students have Whorl fingerprint

Explanation:
Big Ideas Math Answers Grade 4 Chapter 6 Factors, Multiples, and Patterns chp 31
From the given figure,
Each half-circle = 1 student
The number of fingerprints of loop-type fingerprint: 9
The number of fingerprints of Arch-type fingerprint: 12
The number of fingerprints on Whorl-type fingerprint: 7
Hence, from the above,
We can conclude that the prime number of students has Whorl-type fingerprint

6.5 Number Patterns

Write the first six numbers in the pattern. Then describe another feature of the pattern.
Question 32.
Rule: Subtract 11.
First number: 99
99, ___, _____, _____, _____, _____
Answer:
The first 6 numbers in the given pattern are: 99, 88, 77, 66, 55, 44

Explanation:
For the formation of the pattern,
the given rules are:
Rule: Subtract 11
First number: 99
Hence,
The pattern we will obtain is:

Question 33.
Rule: Multiply by 5.
First number: 10
10, _____, _____, ____, _____, _____
Answer:
The first 6 numbers of the given pattern is: 10, 50, 250, 1,250, 6,250, 31,250

Explanation:
For the formation of the pattern,
the given rules are:
Rule: Multiply by 5
First number: 10
Hence,
The pattern we will obtain is:

Question 34.
Rule: Add 8.
First number: 15
Answer:
The first 6 numbers of the given pattern are: 15, 23, 31, 39, 47, 55

Explanation:
For the formation of the pattern,
the rules are:
Rule: Add 8
First number: 15
Hence,
The pattern we will obtain is:

Question 35.
Rule: Divide by 4.
First number: 4,096
Answer:
The first 6 numbers in the given pattern is: 4,096, 1,024, 256, 64, 16, 4

Explanation:
For the formation of the pattern,
the given rules are:
Rule: Divide by  4
First number: 4,096
Hence,
The pattern we will obtain is:

Open-Ended
Use the rule to generate a pattern of four numbers.
Question 36.
Rule: Divide by 2.
Answer:
Let the first number be 16
Hence,
The first 4 numbers of the given pattern are: 16, 8, 4, 2

Explanation:
For the formation of the pattern,
the given rules are:
Rule: Divide by 2
Let the first number be 16
Hence,
The pattern we will obtain is:

Question 37.
Rule: Add 3.
Answer:
Let the first number be 3
Hence,
The first four numbers of the given pattern are: 3, 6, 9, 12

Explanation:
For the formation of the pattern,
the given rules are:
Rule: Add 3
Let the first number be 3
Hence,
The pattern we will obtain is:

Question 38.
Rule: Multiply by10.
Answer:
Let the first number be 10
Hence,
The first  4 numbers of the given pattern are: 10, 100, 1,000, 10,000

Explanation:
For the formation of the pattern,
the given rules are:
Rule: Multiply by 10
Let the first number be 10
Hence,
The pattern we will obtain is:

Question 39.
Rule: Subtract 6.
Answer:
Let the first number be 36
Hence,
The first 4 numbers of the given pattern are: 36, 30, 24, 18

Explanation:
For the formation of the pattern,
the given rules are:
Rule: Subtract 6
Let the first number be 36
Hence,
The pattern we will obtain is:

6.6 Shape Patterns

Question 40.
Extend the pattern of shapes by repeating the rule “trapezoid, circle.” What is the 57th shape in the pattern?
Big Ideas Math Answers Grade 4 Chapter 6 Factors, Multiples, and Patterns chp 40
Answer: The 57th shape in the pattern is: Trapezoid

Explanation:
Given pattern is:
Big Ideas Math Answers Grade 4 Chapter 6 Factors, Multiples, and Patterns chp 40
The rule given for the pattern is: trapezoid, circle
So,
The total number of figures in the given pattern is: 2
So,
for this, when we divide any number by the total number of patterns, the value of the quotient will be a total repetition of the patterns and the remainder will be the counting of the pattern from the 1st figure.
So,
By using the partial quotient method,
57 ÷ 2 = ( 40 + 16 ) ÷ 2
= ( 40 ÷ 2 ) + ( 16 ÷ 2 )
= 20 + 8
= 28 R 1
Hence, from the above,
we can conclude that the 57th shape in the pattern is: Trapezoid

Question 41.
Extend the pattern of shapes by repeating the rule “top left, top right, bottom right, bottom left.” What is the 102nd shape in the pattern?
Big Ideas Math Answers Grade 4 Chapter 6 Factors, Multiples, and Patterns chp 41
Answer: The 102nd shape in the pattern is: Bottom left

Explanation:
The given pattern is:
Big Ideas Math Answers Grade 4 Chapter 6 Factors, Multiples, and Patterns chp 41
The rule given for the pattern is: top left, top right, bottom right, bottom left
So,
The total number of figures in the given pattern is: 4
So,
for this, when we divide any number by the total number of patterns, the value of the quotient will be a total repetition of the patterns and the remainder will be the counting of the pattern from the 1st figure.
So,
By using the partial quotients method,
102 ÷ 3 = ( 99 + 3 ) ÷ 3
= ( 99 ÷ 3 ) + ( 3 ÷ 3 )
= 33 + 1
= 34 R 0
Hence, from the above,
We can conclude that the 102nd shape in the pattern is: Bottom left

Question 42.
Describe the pattern. How many squares are in the 61st figure?
Big Ideas Math Answers Grade 4 Chapter 6 Factors, Multiples, and Patterns chp 42
Answer: The number of squares in the 61st figure is: 3

Explanation:
The given pattern is:
Big Ideas Math Answers Grade 4 Chapter 6 Factors, Multiples, and Patterns chp 42
In figure 1, the total number of squares = 3
In figure 2, the total number of squares = 6
In figure 3, the total number of squares = 9
So,
The total number of figures = 3
So,
for this, when we divide any number by the total number of patterns, the value of the quotient will be a total repetition of the patterns and the remainder will be the counting of the pattern from the 1st figure.
So,
By using the partial quotients method,
61 ÷ 3 = ( 57 + 3 ) ÷ 3
= ( 57 ÷ 3 ) + ( 3 ÷ 3 )
= 19 + 1
= 20 R 1
Hence, from the above,
We can conclude that the number of squares in the  61st figure in the pattern is: 4

Question 43.
Structure
Draw the missing figure in the pattern. Explain the pattern.
Big Ideas Math Answers Grade 4 Chapter 6 Factors, Multiples, and Patterns chp 43
Answer: The missing figure has: 7 squares

Explanation:
The given pattern is:
Big Ideas Math Answers Grade 4 Chapter 6 Factors, Multiples, and Patterns chp 43
The rule-following in the given pattern is: Add 2
The first number: 1
According to the rule,
The number of squares in the pattern will be like 1, 3, 5, 7, 9

Conclusion:

We hope the details mentioned in the Big Ideas Math Answers Grade 4 Chapter 6 Factors, Multiples, and Patterns are helpful for you to finish your homework in time. Learn the concepts in depth so that you can solve any kind of problem easily. If you have any queries you can post your comments in the below section.

Big Ideas Math Geometry Answers | Download Geometry Big Ideas Math Book Answer Key

Big Ideas Math Geometry Answers

Big Ideas Math Geometry Answers bridge the gap to your success. You can make use of the Geometry Big Ideas Math Textbook Solution Key PDF via quick links available and score better grades in exams. Don’t bother about the accuracy of the BIM Math Book Geometry Answers as they are given by experts after ample research. You can assess your strengths and weaknesses using the Geometry Big Ideas Math Answers and ace up your preparation accordingly.

Big Ideas Math Geometry Answer Key | Geometry Big Ideas Math Solutions

High School Students seeking homework help on the concepts of BIM Geometry Solutions will find the study material over here of great help. Utilize the quick links over here and start practicing from now itself. Geometry Big Ideas Math Solutions Key provided covers questions from all the Chapters aligned as per the Textbooks. Experts have prepared these Big Ideas Math Geometry Answer Key as per the latest Common Core Syllabus Guidelines.

Advantages of referring to Big Ideas Math Answer Key for Geometry

There are plenty of advantages of referring to our Geometry Big Ideas Math Solutions. They are outlined in the following fashion

  • Geometry BIM Textbook Solutions are given by experts meeting the Common Core State Standards Curriculum.
  • Answer all kinds of questions right from Performance Tests, Chapter Tests, Practice Tests, Review Tests, easily by solving from our Big Ideas Math Answers.
  • Become Pro in the Subject and develop problem-solving skills among you.
  • You can download the Geometry Big Ideas Math Answer Key PDF over here for free of cost and ace up your preparation.

FAQs on High School BIM Geometry Solutions

1. Where do I get Chapterwise Big Ideas Math Geometry Answer Key for free?

You can get Chapterwise BIM Geometry Answers for free of cost on Bigideasmathanswer.com

2. How to access the High School Big Ideas Math Geometry Solutions?

You can access the High School Big Ideas Math Geometry Answers via quick links available on our page. Just tap on them and prepare accordingly.

3. Can I Score better grades by practicing from BigIdeas Math Textbook Geometry Answers?

Yes, can score better grades by practicing from BigIdeas Math Textbook Geometry Answers.

Big Ideas Math Answers Grade K | Big Ideas Math Book Grade K Answer Key

Big Ideas Math Answers Grade K

Download Kindergarten Big Ideas Math Textbook Answer Key from here. You have different opportunities to problem-solving and communication skills through explore and grow, think and grow. Apply the concept of maths in real-time and enhance your skills. With the help of Big Ideas Math Answers Grade K, you can perform well in practice tests, Assessment tests, and chapter tests. BigIdeas Math Answers Grade K is an essential teaching practice for elementary school students.

Download Big Ideas Math Answers Grade K Pdf | BIM K Grade Answers

Students can learn the basics or fundamentals of maths from our BIM Grade K Solution Key. Download Big Ideas Math Answers Grade K for all Chapters for free of cost. Get BIM K Grade Answers Chapter Wise from this page. Students or parents can make use of the resources on this page to complete their homework or assignment in time. All you have to do is to click on the below links and start preparing for the exams. Make yourself comfortable by posting your doubts @ bigideasmathanswers.com

Common Core 2019 – Grade K

Top 5 Preparation Tips for Exams

  1. Study the subject from the best material.
  2. Write revision notes after the completion of each chapter.
  3. Fix the timetable and plan for preparation.
  4. First, you have to organize the study space
  5. Take breaks regularly for your relaxation.

Importance of Bigideas Math Answers Grade K Solution Key

The advantages of using Bigideas Math Answer Grade K are as follows,

  • Students can understand the concepts in-depth and also score maximum marks in the exams.
  • By referring to our BIM Grade K Answers students can enhance the performance skills and become masters in math.
  • Big Ideas Math Answers for Grade K helps parents to make their kids learn the concepts with more concentration.
  • Test your knowledge by solving the question given at the end of the chapters.

FAQs on BIM Answers Grade K

1. Where can I Get the best Bigideas Math Answers for Grade K?
Students of Elementary School Grade K can get the best Answers with explanations on the bigideasmathanswers.com website.

2. What are the benefits of Bigideas Math Kth Grade Solution Key?
It helps you to secure the highest marks in the exams and also improve your knowledge by referring to Big Ideas Math Answer Grade K from bigideasmathanswers.com

3. Can I get the Download BIM Answers for Grade K Chapterwise in Pdf Format?
It is very easy to Download BIM Grade K Answer Key according to the list of the chapters in pdf format. Just tap the chapters link your chapter will be downloaded in pdf format.

Big Ideas Math Answers Grade 1 | Big Ideas Math Book 1st Grade Answer Key

Big Ideas Math Answers Grade 1

Learning maths is always fun and interesting for kids. Big Ideas Math Book 1st Grade Answer Key is the best option to educate elementary school kids and make them understand the basic concepts of maths like addition, subtraction, multiplication, division, etc. We are using the latest research to make you understand the concepts and achievement of maths. The solutions are prepared from Big Ideas Math – Modeling Real Life Grade 1 Student Edition Set. Learning targets and success criteria help the students to focus on learning the subject. So, Download Big Ideas Math Answers Grade 1 Chapter Wise from this page.

Big Ideas Math Book Grade 1 Answer Key | BIM Answers 1st Grade Pdf

Get the Solutions for all Primary School Big Ideas Math Grade 1 Chapters from here. This will help you to learn the Real-time math applications by this you relate the math with everyday life. Solving the problems is not important you have to understand the concepts and know-how to solve the problems from the basic level itself. If you try to understand the concept you will become a master in maths. Just click on the chapter you wish to practice and Download Big Ideas Math Grade 1 Answers Pdf for free of cost. Also, know the preparation tips and benefits to score maximum marks in the exams from this page.

What is the Advantage of BigIdeas Math 1st Grade Answer Key?

There are a lot of benefits of referring to Big Ideas Math Answer Grade 1. We have enlisted few advantages of the BIM 1st Grade Answer Key in the below section.

  • Big Ideas Math Answers Grade 1 is designed by the subject experts.
  • These BIM 1st Grade Answers are very easy to grasp and educate the kids with all basic math concepts.
  • Quick learning is possible with Big Ideas Math 1st Grade Solution Key Chapterwise in the pdf format. Get Free Access to Download BIM 1st Grade Chapter Wise Answer Key from here.

FAQs on Elementary School Big Ideas Math Grade 1 Answers Pdf – A Common Core 2019 Curriculum

1. Where do I find Chapter-wise Elementary School Big Ideas Math 1st Grade Answer Key?

You can find step by step explanations of all BigIdeas Math Modeling Real Life Grade 1 Answers for all chapters from our site bigideasmathanswers.com

2. Is it possible to download A Common Core 2019 Curriculum Bigideas Math Grade 1 Answer Key PDF?

Yes, it is possible to download BIM Grade 1 Answer Key by accessing the quick links available on bigideasmathanswers.com With one click you can access & download the respective chapter Big Ideas Math Answers Grade 1 in pdf format.

3. Our provided Free Easy Access Student Edition of Bigideas Math 1st Grade Solutions are enough to practice?

Yes, bigideasmathanswers.com provided Free Access Student Edition of Big Ideas Math Grade 1 Solution Key are enough to practice more & get a good knowledge on the subject.